t w j«myffN4»;i4iiiimHii i FOR S ECO^Mf^ SGH 00 IJ THURSTON CMILLAN'S OMMERCIAL GIFT OF f BUSINESS ARITHMETIC FOE SECONDAEY SCHOOLS ^"^^v^- BUSINESS AEITHMETIC FOR SECONDARY SCHOOLS BT ERNEST L. THURSTON SUPEBINTBNDENT OF PLBLIC SCHOOLS, DISTRICT OF COLUMBIA "^ ^•> . « ' Neto ¥0rlt THE MACMILLAN COMPANY LONDON: MACMILLAN & CO., Ltd. 1917 Copyright, 1913, By the MACMILLAN COMPANY. Set up and electrotyped. Published March, 1913. Reprinted August, September, igi3; April, August, 1914; February, 1916; August, 1917. • • * t • • • • Norteooti ^rtjjji: Berwick & Smith Co., Norwood. Mass., U.S.A. PREFACE. The use and applications of number vary with the changes and development of our industrial, commercial and social relations. The arithmetic of to-day is not the arithmetic of ten years ago. The fundamental principles and processes do not alter, but new applications constantly develop and older applications lessen in usefulness. In this sense, arithmetic is a living subject which deals with living phenomena. In these days arithmetic is commonly used not only for the necessary routine computations incident to our private, busi- ness, or scientific affairs, but its processes are called upon to study facts through number and to interpret these facts — to put number into things rather than to take number from them. Moreover, the modern use of number often takes the form of a language, as in the construction or reading of numerical illustrations, and statements or statistics which are designed to impress upon others certain facts or conditions. Thus modern arithmetic may be said to have a use as a tool, i. e., for routine computation, an interpretive use, and a language use. But accuracy and facility in the use of most working instru- ments presuppose a knowledge of the fundamental parts of the instrument, of the interrelation of these parts, of their working principles, and of the classes of material or work to which they may be applied. Thus, facility in the three " uses '* of the arithmetic instrument is founded, primarily, on a logical working knowledge of the fundamental operations and prin- ciples. This is the more necessary because the arithmetic problems of actual life do not have a text-book form of state- ment. If formulated at all, they may offer no direct clue to a particular subdivision of the subject. Often the problem must first be formulated, at least mentally, the power of 4f?9940 VI PREFACE. selection being exercised to choose from a group of number facts those essential to the computation desired. This book is an attempt to construct the arithmetic instru- ment logically, in its simplest form, as a branch of mathematics; then to apply this tool to general computation, and to the study, preparation and interpretation of varied number material. Involved in this is the purpose to develop arith- metic as a language of business, or as a means of interpretation and study of business, and economic conditions; to acquire the capacity properly to present numerical facts by tabulations or graphs; to cultivate clearness of thought and expression, and an appreciation of the value of order and system in applied number work. In the preparation of the subject matter of this book, no attempt has been made to draw from other text-books, al- though much may be found here that is in harmony with them. Material has been gathered only from living and reliable sources. The material has been selected with the purpose of emphasizing how arithmetic is applied for a definite object. The problem exercises are not intended, as is so often the case, as a means of conveying to the mind indigestible information on most known subjects. The author believes that the book will be found none the less rich in content because it aims at the digestion of facts, rather than at surfeiting with them. Much thought has been given to the problem work and, as one result, considerable variety will be noticed in form of statement. Thus the old time "problem in question form" is used, also the incomplete statement, the statistical table, the business form, the written article, the memorandum, etc. Place is also given to series or related problems, composite problems, and "central" problems, to be viewed from different standpoints. Exercises in the preparation of original problems are included for the purpose of cultivating powers of selection PREFACE. vii and statement, and of distinguishing between essentials and non-essentials. Mental development, and growth in power of analysis, must necessarily follow the doing of the work that these exercises require. A method of development has been followed, in condensed and simplified form, and rules have been reduced to an absolute minimum. It has been assumed, however, that the pupil has had considerable experience in grade arithmetic, before under- taking the study of this book. For this reason illustrative examples are frequently given only in outline. To give facility in the fundamental operations and main processes, a considerable quantity of abstract work has been introduced. In this line, special emphasis has been laid on the fundamental processes as applied to whole numbers, decimals and fractions. Short methods have been emphasized for paper work, and methods of checking have been introduced or suggested. It is felt that every important computation should be checked. Oral exercises, planned with the utmost care to serve as models, have been given throughout the book, but they should be supplemented by further work along the lines that they suggest. The exercises should vary in form, being, at dif- ferent times, sight exercises, exercises by dictation, and exer- cises based upon some fundamental expressions that are placed upon the board, and around which individual examples are built. Many types of exercises will be found illustrated in the following pages. In all oral work, it should be remembered that facility will come (1) from constantly varying the simple number com- binations, or "ringing the changes" on fundamentals, and (2) from fitting the examples to the individual pupil, giving him something that he can do and that will develop him in the doing. Attention to these requirements will bring a rich return in the increased capacity of the pupils. The author wishes to acknowledge his indebtedness to Mr. viu PREFACE. Charles Hart, head of department of business practice, Busi- ness High School, Washington, D. C, who read the first proof, and whose wise counsel and careful criticism have been helpful. E. L. T. Washington, D. C. EDITOR'S INTRODUCTION. No subject of the curriculum has received recently more discussion and criticism than has arithmetic. In that most suggestive book "School and Society'' John Dewey dwells on the facts that there should be established a natural connection of the everyday life of the child with "the business environ- ment of the world around him," and "it is the affair of the school to clarify and liberalize this connection." Observation confirms the dictum of John Stuart Mill that the child is by nature logical — he seeks the reasons for things. The teaching of mathematics as an abstract science based on phenomena outside of the child's experience serves to dull his natural logical habit of mind. The three-fold aim of teaching arithmetic is now generally conceded, and a new text can scarcely fail to take these into account. First, it should give the ability to compute correctly and with some rapidity. Second, it should sustain the interest of the learner, that is, arithmetic should present a body of knowledge that is fresh and attractive. Third, the teaching of arithmetic should develop the power of applying number concepts to the activities of everyday life. The statement of these ideals is easy: the difficulties come in their realization. Arithmetic can be made an instrument of education, both for what has commonly been termed culture and for practical affairs. No school study offers better oppor- tunities for teaching pupils to think or for directing their thinking to the sort of phenomena with which they will deal when school is at an end. The author and the editor of this book recognized these ideals when the book was planned, eight years ago. In preparing the book certain fixed prin- ix X EDITOR'S INTRODUCTION. ciples have been borne in mind. One is Herbert Spencer's — that "so far as possible the child should make his own in- vestigations and draw his own conclusions." Constantly in this book the pupil is asked to investigate for himself, to for- mulate or arrange his data, and to deduce his own conclusions. Secondly, this book is built on the assumption that through the use of concrete problems, it is possible to do three things at once, namely, to give drill, to keep up interest, and to train in the facility of practical application. In the selection of material little or no regard has been given to text-books here- tofore existing. Problems have been drawn almost entirely from home, store, farm, shop, the engineer's office, and other like sources. We have progressed too far educationally to spend time in arguing that it is saner to teach pupils to think on the data that they will later use, than to try to teach them from the obsolete data of the books. Not only does the first method arouse more interest; it gives the only assurance that the pupils will be trained for practical affairs. The author would seem to have had an almost ideal prep- aration for the writing of this book. In the first place he was broadly trained in mathematics, and for several years had experience as a teacher in an institution of higher learning. In addition to this he had some years experience as a teacher of arithmetic in the Business High School at Washington, and more recently has served as a supervisor in methods of teaching in elementary schools. Best of all he is a trained and careful investigator, who has been willing to forsake the traditions of arithmetic to work out by practical investigation a book along new lines. This book gives but limited attention to the fundamental operations, and the attention that is given is only to emphasize the relative value of the fundamental processes. As with the other book of the series, decimals are made to follow im- EDITOR'S INTRODUCTION. xi mediately after whole numbers, thus combining these closely related parts of our notation. The difficulty with much arithmetic teaching is that the pupil considers arithmetic entirely a matter of the book, having nothing to do with practical affairs. The pupil who works through this book should have an intelligent notion of the modern organization and operation of business, a grasp of himself and arithmetical processes, and such facility in the application of these processes as will enable him to go out and make himself useful in the work-a-day world. C. A. H. GiRARD College, January 1, 1913. TABLE OF CONTENTS. Chapter Pagb I. Units and Numbers 1 II. Notation and Numeration 3 III. Addition . . 7 IV. Subtraction 18 V. Multiplication 25 VI. Division 35 VII. Arithmetical Averaging 43 VIII. The Equation 46 IX. United States Money 49 X. Making Change 54 XL Postage 56 XII. Payment for Services 61 XIII. Business Terms and Accounts 72 XIV. Advertising 77 XV. Factors and Multiples 86 XVI. Reduction of Fractions 91 XVII. Fractions — Fundamental Processes — Con- version « 95 XVIII. Fractional Relations of Numbers 106 XIX. Aliquot Parts 110 XX. Problem Analysis and Solution 118 XXI. Involution and Evolution 121 XXII. Denominate Numbers 132 XXIII. Practical Measurements 143 XXIV. Measurement of Time '. 164 XXV. Practical Measurements — Temperature, Composite Units, Formulae 175 XXVI. The Metric System 182 xiii XIV TABLE OF CONTENTS. Chapter Page XXVII. Ratio and Proportion 186 XXVIII . Graphic Arithmetic — Scales, Plotting, Graphs 193 XXIX. Percentage 213 XXX. Profit and Loss 225 XXXI. Commercial Discount 236 XXXII. Agency 245 XXXIII. Insurance — Personal and Property 255 XXXIV. Taxation and Public Revenue — Local, State and National 269 XXXV. Interest — Simple 285 XXXVI. Interest — Compound 302 XXXVII. Loans and Payments 306 XXXVIII. Savings Accounts 326 XXXIX. Stocks and Bonds 332 XL. Domestic and Foreign Exchange 355 XLI. Depreciation 375 XLII. Cost-Keeping 379 XLIII. Bids and Estimates 383 XLIV. Partitive Proportion and Partnership 387 XLV. Equation of Payments and of Accounts. . . . 397 XLVI. Billing 403 XLVII. Storage. . .^ 414 Appendix I. Signs and Symbols 419 II. Standard Abbreviations 420 BUSINESS ARITHMETIC FOR SECONDARY SCHOOLS BUSINESS AEITHMETIO FOE SECONDAEY SCHOOLS CHAPTER I. UNITS AND NUMBERS. In applied arithmetic, constant use must be made of units of value and of measure. Unity means one-ness, but a unit is not necessarily "one," or a single thing, for it may be any quantity definitely measured or stated. One dollar is a unit, but the dollar is one hundred cents, or a certain number of unit grains of gold and alloy, that is, a collection of other units. In practice, units vary with the things with which they are associated. A retailer sells goods by the can — the can is his selling unit; but he buys by the case, say of 24 cans, and the case may be termed his buying unit. The idea of a unit as a definite quantity is emphasized in many occupations, as, for example, in engineering, where a unit distance is often taken as 100 or 1000 feet. It is evident that all units are divisible. If the unit is one or a quantity considered as one, it is termed an integral unit. If an integral unit is divided into tenths, hundredths, etc., one of the parts so obtained is termed a decimal unit. If the integral unit is divided into any number of equal parts, it is termed a fractional unit. A number is a unit or a collection of units. It is therefore integral, decimal or fractional according to the character of units of which it is composed. If the number is entirely unaf- fected by association with any object or expression it is an 2 1 2 • BtSINESS ARITHMETIC. ahstr'act ''riurrvber: ' (]6±ainples : 87, 34-) If the number is associated with some idea it is concrete. (Example, 8 foxes.) If the associated expressions are those of standard measures, the number is denominate. (Example, 12 yards.) Numbers are like or unlike, according as they have the same or different unit values. Thus "five months" and "eight months" are like numbers, while "Rve months" and "eight pounds" are unlike. Numbers are simple if they contain units of the same kind; they are compound if they contain different forms of units which are reducible to a single form of unit. Thus "4 lb." is simple, but "4 lb. 8 oz." is a compound number which may be reduced to the simple number "72 oz." EXERCISE. 1. Name six units of measure. 2. Name five units employed in different trades or professions. 3. Classify these numbers and name the unit in each case: (a) 976. (d) $3.85. (g) 53^ gal. (6) .08. (e) 5 yd. 2 ft. (h) U lb. (c) 45 lb. (/) 276 payments. (i) 4 horses, 2 mules. 4. Name and classify the numbers and units in these expressions: (a) "The cost of this cloth is 55 cents per yard." (6) "The goods are packed in cases of 48 boxes each." (c) "The temperature reached 96° to-day." id) " It will take at least half-an-hour and probably forty minutes." (e) 240 is H of 480. CHAPTER II. NOTATION AND NUMERATION. INTRODUCTORY EXERCISE. 1. How do the expressions "3 yd." and ".03 yd." differ in meaning? 2. Add ciphers to the numbers "40" and ".04" in such a way as not to alter their values. 3. Where does the insertion of a cipher cause a change of value? 4. In expressing numbers orally, what are the values of the syllables "teen" and "ty"? Arabic Notation. The Arabic or Hindu notation is the system universally used in civilized countries. The significant figures or digits, 1 to 9, were in use in India 2,000 years ago, but calculations were performed with extreme difficulty until the invention of the tenth symbol, the cipher or zero, 800 years later. The zero made modern computation possible. In the seventeenth century, the invention of the decimal point made possible the present varied use and application of number. The present tendency is toward a continually broadening use of decimals. The "significant" figures of the system are those that have a name or face value, and a position value. The zero serves only to alter the position value of other figures. PRINCIPLES OF USE. 1. The name value of each significant figure 'm constant. 2. The position value increases in ten-fold ratio from right to left. Note. Hence called the "decimal system," from the I^atin decern meaning "ten." 3. A decimal point, or period, separates units and tenths of imits. Note. The portion of the number to the left of the decimal point is termed the whole number or integer; the portion to the right is the decimal. 3 4 BUSINESS ARITHMETIC. 4. The zero increases the position of significant figures to its left ten fold; it decreases the value of significant figures in a decimal, to its right, to one-tenth their previous value. 5. In any number, where a significant intermediate figure is lacking, a zero is written in its position. 6. Place values are named under the French or English system. Under the French system, used in this country, the figures of large numbers are divided into groups or periods of three figures each, as follows: Place 131211109 8 7 654 321 . 123 456 Name g a fl 2 iSl Jl_ 211 ^ ^ II Um C^Srs !=^c3li: SS^ J^sa a». o ^,C S^'^i H WH« «h:^ teHH wh;i3 q hwh hw^ Period BilUons Millions Thousands Hundreds Thousandths Millionths Illustration of Numeration. The numbers should be read in the simplest way, the word "and," or "decimal," being used for the decimal point, thus: 14,264.216 is read: Fourteen thousand, two hundred, sixty-four and two himdred sixteen thousandths. 2,132. is read: Twenty-one hundred, thirty-two. 3.02 is read: Three and two hundredths. 30,000.003 is read: Thirty thousand and three thousandths. Note. In English tables, the periods contain six places each, called units, millions, etc. Under this system the number 36,000,000,000 would be read: Thirty-six thousand milHons. EXERCISE. 1. Which is the larger, 3 yd. or .030 yd.? 2. How does the decimal point determine the size of the part taken? 3. Read: 4. Read: 6. Read: 52,608 2.43 30.0009 1,292,723 17.061 2.0703125 329,624,006 228.0406 .627724 50,007,064,329 5,060.38001 2.390 6. Read by both English and French systems: 472,693,684,721,125.001. NOTATION AND NUMERATION. 5 7. Insert a cipher in the number 17,246 in five different positions in turn, and read the results. In which position has the cipher the least effect? The greatest? 8. Insert a cipher, in turn, between each two significant figures in the number 86.043201, and read the resulting numbers. 9. Read .4 as hundredths; .65 as ten-thousandths; .025 as hundredths; .002 as millionths. 10. What is the decimal of lowest value, that can be written with the figures 3 and 7 and foiu- ciphers? Note. In writing large numbers, separate periods by commas, or by spacing. 11. Write in colunm: Seven billion, two hundred forty-eight million, seventy thousand, eight hundred twenty-eight; two hundred twenty-six million, four hundred thousand, thirty; eighteen hundred thousand, four; six ten-thousandths; five and four milUonths; sixty-three thousand and six hundred thirty hundred-thousandths; four hundred fifty-nine and eight hundred eleven ten-thousandths; fifty-four tenths; fiftj^-two hundred- thousandths. 12. Write, substituting numbers for numerical words: (a) The cir- cumference of a circle is three and fourteen thousand, one hundred fifty- nine hundred-thousandths times the diameter. (6) A nautical mile is one and one hundred fifty-two thousandths of a statute mile, (c) The total elongation of a bar on a steel bridge, under a maximum load, was two hundred fifty-six thousandths inches. Roman Notation. The Roman system employs as number symbols seven letters of the Roman alphabet. These characters have a name value and a variable position value. I V X L C DM 1 5 10 50 100 500 1000 PRINCIPLES OP USE. 1. Combinations of symbols are written from left to right in order of value. The total value is the sum of the constituent values. Illustrations. CX = 100 + 10 = 110. MDCC = 1000 + 500 -f 100 + 100 = 1700. 6 BUSINESS ARITHMETIC. 2. To avoid the consecutive use of symbols four times, a letter of less value may be written before one of greater value, the combination being equal to the difference of the name values. Illustrations. L = 50, but XL = 50 - 10 = 40. IX = 10 - 1 = 9., 3. The mark " — " over a symbol multiplies its value by 1000. Illustration. XIV = 14, but XIV = 14,000. EXERCISE. 1. Write the Roman numerals from 1 to 100. 2. Spell the Roman numerals from 1854 to 1913. 3. The corner stone of a pubUc building is marked MDCCCLXVIII. It was laid in what year ?. 4. Write in Arabic notation: XIV; XCVI; MDVIII; MCDIX: XXlfl; CX; DCXII. 5. Name three common uses of the Roman notation. 6. Name objections to the general use of the system. CHAPTER III. ADDITION. The fundamental operations on which the working methods of arithmetic are based are: (1) addition; (2) subtraction; (3) multiplication; and (4) division. These processes are applied to whole numbers, decimal fractions and common fractions. They will be considered, first, in their relation to whole numbers and decimals. Addition is the process of combining two or more numbers into a single equivalent number, called the sum, the amount, or the total. It is the most universally employed fundamental process, and should be performed with absolute accuracy and reasonable speed. From a business standpoint, this accuracy and speed depend on: (1) careful formation of figures; (2) alignment of figures; (3) capacity to give and take number dictation; (4) capacity to copy figures accurately; (5) working knowledge of "grouping"; and (6) continued, systematic practice. Careful Writing. Figures should be uniform in size, legible, simple and distinct in form, evenly aligned vertically and horizontally, and closely spaced but not crowded. EXERCISE. 1. Give reasons for the requirements just stated. 2. How do carelessly formed figures reduce speed in addition? 3. Write an original column of 6-place figures. (Exchange papers and criticise for form.) 4. The money column on page 331 is "unit ruled." What is unit ruling? Of what value is it in addition? Number Dictation is frequently incidental and preparatory to addition. One clerk may dictate values to another, who 7 8 BUSINESS ARITHMETIC. writes them in column, or "checks off" a list he holds. In dictation, numbers must be clearly enunciated in an even tone, without unnecessary words, and at uniform speed. SUGGESTION FOR TEACHERS. 1. Dictate a column of figures. Have pupils check accuracy by "dictating back." 2. Let pupils dictate original columns to each other. Later, check by addition. Copying is more common than dictating. Office clerks are continually transferring figures from one record to another. Thus a bank clerk lists a depositor's checks, or a book-keeper posts from a Cash Book to a Ledger. SUGGESTION FOR TEACHERS. 1. Have the class copy a column of blackboard figures. Check by dictation. 2. Exchange original columns for copjring. General Addition. The mental process of adding consists in grouping digits of the same order. The thought should be placed on the result. In adding 5, 8, 9 and 7, think of each sub-total (13, 22, 29). Facility in addition is gained by reading instantly any two or three digits as a sum. Thus, "8 + 9" should be read "17." DRILL TABLE. The forty-five two-figure combinations: Name sums at sight. 74241343314221189856455 76537623251213199851434 7 1 5 6 6 8 9 8 7 7 4 9 ' 7 6 7 5 3 2 4 5 7 6 2 8 6 6 9 6 1 2 3 5 8 3 8 7 9 9 8 9 9 8 4 2 ADDITION. EXERCISE. 1. Name the totals. 4 + 8, 6 + 9, 3 + 1, 4 + 7, 2 + 8, 1 + 7, 3 + 9, 4 + 5, 4 + 4, 14-2 + 3, 2+4 + 3, 5 + 2 + 7, 6 + 7+3, 8 + 2+4, 1 + 1+3+2, 1+8 + 8. 2. List all possible groups of two or three digits equaling ten. Leam them. 3. Develop systematically a list of the 165 possible combinations of three digits. Illustration. 1 + 1 + 1, 1 + 1+2, 1 + 1 + 3, etc. 4. Give, orally, the consecutive totals obtamed by adding: 3s to 58 (58, 61, 64, etc.), 4s to 127, 9s to 156, 8s to 69, 7s to 32, alternate 3s and 8s to 14, alternate 2s and 4s to 52. (Limit 300) Grouping. In adding columns of figures, it is advisable to group digits into sub-totals, and to combine these sub-totals. The grouping may be consecutive, irregular or by tens and twenties. Illustrations. L 11. III. 3\ 4\ 5 (I.) Read instantly the joined numbers as single 6 / 9 J 9 \ numbers. The consecutive totals, downward, are : 9\ 3/ /4j 9,20,30,38,48. 2 y 8 \ ^ ^ (I^*) ^^^^ upward, grouping simple numbers, 4\ 7\ ^6 even if non-consecutive. Totals: 7, 16, 25, 33, 40,. 6/ 2) 8\ 49. 3 \ 9 / 3 I (III.) Since "tens" are easier to add than irreg- 6/ Q\ I 2' ular combinations, group "tens" as shown. 2\ 1 / \ 7 Totals upward: 5, 15, 25, 35, 45, 50. 1 ) 49 _5 J^J 50 48 It is frequently necessary to combine groups of two-place numbers. Some accountants regularly add double columns in place of single columns. Until totals are possible by in- spection, add the tens and then the units, thus: 46 + 57 = 40 + 50 + 6 -t- 7. 10 BUSINESS ARITHMETIC. ORAL EXERCISE. Name the sums of the following: 36 21 84 73 27 62 81 94 78 55 43 16 21 62 43 67 43 69 72 69 41 25 73 18 63 57 54 11 21 29 13 58 — — — — — — — — — — — 14 10 71 2 11 Complements. The complement of a number is the dif- ference between it and the lowest number of the next higher order. Thus the complement of 8 is 10 — 8, or 2; the com- plement of 87 is 100 — 87, or 13. Complements are an aid to rapid addition. Illustration. 376 + 96 = 376 + 100-4. ORAL EXERCISE. 1. Name the complements of the following: 7, 21, 36, 64, 77, 129, 973, 9959. 2. When is it advisable to use the complement in mental addition? 3. Add by complements: 64 + 97, 88 + 93, 173 + 98, 77 + 84, 568 + 92. EXERCISE. Add these colunms without copying figures, groupmg simple numbe 1. 2. 3. 4. 5. 524 3164 629 234,567 1 234 567 689 43 61324 123,456 2 456 673 306 721 399 912,345 5 928 431 1247 2903 2401 891,234 4 241 326 1201 8127 88 789,123 5 261 252 3824 655 1120 678,912 4 683 392 6735 432 21205 567,891 9 221 243 3289 288 4630 456,789 4 322 881 6476 179 880 345,678 3 333 333 6. Why is column 1 easier to add than column 3? 7. What aids to quick addition are found in columns 4 and 5? 8. Write two original columns, one of 20 six-place figures, the other of 60 two-place figures. Add. Which example is more difficult? Why? 9. Name ten simple applications of addition, in everyday life. ADDITION. 11 Checking. All addition should be "checked," or tested by one of the following methods. (a) Reverse direction. Add down after adding up, etc. Why? (6) After adding by consecutive numbers or groups, group by tens. Why? (c) Divide the column at one or more points by horizontal lines. Determine the total of each partial column. Set out these totals and add. (d) Add the unit column, writing the total at one side; add the "tens" column independently, offsetting the total as shown. Continue the process. The total of the sub-totals should equal the column total. This method is also used for direct addition by one who is likely to be interrupted frequently. One may stop add- 13168 13168 ing at any point, and continue again when opportunity offers. (e) Casting or counting out nines. To cast out nines from a number, add its digits and when- ever the sum equals or exceeds 9 discard 9, adding the re- mainder to the succeeding digits, etc. To check addition, discard the nines from the numbers added and also from their sum. Discard the nines from the remainders of the numbers added. If the final remainder equals the remainder of the sum, the sum may be presumed to be correct. 3216 18 2479 15 3821 20 3652 11 Illustration. Final Remainder. 426 3 824 5 936 1542 3 3728 2, check remainder. 12 BUSINESS ARITHMETIC. WRITTEN EXERCISE. Below is shown a portion of the circulation statement of a newspaper, for a recent year. Find and check the monthly totals, using the methods just outlined. Date January February March April May June 1 45578 59471 60105 60150 58408 69636 2 505Jt2 59723 59886 48302 59392 66880 3 56551 59116 60036 59937 59029 56746 4 57318 59400 61096 59887 58713 46006 5 57273 49249 49512 61030 58395 57192 7 57942 59522 59469 60105 47000 57745 8 46937 59400 60107 6O4OO 68410 57679 9 57771 59281 60484 47861 58910 67163 10 57909 59613 59960 6O415 686I4 66662 11 58330 59395 59837 60634 58164 46349 12 58533 59035 48821 60485 68084 57165 13 58106 59372 601 49 60332 67966 66571 U 58428 59306 59575 59953 47006 57876 15 48O47 59536 59680 59939 67966 57957 16 57797 59469 59839 47505 58102 57859 17 57945 59470 60663 58577 57664 66226 18 58443 59477 59950 60825 57335 46547 19 58696 49410 48875 60134 57349 57287 20 58393 59526 6OI43 61332 56774 57684 21 58697 59394 60329 60874 46346 57107 22 48676 53016 60260 60504 67296 67208 23 58396 60116 59825 48023 57197 56767 H 59334 59823 60366 59989 57927 56350 25 59015 59934 60060 69751 67884 46069 26 59453 49187 48110 59658 57362 56293 27 59544 59981 60250 59170 57281 56989 28 59075 69732 60023 69147 46522 67203 29 49327 60569 68858 57068 66927 30 59580 60316 47412 49684 56919 31 59381 60064 57070 JUiAfy ibia£ Horizontal Addition. It is often necessary to find the sum of several numbers written in a horizontal row, without re- writing them in vertical columns. Such addition is common in billing, accounting and statistics. Illustration. 2472 3014 5207 Total Col. 13962 ADDITION. 13 In adding, follow the usual rule. Add the units (22) writing the unit value of the total and carrying '2'; add the tens, etc. Be careful of positions after passing the second place. EXERCISE. Sales recapitulation sheets are used in department stores, factories, etc., to sum up, for jBxed periods, the total sales of each department, of each class of output, of each salesman, etc. Their totaling often involves both horizontal and vertical addition, as in the following: Recapitulation of Department Sales. February, 19 — . Day Dept. A Dept. B Dept. C Dept. D Dept. E Dept. F Totals 1 $460.77 $344.00 $450.55 $305.50 $355.06 $455.56 ? 2 544.04 550.00 405.60 366.67 460.60 758.24 3 477.07 637.89 400.33 536.56 367.89 457.65 5 477.08 460.70 512.45 435.56 404.40 460.70 6 546.87 315.85 428.67 345.67 546.82 467.19 7 478.89 211.81 543.78 377.69 512.87 601.03 8 213.56 110.87 232.16 215.46 127.83 105.68 9 136.46 98.67 205.38 122.69 200.80 149.03 10 568.67 511.84 690.57 477.89 624.35 589.98 12 477.21 647.29 489.39 436.29 527.69 672.39 13 567.85 437.27 512.11 532.56 455.21 532.24 14 547.54 512.37 544.65 522.22 499.05 542.26 15 600.25 590.01 504.71 488.34 473.56 532.56 16 580.35 543.26 379.67 476.58 532.23 433.59 17 555.88 496.36 665.00 299.99 548.84 477.25 19 788.27 564.88 675.50 590.54 693.47 657.76 20 433.19 459.23 435.67 467.89 590.12 534.56 21 512.25 511.11 561.00 500.15 509.90 503.32 22 485.20 500.44 503.67 483.74 468.49 589.70 23 522.16 460.69 544.36 467.78 488.29 512.61 24 501.44 577.10 600.00 503.92 605.58 505.45 26 473.36 546.46 516.17 563.10 718.14 503.25 27 494.58 536.25 500.26 468.87 600.21 580.41 28 456.67 478.75 601.37 503.22 496.59 502.45 Totals (a) Total the vertical columns. What do they show? (6) Total the horizontal columns. WTiat do they show? (c) Suggest a "check" for the vertical and horizontal totals. 14 BUSINESS ARITHMETIC. (d) Indicate the possible effects of weather, season, special sales, and other causes, on values appearing in departmental columns. Show what values in the above table might be specially accounted for. Note. Most tabular addition in business offices, and considerable computation requiring other fundamental operations, is now done by machinery. As early as A.D. 968, Gilbert, Archibishop of Rheims, per- fected some ingenious computing machines, which, however, sometimes failed to compute correctly. Pascal (1642-45) invented a machine correct in theory, which, while not practically useful, formed the basis for some modern machines of great capacity. The modern machines generally perform the fimdamental operations. They are either "listing" or "non-listing" machines. One standard machine has eight or more "banks" of keys, each containing nine digits and having its own place value. The keys to represent a certain number are pressed down and the movement of a lever, or the pressing of an electric button causes the printing of the number on a continuous roll of paper. The next number is printed below the first, the changing total appearing in the machine. A special movement prints the final total on the paper, and "clears the machine" for the next operation. Some machines list abstract numbers; others list money values, fractions, etc. Some are at- tached to typewriters so that bills of merchandise may be written and automatically totalled. TABULATIONS AND STATISTICS. EXERCISE. These two news items have appeared in print: (Tabulated.) (Untabulated.) The Department of Agriculture The losses caused by insects estimates the minimum annual to the products of the country, damage done to crops by our most during an average year, are es- destructive insects, as follows : timated by the Department of Chinch-bug $60,000,000 Agriculture, as follows: Cereals, Grasshopper 50,000,000 $200,000,000; animal products, Hessian fly 40,000,000 $175,000,000; forests and forest Corn-root worm 20,000,000 products, $111,000,000; products Corn-ear worm 20,000,000 in storage, $100,000,000; hay Cotton-boll weevil 20,000,000 and forage, $53,000,000; truck Codling moth (Apple) . 20,000,000 crops, $53,000,000; cotton, $50,- Armyworm 15,000,000 000,000; tobacco, $5,300,000; Cotton-boll worm 12,000,000 fruits, $27,000,000; sugars, Grain weevil 10,000,000 $5,000,000; miscellaneous crops, San Jose scale 10,000,000 $5,800,000. Total, | ? . Cotton-leaf worm. .... 8,000,000 Potato-bug 8,000,000 Cabbage worm 5,000,000 Total ? ADDITION. 15 (a) Find the missing totals. In which case is it easier? Why? (6) Wliich statement is read more easily? Which is more noticeable? (c) From which statement is it easier to select, or reclassify values? Why? (d) Compare capitahzation in the two cases. Compare use of "$." (e) Tabulate the second illustration, using a brief title in place of the introductory clause. The illustrations given above show the value to the statis- tician of the "column" and "total" in conveying numerical information. The column tabulation attracts attention, and brings each numerical value in sharp contrast with other values, while allowing comparison with the total. In tabulat- ing, names attached to values should be brief, but definite. Each tabulation should have a brief descriptive title. EXERCISE. 1. Tabulate and find the lowest cost of construction of a building for which the following bids were submitted: For masonry, iron work, carpentry, glazing, tinning, plastering, etc. — A. B. Ryan, $49,845; J. D. Norton, $41,900; C. A. Olhaus, $42,344; Robert Adams, $48,275; Brown & Mann, $35,691; Wm. Harris, $37,573; W. B. Norton, $44,678; A. S. Keene, $43,536; Northern Cons. Co., $40,547; Jas. Davis, $35,386. For materials and labor necessary to furnish, install and complete a low pressure, steam-heating apparatus: — Robert Davis, $4,646; Morris Heating Cons. Co., $3,674; Evanson & Son, $3,940; E, T. Roberts Co., $3,970; Barton Mfg. Co., $4,180; Geo. A. Sanderson, $4,900; Brown & Mann, $4,950; Wm. Harris, $3,398. For materials, labor, etc., for plumbing work: — D. F. Kean, $3,325; Robert Harris Co., $5,132; Wm. Harris, $4,736; A. B. Ryan, $4,728; Harper & Son, $4,198; Brown & Mann, $5,763. From the tabulation just made, find : (a) what bidder will complete the work for the lowest sum; (h) the lowest sum for which the work may be done by giving sub-contracts to different bidders. Decimal Addition. INTRODUCTORY EXERCISE. 1. Annex a cipher to .6, and read result. 16 BUSINESS ARITHMETIC. 2. Reduce 4 to tenths. 3. Reduce 53 to hundredths, 4. Add .3 and .6. 6. Add A, .6, .9, .8. 6. Add .04, .83, .07, 001. 7. Add .047 and .07. (.07 = how many thousandths?) 8. How does reduction to a common order aid in decimal addition? There is practically no difference between integral and decimal addition. Integral or decimal units of the same order are added. In writing in column, be sure that the decimal points are in vertical alignment, as this will ensure proper "order" position, and also the position of the decimal point in the total. Illustration. 46.205 392.0872 1245.01232 254.67005 1937.97457 ORAL EXERCISE. 1. How does the number of decimal places in the total compare with the nimiber of places in the constituent numbers? 2. May the total contain a less number of decimal places than any significant number? Add: 3. .4 and .85. 8. 2.33, 1.062 and 2.0382. 4. .072 and 0096. 9. 3.25, 1.0006 and 8.0129. 6. .12, .065 and .363. 10. .46, .05, .8 and .002. 6. .013, .21, and .220. 11. 6.3205 and .00061. 7. .01, 2.006 and 15.0126. 12. .001, 2 01 and .0352. ADDITION. EXERCISE. Total these columns: (1) (2) (3) 23.075 123.456729 4.37 06.5082 54.08296 83.070096 16.09327 5.4729825 5.9276 5463.382473 .007369 329.832 1.29634 527.8308896 4.00892 50069.8002173 5.2100009 .0006732 542.06345276 67.31438 1.01 12.47234 5729.61 15.2729 5067.30821 80.8 .888005 Note. A large proportion of decimal addition deals with mone- (4) (5) (6) $ 326.925 £ 456.29 45.B25 fr. 48.065 39.625 129.8 723.8 1233.40 46239.03 9654.47 56.1235 596.008 830.425 5.60 63.9 65.72 1283.55 1321.48 12756.23 14756.125 32635.835 52.08 677.0875 4.006 926.00 1239.05 5282.56 7259.12 36.075 43.45 5.69 5296.5125 6.735 627.33 83.0125 529.62 46.09 962.825 88.9 17 7. Find the totals, in units of a thousand feet, of these orders for lumber: 6246 ft., 15,389 ft., 7500 ft., 4382 ft., 965 ft., 45 ft., 12,925 ft., 6750 ft. 8. Total these engineering measurements, and express total in units of 1000 ft.: 564', 296.5', 4521', 885.2', 750', 1325', 632.8', 983.7', 66.8'o CHAPTER IV. SUBTRACTION. INTRODUCTORY EXERCISE. 1. Name the difference between 2 and 9, 16 and 21, 2. From 129 subtract 4s. 3. From 138 subtract alternate 3s and 5s. 4. Give several illustrations of the use of subtraction in everyday life. Subtraction is one phase of addition. It is the process of finding the number to be added to the lesser of two given numbers in order that the sum may equal the greater. lLL'j3rRATiON. Find the difference between 774 and 1969. Process. Add to the subtrahend, digit by digit, until iQftQ miniioTiri *^® miuucud is obtained, thus: 4 (sub.) +5 = 9 774 SSend ("^^^•)- '^"^^ ^' ^« "^^^ difference. 7 (sub.) + 9 = 16 n^ suDtranena. ^^^^^^ ^^^^^ 9 ^^ ^^^,^ pj^^^ ^^ difference. 7 (sub.) 1195 difference. + 1 = 8 (9 - 1, min.). (sub.) + 1 = 1 (min.). Note. In formal work, it is often inconvenient to write the lesser number under the greater. Position, however, makes no difference in method. The above example might be written: (a) 774 sub. (b) 1969 - 774 = 1195. 1969 min. 1195 diff. EXERCISE. 1. Show that these subtraction "checks" are true. (a) The sum of subtrahend and difference equals minuend. (6) The minuend less the difference equals the subtrahend. Solve by inspection: 2. Find by addition the complements of 36, 29, 64, 83, 91, 896. 3. Solve by addition: 54 - 13 76 - 23 148 - 93 168 - 72 59 - 37 43 - 29 124 - 58 249 - 68 18 SUBTRACTION. 19 Note. Oral work is sometimes simplified by subtracting the orders of the subtrahend independently, beginning with the highest. This is termed subtraction from the left. Thus 578 - 364 = 578 - 300 (278) - 60 (218) - 4 = 214. 4. Solve, by subtraction from the left : 249 - 67 328 - 243 1527 - 892 968 - 871 1116 - 452 1793 - 896 529 - 381 4473 - 269 Note. Irregular numbers may be subtracted mentally by subtracting an even number of the next higher order, correcting the difference of the numbers by adding the difference of the subtrahends. Thus 384 — 47 = 384 - 50 (334) + 3 = 337. 5. Solve, explaining the process: 362 - 98 473 - 57 734 - 96 263 - 99 654 - 319 328 - 246 899 - 473 346 - 228 Note. Often, in practical work, the subtrahend must first be obtained by addition. 6. 624 - (32 + 16 + 11) = ? 7. 1754 - (200 + 311 + 40) = ? 8. Sales of 72, 14 and 11 pounds from a stock of 360 lb. leave how many pounds on hand? EXERCISE. 1. 59,768,003 - 407,019 = ? 2. $3,876,547 3. $567,839 4. 987,654,321 108,976 203,678 123,456,789 ? ? . ? In the following, subtract the last number from the sum of the balance of the colunm: 7239 6. 1209 7. 213 8. 32139 2146 9660 32 4781 1299 5138 921 61020 881 1234 4238 3121 4731 8108 1119 5288 9-12. Repeat the last four examples, subtracting the first number from the sum of the balance of the colunm. Note. Subtraction is often combined with direct addition, especially in business forms and statistical tables. 13. This form is one of many used in business to keep account of stock on hand. 20 BUSINESS ARITHMETIC. STOCK RECORD OF LIMIT MO WHERE KEPT STOCK RECEIVED STOCK SOLD ON HAND DATE FROM WHOM QUAN. DATE TO WHOM QUAN. QUAN. 19— 19- 3-3 ^rtAP&nid^'u, 172 3-5 *^3762 9 163 3-8 3964 27 3-9 4287 15 3-12 4561 42 3-21 ja/rtAoifn, Vo. 50 3-21 5265 18 3-24 5290 54 3-26 5380 21 3-29 5620 20 (a) Extend the form. (6) Explain the writing of the date. (c) How are entries in the "On hand" column obtained? (d) Explain the solution for Mar. 21. (e) How many desks are on hand on Mar. 11? (/) How may the "Received" and "Sold" columns be checked? 14. Many private families, living on a "cash" basis, keep simple record, showing in detail household income and expenditures. (Page 21.) (a) Explain the first items in the "Total paid out" and "Balance" columns. (6) Find the remaining values for these columns. (c) Total all the vertical columns except the "Balance" column. (d) Why is not the "Balance" column totaled? (e) Find a check for the "Total paid out" total. (/) Suggest a check for the final balance. (g) Suggest how amounts in the different columns might vary with the season of the year. The statistician who is tabulating crops, wealth, railway mileage, bank clearings, exports, etc., frequently gives figures for earlier periods and shows by a difference column the amount of change, the differences often lending special significance to the gross values. The business man, also, gives special atten- * Order number identifies purchaser. SUBTRACTION. 21 ^-^ ^ «^ N ^ > N cs k> «^ N >s v^ ^^ ^"^ 0^ ^ Ss '^ >N X "*^ •^^ ^ ^ N >N ^ N ^. s -< USE- NTS. URIES, AVEL ^ ^ ^•>> ^ *^ '\> ^^ ^ •^^ "N ^ S ^ X oc •< 2 = 1- —1 i ^ >s ^ ^ .V N =^ ^ ^ ^ o s o ■ cv ^ ^ N ^ ^ ^ ^ ITUR ND PLIES ^ ^ ^ >s <3V ^ *^ Va >N ^ «^ Sy "^^ >s 5^ 1 ^ ^ 1 <^, ^ 1 S ^ ^ s ° s ^ «^ ^ 'S 5" ^ ^^< ^ ©y *^ •^^ LIGHT AND FUEL n] ^ ^a tv ^-i "N ^ >N 'V. 1 RENT TAXE REPAU ^ 5^ UJ , a= ^ =v ^ 5^. ^. 5^. ^ ^ 5^ =^. ^ ^ ^ «V ^ =5 ca »— _l =3 J* o ^ ^. 5V «V. =v. <\ 5^ =^ <\ »^. =^ «^. 5^ «V. <\ ?i ea 1 1 UJ > ^ ^ ■N, o ^ ^ CO 1^ ^ ^ N? s 3 ^ ' "^ ^ 1 ^ ^ ^ ^ ^ ^ ' >N ^ erj •*^ ^•^ ^ «0 =5^ ^ ^ '^ i 1 ^ 22 BUSINESS ARITHMETIC. tion to the increases or decreases, as well as to the total of his sales, profits or expenses. In the exercise that follows, note how the differences add weight to the presentation of facts. EXERCISE. 1. These figures from a financial article show certain facts concerning the business of the country for two consecutive years. Fmd the differences, placing decreases in the "increase" column, but marking them with a minus sign. Two Years' Record. 1910 1911 Increase Bank clearings $158,450,000,000 $162,914,100,000 ? Railway earnings 2,395,800,000 2,423,400,000 ? Exports 1,867,605,000 1,637,256,000 ? Imports 1,392,500,000 1,426,200,000 ? Grain crops 2,702,458,000 2,489,000,000 ? EXERCISE. 1. Bring to class four illustrations of " difference " columns in statistical tables. 2. Sum up the different methods of checking addition and subtraction used or suggested in this chapter. Subtraction of Decimals. INTRODUCTORY EXERCISE. 1. From 36 hundredths subtract 12 hundredths. 2. 5.46 - 2.39 = ? 3. .62 = ? thousandths. .52 - .412 = ? The only difference in process between whole nun^iber and decimal subtraction consists in the necessity for a reduction, in the latter, of both subtrahend and minuend to decimals of the same order. Illustration. Find the difference between 5.029 and 3.0014. Solution. 5.029 is the same as 5.0290. 5.0290 3.0014 2.0276 SUBTRACTION. 23 EXERCISE. By inspection find the difference between: 1. .51 and .402. 6. .01024 and .08. 2. 3.6 and 1.32. 7. .000004 and .0004. 3. .9 and .09. 8. 10.003 and .230. 4. 1. and .0321. 9. From 3.75 subtract .05s. 5. 2.0004 and 1.3. . 10. From .324 subtract .004s. 11. From one and one-hundredth, subtract nine ten-thousandths. EXERCISE. 1. 4.0739625 - 2.00100376 = ? 2. 5506.38 - 296.54325 = ? 3. 100,109.0001003 - 1202.12345 = ? 4. From 46 subtract the first of the following decimals; from the remainder subtract the second, and so on: 5.0621, 2.010309, 1.0042, .98396, 25. 1013096, 1 .24965. How may the accuracy of the final answer be checked? Decimal subtraction is common in higher mathematics and in scientific and estimating problems. Two-place decimal subtraction is common in money computations. Usually sub- traction is incidental to some other process. EXERCISE. 1. A farmer's poultry account for January is as follows: Value of fowls, Jan. 1, $1800; sales of fowls during the month, $247.53; value of fowls at close, $1750; cost of food and care, $112.50; sales of eggs, $105.82; miscellaneous expenses, $54.27. Make a neat statement showing the net profit for the month. 2. A metal bar 16 inches long, is contracted under pressure .8583765 inches. What is its length imder pressure? 3. A yard measure is equal to 0.914,402 meters. A meter exceeds a yard by what decimal of itself? 4. A certain cloth shrinks .1525 of a foot per yard in dyeing. A yard of the undyed cloth produces how many yards of dyed material? 5. The needle on a recording machine travels from 3.256 ft. on the scale to 4.105 ft., a distance of ? ft. 24 BUSINESS ARITHMETIC. 6. In one city, having 5c car fares, the cost of carrying each passenger is 3.9246c. In another city, having a 4c fare, the cost is 3.1024c. Find the profit per passenger in each case, and the difference in cost per city. 7. Complete this memorandum of costs and profits. The Novelty Manufacturing Company. Memorandum of Costs and Profits per Article. Article. Cost to Make, ^. Net Sell. Pr. ff. Net Profit. Butterfly cards Dime savings banks Remembrance cards ... 3.0245 14.308 .0965 4.85 13.0625 7.5 16.5 2.15 6.1125 15. ? ? ? Surprise box ?" Three of a kind ? CHAPTER V. MULTIPLICATION. Multiplication, as a number process, is second in importance to addition, and has extensive and varied applications. It is really an addition "short cut," for expressions of multiplica- tion, such as "5 X 381" may be written in the form "381 + 381 + 381 + 381 + 381." As an independent process, also, it lends itself to so many short methods as to make it worth a special study. Illustration. Multiply 3264 by 529. 3264 (multiplic^d) 529 (multiplier) 29376 (units) 6528 (tens) 16320 (hundreds) 1726656 (product) In the product there are 9 X 3264 units; 2 X 3264 tens and 5 X 3264 hundreds. Note. The multiplicand and multiplier are called factors of the product. While the multiplier is invariably an abstract number, the multiplicand and product may be either abstract or concrete, but they are always like numbers. As a result, the product is not altered by changing the order of the factors considered as abstract numbers. Illustration. The weight of 8724 bx. of 65 lb. each is either (8724X65) lb., or (65 X 8724) lb. FOR DISCUSSION. 1. What is the objection to solving the examples illustrated above by addition? 2. At what point, in regular multiplication, does addition occur? 25 26 BUSINESS ARITHMETIC. 3. In finding the product of 873,926 by 1001, which factor should be used as multiplier? Why? 4. Why is this statement incorrect: "$16 X 15 bbl. = $180"? ORAL EXERCISE. 1. Find the products of: 3 X 14, 7 X 18, 3X3X6, 4 X 2 X 22, 8 X 40 bu , 5 X 81c, 2 X 3 X 8 X 4, 2 X 24 ft. 2. Multiply each integer under 21, by itself and by each smaller integral number. These are the most used factors. 3. A camping party of three men, planning a sixty day trip, order rations in the proportions given below. Determine the required quantity of each article. Supplies per Man per Thirty Days. Flour 241b. Oatmeal 7 " Beans 6 " Sugar 12 " Cond. milk. ... 3 en. Tea, cocoa.. ,1 lb. Coffee 2 " Rice 4 " Salt 1 " Prunes 2 " Butter.... 21b. Bak. pow.. 1 " Bacon.... 11 " Pepper ... 1 " (6 men) 4. A certain contractor foimd constant need for the products of the factors 80 to 100 by the factors 20 to 40. He prepared the form outlined below, the product of each two factors being written at the intersection of the proper horizontal and vertical columns. The product of 84 by 22 is shown. Compute the other products mentally. Show how to use an addition method for the work. Suggest a method of checking the compu- tations. 80 81 82 83 84 85 Etc. 20 21 22 1848 Etc. 5. Construct a similar multiplication table for the factors 1 to 25 in- clusive. EXERCISE. Find the products of the following and check accuracy by reversing factors: MULTIPIJCATION. , 27 1. 726,943 X 120,301. 4. 373,259 X 129. 2. 3,629,007 X 20,905. 5. 12,345 X 67,890. 3. 16,730,021 X 802,001. 6. 20,103 X 1,387,654. 7. A railway company sends out eight engineering parties of 19 men each, on reconnoisance and track location work. Rations are required for 200 days, based on a U. S. Government ration Ust. Prepare a tabu- lation showing the quantity of suppUes to be provided. Ration List for One Man for One Hundred Days. (Take first named article.) 100 lb. fresh meat, including fish and poultry. 50 " cured meat, canned meat, or cheese 15 " lard. 80 " flour, bread, or crackers. 15 " corn meal, cereals, macaroni, sage, or com starch. 5 " baking powder, or yeast cakes. 40 " sugar. 1 gal. molasses. 121b. coffee. 2 " tea or cocoa. 10 en. condensed milk, or 50 qt. fresh milk. 101b. butter. 20 " dried fruit, or 100 lb. fresh fruit. 20 " rice or beans. 100 " potatoes or other fresh vegetables. 30 " vegetables or fruits. 4 oz. spices. 4 " flavoring extracts. 8 " pepper, or mustard. 3 qt. pickles. 1 " vinegar. 41b. salt. 8. As a first result of the survey, the Company decides to build 19 miles of single and 38 miles of double track. Compute the total quantity of the following supplies which are to be used in the construction: Cross- ties, at rate of 2640 per mile of single track; steel rail, at 90 pounds per yard; spikes, at the rate of 11,380 per mile of single track; 68 angle irons for special construction, 34 feet long, weighing 16 poimds per foot; 240 I-beams, 28 feet long, weighing 65 pounds per foot. Give quantities of iron and steel by weight. 28 BUSINESS ARITHMETIC. Short Methods. Most fundamental processes of number may be abbreviated — often with marked increase in rapidity of solution and accuracy of result. The short methods used, however, should be such as one naturally develops for himself, or those that instinctively appeal to one as being simpler and more direct than the regular methods. Memorized rules for short methods are dangerous aids, as an inexact memory may cause an in- correct application of the rule at some critical moment. Short methods are commonly used to do away with computation on paper, or to simplify such computation, or to check the accuracy of computations performed by other methods. Short Methods in Multiplication. Some typical cases follow. Suggest other methods. In each case, show wherein the value of the method consists. Multiplying by 10, 100, etc. 726 X 10 = ; 1428 X 100 = ; 345 X 1000 = . Compare the significant figures of the product with those of the multiplicand. How may the product of such factors be written at sight? What effect has the annexing of a cipher? Of two ciphers? ORAL EXERCISE. 1. 167X10 = ? 4. 100X26 lb. = ? 7. 1,000X80,201=? 2. 100X3,802 = ? 5. 10,000X131 ft. = ? 8. 70,073X100,000 = ? 3. 100X50,201=? 6. 10X7,384 yd. = ? Find the weight of: 9. 246 bbl, averaging 100 lb. ea. 10. 1,246 loads, averaging 1000 lb. Multiplying by 11, 21, SI, etc., and by multiples of 11, Outline a short method for these multipliers, suggested by the follow- ing illustrations: (1) 11 X 64 = 10 X 64 + 64 = 640 + 64 = 704. (2) 31 X 126 = 3 (10 X 126) + 126 = 3780 + 126 = 3906. (3) 101 X 48 = 100 X 48 + 48 = 4800 + 48 = 4848. MULTIPLICATION. 29 ORAL EXERCISE. Find the product of: 1. 11 X 16. 3. 11 X 242. 5. 21 X 360. 7. 21 X 19. 2. 21 X 28. 4. 31 X 18. 6. 41 X 22. 8. 1001 X 287. Find the weight of: Find the cost of: 9. 240 ft. iron at 111b. per. ft. 11. 12 lb. butter, @ 31c. 10. 165 kegs, 31 lb. each. 12. 142 sq. yd. tiling, @ 21c. Multiplying by 9, 19, 29, etc. After studying the illustrations, outline the short method and compare with the previous case. Illustrations. (1) 9 X 148 = 10 X 148 - 148 = 1480 - 148 = 1332. (2) 19 X 320 = 2 (10 X 320) - 320 = 6400 - 320 = 6080. Used in paper work, also. (3) Example. Multiply 32,689 by 19. Long method. Short method. 32689 Factors not re-written. 19 653780 (2 X 10 X 32689, written at sight) 294201 32689(1X32689) 32689 621091 (by subtraction) 621091 Note. The short method substituted the multiplication by 2 for the more difficult multiplication by 9. ORAL EXERCISE. Find the product of: 1. 9 X 87. 3. 29 X 16. 5. 9 X 269. 7. 29 X 17. 2. 19 X 42. 4. 109 X 120. 6. 9 X 424. 8. 59 X 11. 9. Find the weight of 720 pc. castings, weighing 9 lb. each. EXERCISE. Using short methods, find the product of: 1. 19 X 32,967. 4. 29 X 6,376,951. 7. 99 X 8,703,129. 2. 9 X 1,213,027. 5. 19 X 12,587,328. 8. 1,009 X 784,562. 3. 39 X 47,201. 6. 109 X 630,851. Multiplying hy 5, 15, etc. Illustrations. (1) 5 X 620 = One-half of 10 X 620 = ^^ = 3100. (2) 15 X 48 = 10 X 48 + H of 10 X 48 = 480 + 240 = 720. 30 BUSINESS ARITHMETIC. ORAL EXERCISE. 1. In multiplying a number, such as 7,268,316, by 5, why may it b« simpler to use the short method? 2. Multiply by 5: 16, 21, 32, 54, 123, 1309, 677, 1893. 3. Multiply by 15: 24, 68, 152, 19, 137, 240, 62, 192, 1012. 4. Find the costs of: 124 en. at 5c; 16 lb. nails at 5c; 68 yd. dress goods at 15c. EXERCISE. Using short methods, and omitting the re-writing of factors, find the products of: 1. 5 X 7236. 3. 15 X 724 yd. 5. 15 X 8039 qt. 2. 15 X 83,963 lb. 4. 5 X 12,463 qt. 6. 15 X 7204 oz. Multiplying by factors of multiplier. Illustration : 36 X 120 = 6 X 6 X 120 = 6 (6 X 120) = 6 X720 = 4320. ORAL EXERCISE. i. Compare the short and the direct methods. 2. Is the short method of real advantage in oral or in written work? 3. How do the factors themselves limit the use of the method? 4. Reduce to two or mor€ simple factors: 65, 24, 49, 18, 72, 96, 48, 45, 54, 84, 110, 39, 92, 120. 5. Determine these products by factoring multiplier, and check by other short methods: 25 X 160 122 X 12 99 X 48 16 X 32 240 X 18 12 X 118 15 X 72 20 X 155 ' Multiplying by parts (multiplication from the left). Illustration. (1) Find the weight of 722 bbl. of 112 lb. each. 112 X 722 = 100 X 722 + 10 X 722 + 2 X 722 = 72200+7220+1444 = 80864 (2) Written work. Multiply 738,421 by 321. Regular method. Method by parts. 738,421 Factors not re-written. 321 221,526,300 (3 X 100 X number) 738421 14,768,420 (2 X 10 X number) 1476842 738,421 (1 X number) 2215263 237,033,141 237033141 MULTIPLICATION. 31 Note. The advantage of this method for oral work, lies in adding con- tinuously decreasing sub-totals to a growing total, each new product being added as found. ORAL EXERCISE. Find the product of: 1. 115X36. 3. 23X42. 5. 62X312. 7. 103X428. 2. 111X249. 4. 22X124. 6. 210X122. 8. 51X122. 9. What is the output in 16 hours of a machine making 240 doz. cans per hour? EXERCISE. Find the product of: 1. 212X324,567. 2. 1325X88,459,876. 3. 1245X87,960,345. SPECIAL EXERCISE. 1. Write a brief illustrative paper on "Short methods in Multipli- cation." 2. Write a paragraph on the dangers and sources of error in the use of short methods. 3. ^rite a brief paper on "Methods of Checking Multiplication." DECIMAL MULTIPLICATION. INTRODUCTORY EXERCISE. 1. One-tenth of 6 = ? One-hundredth of 6 = ? 2. Ten times .6 = ? One hundred times .6 = ? 3. Compare these numbers as to value: .06, .6, 6, 6000. 4. How is the value of a significant figure affected by moving the decimal point to the right. 5. Suggest a short method for multiplication of decimals by 10, 100, 1000, etc. Illustrate, using .006 and 4.2 as multiplicands. 6. One-tenth of 68 = 68 ^?; or 68 X ? 7. Using 72.6 show the effect on its value of moving the decimal point to the left. 8. Suggest short methods of multiplication by .1, 01, 001, etc. The process of multiplication of decimal fractions differs in no essential from the midtiplication of integers, except in the determination of the decimal point of the product. Notic- ing that .1 X .01 = 1/10 X 1/100 = 1/1000 or .001, it is 32 BUSINESS ARITHMETIC. evident that the number of decimal places in the product equals the sum of those in the factors. In multiplication, therefore, decimals may be treated as integers, the proper number of places being pointed off after the numerical product is obtained. Illustrations. (1) Multiply 5268.39 by 1.6. Solution and analysis. 5268.39 contains 526,839 hundredths. 1.6 contains 16 tenths 526,839 hundredths X 16 tenths = 8,429,424 thousandths, or 8429.424 (2) Find the product of 7.26 multiplied by 3600. Solution. 3600 X 7.26 = 36 X 100 X 7.26 = 36 X 726. Note. Notice how the decimal is cancelled before the multi- plication of significant figures is begun. This is often possible. 5268.39 1.6 3161034 526839 8429.424 726 36 4356 2178^ 26136 (e) .00008X4.00007 (/) 9.001X1.0000401 (g) 1.002 X. 021 (h) .06X6000 (i) 7.25 X. 0002 ORAL EXERCISE. 1. How many decimal places in the products of: (a) 3.6 X. 004 (c) 58 X. 020 (6) 50.29 X. 00609 (d) 1573.829 X .032 2. Find the products of: (o) 1.2X.12 (d) 4003X.0002 (6) .003 X. 0004 (e) 6.01 X. 005 (c) 2.01 X. 00002 (/) .000003 X. 3 3. Compute the cost of: (a) 700 lb. @ 32c. (b) 2000 bu. @ 45c. (c) 2960 lb. @ Jl.lO. EXERCISE. (Check all products.) 1. Multiply 7206.021 by 3.001029. 2. Multiply 739.65482 by 549.03296. 3. Multiply 7.2098 by .02604, and the product by 100.09. ORAL EXERCISE. * (Use the short methods given for integral numbers.) 1. 100X26yd. = ? 4. 1000 X. 005 yd. = ? 7. 9X.8 = ? 2. 380X.2X.01 = ? 5. 21X.4 = ? 8. 19Xl.2 = T 3. 11X3.6 = ? 6. 101 X. 08 = ? MULTIPLICATION. 33 GENERAL EXERCISE. Note. In the computation of costs and quantities by multiplication, decimals are usually involved. The following are tj'pical illustrations. 1. For accounting purposes, Martin & Co. take an inventory, or valuation of their stock of skins, pricing them at the lowest market quo- tation, except for the gray fox which they price at the highest rate. Using the following quotations, make a neat tabulation showing the valuation of the entire stock. WOOL AND HIDES.— Quotations for fm-s on No. 1 articles only. Wool, washed, free of burrs, per lb., 38a40; wool unwashed, per lb., 30a33; hides, green, per lb., 10; dry, per lb., 15al7; sheepskins, green, each, 1.00a 1.25; dry, each, 25a75; calf -skins, green, each, 1.00al.50; muskrat, each, 12al8; skunk, each 25a75; mink, each, 3.00a4.00; rabbit skins, each, 10; opossum, each, 25a27; raccoon, each, 25a90; fox, red, each, 2.00a2.50; gray, each, 75a90. The stock of Martin & Co. consists of: 2480 lb. wool, washed; 1600 lb. wool, unwashed; 750 lb. hides, green; 2375 lb. hides, dry; 128 calf skins; 46 muskrat skins; 256 rabbit skins; 13 gray fox skins; 129 opossum skins; 382 raccoon skins. 2. Tabulate individual and gross weights of the following order: 200 16' I-beams, weighing 85.2 lb. per ft. 162 19' I-beams, weighing 23.5 lb. per ft. 200' trough plates, weighing 16.32 lb. per ft. 1150 18' channel irons, weighing 33 lb. per ft. 450' deck beams, weighing 27.23 lb. per ft. 620 14' angle irons, weighing 17.2 lb. per ft. 145 Z-bars, 12', weighing 29.8 lb. per ft. 3. This is an itemized tabulation of the cost of a masonry wall con- taining 12,642 cu. yd. of masonry. Compute the total cost. Per cu. yd. Total cost. Quarrying stone $0,415 $ ? Loading and hauling stone 912 Hoisting stone 605 Excavating 300 Loading and hauling sand 355 Cement 345 Mixing and dejivering mortar 205 Masons and helpers 915 Carpentry 885 Blacksmithing 190 Tools and general supplies 175 Superintendence, foremen, etc .725 (How check?) 4 34 BUSINESS ARITHMETIC. 4. Bids a, h, c, d and e have been submitted by different parties, ttj cover the cost of a granite block pavement. Find and tabulate the tota.« cost of each item and of each bid. Determine what combination of dif- ferent bids will give the lowest cost. a h c d e 400 cu. yd. concrete $4.00 S3.00 $3.90 $2.55 $3.98 135 lin. ft. granite crossings 1.20 1.13 1.25 1.00 1.24 1178 lin. ft. 5" granite curb 1.275 1.30 1.30 1.29 1.30 162 lin. ft. circular curb 2.00 2.10 1.95 2.00 2.0G 65 lin. ft. curb, reset, etc 21 .15 .20 .32 .18 48250 cu. yd. grading , 43 .425 .41 .45 .50 2476 sq. yd. granite paving 3.25 3.18 3.25 3.00 3.30 2476 sq. yd. sub grade 06 .06 .06 .06 .06 268 lin. ft. 24" dram pipe 2.78 1.75 2.00 2.10 1.95 73 cu. yd. brick masonry 12.00 11.25 12.50 10.90 11.75 21 cu. yd. rubble masonry 5.75 6.00 4.25 5.00 4.80 12 pc. Palmer inlets. 40.00 40.00 40.00 40.00 40.00 Totals CHAPTER VI. DIVISION. INTRODUCTORY EXERCISE. 1. Add by 4s from to 48. There are how many 4s in 48. 2. Subtract by 9s from 189 to 0. There are how many 9s m 189. 3. Compare division with addition and subtraction. 4. I agree to pay $40 cash and $6 per week for a $100 tjrpewriter. Name the amount due after each payment. Determine the number of payments (1) by addition, (2) by subtraction, (3) by division. Lowering the weekly payment to $4 extends the time of settlement ? weeks. 5. 172 -^ 4 = ? ? X 4 = 172. 6. Compare multiplication and division. Division may be termed a short method in addition and subtraction, and the reverse of multiplication. It is the process of determining the unknown factor of a product from the product and a known factor. The known factor is the divisor, the product is the dividend, and the factor to be deter- mined is the quotient. The remainder is the portion of the dividend that is left when division is not exact. ORAL EXERCISE. If the dividend is 64 and the divisor 8, show that: 1. The dividend divided by the divisor equals the quotient. 2. The quotient times the divisor equals the dividend. 3. The dividend divided by the quotient equals the divisor. FACILITY EXERCISE. (Solve by inspection.) 1. Divide by 2: 16, 38, 57, 96, 248, 1749, 62,482. 2. Divide by 3: 12, 21, 48, 69, 125, 683, 952, 667. 3. Divide by 2 and the quotient by 3: 36, 124, 88, 962. 4. Divide by 4: 76, 320, 888, 96, 154, 723. 35 36 BUSINESS ARITHMETIC. 6. Divide by 5: 25, 65, 83, 195, 217, 1259. 6. Divide by 6: 42, 80, 172, 620, 38, 415. 7. Divide by 7: 49, 35, 73^, 1420, 203, 633. 8. Divide by 8: 72, 30, 960, 1240, 884, 1720. 9. Divide by 9: 30, 75, 810, 640, 1850, 27630. 10. Multiply 24 and successive products by 2 (6 times). Divide by successive 4s. Short Division. In short division the divisor and dividend are written, and the figures of the quotient, as obtained, but intermediate remainders and products are omitted. Illustration. Divide 7248 by 8. Solution. Analysis. 72 hundreds -^ 8 = 9 hundreds; 4 tens are 8 )7248 not divisible by 8; 4 tens + 8 units = 48 units. 48 units 906 -^ 8 = 6 units. Note. At times, when the divisdr is reducible to very simple factors, it is possible to employ continued short division by the factors in turn. In such cases, of course, each factor must be an exact divisor until the last. EXERCISE. Divide: 1. 239,872,452 by 4. 5. 6888 by 56 (continued division), 2. 8,703,265 by .5. 6. 24,496 by 16. 3. 364,285,968 by 6. 7. 193,625 by 25. 4. 123,456,789 by 7. 8. 4,829,672 by 32. ILLUSTRATIVE EXERCISE. 1. Solve: (a) 36 -^ 4 = ? (6) 36 ft. -^ 4 = ? (c) In 36 bbl. there are ? lots of 4 bbl. each. 2. What is "common" to these problems? What changes? It is essential, in division, to determine whether the 'quotient and the remainder are abstract or concrete. Division by an abstract divisor is called partition, and by a concrete divisor, measuring. Thus, in 1 (6) in the preceding exercise " 36 ft." is parted into 4 parts, and in 1 (c) "4 bbl." is used as a measure to determine the number of "4 barrel" lots in 36 bbl. It DIVISION. 37 should be noted that if divisor and dividend are concrete, the quotient is abstract; while if the dividend is concrete and the divisor abstract, the quotient is a concrete number of the same order as the dividend. ILLUSTRATIVE EXERCISE. 1. Find the remainders in the following: (a) 725 ^ 4 = ? (6) 420 yd ^ 8 = ? (c) For 88c I can buy ? yd. of cotton at 8c per yd. 2. When is the remainder abstract? When concrete? Long Division. In long division, intermediate products and remainders are shown. Illustrations. (1) Divide 76,945 by 162. (2) Solution. 474 (Quotient) Table of Multiples of 162. Byl 162 2 324 3 486 4 648 5 810 6 972 7 1134 8 1296 9 1458 (Divisor) 162)76945 (Dividend) (400 X 162) 648 1214 (70 X 162) 1134 805 (4 X 162) 648 157 (Remainder) Check. 474 X 162 + 157 = 76945. If, as often happens in business and statistics, the same divisor is constantly employed, a table of its multiples (see (2) from 1 to 9, is prepared, and used for reference. This avoids repeated multiplication, and serves to show, without trial, each successive figure in the quotient. EXERCISE. Divide, expressing remainder separately: 1. 146,732 by 1396. 3. 9,820,605 by 89,243. 2. $372,486 by 2013. 4. 620,548 lb. by 63 lb. 5. In 468,246 lb. of grain there are ? bu. of 56 lb. each. 38 BUSINESS ARITHMETIC. 6. Divide the following by 1237, using a table of multiples: (a) 673,892. (6) 960,478. . (c) 1,214,734. 7. Divide the following by 864, using a table of multiples: (a) $4,327,432. (b) 8,921,065. (c) 1,234,056,789. 8. Find the number of tons of coal (2240 lb. ) in each of these shipments, ignoring fractions of a ton. Car No. Wt. of Loaded Car. Empty Car. Net Weight. Tons. 32,064 134,200 26,420 ? ? 38,296 129,100 27,020 43,129 126,000 24,320 ? ? Short Methods. Division does not lend itself to so many easily applied short methods as does multiplication. The following are the most common : Division by 10 y 100 ^ etc. ORAL EXERCISE. 1. What effect has the decimal point on the comparative values of these numbers: 6800, 680 0, 68.00, 6.800? 2. Suggest a short method of division by powers of 10. Find the quotient of: 3. 720,000 ^ 10. 6. 30,000,000 lb. ^ 10,000. 4. 726,000 -^ 1000. 7. 368,000 ^ 100. 5. $725,900 -h $100. 8. $850 -^ 10. Division by multiples of 10. When a power of ten is a factor of both dividend and divisor, the example may be simplified by the principle of continued division. Illustration. 4720 -^ 80 = (4720 -^ 10) -^ 8 = 472 4- 8 = 59. EXERCISE. Find the quotient by inspection. 1. 840 ^ 70. 4. 320 lb. -=- 20. 7. $650 ^ 130. 2. 725,000 -^ 600. 5. $7,600 ^ 40. 8. $987,500 -J- 500. 3. 870 -^ 30. 6. 800,000 lb. -^ 40. DIVISION. 39 DECIMAL DIVISION. INTRODUCTORY EXERCISE. 1. 4.2 is equivalent to ? tenths 2. 4.2 -i- 6 may be expressed: ? tenths -^ 6. The quotient is — . 3. Divide by 6: $480, 48 lb , 48 T., .048 cu. yd., .00048. 4. Divide these same values by .6. What remains unchanged in the quotient? What changes? It is evident that the location of the decimal point in the quotient is the new feature of the division process met with in passing from whole numbers to decimals. Illustrations. (1) Divide 49.776 by 16. Hon Condensed solution. 3111 3,111 16)49.776 16)49.776 (3 X 16) 48 , - 48 1 776 remaining. 17 (.1 X 16) 16 16 .176 remaining. 17 (.01 X 16) .16 16 .016 remaining. 16 (.001 X 16) .016 16 Note. If the divisor is integral, the position value of each figure in the quotient is the lowest position value in the partial dividend used in obtaining it. Thus "49" is the partial dividend used in obtaining "3" of the quotient, and its lowest position value is ''unity." The advantage of writing quotient over dividend is evident, since the "placing" of the first figure of the quotient practically determines the place value of the rest. (2) Divide 7.6845 by .15. Solution. Multiply each term by 100, by short method. 7.6845 H- .15 = 768.45 -^ 15. • 51.23 15)768.45 Note. In case the divisor is decimal in form, it may be cleared of decimals, by mo^dng the decimal point to the right in both dividend and divisor. The example then comes under the class shown in the previous illustration. 40 BUSINESS ARITHMETIC. EXERCISE. Using short methods, if convenient, find by inspection the quotient of; 1. 3.6^6. 4. .0004^2. 7. 3.824-r-8. 10. 52.08 T.-^100. 2. .427-^7. 5. 2.644-3. 8. 16.860-J-12. 11. 3.62-^10. 3. .084-^16. 6. 1.64-4. 9. .00129-^30. 12. 29000.01 -MOOO. In the following, clear the divisor of decimals: 13. .844-1.2. 14. 63.08-i-.002. 15. .00034- 006. 16. 2.40604-1.2. Divide, by inspection: 17. 1 by .01, .001, .0001, 10000. 22. 200, 4000, .08 and .0006 by 2. 18. .01 by 10, 100, .01 and .00001. 23. .00012 by 6, 600, .3, .03, .006. 19. .0001 by .005, .05, .02, 100. 24. 4.24 by .0006 and by 6. 20. 4.8 by 1.2, .012, .0006, .3. 25. 63 by .007, .09, .7, .0003. 21. 60 by .002, .3, .0005, .12. 26. 660.15 by .015. Exact and Approximate Results. Decimal ciphers may be added to the' dividend indefinitely, to secure an accurate quotient to any number of decimal places, the final remainder being expressed fractionally. Commonly, however, results are required exact to some stated number of places, the remainder being unstated. Thus a business man may compute costs "to the nearest cent," or an engineer make measurements to "hundredths of an inch." Many working tables, also, are computed to four, five or six decimal places, remainders being ignored because, as a rule, they are too small to affect appre- ciably results computed by them. Often, if the remainder is one-half or more of the preceding decimal figure, that figure is increased by 1. Thus a quotient of 7.3086 may be written "to three decimal places" as 7.309. When merely an approximate quotient is desired, the approximate method is sometimes used. This consists usually in direct division until the integral portion of the quotient is obtained. From that point, no further figure of the dividend is brought down but, at each step, the right hand figure of the previous divisor is cancelled. DIVISION. 41 Illustration. Divide 684.394683 by 56.138. Contracted Soluiion. 12.1914 56.138)684.3946839 56138 123 014 112 276 10 738 (Div. 56.13) 5 613 5 125 (Div. 56.1) 5 049 76 (Div. 56) 56 20 (Div. 5) 20 EXERCISE. Divide: 1. $726 48 by 12. 4. .0297 by .31. (2 places ) 2. 114,269 lb. by 11. (2 places.) 5. 12.873 by 151.3. (1 place.) 3. 4629.2 cu. yd. by .24. 6. 842,097 by .0036. (Approx.) 7. 23.0567385 by .07525. (3 places approximate.) 8. $7296.47 by 123. (3 places.) 9. 91.38 by 1.02545. (3 places, approximate.) 10. A speed of 592 knots in 24 hours is a speed of ? knots per hour. (2 places.) EXERCISE. Note. The weight of any material, as compared with an equal bulk of pure water, is termed its specific gravity. Thus a brick having a specific gravity of 1.602 weighs 1.602 times as much as the same volume of water. Assuming the weight of a cubic foot of water as 62.355 lb., find the specific gravity of the following building materials: Material. Pounds per Cu. Ft. Specific Gravity. Asphaltum 87 Hard brick 125 Clay 120 to 145 to Limestone 170 to 210 to Plaster of paris 75 to 80 to Slate 165 to 180 to Tile 106 to 122 to Sand stone 140 to 155 to ■ 42 BUSINESS ARITHMETIC. Selected Individual Assignments. 1. Write a brief on "Methods of Checking Arithmetical Computa- tions/' giving illustrative examples. 2. Write a brief on "Short Methods in Computation." Illustrate short methods in common use. If possible, report on business men's attitude towards the use of short methods. CHAPTER VII. ARITHMETICAL AVERAGING. INTRODUCTORY EXERCISE. 1. What six equal numbers have a total equal to the sum of 12, 14, 30, 42, 18 and 10? 2. Five men give to a public cause, respectively, $40, $35, $20, $60, $25. The same sum would have been secured by their contributing equally, $ — . 3. The total profit of a business for three years was $19,654.21. As- siuning that there was no variation from year to year, what was the annual profit ? 4. The values just determined are called averages, or average numbers. What is an average number? What processes of arithmetic are employed in averaging? 5. What does a physician mean when he says "My office calls average 12 per day"? Averages offer a common and effective means of comparing values, and are used constantly in statistical work and in business. Thus the average output, expense and profit of a business, for past years, will guide a possible purchaser to a decision, or aid an owner in remedying defects. The average monthly rainfalls, in a certain farming section, will guide an intending purchaser in his decision as to possible crops. Other illustrations will be found throughout the book. Ordinarily, the average, mean, or normal, is obtained by a process of addition, followed by division. At times, the addi- tion may be mechanically accomplished, as in the case where a number of castings are placed on a foundry scale and weighed at one operation, the total weight being then divided by the number of castings to determine the average weight. In determining averages care must be used to include all influencing factors. . 43 44 BUSINESS ARITHMETIC. Illustration. On April 1, Brown deposits, in a bank that pays interest on average monthly balances, the sum of $2000; a^d on April 21, reduces this sum lo $1600. Determine his average balance for April. Solution. The average is not $2000 + 1600 divided by 2, for the individual balances existed for different times. $2000 was the balance for 20 days, and $1600 of 10 days. There were 20 daily balances of $2000 and 10 of $1600. The average is, therefore, 20 X $2000 4- 10 X $1600 $56000 ^.^^^^^ 30 ="^0~ =^1866.67 -. EXERCISE. Note. These sentences are taken at random from news reports and general publications. Determine the missing values. 1. "The Lusitania made 598, 606, 612 and 603 knots in four successive days, averaging — knots per day." 2. "England records a West Coast train from London to Aberdeen, September, 1895, 540 miles in 512 minutes miles per hour." 3. "In 1895 the number of depositors in the savings banks of the country was 7,696,000, having total deposits of $3,093,236,119, or an average of each." 4. "The distance is 192 miles, but I can average 12 miles an hour. If I leave at 7 o'clock to-morrow, that would get me there by -" 5. "Those cattle ought to average 940 lb. each, or lb. for the lot of 37." 6. "The output with the old machinery averaged 46,000 yd. per 10 hour day. With the new equipment, it is 48,000 yd. in 9 hours. The average hourly output has increased yd." 7. "Mr. Newbold testified that the improvement had increased their average daily output from 143.2 tons to 158.5 tons — an increase of tons per year." (1 year = 300 days.) EXERCISE. Note. Each example shows a different application of the average. 1. In a history test, the individual marks obtained were: 47, 90, 87, 82, 100, 100, 96, 90, 40, 35, 82, 87, or a class average of—. (2 dec. places.) 2. Twenty pupils in a class were present on the twenty-one school days in March, four were absent one-half day, three for one day, two for three days, and one for five and one-half days. Find the average daily attendance. AVERAGING. 45 3. In an Eastern city, having a normal maximum temperatm-e for July of 76.5" a hot wave caused maxima, for successive days, of 90°, 93°, 95.5°, 89°, 96°, 98°, 101°, 92°, an average of — ° above normal. 4. During November, 1907, individual rainfalls were .45 in., 2.15 in., .08 in , 1.37 in., and .78 in. Was the rainfall excessive as compared with (4.88 in.) the normal for past years? 5. The Rankin Mfg. Co. pays 60 men $2.15 per day; 80 men, $3 00; 12 men, $4.50; and 8 men, $5.75. How does its average daily wage compare with that of the Morgan Co., which has an average for the same class of work of $3,225? 6. Find the average daily and monthly circulation of the newspaper whose circulation statement is printed on. page 12. 7. Find the average speed on these long-distance runs: Course. Distance. Hours and Min. Average. Jersey City and Chicago 912 mi. 17:41 Jersey City and Pittsburgh ... 443 8:41 New York and Buffalo 440 8:15 New York and Chicago 979.52 18:00 London and Edinburgh 393 7:45 London and Glasgow 401.50 8:00 Paris and Bayonne 486.25 8:59 8. Compare the profits of the company mentioned below for the last year with the average for the past five years. Net Profits of the Cropleigh Wheel Company. 1906 $17,246 1908 $18,076.50 1910 $20,132.61 1907 21,032 1909 16,548 1911 23,328.46 1912 24,736.59 9. Explain the difference in meaning of these expressions: (a) "126 pieces of castings, averaging 84 pounds each." (6) " 126 pieces of casting weighing 92 pounds each." (c) "45 pieces of castings weighing 2746 pounds." CHAPTER VIII. THE EQUATION. INTRODUCTORY EXERCISE. 1. Use the word "equal" in comparing the cost of two articles. 2. Use the same word in expressing the cost of one article. 3. Use it in comparing the time necessary to do two pieces of work. 4. Use the word in expressing the time it takes to go from Philadelphia to New York, by rail. 5. Use it in comparing the size of two objects, one greater by one-half than the other. An equation is formed by using the equality sign (=) be- tween two equal numbers or expressions, called the members or sides of the equation. Illustrations. My business for May = that for April. The cost of 5 bbl. at $6 = the cost of 6 bbl. at $5. The quotient X the divisor = the dividend. Certain self-evident truths (axioms) aid us in the practical use of the equation: 1. Things equal to the same thing are equal to each other. 2. The equality of two expressions is not destroyed: (a) By adding equals to each term. (6) By subtracting equals from each term. (c) By multiplying each term by the same factor. {d) By dividing each term by the same factor. Illustrations. 1. If one barrel of apples cost $3 and one barrel of sweet potatoes costs $3, then — The cost of 1 bbl. apples = the cost of 1 bbl. sweet potatoes. If a = 6 and 6 = 6, then a = 6. 46 THE EQUATION. ' . 47 2. If the cost of 1 bbl. apples = cost of 1 bbl. sweet potatoes, then {a) Cost of 1 bbl. apples + $4 = cost of 1 bbl. potatoes + S4. (6) Cost of 1 bbl. apples — $1 = cost of 1 bbl. potatoes — $1. (c) Cost of 6 bbl. apples = cost of 6 bbl. potatoes. ,r^ Cost of 6 bbl. apples cost of 6 bbl. potatoes id) 2 = 2 • 3. If a = 6, then — (a) a + X = b -\- X. (]b) a-y =h'-y, (c) 6a = 66. (d) a/4 = 6/4. Transpositicm. A term in an equation may be changed from one side to the other provided its algebraic sign is changed from plus to minus, or from minus to plus. Illustration. Ifx+I0 = y a; + 10 - 10 = ?/ — 10. (Subtract 10 from each side.) X = y — 10. That is, the + 10 on the left has been changed to a — 10 on the right. In a similar way, the equation " a—b=c" may be written "a=c+6." If in an equation there occurs an expression whose value is not given, the value may be determined, in most cases, by transposition. This is termed solving the equation. Illustration. Let a + 6 = 9 + 4, to find the value of a; then a = 9 + 4 — 6 (transposing 6), and a = 7. In handling "signs," it is essential to have their order of rank, or precedence, understood. Of the four signs, +, — , X and -^ , the multiplication and division signs have precedence over those of addition and subtraction. Illustration. Let a =« 52 — 6 X 8. The multipUcation sign is considered first, and we have a = 52 — 48 = 4. The signs X and -i- have equal weight, and terms contain- ing them are reduced to simpler form by performing each 48 . ' BUSINESS ARITHMETIC. operation as its sign is met. The signs of addition and sub- traction also have equal weight. Illustrations. 9 X 8 4- 3 = 72 -i- 3 = 24. 4 + 7-3 = 11 -3 = 8. Note. The signs of addition and subtraction refer to all following values until the next plus or minus sign. Thus in"3+8X6-^2+ 8," the *' + " refers to the result of "8 X 6 -r 2." The expression equals 3 + 48/2 + 8 = 35. • EXERCISE. 1. Illustrate the five axioms, using the equation a + 8 = 6 — 4. 2. Find the "cost" from the equation "The cost of a house + $275 = $8450 - $90." 3. Find the value of 1 A. from the equation: Cost of 21 A. = $640.50 4. Solve for ar, if a; - 70 + 4 = 20 X 6 -^ 5. 5. Dividend X 8 = 64. What is the dividend? 6. Solve: 2+4-7 = aX6-^2-5. 7. A number + 16 = 128. Find the number. CHAPTER IX. UNITED STATES MONEY. INTRODUCTORY EXERCISE. 1. What is your own definition of a coin? 2. What denominations of coin have you handled? What others can you name? 3. Read the wording on a one-dollar bill. What is the difference between it and a silver dollar? What makes the paper money of value? 4. WTiat causes the difference in size between a silver dollar and a gold quarter eagle? 5. WTiy should not the Government coin iron, as money, in place of gold? 6. In what ways would a man be inconvenienced, who had much property but absolutely no money? Could he get along comfortably without money? 7. What is the disadvantage in trading one kind of property for another — instead of using actual money to pay for the desired article? One function of a modern government is to provide a system of money or currency to serve as a " medium of exchange.'* A primitive people may use anything as a standard, as corn, wampum, horses. As commerce grows, and one who desires an article cannot meet directly a distant producer who has it, property must pass through several hands; the primitive property standards then become difficult of use, and also subject the users to loss and fraud. Then the government, as a representative of all the people, adopts as standard something easily transferable, and puts its mark upon it with such care and in such a way that to every holder a certain quantity may represent the same value. Thus gold is the standard of this country, and the coming standard of the world, because high values are imputed to small quantities of it, making it easy of transfer, and because, when coined, exact uniformity of 5 49 50 BUSINESS ARITHMETIC. weight and value may be given artificially to each piece. Small denominations are coined in metals of less value than gold, because otherwise the coin would be too small to handle and it would be easily lost. Such coin, in suflficient sums, may be exchanged for gold. Likewise, principally for convenience in transferring large sums, paper certificates are issued in ex- change for gold and silver and are payable on demand in coin. The currency now in use in the United States consists of metal coins, and paper notes and certificates. UNITED STATES CURRENCY. Metal. Gold. Silver. Nickel. Double eagle . . $20.00 Dollar $1.00 Nickel $.05 Eagle 10.00 Half dollar 50 Brmize. Half eagle 5.00 Quarter dollar. . .25 Penny (one cent Quarter eagle. . 2.50 Dime 10 piece) 01 Paper. 1. Gold certificates. Denominations: $20, upward to $20,- 000 . Issued for gold deposited in the United States Treasury, and payable in gold on demand. 2. Silver certificates. Denominations: $1.00, $2.00, $5.00, $10.00, $20.00, $50.00, $100.00. Issued for silver deposited in the Treasury, and payable, on demand, in the same metal. 3. United States notes (Greenbacks). Denominations: $10, $20, $50, $100, $500, $1000. They are a part of the debt of the United States and consist of promises to pay in coin the number of dollars specified on demand. 4. National bank notes. Denominations $5, and upward to $1000. They are issued by national banks, under government supervision, and are redeemable in lawful money. Note. Legal tender is a term applied to money that may be legally ofiFered in payment of debts. Gold and silver coin of standard weight, and gold and silver certificates, are legal tender for all debts. United States UNITED STATES MONEY. 51 and national bank notes are legal tender, except for duties on imports and interest on the national debt. The Government has five mints, or coin factories, where gold or silver bullion is received for coinage, and seven assay offices. The assay offices, as well as the mints, receive deposits of gold and silver, and also manufacture refined bars of standard purity. The bars, when returned to the depositors, are used in the arts, or are exported. The metal left in bars far exceeds in value the metal coined. The mints are situated at Phila- delphia, San Francisco, New Orleans, Carson City and Denver. The assay offices are scattered throughout the country at the main points of metal production and importa- tion. Some idea of the volume of business may be obtained from these figures for a recent year: Coinage of the United States. Denomination. Pieces. Value. Gold Double eacles . • 1 495 035 1 494 795 1 559 435 142 509 $ ? Eagles Half eagles Quarter eagles Total ? 1 621 700 4 422 700 20 998 431 ? Silver. Half dollars ? Quarter dollars Dimes Total ? 46 047 950 89 588 480 ? Minor. Nickels ? Pennies Total ? ? Total Coinage ? ? EXERCISE. Find the missing values in this and the following tables. Worn or abraded coins are collected through the banks and 52 BUSINESS ARITHMETIC. are recoined. The loss through abrasion is considerable, as the following table shows: Recoinage of Uncurrent Coin. (Six recent years.) Face Value. New Coin. Loss. 1 2 3 4 5 6 $3 832 280.69 3 333 437.06 3 008 747.98 2 828 384.90 1964 476.11 1 414 963.90 $3 613 021.59 3 141 548.04 2 829 890.71 2 656 104.21 1 839 219.24 1 322 834.27 ? Total, 6 yr. Average ? ? ? ? While the total amount of money in circulation seems very large, the actual amount of money in circulation per individual of our population seems remarkably small, and shows to what extent we carry on transactions by. means of checks and other forms of credit, without actually handling currency. Money in Circulation. (Five recent years.) Population. 84,662,000 86,074,000 87,496,000 88,926,000 90,363,000 Money in Circulation. $2,736,646,628 2,772,956,455 3,038,015,488 3,106,240,657 3,102,355,605 Circulation per Capita. $32 32 ? ? ? ? IS a As has already become evident, the "money value" common factor in a large part of the arithmetic calculations incident to our daily life, yet in the majority of cases the actual calculations are based on dollars and decimals thereof — and no knowledge of denominations is essential to solutions. The following classification of United States money is common, UNITED STATES MONEY. 53 although the dime and eagle are usually omitted, being con- sidered simply the names of coins. Table of United States Money. 10 mills = 1 cent. Expressed c, ct., ^, $.01. 10 cents = 1 dime.* " d. or $.10. 10 dimes = 1 dollar. " $1.00. 10 dollars = 1 eagle. " E. When, as a result of a calculation (as, for example, the deter- mination of cost or wage) a payment of money must be made, knowledge of denominations of coins and certificates becomes of distinct value. A cashier, a bank teller, or a pay clerk, must have extended knowledge even of issues of coins and notes and of the minute details that distinguish the genuine from counterfeits. Moreover, such employees must be able to make change quickly and must be able to plan out and arrange for the quantities of coins of different denominations necessary during each day for the easy transaction of business. In connection with the cash payments, many notes and checks are handled. These, and similar papers, usually have the money value stated twice, once in words, as briefly as pos- sible, and once in figures. Portions of a dollar are usually expressed in figures, fractionally. See page 323. CHAPTER X. MAKING CHANGE. Speed and accuracy in change making and in making money payments are essential in an increasing number of positions. Making change is a process of addition. The cashier adds mentally to the amount of the purchase each coin or note he takes from his tray, until he reaches the total of the sum given in payment. In most cases he does not know the exact amount of the change returned. iLTiUSTRATioN. A $10.00 note is given in payment for a purchase of $2.37. The cashier takes from his tray three pennies, a dime, a half- dollar, a two-dollar note and a five-dollar note, saying aloud, if dealing with a customer, "Thirty-eight, thirty-nine, forty, fifty, three dollars, five, ten dollars." Question. Why is this better than a subtraction method? The careful salesman, on receiving a note, states its value to the one who gave it to him, before sending it to the cashier, in order to avoid argument when the change is returned. In the above case, the salesman might remark, "You gave me $10.00." Question. State some simple misunderstanding that might arise from neglect of this precaution. ORAL EXERCISE. Note. In practising change-making, name the sub-totals until the amount is reached. When not reciting, watch the totals given to determine the currency denominations used. Suggest other combmations. 1. Name the common United States coins, certificates and notes. 2. Make change for the foUowmg: irchase, . Pavment. Purchase. Payment. Purchase. Payment. $5.47., . . .$io.oo $15.43.... $20.00, $.01.... $.10 3 19. ... 20.00 .08 25 .18 50 .67., . . 2.00 1.32.... 5.00 3.21.... 3.50 .11., . . . .50 .21 50 .83..., 5.00 1.14. ... 5.00 .43... 2.00 64 2.17.... 4.00 MAKING CHANGE. 55 3. In each of the above cases, name the denomination of coin or notes given in payment. 4. Why should not a customer give a salesman $4.27 for a purchase of $3.09? Make change for: 5. $10.00, given in payment for two chairs @ $1.17. 6. $5.00, in payment for May gas bill, $2.53. 7. $10.00, in payment for one pair of shoes, $3.75. 8. 50c, in payment for 17 Ic stamps. Note. Cashiers may have to break notes into smaller denominations, or to make specified payments. 9. Change a $1 00 bill into nickels and dimes. 10. " Break up " a $10.00 bill. 11. Pay a check for $124, in denominations of $5.00 and under. 12. Count out $15.42 for a pay envelope. 13. Pay your milk man, $3.41. 14. Pay John Brewer $700 in Treasury notes. 15. Pay John Smith $445 in gold coin. FOR DISCUSSION. 1. Do all businesses require, for change-making, the same proportions of coins of each denomination? 2. Inquire of employees of ten distinctive businesses what coins are most needed for change-making. Report to class. 3. What businesses receive more change than they need? 4. Name businesses receiving large quantities of pennies, nickels and dimes. 5. Find out the proportions of coins required by a commercial bank in meeting the demands of its customers for change. 6. At what seasons of the year do banks make special preparations for supplying small change and bills? 7. Investigate and report on money machines, such as coin trays, cash registers, change-making machines, etc. CHAPTER XI. POSTAGE. INTRODUCTORY EXERCISE. 1. Is postage a tax? 2. Are stamps used on all mail matter? 3. Why do the stamp denominations differ in part from our coin denominations? 4. What government stamps are issued for other objects than the direct payment of postage? 5. Name some limitations as to acceptable mail matter? 6. How does the modern postal service help business? 7. What is "rural free delivery"? 8. Why is the regular postage rate in a country the same, irrespective of distance the matter is carried? 9. TMiat is the " parcels post "? The Post Office Department is one of the greatest of business organizations under government control. It is not run for profit, but its enormous expenses are met, largely, by postage and money order receipts. Some idea of its cost may be obtained from these figures for five recent years: Post Offices and Post Routes. Year. Post Offices Num- ber. Extent of Post Routes Miles. Revenue of Department Dollars. Expended for Trans- portation. Total Ex- penditure of Department. Defi- Domestic Mail. Foreign Mail. cit. 1 2 3 4 5 65,600 62,659 61,158 60,144 59,580 478,711 463,406 450,738 448,618 447,998 167,932,782 183,585,006 191,478,663 203,562,383 224,128,637 72,944,352 77,471,917 78,174,988 80,901,899 81,709,433 2,895,756 2,988,849 2,982,732 2,804,170 3,164,254 178,449,779 190,238,288 208,351,886 221,004,102 229,977,224 Total ? ? ? ? 56 POSTAGE. 57 POSTAL INFORMATION. 1. Domestic Mail Matter. Postage rates apply also to mail matter for Canada, Canal Zone, Cuba, Guam, Hawaii, Mexico, Philippines, Porto Rico, Republic of Panama, Tutuila, and the U. S. postal agency at Shanghai. The domestic rate, for letters only, applies to Germany, Great Britain and Ireland, and Newfoundland . First Class. Letters and sealed matter: 2c for each ounce or fraction. Postal cards and post cards: Ic eacL Limit of weight, 4 pounds. Second Class. Newspapers and periodicals: Ic per 4 ounces or fraction. No weight Umit. Publisher's rate, Ic per pound. Third Class. Books and miscellaneous printed matter: Ic per 2 ounces or fraction. Limit of weight, 4 pounds, except for a single book. Fourth Class. (Parcels-Post.) All matter not in other classes. Seeds and plants, Ic for 2 oz. For other matter the coimtry is divided into unit areas and ei^t zones. The first zone includes territory within a radhis of 50 miles of any unit area. The radius of the second zone is 150 miles, of the third zone, 300 miles, etc. Postage rates: Under 4 ounces, Ic per ounce; over 4 ounces, for delivery in first zone, 5c for first pound or fraction, and 3c for each additional pound. For each additional mile of delivery there is an increase in the rate. Special stamps must be used. Limit of weight, 11 pounds. Consult your postmaster as to rates and size limitations. Special Rates. For registering for safe delivery, 10c in addition to regular postage. Indemnity limit, $50. For "special delivery" or im- mediate dehvery within the carrier-delivery limit of city free delivery and within one mile of any other United States post oflice, 10c in addition to postage. Notes. Matter that is harmful, libellous or threatening, which has to do with lotteries or fraudulent schemes, is unmailable. At least 2c postage must be prepaid on first class matter. All other matter must be prepaid in full. Stamps are not affixed to publications mailed by the publishers, which are weighed in bulk. Stamps need not be affixed to third and fourth class matter consisting of at least 2000 identical packages. In these cases, a special "permit" is printed on the wrapper to show that postage has been paid. Postage Stamps and Postal Cards. The denominations of stamps are 1, 2, 3, 4, 5, 6, 8, 10, 15 and 50 cents; and 1 dollar. Stamped envelopes are issued of different sizes, for de» nominations of 1, 2, 4 and 5 cents. Postal cards are issued for Ic, single, 68 BUSINESS ARITHMETIC. and for 2c, reply. Private post cards must be within fixed limits as to size and must have a Ic stamp attached. 2. Foreign Mail Matter. Rates and Regulations. Letters and Sealed Matter. 5c for first ounce; 3c for each additional ounce or fraction. See Domestic Rates, for certain coimtries. Postal Cards. 2c, single; 4c, double- Commerical Paper. Ic for each 2 ounces or fraction; not less than 5c per package. Printed Matter. Ic for each 2 ounces or fraction Samples of Merchandise. Ic for each 2 ounces or fraction, but at least 2c per packet. Parcels-Post. 12c per pound. There are so many conditions as to such mail that it is advisable to consult your postmaster before sending package. Conditions are not uniform for all countries. (For other information see United States Official Postal Guide, or a booklet on Postal Information, issued for public distribution.) EXERCISE. Classify this mail matter. Determine rate and cost of postage. Des- tination, the United States unless otherwise stated. 1. A 2 oz. letter to London. 9. 9 oz. of seeds. 2. 40H oz. sealed ckculars. 10. 12 oz. seeds, registered. 3. 18 oz. merchandise samples H* A. 6 lb. package to Bremen. toBerlm. 12. 3 lb. 7 oz. of merchandise. 4. Issue of 60,000 4-oz. maga- (^h-st zone.) zines. 13. A Christmas card, IH oz. 5. 8 souvenir postals to London. 14. 4750 lb. of newspapers. 6. IH lb. of gunpowder. 15. 9 oz. of proof sheets. 7. Mdse., 3 lb. 9 oz., to Zurich. 16. A book weighing 50 oz. 8. 83^ oz. commercial paper to Paris. 17. Estimate the total postage on: 21 letters; 150 circulars, single page; 4 pkg. merchandise, respectively 9 oz., 21 oz., 16 oz., 13 oz.; 1 pkg. seeds, 14 oz.; 2 registered letters; 5 special delivery letters; 25 postals. (Pkg.— first zone.) 18. Estimate the publisher's postage bill for his December issue of 147,456 copies, weighing 11 oz. each. 19. Compute postage on the following, and state the simplest stamp denominations to use: (a) On a 4-oz. letter, by special delivery, (h) On a 10-oz. re^stered package of merchandise. (Second zone.) POSTAGE. 59 (c) On a 53^ oz. sample of merchandise. (Fifth zone.) (d) On a book weighing 47 oz. (e) On a 3^ oz. registered letter to London. Most business organizations, sending out hundreds or thousands of pieces of mail matter per day, keep accurate postal records, such as the following : Postage Report. For the month ending January 31, 190-. Dr7 Total Bought. Departments Total of Mo. Balance. Hand. Pur- chase. Adrer- tising. Sales. Acct'ng. Miscel- laneous. for the Day. 1 2 3 4 5 145 115 90 200 120 20 90 25 ? ? ? ? ? ? ? ? ? 9 4 8 6 16 16 90 52 10 82 7 5 12 9 13 25 40 82 75 40 Ill 92 47 105 116 40 85 32 86 45 6 3 4 8 11 15 27 52 21 06 3 1 10 09 96 72 52 ? ? ? ? ? Note. At the close of the day the maiUng clerk enters under each department its cost for postage. The ''Total Bought" column shows each day's purchases; "Balance" shows the quantity from the previous day. Together, they equal the amount on hand. The subtraction of the total for the day determines the new balance. Two forms follow, which are designed to show even the denomination of stamps used, and a classification of the mail matter (page 60). On hand Received Total Used Balance Value of bal. Denominations. Ic 54 700 ? 528 ? ? 2c 110 500 ? 602 ? ? 3c 4c 90 400 ? 128 ? ? 5c 45 100 ? 46 ? ? 6c 150 ? 51 ? ? 8c 200 ? 40 ? 10c 300 400 ? 161 ? ? 15c 152 50c 40 100 ? 45 ? ? 1.00 The Postage Account On hand Rec'd Total Used Bal. EXERCISE. Complete the extension of the above forms, and of the form on page 60, 60 BUSINESS ARITHMETIC. is" g o QQ I o ^ CHAPTER XII. PAYMENT FOR SERVICES. INTRODUCTORY EXERCISE. 1. Name advantages and disadvantages of being a wage-earner. 2. What payment for services does a private householder make? 3. Name typical cases in which compensation is spoken of as a wage As a salary. 4. What name is given the payment of a lawyer? Of a physician? How do these payments differ from wages or salary? The modern organization of business demands a careful accounting of all sources of expense. Of these, the cost of labor is a leading item and requires special attention. Some idea of the sums paid for services may be obtained from the following recent Census figures for a few selected industries: No. of Estab- lishments. Wage-Earners. Ayerage Wage. Industry. Av. Num- ber. Wages. Agricultural implements 648 1316 18227 4956 4504 9423 605 624 47,394 149,924 81,284 60,722 137,190 402,914 242,640 79,601 $25,002,650 69,059,680 43,179,822 30,878,229 57,225,506 229,869,297 141,426,506 26,767,943 $? ? Bakery products Carriages, etc '. ? ? ? ? General iron and steel works ? Silk, gilk goods ? Methods of payment vary, and the calculations involved are often so numerous and technical as to rank this class of com- putation as an independent subject. With millions of wage- earners, it is evident that the subject affects, directly or in- directly, our entire population. 61 62 BUSINESS ARITHMETIC. In business, the recompense paid, or stipulated to be paid, to persons, at regular intervals, for services rendered, is called salary or wage. In general, a stipend based on long intervals (i. e., year or month) is called a salary. Recompense reckoned by brief intervals (i. e., week, day, or hour) is usually called a wage. There are many exceptions to this classification. In some businesses, payment for distinctively mental labor is termed salary and manual labor wage. Thus a clerk in an office receives a salary, while a high-class mechanic, working at a machine, may receive a far higher wage. In manufacturing establishments payment for services is frequently based on the quantity of work done. This work is called " piece work." Salaries. Salaries are usually paid monthly or semi-monthly. The rate is commonly expressed in dollars per year. A month is considered one-twelfth of a year, but in many cases, pay for a fraction of a month is based on the exact number of days in the month. (How would one day's earnings in February compare with one day's in January?) Payments are usually made by check or in cash. Sometimes employees are credited on the books and may draw funds as needed. Usually a pay roll is prepared and is signed by each person when he receives payment. A pay roll is shown on the opposite page. PAYMENT FOR SERVICES. 63 o CO 6 6 ^ o ^ ^ o '"'*"' o '"' o o '"' a> Sis fe 2^ 2 q3 2 o3 o g £ > ^ c^ H u o i I a a ^ o ai rt Eh OJ r! ^ 9 fl a E-< P^ H? W 4 O •c.a c- o gl § 8 05 05 05 05 11 5 r CO i-H 1 CO T^ 1 i i CO i-iO fi ^ u a ^ !fl p ^ > •>-> ■4^ &• g J^ -t-" u a> 1 u o 1 -2 fs 'S o o a u riiJ hf o u 01} -^ >1 ^ a ri 0) ■s :S i .S' o so c .S . •5 ? " ^ .^ -►^ a» 3 2 ^Q 1=1 « So 1' bS HS Si 00 s p^ .s J 4r?i^«4 s c-e^c- c^ IS ":»!l< O 0000 0000 o ^ 00 00 0000 00 00 Cffi 00 0000 0000 00 00 00 00 0000 0000 v^ w ^v^^ a ^ u u G o o if bC O o i CO l>00050 r-(C^ cc^ cof;- 66 BUSINESS ARITHMETIC. From the "time in hours" and "remarks" columns explain how each person worked, and how his wage was determined? {Note. In the above case, Sunday time and overtime is paid at double rates.) Do you notice anything unusual in the signatures? ORAL EXERCISE. Reckon wages for the week ending March 9, as follows: 1. 36 hr. at 15c; 42 hr. at 18c; 48 hr. at 20c; 47 hr. at 27c; 39 hr. at 19c; 42 hr. at 30c; 54 hr. at 33ic; 46 hr. at 25c. 2. Reckon wages in the following cases, basing calculation on a 54 hour week, with double pay for overtime: 42 hr. at 15c; 54 hr. at 30c; 54 hr. at 32c; 48 hr. at 17c; 60 hr. at 25c; 56 hr. at 30c; 45 hr. at 35c. EXERCISE. Rule up and extend a pay roll for the current week for some shop, factory or contracting firm. Add to the pay roll a column to show work- man's position or department. Follow directions in example 5, page ^. Schedule of Change Desired. By Address In 1-cent pieces ^ " 5-cent " ■ " 10-cent " • " 25-cent " • " 50-cent " • " Silver Dollars 1 1 2 14 14 40 60 75 50 In Notes or Certificates- Si 41 36 20 100 2 ' 5 io 20 50 100 Total S 217 39 When the pay envelope method is used the pay clerk com- putes, or if he is an expert he approximates, the quantity of each coin denomination required to enable him to *'make- PAYMENT FOR SERVICES. 67 up" each employee's exact "amount due." Sometimes this coin classification is incorporated in the pay roll. Often a currency memorandum similar to that shown opposite is prepared, to be presented at the bank when the check for the total of the pay roll is cashed. EXERCISE. 1. Prepare a denomination slip for this series of payments: $4.78, $9.62, $11.21, $11.45, $12.00, $12.65, $13.11, $14 29, $14.43, $15.80. 2. Prepare a currency memorandum for the pay roll shown on page 65. 3. Prepare a wage pay roll and currency slip for a contractor's force for the current week. The force consists of 1 foreman, 4 drivers, and 17 loaders. One man is discharged and two are hired during the week; one is out sick for one day. The pay clerk who is obliged to handle a large pay roll based on fixed rates, often buys or computes wage tables as aids to rapid work. These give the wages due for any number of hours or half hours, at given rates per week. The original computations are made with absolute accuracy, and are then written in the table to the nearest cent. EXERCISE. The Rankin M.^j^ufacturing Company. $12.00 48 hours. Hour. Wage. Hour Wage. Hour. Wage. Hour. Wage. Hour. Wage. Hour. Wage. k 13 8^ 2 13 16i 4 13 24i 6 13 32i 8 13 m 10 13 1 25 9 2 25 17 4 25 25 6 25 33 8 25 41 10 25 1* i38 9^ 2 38 17^ 4 38 25^ 6 38 33^ 8 38 4U 10 38 2 50 10 2 50 18 A 50 26 6 50 34 8 50 42 10 50 2^ 63 10^ 2 63 18^ 4 63 26^ 6 63 34-^ 8 63 42-^ 10 63 3 75 11 275 19 75 27 6 75 35 8 75 43 10 75 3^ 88 IH 2i88 19^ 88 27-^ 6 88 35i 8 88 43^ 10 88 4 100 12 300 20 5 00 28 7 00 36 9 00 44 11 00 4^ 1 1 20^ 1. Complete the above table. (The upper half only is given.) 2. What error would result from constructing the table by com- putations based on the approximate value for the first half hour? 68 BUSINESS ARITHMETIC. 3. Read off a man's wage, at the given rate, for 37 hours, 21 hr,, 13 hr., 12 >^ hr., 19 hr., 363^ hr., 50 hr. (double), 52 hr. 4. How might the table be used for a wage rate of $6.00 per week? Of $18.00? 5. Construct a table for a wage of $15.00 and a week of 54 hr. 6. Construct a 48 hour wage table showing overtime at double rate up to 60 hours. Teacher's Note. Assign each pupil a different rate. Have tables prepared and exchanged for testing. In some factories and large stores time clocks are used to record the arrival and departure of employees. In some systems, the employee inserts his key in the clock, thus auto- matically recording his number and the time. In other systems, the employee inserts a card in the clock, the time being stamped automatically. Generally, the form is used to show whether or not the employee is on time — small frac- tions of hours being ignored. In other organizations, a few moments* tardiness may cause the docking of the employee to the extent of one-half, or one hour. EXERCISE. 1. Compute the cost of labor on the piece of work shown by ticket A. Reckon to the nearest one-fourth hour. The rate given is the rate per week. 2. Compute the time of eight men for Mon- day, and determine the wages due at 31 ^c per hour: FM>r kCT OYE CRY EXPENSE .0. ^7S WOR f €2e-^ DATE V27 1 DAY IN OUT IN OUT rOTAl s A.M P.M s A.M P.M M A.M. 6A« 11^1 P.M. 2253 5«2 T A.M. 6l» &*2. 11^1 V>J1 P.M. -. W A.M. :^c^ wiA ed P.M. T A.M. P.M. F A.M. P.M. p //S^ff fiOUT ' Ticket A. No. 1 2 3 4 5 6 7 8 In. 6:59 7:01 6:58 6:59 6:58 6.56 6:50 Out. 12:01 12 .02 11.59 10:01 12:00 12:01 12:02 In. 1:00 1:01 12:58 2:00 12:59 12:58 12:57 12:55 Out. 4:59 5:03 5.01 5:02 5:03 3:02 4:59 5:04 PAYMENT FOR SERVICES. 69 Premiums or Bonus Work. The premium plan of labor payment is intended to reduce the time of production in factory work, by the payment of a bonus for extra efficiency. Usually, as a result of hundreds of experiments, a fixed average time for a certain operation is determined. The employee is then told that his employer will share with him any saving in working time as compared with this average. For example, the offer might be "one-half of the wage saved." If the average time to make an article is 10 hours, and the wage $4.00, the employee earns 20 cents extra for eVery hour saved. The table below shows the effect of the plan. Time Con- sumed in Hours. Wage per Piece. Premium. Total Cost of Work. Workman's Hr. Rate. Workman's Day Rate. 10 9 8 7 6 $4.00 3.60 3.20 2 80 ? .20 .40 ? 9 $4.00 3.80 3.60 ? $.40 .422 .45 ? ? $4.00 4.22 4.50 ? ? Note. Often a set number of pieces or operations is required per hour, and any increase in quantity is paid for at a special rate. Often, if the wage is stated as a price per piece, the wage rate is increased for a quantity over a certain fixed number. EXERCISE. 1. Complete the above form. 2. Prepare a similar table showing one-half hour time reductions from 6 hours to 4 hours, the 6-hour standard rate being $3.60. 3. A workman, whose wage is 38c per hour, reduces his working time on a stove from 4^ to 3 hours, earning a premium of 16c per hour. He raises his wage for a 9-hour day to $ — . 4. The standard number of pieces required of a polisher per hour is 12. The rate is 5c each, or 6c for extra efficiency. He averages 13^ per hour, for an 8-hour day, earning $ — . Bills for Professional Service. Professional men, as doctors, and lawyers, and many inde- pendent artisans, charge fees for their work, and render hills for services. As a rule the computations are very simple. 70 BUSINESS ARITHMETIC B. C. Student & Co., No. 848 Payne Ave., To Charles Rawlingson, Dr., Attorney at Law. Lawyer's Bill. Cleveland, Ohio, Dec. 21, 19—. For professional services rendered in suit against Salton & Bro. Court Costs, Stenographer, 26 hr. at 30c Note. Bills for the service of an agent are considered in the chapter on Agency. EXERCISE. 1. Draw up a lawyer's bill for services. Fee, $275; court costs, $30; stenographer, 17 hr. at 32c; registration fee, $3.50. 2. Draw up a cabinet maker's bill for remodelUng an antique side- board. Material, $1.75; delivery, 50c; labor $ — . 3. Draw up an original bill for services. 4. From the doctor's memorandum card illustrated on the following page, showing house treatments (numerals), and office treatments (O), draw up a bill. The rates are $2 and $1 respectively, for house treat- ments and office visits. Supply names. PAYMENT FOR SERVICES. 71 T-H CO g §5 1 Oi T-l 00 1—1 1—1 CO rH T— I o tH Oi 00 t>. CO lO CO (N 1— < 1-9 < O o o o o 1—4 P O >-> o o 1-H l-H 1—1 (N T— 1 (N (N rH l-H 1—1 6 O o — 1— 1 1—1 1-H 1—1 o 8 o i o o d CHAPTER XIII. BUSINESS TERMS AND ACCOUNTS. All business is an exchange of one kind of value for anothei kind. The values exchanged may be general property, funds, or services. In this connection, the business man employs two terms: (1) Debit, as an expression of value received or due; and (2) credit, an expression of value delivered or owed. Illustrations. (1) A dealer gives $200 (a credit) for a horse (debit). (2) The dealer pays his clerk $20 salary. The debit, or value received, is services; the credit is $20 cash. (3) He buys 40 bbl. of flour from Norton, giving his note for 30 days. The debit is merchandise;^ the credit a note payable. Notice that in every transaction the debits equal the credits. Accounts are collections of debits or credits relating to a particular person or thing. Thus the Cash Account is a record of all cash received or paid out. Other accounts with which the average person meets are: Merchandise, Expense and Personal. If, in the course of business, a transaction, or a series of transactions, produce more than they cost, a gain results; if the sum spent exceeds the returns, a loss results. The excess of gains over losses, for an entire business, or for a certain series of transactions, is termed the net gain; the excess of losses over gains is a net loss. Any property owned by, or owed to, a person is termed his resource; any amount he owes is termed his liability. If resources exceed liabilities, a person has a net capital, or present worth, equal to the differ- ence. If liabilities are greater, the excess represents his net insolvency. Properly kept accounts show the resources, lia- bilities, losses and gains of a business. The Cash Account shows the debits and credits of cash, the 72 BUSINESS TERMS AND ACCOUNTS. difference, or balance, representing the amount of cash hand, as follows: Cash, 73 on 19— RECEIVED 19- 2000 — €ct. 18 Vfi — 20 485 60 24 31 2625 60 PAID * nee 30 m 2625 60 Note. The halnnce is entered on the smaller side, so that the sides "total" the same. The source of receipt or payment is sometimes entered in the explanatory space. The Merchandise Account shows the cost of merchandise bought (debits) and the receipts from sales (credits). In general, explanations are not written. Here, however, they show, among other things, the people who owe, or are owed, for merchandise. The balance shows a loss, if the debit side exceeds the credit, or a gain, if the credit side is in excess. See below. Merchandise, 19- MDSE. REC D. ^oodi. on, Aa/nc6 5.K>. So. ^ane i6\ifr^. ^ou^ 18,^i^uA, 3lf^aMt 19— 845 20 J^' 5 278 40 8 840 60 15 500 — 29 ??? — 31 2683 SO MDSE. DELIVERED. 40 120 H 2146 50 60 20 30 Note. If unsold goods are on hand their cost value (inventory) is entered on the credit side. This is equivalent to subtracting the inventory from the debit side, in order to find the cost of the goods sold, and thus to determine the gain. The Expense Account, as illustrated on the following page, shows the cost of running a business: 74 BUSINESS ARITHMETIC. Expense, 19— 19— ~ J5^. 2 ^o^la^ ^^ 20 ^l 28 ^'n/tten/c^tt^ 2 m 6 "^oa/ SO — {^43S^Cl^) 15 ^^^ 60 — 28 ,^'n/v^9tto'?^ 8 50 21 C^^'yleA. 38 50 {cc^) 26 J^^^ 12 47 28 %^S m 156 17 156 17 A Personal account shows the amounts owed to, or by, the person whose name appears in the title. Henry Brown, 1456 K St., N, W. 19— ^Md^. ^Mc{&e. 240 165 — 19— 10 10 25 405 — 176 50 176 50 5 JfiO 105 48 'i'i Note. Whenever the two sides of a personal account are equal, it is ruled up to show no balance or debt. If debits exceed credits the person named owes the firm the balance; if the credits are in excess the firm owes the person named. The Proprietor's or Partner's Account shows his relation to the business. The credits usually represent investments and gains; the debits show the losses and withdrawals. A credit excess represents the present worth, or net capital; a debit excess, the net insolvency. A proprietor's account is shown on the next page. BUSINESS TERMS AND ACCOUNTS. S. p. Mason, Proprietor. 75 19— 11 85 f 60 19— >-- 1 1 31 12865 SI JfiOO 80G0 865 12865 ORAL EXERCISE. 1. What are debits? Credits? WTiat is an account? 2. When is an account debited? When credited? 3. What sum of cash was received, according to the cash account? What sum paid out? What sum was on hand on October 21? For what was cash paid out during October? 4. From the Merchandise account it is evident that, $ ? worth of merchandise was bought during January, and that, $ ? was sold. $ ? was sold for cash; $ ? for a note; and $ ? to be paid for later by ?. 5. Henry Brown's account shows that he bought merchandise on what dates? For which purchases has he paid in full? For which has he evidently paid in part? 6. 8. $2500? Name the debit and the credit in each of the following transactions: (a) The fii-m buys merchandise for cash, $200. (6) The firm sells A. B. Bolton, merchandise, $400, for which he agrees to pay in ten days, (c) The firm pays cash for office furniture, $120. {d) The firm receives cash for some real estate. (Real Est. Ac.) (e) The firm pays cash in settlement of a gas bill. Classify the followdng as resources, liabilities, losses and gains: (a) The balance of the Cash Account. (6) A personal account with debits of $500 and credits of $400. (c) Merchandise account with debits, .$600, credits, $700. No goods on hand. id) Expense account with debits of $46.50 and credits of $13.00. (e) Merchandise on hand, $125. What is the present worth, if resources are $12,800 and* liabilities 9. What is the net loss if gains are $560 and losses $764? 76 BUSINESS ARITHMETIC. EXERCISE. 1. Copy the Cash Account on page 73 through Oct. 24. Add these entries: Oct. 25, received for merchandise, $250; Oct. 26, paid for horse and wagon, $185; Oct. 27, lost $2; Oct. 29, received from C. B. Drew, $89.60. Balance the account. 2. Alter the Merchandise Account as follows: Goods unsold on Jan. 31, $2003.15; amount paid by A. B. Brown, Jan. 15, $36.40. Balance the account. 3. Find the balance again, assuming that all goods had been sold. What does the account show? 4. Write an account for D. Poynter, Proprietor, showing invest- ments on Jan. 1, of cash, $2500; merchandise, $3867; and real estate, $4500. Withdrawals, Jan. 10, $265 cash; Jan. 18, $1645 in merchandise. Net gain, Jan. 31, $211.46. NAME ADDRESS t^Hewi^Ae^ ^e^t9t. FILE NO. 19— 16 21 ^«fe^ ^ 3620 4125 360 — 91- 17 31 30 d. rto£i^ 360 214 — 214 85 85 30 4356 561 40 J^.' 9 " 245 — J^. 3 ■ 4581 311 27 20 60 d. rvole 900 — 19 4936 1216 03 20 SS^ ccoA, 316 03 21 26 6011 5203 1008 65 60 40 25 28 17 f f 25 f (a) Brewer still owes $ ? (6) Between what dates did Brewer owe nothing? (c) Between what dates did he owe the least amount? {d) How do his cash payments compare with his payments by note? (c) Explain the credit item for Feb. 25. CHAPTER XIV. ADVERTISING. Modern advertising, or solicitation through public notice, really dates from the insertion of such notices into the maga- zines in 1864. In that year, the most prominent monthly con- tained, in its October number, three and one quarter pages. During this period advertising rates rose enormously, as did the number and circulation of periodicals. Advertising ex- tended in heavy volume to newspapers, and then to bill boards, complicated catalogues, street car signs, show window dis- plays and demonstrations. In a recent year it was estimated that $600,000,000 was spent in these main sources of adver- tising. Practically everyone reads advertisements; and many people, regularly or on some special occasions, advertise. The cost of advertising is one of the heaviest expenses of many a modern business. The methods of computing costs and returns, how- ever, are based on the simplest arithmetical processes. INTRODUCTORY EXERCISE. 1. What is advertising? 2. Name at least five ways of advertising merchandise in print. 3. Name other ways of advertising. 4 What is a trade mark? Is it a form of advertising? 5. What effects have climate, season and prosperity on advertising matter? 6. Is the paper of largest circulation necessarily the best advertising medium? 7. What is the value of a "catch phrase" or title? 8. Why do not local dealers advertise in magazines? 77 78 BUSINESS ARITHMETIC. 9. Does the character of newspaper advertising differ from that oi magazines? 10. Can you name any article brought into general use by advertising? 11. Some advertisements give no dealer's name. Why? 12. What need has the salaried man or private householder to adver- tise? 13. Special, Bring to class specimen advertisements, securing cost of insertion, where possible. Wall Signs, for advertising purposes, are painted on blank walls of buildings, fences, bulletins, etc. Contracts are usually for the year, including painting, and exclusive of the vary- ing commissions or fees for rental. For bulletins, the price is quoted per running foot. EXERCISE. 1. A manufacturer of breakfast foods rents for $200, a wall space of 40 ft. by 24 ft., and pays 6c. per sq. ft. for the sign. Find the entire cost. 2. Lawton & Co. rent spaces as follows: 26' by 20' for $250; 20' by 18' for $4 per sq. ft. ; 32' by 21' $2.50 per sq. ft. The charge for placing signs is $4.25 per sq. ft. What is the entire cost? 3. I contract for six months, for a 20' sign on a standard bulletin board (10' high) at 45 c per running foot per month. What is the cost of ad- vertising? 4. What is the cost of a one-year contract for 60 feet of bulletin space at 55 c per foot per month? Posters for advertising purposes are shown on bill boards. These boards are designed to accommodate posters four sheets high and one to six sheets wide. Complete posters are spoken of as 4s, 8s, 16s, etc., according to the number of sheets. The sheets measure 28 in. high by 42 in. wide. A bill-poster's space is rented by the month of four weeks, or the year of forty-eight weeks. Prices are usually quoted for four weeks' display. EXERCISE. 1. Find the dimensions in feet and inches of 4, 8, 12 and 16 sheet posters. ADVERTISING. 79 2. Allowing for 1/10 rene\\:als of damaged sheets, how many sheets are required for 450 24-sheet posters? 3. To post 100 16-sheet posters for ten weeks cost a cereal company 16 c per sheet per month, or $ ?, 4. Extend the following: Estimate of the Cost of Two Thousand 16-sheet Posters. Number required 2000 For renewal, 1/5 ? For 16 s. at $269 per M $ Freight and drayage 26.45 To be posted 12 weeks 560 in small towns at 14 c $ 1200 in specified cities at 16 c 240 in San Francisco at 12 c $ = Street Car Posters are printed on cards, usually 1 1 in. by 21 in., though some of other size are used. The rates average about 40 c per card per month, on yearly contracts, 45 c on contracts for six months, and 50 c on three months contracts. EXERCISE. 1. The manufacturer of a toilet soap contracts for 3500 cards at $47.50 per M., paying S18.50 for the design, and $16.00 for the necessary plates, a gross cost of $ . 2. He distributes 3/5 of his supply (on six months contracts) throughout Middle West cities at prices given above. This cost him $ ? 3. The balance he distributes in Pennsylvania and New York on a one year contract, at the above rate, except for 80 for which the price is 35 c. Find the cost of the year contract? 4. Prepare a statement showing the entire cost to the manufacturer. Newspaper and periodical advertising absorbs the larger portion of the money spent on publicity. Periodical adver- tising is usually the more costly. The common measure of space is the agate line, of which fourteen equal one inch of column space. One periodical charges $6.00 per insertion per agate line, or $4000 for a full page of 9f '' by 14j". 80 BUSINESS ARITHMETIC. For a standard magazine page, 5^" by 8", rates may run up to $500 per insertion. Extensive general advertisers usually engage agencies to prepare copy, select publications, and arrange for insertion. The agency is paid either a lump sum or a commission. In some cases the commission is paid by the publisher. EXERCISE. 1. yxom the following table find to two decimal places the average number of copies issued for each member of our population. Newspaper and Periodical Circulation. * (Round numbers). Year Population Gross Circulation Copies per capita. 1850 23,000,000 426,000,000 ? 1860 31,000,000 928,000,000 ? 1870 39,000,000 1,509,000,000 ? 1880 50,000,000 2,068,000,000 ? 1890 63,000,000 4,681,000,000 ? 1900 76,000,000 8,168,000,000 ? 1910 92,000,000 13,105,000,000 ? Find necessary values for the following : 2. The recent holiday numbers of ten standard magazines contained respectively, 192, 181, 175, 171, 150, 150, 148, 193, 138, and 127 pages of advertising, a total of pages, and an average of pages. 3. In the same issue, the agate lines of advertising were 43,776, 41,496, 39,900, 39,047, 34,980, 34,392, 34,088, 33,306, 32,172 and 27,132 Imes. Total lines . Average lines . 4. Five great railway systems recently spent for advertising in one year, respectively; $348,457; $150,647; $84,335; $147,564; $25"l,532— an average of $ . 5. A half-page advertisement in a monthly, $448 per page less one-fifth for time, costs a dealer $ for twelve insertions. 6. The magazine referred to above contained 168 pages of advertising in a given number. Estimate the minimum income. Why might the income be larger? 7. A novelty desk pad is advertised by its maker in a 20-line space for 3 months, in five monthlies, having line r^ies as follows: $3.50, $2.75, $2.50, $2.00 and $1.20. Compute the cost. ADVERTISING. 81 Note. Agencies receive net or gross rates from publishers. They charge advertisers the net rate, plus a commission ; or, if under the gross rate agreement, charge the advertiser this rate, receiving a commission or rebate from the publisher. 8. Complete this statement. New York, N. Y. Nov. 30, 19 . Mr. Brown C. Norton To J. F. Jamison, Dr. Advertising Agent. Net Gross The Johnstone Weekly ip. 12 times 5400 The World Magazine 1 col. 12 times 2700 The Hour ip. 12 times 3880 Every Moment 1^ col. 12 times 7650 The Monthly Ip. 12 times 3648 Literary Magazine Ip. 12 times 960 Livelihood Icol. /lO 12 times 1260 Commission 1 ? ? ? ? In newspaper advertising, which is largely local, many dealers make contracts for a fixed amount of space per annum, to be used as required. This space is usually expressed in line measure. For "want" advertising and minor notices, rates vary with the advertising matter. The rate cards of newspapers are often very complicated. Opposite is a small section of one. Extract from a Newspaper Rate Card. General Display Agate Line. Run of paper 50c. Special portion, last page, opp. editorials, etc. 55c. Space Discounts. One Year Limit. 5000 Unes 1/20 off. 20000 lines 3/20 ofif. 10000 " 1/10" 40000 " 1/5 " Classification Rates. Agate Line. Amusements 50c Une. Public notices 20c line. Art sales, etc 20c " Railway notices 15c " Auctions 20c " Birth, marriage, death notices $1.00 ea. Financial notices 45c line. Reading notices, financial page, $2.00. Railway time-tables, daily by the month 20c line. Railway time-tables, daily by the year 15c line. 82 BUSINESS ARITHMETIC Want Advertisements. Per Line 1 time 3 7 Per line 1 time 3 7 For sale 18c 25c 40c Lost and Found. . .8c 20c 35c Help wanted 8 20 35 Positions wanted. .5 10 15 Note. Eight words to the line: five words if set in caps. ORAL EXERCISE. Use above rates where none are stated in problem. Find the cost of: 1. 60-line advertisement, run of paper. 2. 28-line advertisement, general display. 3. 5-inch advertisement, special position on last page. 4. 100-line advertisement, special position surrounded by reading matter — the charge being 5c. extra per line. 5. 5000 lines, contracted for 55c. 6. 20,000 lines, contracted for 35c. 7. 10,000 lines, quoted rates. 8. 4-inch public notice. 9. Three insertions of a four-line liOst and Found notice. 10. One month's insertion of a 56-line railway time table. 11. A six-inch reading notice on the financial page. 12. Seven insertions of a six-line " position wanted " advertisement. 13. A ten-inch advertisement of a circus. EXERCISE. 1. A 12-in. advertisement in a Chicago newspaper @ $4.92 per in. costs $ ? per insertion, or for 26 insertions @ $3.92 per inch, per in- sertion, $ ? 2. A Seattle dealer contracted for 12,000 lines @ 49c. During January he inserted advertisements as follows : 120 lines, 65 lines, 140 lines, 210 lines, 560 lines (twice running), 28 lines. What is his bill for the month? 3. A 70-line double column advertisement in a Kansas City paper, @ $1.40 per inch, costs how much for two insertions? 4. Compute the cost of this advertisement at quoted rates. WANTED— EXPERIENCED SALESWOMEN IN books, stationery, handkerchiefs, notions, neckwear, toys, dolls, umbrellas, &c. Also bright young women from 8th grade and high school. Box 462, Boston, Mass. ADVERTISING. 83 5. An agency agrees to insert the above "want ad" in 103 city papers for $35.00 for 21 words, and $11.65 for each additional 7 words. This will cost $ ? Advertisers watch the returns from advertising and dis- card pubHcations that fail to bring customers. Returns are traced in many ways. For example, an advertiser may alter his street number, writing it "234 State Street" in one period- ical, and "236 State Street" in another; or he may offer a different bargain in each magazine; or he may insert in the advertisement a reply-slip, properly keyed with a number or letter for identification. The replies are then classified, and the papers yielding the lowest cost per inquiry, the greatest number of orders, or amount of sales, are rated as the best advertising media. EXERCISE. 1. The form on p. 84 illustrates a cost per inquiry card. It is arranged as a calendar showing the number of inquiries received on each date. Why are no replies received on Jan. 8, 15, etc.? Each horizontal line total represents what? The vertical total? Extend the form and find all values. 2. To trace advertisements month by month, and to see which of a series of advertisements draws best, the key in a magazine may be changed in each issue. For example, the magazines may be numbered from 10 to 99. To them is affixed a number from 1 to 12 to show the issue. Magazine No. 10, for a Februarj^ issue would be represented by 102, because February is the second month. The form on p. 85 is made out for June, and shows on the left the classification of calendar replies. The blocks on the right (1 to 12) are a classification of replies by the advertisements that called them forth. Find the totals and explain the form in full. 84 BUSINESS ARITHMETIC. o 0) CO •- 'h:^ xi >! CO o 6 S I o " — d (5 cc O O « I^Q ^ <; 1 c^ C-- ff- o- (N- c^ e^ CO T-H T (M CO ^ T 1 00 ~p T uo C5 1 _!_ CO CO CO I 00 CO !=> CO T T 00 t^ T l»o !(N o 00 1-H o OQ co T-H CO co" 1 (M T-H 1-H 1-H 1 Ttl CO 1— 1 o" cT t^ rt< (M (N (M CO 1-H 1-H CO ^ »o 00 1 cT CO (N CO 01 1 OJ C3_ C a i-H 1 t^ CO lO rf! J^ TJH CO Tt^ (N t^ 1 (N *o 1— 1 (N 00_ t^ CO 1 05_ CO ,o (N 05 o ■^ CO o ^ 'CO 05_ 1^ 05 iO CO ^ (N :0 1 1 CO CO o .-1 1-* 1 _L 05 00 05. rH JTt^ CO CO 00 00 1 r^'iO b- t- 00 CO 1 2,S,S (M (N o 00 1> 1-H l> 05 ^11 f 1 t^ ^ FT 00 I :_ ai l> 00 05_ 1 CO r-i CO (N lO T-H (M CO a> CO iC t^ 1> (M (N CO o 1 05 CO ■^ iO Tt^ 1 «o CO 1 CO ,_l Tt< 00 ,_, — J_ 1-H £1 T-^ t^ lO hJ z m 5 (4 t H z -3 ■ S < < p ►■s i M "<3 H C-. ="• il H <^> ^ r± fe ;::: o -^^ —< O 03 to ^ -^j P ^ H m S 00 £ - 1 o S3 F: ^ -. g 1 ^S ^- ^5l r^ ^ O CD lO 00 TfH fe ^ TjH 1-1 og „^0000_ ^ „ O -H O O l-H Q CM^ ,-H CO o o o o ^ ^ ^ lO Tj< 00 O S (N Tf^ (N 05 rH Z— 1 25 O 05 00 CO »o C^ 05 (M CO lO CO ^ 00 00 -t 00 t- lO CI ^ I> ^ lO t- 00 g^ CO CO 00 00 OS ^S^oT© co__ rH rtl Ttl CO O 00 01 ^rL O 00 'O Tt< CO '^ . 05 00 CO O »0 oT =^ — § ^ (N 05 CO 05 «C> "^ ;2 00 t- ^00 o O TJ^ t^ oT^g 00 1—1 ^J J S2cOC^^^^_ •§ (N CO 00 00 c;3 "^ & nn =L____ d ^ h5 :± O '^ CO CO 5 t^ tH J^j 05 00 Tj< TiH 00 00 00 lO 00 lO OS o t^ CO t^ CO O O '^ Os" oT^ OS CI CO ^^oTcToTo "" !^i'"iii g CO <^ ^ 00 ^ =^2 i c^2 ^^^§~ •B ^^^^'^;^'~ OQ — ^ i-S CHAPTER XV. FACTORS AND MULTIPLES. Thus far, attention has been given to numbers, their expres- sion, the units of which they are composed and the funda- mental operations involving them. Certain properties com- mon to numbers should now be noted. 1. Any number may he considered as the product of two or more factors. The process of determining these factors is termed factoring. ORAL EXERCISE. 1. Define a factor. 2. Name two factors of 35, 64, 120, 42, 18, 168. 3. 2 X 3 X ?= 48. 4. Name three factors of 36, 160, 54, 846. 5. 2 is a factor of which of these numbers: 68, 45, 32, 19, 127, 388? 6. Name some factor that is common to 48 and 36; 42 and 280. 7. Name two factors of 17, 29, 41, 53, 13. ^Vhat is conmion about the factors of these numbers? Classified according to factors, integral numbers are even if two is a factor, and odd if two is not a factor. Integral numbers are prime numbers if the number itself and one are the only factors. A series of numbers are mutually prime if one is the only common factor. All numbers not prime are composite and contain two or more factors. ORAL EXERCISE. 1. Name and classify the numbers from 1 to 150, as even, odd, prime or composite. 2. Name four groups of numbers that are mutually prime. 3. May the factors of a number be composite? Note. There are certain tests of divisibility that are useful in factor- ing and general computation. Thus a number is divisible by: 86 FACTORS AND MULTIPLES. 87 2, if it is even. 3, if the sum of its digits Ls divisible by 3. 4, if the number represented by its two right hand figures is divisible by 4. 5, if the right hand figure is 5 or 0. 6, if even, and the sum of its digits is divisible by 3. 7, (No simple test). 8, if the number expressed by the three figures to the right is divisible by 8. 9, if the sum of the digits is divisible by 9. 10, if the last figure is 0. EXERCISE. 1. Suggest tests of divisibility by 100, 1000, 10000, 20, 90, 18, 12. 2. If a number is divisible by 8, it is necessarily divisible by ?, ?, or ? 3. If a number is divisible by 12, it is necessarily divisible by ?, ?, ?, or ? 4. 9,684,324 is divisible by what factors under 13? Using divisibility tests, determine at least two factors of: 5. 144. 7. 1728. 9. 2240. 11. 5280. 13. 4589. 6. 320. 8. 19,200. 10. 63,360. 12. 12,345. 14. 12,726. EXERCISE. Reduce to prime factors: 1. 7668. Being even, 2 is a factor. The resulting factor, 3834, is even, and is therefore again divisible by 2; the sum of the digits of 1917 (18) is divisible by 3, a prime factor. The remaining factors are found by inspection. Check 2X2X3X3X3X71= 7668. 2. 246. 4. 4225. 6. 1728. 8. 2496. 3. 389. 5. 4128. 7. 63,360. 9. 7884. Find the prime factors common to: 10. 378 and 864. Since 7 and 16 have no common factor, division need not be carried further. It is evident that the factors common to 378 and 864, are 2, 3, 3 and 3. 11. 984 and 676. 12. 1425 and 5000. 13. Solution. 2 7668 3834 1917 639 213 71 Solution, 2 378 864 3 189 432 3 63 144 3 21 48 7 672 and 396. 16 88 BUSINESS ARITHMETIC. The product of the common factors (in Ex. 10, 2 X 3 X 3 X 3) of two or more numbers is termed their highest common factor (h.c.f.). The highest common factor was once of extreme importance in compHcated fractional work, but it is now little used, owing to the substitution of decimals for all but simple fractions. EXERCISE. Find the highest common factor of: 1. 678 and 420. 2. 5280 and 320. 3. 546 and 384. EXERCISE. Factoring is often employed to simplify and shorten the processes of multiplication and division. This is done by rejecting common factors from numbers that stand in the relation of dividend and divisor. 1. Why is the quotient of 5 X 8 divided by 5 X 4 the same as 8 -i- 4? 2. Divide 4X7X8 by 3X7X9. Reject the common factor and divide. What is the effect of dropping the common factor? 3. Divide 12 X 42 X 18 X 6 by 6 X 8 X 9 X 11. Solution. 3 ^ *gX 42 X IS X^ ^ 3 X42 ^ 126 ^X-&XS^X11 11 11 ■2- 11 A Indicate the products and division as shown. Cancel the common factor, 6. Cancel the factor 9 of the divisor with the 9 in 18 of the dividend, leaving 2. Cancel the 2 from dividend and from 8 in the divisor, leaving 4. Cancel 4 from the divisor and from 12 in the dividend, leaving 3. Multiply the remaining factors. 4. 14 X 8 X 64 divided by 11 X 16 X 4 X 7 = ? 6. 3X21X84X9-^69X8X6X14 = ? 6 52 X 16 X 5 X 11 ^ ^ • 13 X 28 X 12 X 15 • 7. The output of a cracker factory is 840 doz. packages per hour, for a nine-hour day. This is equivalent to how many cases of 8 cartons, containing six packages each? FACTORS AND MULTIPLES. 89 8. 420 cases of 4 doz. cans each, are equivalent to how many packages of each containing 6 cans? 9. At the rate of $9.60 per doz., what is the cost of five units? 10. In a given year the coal consumption for heating an office building was 1825 tons. The price paid was $3.90 per ton. What was the average daily cost for fuel? The multiple of a number is any integral number of times the number. A common multiple of two or more numbers is one that is an integral number of times each of the numbers. The least number that is such a multiple, is termed the least common multiple (l.c.m.). EXERCISE. 1. Name multiples of 8, 5, 21, 4. 2. 12 is a common multiple of what smaller numbers? 3. Name a common multiple of 15 and 25. 4. Name two common multiples of 4 and 6. What is the least common multiple? 5. Is the product of two numbers a multiple of them? 6. Is it ever the least common multiple? 7. Find the 1 c. m. of 36, 84 and 96. To be a common mutiple the number must contain the prime-factors of each of the given numbers. Illustration. Resolve 36, 84 and 96 into their prime Solution. factors. The multiple must have the fac- 36 = 2X2X3X3 tors "2" five times because 96 contains 84 = 2X2X3X7 that number; the factor "7" once, because 96 = 2X2X2X2X2X3 84 contains it ; and two " 3's " because both 36 and 96 have them. The least common multiple is therefore 2X2X2X2X2X3X3X7, or 2016. If the factors are not determined by in- spection, arrange the numbers in a hori- 2 zontal line and divide by factors common 2 to any two numbers. 3 Bring down undivided quotients that 3 7 8 are not exactly divisible. Continue until Solution. 36 84 96 18 42 48 9 21 24 90 BUSINESS ARITHMETIC. all quotients are mutually prime. The 1. c. m. is equal to the product of the divisors and the final quotients. 1. c. m. = 2 X 2 X 8 (2 X 2X 2) X 3 X 7 = 2016. Find the 1. c. m. of: 8. 4, 9, 6, 2. 10. 4, 20, 30, 18. 12. 5, 35, 60, 75. 9. 9, 12, 15. 11. 4, 2, 8, 32. 13. 3, 6, 5, 92, 18. 14. A steamship company has two steamers leaving New York, the fii'st maldng a round trip in 10 days and the second in 16 days. How often do they reach New York at the same time? 15. A manufacturing company has on the road, four drummers, each following circuits that take respectively 7, 6, 4 and 12 days. How often can the manager confer with the four men together? 16. Two wheels of 20' and 16 ft. circumference are geared together. How often do the wheels complete their revolutions at the same instant? 17. A novelty manufacturer uses cloth in strips of 3", 4", 9", 6" and 12" in width. What least width of cloth should he buy in order that he may cut it into any of the given widths without waste? FOR DISCUSSION. 1. Find the h. c. f. and 1. c. m. of any two numbers. Compare the product of these values with the product of the numbers. Explain. 2. The dollar of our currency is a multiple of what other denomina- tion of coin? 3. Name some of our systems of measures that are based on a multiple system. 4. What are the advantages of multiple over mutually prime measures? 5. Show that in determining the 1. c. m., all numbers that are factors of other given numbers may be disregarded. CHAPTER XVI. REDUCTION OF FRACTIONS. A fraction is one or more of the equal parts into which a whole may be divided. One of the equal parts is a fractional unit. All fractions may be expressed as common fractions by two numbers or terms separated by a horizontal line. The denominator (lower term) names the parts or size of the frac- tional unit; the numerator (upper term) shows the number of fractional units in the given fraction. In reading, the numera- tor is invariably read first. ^ five (numerator) 5 „ • i ^t Illustration. . , ., ,j . . \ = s = nve-eightns. eighths (denominator) 8 INTRODUCTORY EXERCISE. 1. Name the unit in the expressions: 15 lb., 7 da., 3 wk., 1/2, 5/6, four-ninths, 7/8 yd. 2. Which term of the common fraction is missing in the decimal fraction? How is it expressed? 3. Which is greater, 1/4 or 1/8? How is the value of a fraction afifected by increasing its denominator? 4. Arrange in order of value: 1/5, 1/98, 1/72, 1/5000. 6. Can 1/3 lb. and 1/4 A. be compared as to value? 6. How many thirds is 1? WTiich is greater, 1/3 or 2/3? How is the value of a fraction affected by increasing the numerator? 7. Arrange m order of value: 3/9, 7/9, 11/9, 5/9. It is evident that any increase in the numerator or decrease in the denominator increases the value of the fraction, and that the value is decreased by decreasing the numerator or increasing the denominator. Moreover, if two fractions have like numerators the larger fraction has the smaller denomina- 91 92 BUSINESS ARITHMETIC. tor; if they have like denominators, the larger fraction has the larger numerator. Common fractions are proper fractions if the numerator is less than the denominator; otherwise they are improper fractions. Fractions are compound, if each term is fractional I e. g., 3; 1. A mixed number is an expression containing an integer and a fraction (e. g., 6f ). The tendency in all business and general computation is toward the use of decimals in place of complicated or irregular fractions, so that only the simple fractions are in common use. The business series covers halves, thirds, fourths, fifths, sixths, eighths, twelfths, sixteenths and twentieths. To these our main attention will be given. EXERCISE. 1. This is one inch of a ruler divided into eight parts. Name the unit and the smallest fractional unit. 2. In the whole there are how many halves, fourths, eighths? 3. Multiply numerator and denominator of 1/2 by 4. How is the value affected? 4. Divide numerator and denominator of 6/8 by 2. How is the value affected? 5. Why is value unaffected? {Suggestion. Note independent effect of first dividing numerator and then denominator.) It is evident that a change of expression by multiplying or dividing both terms of a fraction by the same number, causes no change in value. A fraction is reduced to lower terms when smaller numbers are found to express the same value; it is reduced to higher terms when larger numbers are used. ORAL EXERCISE. 1. Reduce to thirds: 8, 17, 21, 14, 8, 64. "!,f ; ■/. '/, % ■'A \ '/, 1 1 REDUCTION OF FRACTIONS. 93 2. Reduce to 16ths: 3, 4, 12, 9, 21, 5. 3. Reduce to 12ths: 7, 13, 5, 3, 12, 15. 4. Reduce to 20ths: 3/4, 5/4, 7/5, 3/10, 3/2. Illustration. 3/4 = ?/20. 20 -^ 4 = 5; 1/4 = 5/20; 3/4 = ^-^ = 15/20. 5. Reduce to 16ths: 3/4, 5/8, 5/2, 7/8, 9/4. 6. 7/6 = ?/36; 5/8 = ?/40; 7/3 = ?/18; 3/7 = ?/49. 7. Is there any limit to the increase of size of denominator in reduction? 8. Reduce to improper fractions: 3i, 5^, 16f, 27f. Illustration. In 3 there are 3 X 4 fourths. In 31 there are 12/4 + 1/4 or 13/4. 9. Reduce to lOths: If, 2f, 12 i (Suggestion. Reduce first to improper fractions.) 10. 3i = ?/12; If = ?/24; 7i = ?/20; 9| = ?/6. EXERCISE. 1. Reduce 76 to a fraction with a denominator of 842. 2. Reduce 117 to 43ds. 3. 3/7 = ?/427; 118/142 = ?/710. 4. Reduce: 16 j^ to 24ths; 192^2^ to 45ths. It is often advisable, in computations, to simplify a fraction by reducing it to lower terms. In final results, proper fractions are usually expressed in lowest terms, and improper fractions are expressed as whole or mixed numbers. It has been noticed that the value of a fraction is not affected by dividing numera- tor and denominator by the same number. Hence reduction is possible by continued reduction by common factors. When the reduced terms are mutually prime, the fraction is reduced to its lowest terms. Illustration. Reduce 585/675 to its lowest terms. Analysis and Solution. By inspection, 5 is a common 585 4- 5 _ 117 factor of both terms. 675 4- 5 ~ 135 By inspection, 9 is seen to be a second common factor. This really amounts to dividing the terms by their 117 -^ 9 _ 13 greatest common di\isor. 135 -i- 9 ~ 15 94 BUSINESS ARITHMETIC. ORAL EXERCISE. 1. Reduce to 8ths: 3/4, 2, 10/16, 15/24. 2. Reduce to 12ths: 3/4, 5/6, 18/24, 12/36, 40/60. 3. Reduce to lowest terms: 15/18 cu. yd., 45/69, 720/2400 T. 4. R-duce to lowest terms: 160/4, 325/5, 56/8, 120/3. 5. Reduce to lowest terms, or to mixed numbers: 9/2, 15/4, 73/3 45/12 lb . 52/8 bu., 162/16. EXERCISE. 1. Reduce to lowest terms: 56/124, 128/264, 985/1200, 172/465. 2. Reduce to whole or mixed numbers: 725/80, 392/56, 1982/1250. 3. Simplify: 285/16 T., 3924/5280 mi., 453/144 sq. ft. CHAPTER XVII. FRACTIONS— FUNDAMENTAL PROCESSES. Addition of Fractions. Since only like numbers can be added, it follows that only fractions or parts of like units can be added. Moreover, since the denominator of a fraction is but a name showing the relative size of the fractional unit, only similar fractions, those having the same denominator, can be added. If fractions differ only in denominator, addition is possible after reduction of the fractions to a common denominator. In the cases commonly arising in everyday usage, the least common denominator is usually determined by inspection. Illustration. Add 3/5 and 5/6. Solution. By inspection, the common denominator is 30. 3/5 = 18/30. 5/6 = 25/30. 3/5 + 5/6 = 18/30 + 25/30 = 43/30, or 1^. ORAL EXERCISE. Which of these groups can be added 1 without intermediate steps? Which cannot be added at all? 1. 4/5 + 8/5 + 5/5. 3. 5/4 ft. + 2 lb. 2. 1/4 + 1/2 bu. 4. 1/4 + 1/5 + 1/6. 5. 1/4 yd. + 31 yd. Find the sum of: 6. 1/4, 3/4, 2/4, 9 fourths. 14. 3/8, 4/5, 7/10. 7. 1/3 yd., 2/3 yd., 5/3 yd., 3/3 yd. 15. 7/12, 2/3, 1/4. 8. 3/16, 4/16, 7/16, 11 sixteenths. 16. 1/2, 2/5, 7/10. 9. 1/5, 3/5, 7/5, 4/5, 11/5, 9/5. 17. 3/20, 4/5, 1/2. 10. 2/3 +4/5+ 9/15 (= ?/45). 18. 1/3, 3/2, 5/6, 1/12. 11. 1/3, 1/4. 19. 1/3, 2/9, 5/6. 12. 3/4 ft., 5/8 ft 20. 3/4 ft,, 2/3 ft., 5/2 ft. 13. 1/2, 3/4, 5/8, 1/16. 95 2. 3i 51, 6f. 4. 5/12 , 1^1, 1.5. 6. 4H, 85/r. 32961 9. 1292U 562i 1828^ 14729t5i 13461 862^ 4621 928xV 18f 6621 96 BUSINESS ARITHMETIC. EXERCISE. Note. If the numbers to be added are mixed numbers, the integers and fractions rnay be added separately and the two results combined. This, however, is not necessary with very simple numbers. 1. U, U, 5i 3. 32i, 7f, 8f. 5. 3i, 41, 12tV. 7. 2141 319xV 4291 296i Note. In totalling lengths of cloth, the exponents, 1, 2, 3, are used to represent one-fourth, two-fourths and three-fourths respectively. Thus 7621 yd. are 7621 yd. 10. 27 pieces of percale are sold a customer on an order. Find the total quantity if the respective piece lengths ai"e; 44, 40, 43, 42^, 41^, 40*, 42S 423, 432^ 431^ 433^ 442^ 42, 43S 422, 431^ 422^ 421, 412^ 403, 44S 43S 42', 43^ 42S 43S 403. 11. Find the cost of thirty-one pieces of Scotch cheviot, @ 37c per yd., the pieces measuring respectively: 38S 37S 37^, 39, 39^, 40S 42^, 40S 38^, 391, 40, 382, 391^ 40, 41, 392, 383, 392^ 40, 401, 402, 39^, 38^, 39S 39^, 40S 39', 392, 391, 40S 412. EXERCISE. 1. Find the sum of any two fractions that have the same numerator. Study the solution and suggest a short method for adding any two such fractions. Add: 2. 1/8, 1/3. 4. 1/11, 1/16. 6. 5/8, 5/11. 3. 1/9, 1/4. 6. 4/9, 4/21. 7. 2/3, 2/9. Subtraction of Fractions. Since subtraction and addition are reverse processes, the principles of units and common denominators apply as in addition. Illustration. Find the difference between 3/7 and 1/6. Solution. 3/7 = 18/42. 1/6 ^ 7/42. 3/7 - 1/6 = 18/42 - 7/42 = 11/42. SUBTRACTION OF FRACTIONS. 97 EXERCISE. (Solve mentally if possible.) Find the difference between: 1. 1/4 and 1/5. 5. 7/3 and 4/5. 9. 7/8 yd. and 2/3 yd. 2. 6/7 and 1/4. 6. 4/3 ft. and 3/4 ft. 10. 7/40 and 3/20. 3. 3/3 and 2/9. 7. 7/4 and 4/5. 11. 1/3 and 1/4. 4. (1/4 + 1/8) and 3/4. 8. 7/8 and 7/20. 12. 6/9 and 6/11. Note. If mixed numbers are given, whole numbers and fractions may be subtracted separately, except that in case the fraction of the subtrahend is greater than the fraction of the minuend, it may be necessary to reduce an integral unit of the subtrahend to fractional units. Find the value of: 13. 5^ - 3i 18. 5| - 2.1. 14. 6| - 1t\. 19. 388f - 172r\. 15. 2f yd. - 1| yd. 20. 32,^^ + 87| - 23^. 16. If - 5/6 21. 582i - 96^. 17. 17/^ - .8. Multiplication of Fractions, introductory. '6/8' may be reaxi in the form '5 eighths.' 16 times 5 eighths are ? eighths. One-eighth of 16 is ? ; five-eighths of 16 are ? . It is evident that the product of a fraction by a whole number is equal to the product of the numerator and the whole number, divided by the denominator. Thus, either fraction or whole number may be taken as multiplier. Illustration. Multiply 5/6 by 124. Solutim. 124 X 5/6 = i?^^ = ^ = 103i EXERCISE. (Solve mentally if possible.) 1. Multiply 320 by each of these fractions: 1/4, 1/2, 3/4, 5/8, 4/5, 3/2, 3/16. 2. Find the following fractional parts of 48: 1/3, 3/4, 5/6, 7/8, 2/3, 7/6. 8 3. Find 2/3 of 7, 3, 1 I 18, 42, 6, 27, 49, 56, 129. Multiply: 4. 425 by 2/5. 8. 2/7 by 147. 12. 5. 25 by 3/4. 9. 2/9 by 117. 13. 6. 37 by 5/8. 10. 5/8 bu. by 726. 14. 7. 3/8 by 5280. 11. 5/6 by 8 X 42. 15. 98 BUSINES ARITHMETIC. 320 rd. by 9/16. 142 by 2/3. 7/12 by 30 X 9. 364 T. by 4/3. If one factor is a mixed number, it may be reduced to an improper fraction, although direct multipHcation is common. Illustration. Multiply 36 by 2^. Solution (1). 36 X 2i = 36 X 11/5 = ?i^JJ: = 395/5 = 791. Solution (2). 36 7i (1/5 of 36) Z2_ 79i EXERCISE. (Solve mentally if possible.) Find the product of: 1. 16 X U. 5. 165 X If 2. 424 X 7f . 6. 3i X 16 T. 3. 21 X ^. 7. 2f X $120. 4. 4i X 160 ft. 8. 168 X 68t^^. Find the cost of: 9. 7| yd. dress goods @ $3; 8^ yd. lining @ 17c; 3/4 yd. velvet @ $3.50. 10. 420 bu. corn @ 42Jc; 652 bu. wheat @ 81|c; 1900 bu. oats @ 32ic. Fractions Multiplied by Fractions. INTRODUCTORY. 1/2 in. = ? fourths of an inch =^ ? eighths. 1/4 of 1/2 in. = 1/4 of ? eighths, or ? eighth. The product of two fractions is evidently a fraction whose numerator is the product of the given numerators and whose denominator is the product of the given denominators. The computation may be simpHfied, in some cases, by cancellation. MULTIPLICATION OF FRACTIONS. 99 Illustrations. Multiply 3/85 by 5/6. 3X5 15 Solution 1. 3/85 X 5/6 =gg-g=^^^ = 1/34. Solution 2. By cancelling common factors. 3/85 X 5/6 = ^^-^ = n X 2 = i 17 2 ORAL FACILITY EXERCISE. Find the product of . 1. 1/4 X 1/3. 4. 4/5 X 7/12. 7. 3/8 X 8/3. 2. 1/2 X 3/4. 5. 5/6 X 6/7. 8. 3/8 X 5/6 X 4/5. 3. 3/8 X 5/2. 6. 3/2 X 8/9. EXERCISE. Simplify 1. 3/4 of 9/24 of 7/15 of 852 bu. 194. 2. 3/84 X 5/72 X ^ X $8290. 15 3. i^ X 84/36 X 21/22. Products of Mixed Numbers. With large integral numbers and small fractions, multiplica- tion by parts is common. In simple cases, the mixed numbers may be reduced to improper fractions and the products obtained as in the previous case. Illustrations. Solviion, (1) Multiply 429f by 127|. 127J i (1/3X3/5) 143 (1/3 X 429) 76i (127 X 3/5) 3003 ' 858 \ (127 X 429) 429 J 54702f Ans. 100 BUSINESS ARITHMETIC. (2) Multiply 4i by Gf. Solution. 4i : = 13/3. 6f = 51/8. 17 13/3 X 51/8 13 X-BT 221 -a-xs ~ 8 ~^^^' 1 EXERCISE. (Solve mentally if possible.) Multiply: 1. U by 2f. 7. 4f by 15i 2. 2f by 3/8. 8. 7f by 14^. 3. If by 3f 9. 2341 by 65i 4. If by li 10. 3451 by 168i 5. 4| by If. 11. 1420f by 17281. 6. li by 2f. 12. 182f by 14^. Short Methods. Few fractional short methods, aside from cancellation, have a broad use. The most valuable method is that applying to mixed number factors having the fraction "J." Case I. Integers alike. Example. Multiply 4| by 4^. Analysis and Solution. By parts, 4^ X 4| = 4 X 4 + 4 X 1/2 + 1/2 X 4 + 1/2 X 1/2. = 4 fours + 2 halves of 4 + 1/4. = 5 fours +1/4 Condensed solution. 4^X4^=5X4 + 1/4=201. In general, multiply the integer by the next higher integer and add 1/4. Case II. Integers unlike. Example. Multiply 85 by 11^. Analysis and Solution. By parts, 8i by lU = 11 X 8 + 11 X 1/2 + 1/2X8 + 1/2 X 1/2- = 88 + 1/2 (11 + 8) +1/4. = 88 + 9^ + 1/4 = 97f . In general, add to the product of the integers one-half their sum, plus II4. EXERCISE. Multiply: 1. \2h by 12i. 5. 6^ by 6i. 9. m by 30i. 2. 8§ by 8^. 6. 61 by 8i. 10. 15^ by 5.5. 3. 15i by 15i 7. 10.5 by 12i. 11. 461 by 46i. 4. 9.5 by 9.5. 8. Uh by 16i. 12. 124^ by 6U. DIVISION OF FRACTIONS. 101 DIVISION OF FRACTIONS. • 1. Fractions by Integers, introductory. 12 ft. 4- 3 = how many feet? 12 thirteenths -r- 3 = how many thirteenths? = ?/13. - 15/16 -^ 5 = 1/5 of 15/16 = 1/5 X 15/16 = ? It is possible, evidently, to divide the fraction by dividing the numerator, or multiplying the denominator by the integral divisor. The reciprocal of a number is 1 divided by that number. Multiplying the denominator by the integral divisor is equiva- lent to multiplying the fraction by the reciprocal of the divisor. Illustration. Divide 18/23 by 6. Solutions. 3 48- 3 23 X-^ 23 1 (i)i-« 18 -J- 6 3 23 23 • (2)1-6 = (3)l|^6 = 3 .^y 1_3 "23^^ 23* Divide: 1. 28/37 by 4. 2. 5/8 by 6. 3. 2/3 by 12. 4. 24/15 by 6. EXERCISE. (Solve mentally if possible.) 5. 3/8 by 5. 6. 21/25 by 7. 7. 2/3 by 7. 8. 5/9 by 8. 9. 5/8 by 15. 10. 142/56 by 14. 11. 36/89 by 18. 12. 22/45 by 33. 2. Division of Mixed Numbers by Integers. Since the mixed number may be reduced to an improper fraction, the method does not differ from that just shown. It is common, however, to divide directly. Illustration. Divide 54 1 by 6. 102 BUSINESS ARITHMETIC. 'Sobkima. :(1] '-t = f. 491 491 1 491 9 • *" 9 ^6" 54 "^^^• (2) 6)541 V 54 f (5/9 - r 6 = 5/9 X 1/6 = 5/54) EXERCISE. (Solve mentally if possible.) Divide: 1. If by 3. 6. 16i by 5. 9. 382f by 26. 2. 12f by 4. 6. 4f by 3. 10. 193A by 15. 3. 27f by 9. 7. 12t\ by 4. 11. 6721 by 12. 4. 8f by 3. 8. 2321 1 by 8. 12. 458^^ by 15. Fractional Divisors. INTRODUCTORY. In 4 there are how many thirds? How many groups of 2 thirds? 4^2=iX?=? * 3 2 Evidently the quotient of an integer by a unit fraction is the product of integer and denominator. If the fractional divisor is not a unit fraction, its terms may be inverted (the reciprocal taken) and the quotient is then the product of the dividend (fractional or integral) and the reciprocal fraction. Illustrations. (1) Divide 425 by 5/6. Solution. 425 -^ 5/6 is equivalent to 425 X 6/5. 85 6 JiQS'X^= 510. 1 (2) Divide ^gby|. 9 3 9 4 Solution, rs -^ r is equivalent to ,-5 X o • lb 4 lb o 3 ^^-3- 4- DIVISION OF FRACTIONS. 103 EXERCISE. (Solve mentally if possible.) Divide: 1. 1/2 by 2/3. 4. 1/6 by 1/7. 7. 5/8 by 7/12. 2. 2/3 by 1/5. 5. 2/5 by 5/6. 8. 3/8 by 9/16. 3. 3/4 by 4/5. 6. 3/7 by 4/11. Note. Simplify the process in the following by reducing to a common denominator. 9. 3/4 by 7/8. 12. 426 by 3/4. 15. 3/15 by 5/36. 10. 2/3 by 5/6. 13. 129 by 2/3. 16. 5/8 by 17/140. 11. 5/12 by 1/3. 14. 3545 by 5/18. 17. 256 by 19/24. Mixed Number Divisors. Since mixed numbers may be reduced to improper fractions, this case reduces to that last given. If the fractions are simple it is often advisable to reduce to a common divisor. EXERCISE. (Solve mentally if possible.) Find the quotient of: 1. U -^ U. 3. U -^ U. 5. 5/2 -^ 1|. 2. 2 ^ If 4. 2| ^ 31. 6. 51 4- 4|. Divide : 7. 5461 by li 11. 416f by 12^ 8. 672H by 14^ 12. 6546^ by 2f. 9. 1291 by 66|. 13. 599^ by 7f. 10. 5821 by 13f . 14. 59^^ by If. Conversion of Fractions. Owing to the growing use of the decimal fraction, it is necessary to have facility in expressing fractions in either common or decimal form. It is evident that the denominator of a decimal fraction, though not expressed, is always deter- mined by the decimal point. 104 BUSINESS ARITHMETIC. Illustration. Express .0825 as a common fraction. Solution. .0825 = 825 ten-thousandths = -|^ = ^. lUOUU 400 EXERCISE. Express as common fractions reduced to lowest terms: 1. .125. 5. 18. 9. .096. 13. .19275. 2. .25. 6. 1.54. 10. .392. 14. .088. 3. .331. 7. 7.45. 11. .875. 15. 62.045. 4. .0084. 8. 375. 12. .0625. 16. 1.0458. The reverse process of converting common fractions to decimal fractions is far more common than that just given. It is performed by fractional reduction, or, more commonly, by executing the represented division. Illustration. Reduce 28/85 to the nearest decimal of three places. Sohdion. Carry out the represented division. .329 Ans. 85)28.00000 25 5 '2 50 170 800 765 35 EXERCISE. (Solve mentally if possible.) 1. Convert into decimal form: 1/2, 1/3, 1/4, 1/5, 1/6, 1/7, (.14|), 1/8, 1/9, 1/10, 1/11, 1/12, 1/15, 1/16. 2. Reduce to decimals: 3/4, 2/3, 5/8, 3/8, 1/30, 5/6, 2/9, 5/16. Note. Common measures are frequently expressed as decimals of other measures. 3. Express 1 ft. aa a decimal of 1 yd.; 1 qt. as a decimal of 1 gal. EXERCISE. Reduce to equivalent decimals: 1. To two decimal places with fractional remainder: 3/12, 5/11, 5/24, 11/25, 13, 16, Ui 17/48, 5^, 8,^^. DECIMALS AND FRACTIONS. 105 2. To the nearest four-place decimal: 47/52, 5/14, 6/28, 9/32, 21/56, 9/125, 12/250, 13/80, 11/60. 3. Express 100' as a decimal of a mile (5280'). 4. Mechanics often use conversion tables, giving fractions of inches as decimals of a foot. Complete the following table for each 1/8" up to one foot, computing to four decimal places. Inch. Fraction of Foot. Decimal. 1/8 1/96 .0104 1/4 1/48 .0208 QUESTIONS FOR DISCUSSION. 1. Why are not 7ths and 9ths common business fractions? 2. What has led to the general use of the common business fractions? 3. Are our common measures adapted to the business fractions? 4. What fractions are used in measuring cloth? Give similar illus- trations of the use of particular fractions in specific trades or professions. 5. Why are decimals used in place of fractions, in statistical tables? 6. Name some fractions that are simpler, for business use, than their equivalent decimals? 7. What is the shortest method of comparing the relative value of several irregular fractions? 8. Why is the value of a fraction altered by adding the same amount to both numerator and denominator? 9. What is true of the product of two reciprocal fractions? Why? CHAPTER XXII. FRACTIONAL RELATIONS OF NUMBERS. It is often necessary to determine the fractional relations of numbers. There are two common cases: I. To Find What Part One Number Is of Another. INTRODUCTORY. 8 is what part of 12? Reduce to lowest terms. Which number repre- sents the part? Which number represents the whole to which the part is referred? 2 sevenths is what part of 4-sevenths? 3/4 in. = ?/8 in. 3/4 m. = what part of 7/8 m.? INCH Vs V4 % h Vs % v& I I I I I -v^ 7/8=- SMALLEST UNITS 3/4= -SMALLEST UNITS H [^ cvidcnt thatthc part that one number is of another is expressed as a fraction having for a numerator the number expressing the part, and for the denominator that whole to which it is referred. This is equally true of integral, fractional, decimal, abstract or concrete numbers. Illustration. Example. I buy 3/8 interest in a business from one who owns 4/5 interest in it. What part of his holdings do I buy? SoMion. ^^^^^ II = relation. 1^ = 3/8 -^ 4/5 = 3/8 X 5/4 = 15/32 the part bought. 4/5 EXERCISE. (Solve mentally if possible.) What part of: 1. 14 is 22? 3. 1§ yd. is 7^ yd.? 5. 14 lb. is 6^ lb.? 2. 11 is 15? 4. 1/2 is 1/3? 6. 7 bu. is 3i bu.? 106 FRACTIONAL RELATIONS OF NUMBERS. 107 7. $2.50 is 70 c? 9. 4/11 is 13/16? 11. 5.01 is 1.29? 8. 3/4 is 5/8? 10. $326.50 is $420? 12. 1728 is 1264? 13. The stands at a ball park have a seating capacity of 13,500, of which 3200, or ?/?, are grand stand seats; 1/4 are bleachers and the balance, or ? seats, are in the paviUon. 14. In a certain coal mine an investigating commission discovered that for every ton of coal secured for market, 6.45 tons are wasted. What part is wasted? 15. Compare the speed of two trains, one of which covers four milea in 3 5 minutes, and the other 7 miles in 6 minutes. 16. 726 tickets were taken up at the entrance to a hall seating 1200. The hall was approximately what part filled? Note. WTien an approximate relation is called for, find the nearest simple fractional relation. II. To Find the Whole When a Part Is Known INTRODUCTORY. If 16 ft. equal 1/5 of a quantity, find 5/5 of it. If 16 ft. equals 4/5 of a distance, find 1/5 of the distance. Fmd the entire distance. Thus the whole may be determined by finding the value of a fractional unit of the given part, and multiplying this value by the number of fractional units in the whole. Illustration. 3/8 is 2/3 of what number? Analysis and Solution. 3/8 = 2 thirds of a number. 1/2 of 3/8, or 3/16 = 1 third of the number. 3 X 3/16, or 9/16 = 3 thirds of the number, or the number itself. In brief: If 3/8 = 2/3 of a number, the number = 3/2 of 3/8, or 9/16. Find missing values: 1. 16 = 1/2 of — . 2. 35 = 5/6 of — . 3. 42 bu. = 3/7 of — . 4. $5.60 = 4/5 of — . 5. 2/3 = 1/2 of — . 6. 1/2 - 2/3 of — . 7. 750 lb. = 2/3 of — . EXERCISE. 8. 4280 = = 1/4 of — . 9. 192 lb . = 4/3 of — . 10. 127 = 3/4 of — . 11. 2420 lb. = 3/15 of - 12. $752 = = 3/8 of — . 13. 592 = 4/13 of — . 108 BUSINESS ARITHMETIC. 14. A railway speed of 14 miles in IS minutes is equivalent to a speed of how many miles per hour? 15. A steam shovel excavated 47^ cu. yd. of earth in 4| minutes. This was at the rate of how many cubic yards per hoiu"? 16. A yield of 26 bushels on a trial plot of 2/5 of an acre is equivalent to a yield of how many bushels per acre? 17. A certain cloth shrinks 1/32 in dyeing. To have 930 yd. of finished product, one must dye how many yards? 18. A man who offered me $750 for a 3/8 interest in a certain patent, evidently valued the patent at % — . 19. A contractor has completed 2/5 of a grading contract in 48 working days. At the same rate, how many days work remain? 20. A manufacturer replaces his old machines with new ones of one- fifth greater productive power. He runs on a ten hour day basis. His new machines should give him his former weekly output with a saving of how many hours. ORAL FACILITY EXERCISE. Find the single term equivalent to each group of two or more tei*ms. Distinguish amounts less than zero by the term minus, thus 3 — 4 = — 1. 1. Halves and thirds. Two is what part of three? Three of two? Fmd 1/2 + 1/3, 1/3 + 1/2, 1/3 of 1/2, 1/3 - 1/2, 1/2 - 1/3, 1/2 - 2/3, 1/3 -^ 1/2, 1/2 -^ 1/3, 1/3 ^ 2/3, 1/2 •- 2/3, 1/3 X 1/2, 1/2 X 2/3, 2/3 of 1/2, 2/3 divided by 1/3, 1/2 of .4, 2/3 of 15, 6 = 1/2 of ? , 12 = 1/3 of ? , 1/2 = 1/3 of ? , 1/3 = 1/2 of ? , 1/3 is what part of 1/2, 1/2 is what part of 2/3? 2. Thirds, sixths and twelfths. Find 1/3 + 1/6, 2/3 - 5/6, 1/6 - 2/3, 1/6 ^ 1/3, 5/6 -^ 5/3, 2/3 -i- 1/6, 2/3 ^ 4/6, 1/3 of 1/6, 2/3 X 5/6, 1/3 + 1/16 + 1/12, 1/6 -^ 1/12, 1/3 ^ 1/12, 7/12 ^ 1/3, 5/12 ^ 5/6, 5/6 - 5/12, 1/3 of 2.4, 5/6 of 72, 7/12 of 18, 4 == 1/6 of ?, 1.5 = 5/12 of ?, 6 = 1/12 of 1/2 of ?, .1/6 is what part of 1/12? Of 1/3? Of 5/6? 5/6 is what part of 2/3? Of 7/12? 1/6 is 1/12 of ? 5/12 is 1/3 of ? 3. Halves, fourths, eighths and sixteenths. Find 1/2 + 3/4, 3/4 + 5/8, 7/8 - U, 9/16 - 5/8, 1/4 -^ 3/16; 3/4 of 3/16, 5/8 X 1/2, 5/8 of 3/4, 1/16 - 3/8, 5/16 ^ 3/8, 3/8 of 5/16, 1/4 + 1/8 -4- 1/16, 1/2 times 7/16, 1/2 -^ 9/16, 1/2 X 15/16. 1/2 is what part of 7/8? Of 15/16? Of 5/4? 3/4 is what part of 3/8? Of 15/16? Of If? What is 1/2 of 3.6? Find 3/4 of 64. What is 5/8 of 12? Find 3/16 of 80. 11/16 of 3.2 = ? 14 = 1/2 of ? 10 = 5/16 of ? 12 - 3/8 of ? 12 = 9/16 of ? 1/2 = 3/4 of ? 1/8 = 1/2 of ? 5/8 = 3/4 of ? 9/16 =- 1/8 of ? 9/16 = 3/4 of ? FRACTIONAL RELATIONS OF NUMBERS. 109 4. Use the answer, at each step, as the basic value for the next com- putation. 4 + 12 = ? Add 8. Divide by 2. Multiply by 1/2. Divide by 1/3. Multiply by 2/3. Divide by 4. Increase by 2/3 of itself. Multiply by 4/5. Divide by 2/3. Multiply by 3/4. Change to eighths. What ia the final result? Teacher's Note. Similar original exercises should be prepared by the teacher and given to the class, in order to cultivate a high degree of accuracy in handling the simple business fractions. CHAPTER XXIII. ALIQUOT PARTS. Numbers that exactly divide a given number are termed its aliquot parts. Thus 1, 1^, 1/5, 2, 3, etc., are aliquot parts of 6. ORAL EXERCISE. 1. Name 6 aliquot parts of 12. What fractional part is each aliquot part? 2. Find 1/4 of 200; of 10. Find 1/8 of 10, of 100, of 1000, etc. 3. What parts of $1.00 are 50c, 10c, 33 ic, 20c, 2c? 4. . Which of these numbers are ahquot parts of 10: 1, I5, 2, 2^, 3, 3i, 31, 4? 5. Complete the following table: Aliquot Part. Number. 1/2- 1/3 1/4 1/5 1/6 1/8 1/9 1/10 1/12 1/15 1/16 1/20 1. 10. 100. 1000. 50 33i 25 20 16! 12^ IH 10 8i 6f 6i 5 1. Multiplication. Aliquot parts are frequently used to simplify multiplication and division. The aliquot parts of 10c and $1.00 are espe- cially useful in business. INTRODUCTORY. 1. Express as aliquot parts of $L00, 25c, 12ic, 2^0, 4c. 2. How does the cost of several articles @ 25c compare with their cost @ $1.00? 3. The cost of 48 yd. @ 25c = the cost of 48 yd. at what price? 4. 622 X 33ic = 522 X $1/3 = 1/3 of $ ? = $ ? 110 ALIQUOT PARTS. Ill 5. The cost of 522 articles @ $1.00 is $ ? The cost at 33 fc is what part of the cost at Sl.OCV It is evident that the cost of an article at any aliquot price of one dollar is its simple fractional part of the cost at one dollar. Illustration. Find the cost of 568 yd. cloth @ 12^c per yd. Analysis and Solution. 12^c = $1/8. Cost of 568 yd. @ 12^c = Cost of 568 yd. @ $1/8 = 1/8 of $568 = $71. Note. Notice that in some of the following examples the aliquot lactor is the figure representing the quantity rather than the price. EXERCISE. Find cost of: 1. 840 yds. wash silk @ 25c; at 33ic. 2. 1620 yds. plaids @ $1.25; at 33i; at 50c. 3. 580 yds. cotton at 6fc, at 6^c. 4. 864 yds. prints at 12§c; 16ic, 20c. 5. 164 yds. Iming at 12|c, 10c, 8ic 6. 422 yds percale at 25c, 20c, 16|c, 12^0. 7. 360 doz eggs at 12^c, at 25c. 8. 560 lb. lard at 10c; 8ic, 12^c. 9. 48 lb. chocolate @, 25c, 16fc, 33ic, 10. 450 lb. tapioca @ Q\c, S^c. 11. 62 hats @ $2.50 (aliquot part of $10). 12. 156 T. old iron @ $16f ton. 13. 1520 A. @ $12.50 per A. 14. 1500 blank books @ 25c. 15. 196 doz. buttons @ 16fc. 16. 124 pr. shoes @ $1.25 ($l+25c). 17. 2500 pineapples @ 12ic. 18. 1425 posts @ 33ic. 19. 625 yds. wire netting @ 25c yd. 20. 364 lbs. coffee @ 33^0. 21. 48 lbs. tea @ 50c; @ 33^0. Note. The principle of interchanging multiplier and multiplicand is frequently employed. Thus the cost of 25 yd. @ 88c is equivalent to the cost of 88 yd. @ 25c. 112 BUSINESS ARITHMETIG. Compute the cost of: 22. 33i yd. @ 78c. 24. 25 bu. beans @ $2.60 (260c) 23. 16f yd. lace @ 96c. 25. 50 bu. clover seed @ $4.20. ILLUSTRATIVE EXERCISE. 100 X 720 = ? 100 25 X 720 = what part of 100 X 720? 25 X 720 = -y- X 28 = ? Suggest the short method. It is evident that the aliquot method greatly simplifies computation, often substituting oral for written work. Illustration. Multiply 672 X 33i Solution-aliquot. Solution-regular. 672 X 33i = 1/3 of 672 X 100.. 672 100 X 672 = 67200. 33i 67200 -^ 3 = 22400 224 2016 2016 22400 ORAL EXERCISE. Suggest short methods for multipHcation by 12|, 6i, 6f , 25, 50, To multiply by 12^, multiply by 100 and divide by 8. 1. Suggest she ►rt me 161. Illustration. Tom WTiy? Find the products of: 2. 25 X 36. 3. 33i X 42. 4. 12i X 184. 5. 61 X 960. 6. 16! X 640. 7. 20 X 85. 8. 33i X 456. 9. 6i X 64. 10. 6! X 90. 11. 50 X 588. 12. 25 X 692. 13. 25 X 12.4. 14. 8i X 72. 15. 3i X 420. 16. 12^ X 8.48. 17. 2i X 880. WRITTEN EXERCISE. Find the products of: 1. 426 X 25 4. 6! X 968. 2. 33i X 2672. 5. 6\ X 884. 3. 4892 X m. 6. 25 X 672. ALIQUOT PARTS 113 7. 2.5 X 876. 12. 6i X $6.40. 8. 250 X 968. 13. 25 X 8.004. 9. 25 X 48/49. 14. 2^ X 8.4. 10. 33i X 66f. 15. 3i X .0696. 11. 6f X $1.50. J6. 333i X 6.0096. MULTIPLES OF ALIQUOT PART. The fraction representing an aliquot part has a numerator of ? Find 1/2 of 100; 1/8 of 100; 3/8 of 100; 1/3 of 100; 2/3 of 100; 1/4 of 100; 3/4 of 100. 12i X 248 = . 37| = X 12^. S7i = what part of 100. 37^ X 248 = what part of 100 X 248? Suggest short methods for multiplying by multiples of aliquot parts. ORAL EXERCISE. 1. Multiply 963 by 66i Solution. 66f = 2/3 of 100. 66| X 963 = 2/3 of 963 X 100 = 32100. Note that the fractional part of 2/3 may be taken before or after mul- tiplication by 100. Also one may multiply by 100 and subtract 1/3 of product. Multiply: 2. 124 X 125. 10. 7i X 640. 3. 68 X 75. 11. 66f X 12. 4. 42 X 66|. 12. 125 X 86. 5. 16 X 37i. 13. 168 X 37i 6. 24 X 66f . 14. 96 X 112|. 7. 24 X 62i 15. 1220 X 150. 8. 48 X 87i. 16, 84 X 75. 9. 54 X 83i 17. 328 X 37i. WRITTEN EXERCISE. 1. 75 X 874. 5. 871 X 920. 2. 661 X 960. 6. 83i X 97.3. 3. 37^ X 8480. 7. 71 X 4520. 4. 62^ X 7244. 9 S. 3i X 872. 114 BUSINESS ARITHMETIC. 9. 125 X 1846. 13. 333iX 942 qt. 10. 133i X 276. 14. 137^ X 840 bu. 11. 1121 X 488. 15. 183i X 6960. 12. 233i X 606. 16. 166f X 825. ILLUSTRATIVE EXERCISE. 1. Find 1/4 of a dollar, 3/4 of it, 2/3, 3/8, 5/8, 4/3, 7/8, 3/40. 2. What part of a dollar are: 25c, 75c, 66|c, 7^c, 15c, 70c, $1.25, $2.50? 3. The cost of 240 yd. @ 75c is what part less than the cost at $1.00? How many times the cost at 25c? 4. Suggest two short methods for finding the cost at 75c. It is evident that the cost at some multiple of an aliquot price is obtained by taking the aliquot fractional part of the total cost at $1.00. Illustration. Find the cost of 726 yd. @ 66fc. Solution. 726 yd. @ $1.00 cost $726. 66fc = 2/3 of $1.00. 2/3 of $726 = $484, cost. ORAL EXERCISE. 1. Suggest short methods for finding the cost @ 80c, 37ic, Sl.lOfc. 83ic, $1.25. Find the cost of: 2. 290 lbs. @ 40c, 10c, $1.50. 3. 42 yds. @ 66|c, 50c, 25c. 4. 85 yds. @ 80c, 10c, 50c. 6. 96 lb. @ 62ic, 75c, 83ic. 6. 24 T. @ $7.50, $2.50, $8.00. 7. 136 qt. @ 25c, 37ic, 75c. 8. 48 oz. @ Sic, 6fc, 66fc, $1.33i 9. 120 sq. yds. @ 7ic, 40c, 75c, 37^c, 66fc. 10. 90 cu. yds. @ $1.33^ $2.66f, $15,661- 11. 16 T. @ $2.50, $25, $3.75, $6.25. 12. 45 ft. @ 20c, 40c, $1.20, $1.60. 13. 75 yd. @ $1.24, $3.36, $2.40, $.88. 14. 12i T. @ $4.80, $5.40, $6.00, $12.00. 15. 37i yds. @ 64c, 72c, 90c, 24c. ALIQUOT PARTS. 115 DIVISION. INTRODUCTORY EXERCISE. 1. $1.00 will buy how many pounds @ 25c; @ 33ic; @ 50c? 2. $1.00 will buy how many yd. of prints @ 25c? $240 will buy 240 times yd. or yd. 3. Suggest a method of finding quantity when cost and aliquot price are given. 4. 37^c = what part of a dollar? $840 -J- 37ic = $840 ^ $? = 840 X? = — 5. Suggest a method of division when the price is a multiple aliquot price. It is evident that by substituting the aliquot fraction, the operation is redi^ced to that of division by fractions, and thus to multipUcation. Illustration. $360 will buy articles @ 33ic; or articles @ 37|c. Solution. (1) 33ic = $1/3. $360 ^ $1/3 = 360 X 3/1 = 1080, the no. of articles that can be purchased. (2) 37^ = $3/8. 360 -i- 3/8 = 360 X 8/3 = 960, the number of articles that can be purchased. ORAL EXERCISE. 1. State rules for division by 375C, 45c, 75c, 66fc, $1.16f. 2. Suggest methods of checking such divisions. Cost. Price. Quantity. Cost. Price. Quantity. 3. $72 25c ? 9. $5.20 $1.25 4. 40 12^c ? 10. 48 .37^ 5. 46 16!c ? 11. 60 1.33i 6. 45 6fc ? 12. 54 1.50 7. 21 8ic ? 13. 918 2.25 8. 126 50c ? 14. 468 WRITTEN EXERCISE. .76 1. $758 will buy — lb. @ 80c. 2. $296 will buy — yd. @ 37ic. 3. $540 will buy — bu. @ 62^ 4. $925 will buy — A @ $12.50. 116 BUSINESS ARITHMETIC. 5. $5296 will buy — bu. @> 75c. 6. — cu. yd. © 62^c cost $1600. 7. — oz. @ 7ic cost $52. 8. — lb. @ 16f c cost $615.50. 9. — lb. @ 12ic cost $6040. 10. — lb. @ $2.75 cost $810. ILLUSTRATIVE EXERCISE. 1. 1.25 is what part of 100. The quotient obtained by dividing by 25 will be times the quotient by dividing by 100. 2. 7620 4- 100 = . 12^ = of 100. 7620 -^ 12^ = times 7620 -h 100. 3. Suggest short methods for division by aliquot parts of 100, of 1000. 4. 37i is contained times in 100. 5. Explain: 9640 ^ 37| = 8/3 of 1/100 of 9640 = . 6. Suggest short methods for division by 62^, 66f , 7^, 3i, 166|, 125, 133|. ORAL EXERCISE. Find the quotient of: 1. 2500 ^ 20. 7. 1600 -J- 80. 2. 4200 -r- 25. 8. 920 ^ 33i. 3. 3850 -r- 50. 9. 600 -^ 66f. 4. 7200 -^ 33i 10. 1200 -J- 133i. 5. 240 ^ 12i by 16!. H. 800 ^ 125. 6. 1200 -5- 75. 12. 2760 ^ 150. WRITTEN EXERCISE. 1. 17800 -^ 37i 4. 82984 -^ 1.33i. 7. 84020 -^ 6i. 2. 52900 ^ 62i. 5. 67500 -^ 125. 8. 94368 -5- 116f. 3. 4890 ^ 66i 6. 1284 4- 87^ 9. 9218 -^ 16i. Price by the Hundred or Thousand. In the many cases where articles are sold by the hundred or thousand, computations are simplified by '^ pointing off." Illustration. (1) Compute the cost of 920 lb. lead @ $4.00 per c. 920 lb. = 9.2 cwt. $4.00 X 9.2 = $36.80 cost. ALIQUOT PARTS. 117 EXERCISE. (Solve mentally if possible.) Compute the cost of: 1. 425 lb. @ $5.00 per c. 6. 4580 lb. @ S5.50 per c. 2. 675 lb. @ $12.50 per c. 7. 546 lb. fertilizer @ 85c per c. 3. 1250 yd. @ $45.00 per c. 8. 2450 ft. lumber @ $33,331 per M. 4. 786 ft. @ $1.25 per c. 9. 5260 ft. pine @ $40 per M. 5. 1900 ft. @ $1.33i per c. 10. 4690 ft. oak @ $50 per M. CHAPTER XX. PROBLEM ANALYSIS AND SOLUTION. In general, five steps should mark the analysis and solution of a problem. 1. A careful reading of the problem to determine what is given and what required. 2. Analysis, to determine the relations between what is given and what required. 3. Selection of process and of factors to be used. 4. The computation. 5. Checking — of reasoning and of computation. A problem should be thoroughly understood before its solution is attempted. The process selected should be the simplest and most direct. The computation should be performed and set down in the briefest and most direct manner — factoring, cancellation and short methods being used to save time and energy. In the written statement, care should be taken to use symbols accurately, and to exercise judgment in the naming of important values. There is no uniformity in the method of solution of problems. It is essential, however, that the selected solution should involve no unnecessary labor, and that the results obtained by it should be easily checked. All solutions of applied arithmetic problems require the application of common sense analysis, but this analysis is a simple matter, as the following illustrations will show. Illustrations. (1) Andrews offers a 3/8 acre plot of land for $72. This price is equivalent to a price of how many dollars per acre? Analysis and Solution. 1. Given the price of a part, to determine the price of the whole. 118 PROBLEM ANALYSIS AND SOLUTIONS. 119 2. If 3/8 of an acre cost $72, 8/8 acres mil cost 8 times one-third of $72. 3. All given factors required for solution. 24 4. 8 X^^ = $192. The value of one acre is $192. 5. Check. 3/8 of $192 = $72. (2) Working at the rate of 100 strokes per minute, a power pump dehvers iOO gal. of water per minute. How much must the speed be increased to pump 30,000 gal. per hour? 1. Given present speed and output per minute, and required output per hour, to find speed increase. 2. If the output is 400 gal. per minute, it is 60 times 400 gal. per hour. The speed must be increased by the same fractional part that the required output is greater than the present output. 3. By multiplication, find the hourly output. Compare the two out- puts to find increase. By fractional multiplication determine increase in speed. 4. Soluiion. 60 X 400 gal. = 24,000 gal., capacity per hour. 94 000 ~ 4 ' B,equired capacity is 5/4 of present capacity, or 1/4 greater. 1/4 of 100 strokes = 25 strokes. Therefore, the speed must be in- creased 25 strokes per minute. 5. Check. An increase of 1/4 in speed wiU increase the delivery per minute 1/4, or from 400 to 500 gal. A deUvery of 500 gal. per minute = a delivery of 60 X 500 gal. per hour, or 30,000 gal. Note. Numbers 1,2 and 3 are usually distinct mental processes. They are not represented in paper solutions. ORAL EXERCISE. 1. 32 is 4/7 of what number? 2. A pump that raises 6400 gal. per day of 8 hours will raise gal. if run 10 hours. 3. At 84c per lb., what is the cost of 3/4 lb. of pepper? 4. A factory is insured in one company for $6000, and for $4000 in a second. If damaged by fire to the extent of $3600, what loss should each company pay? 5. An improvement to a certain steam engine is warranted to reduce the coal consumption 2/5. The present consumption being 15 T per day, 120 BUSINESS ARITHMETIC. this means a saving of T., or a saving in money, at $3.40 per T, of $ . 6. A train running 1 mile in 4/5 of a minute is running at the rate of miles per hour? 7. The report of a cannon is heard 20 sec. after firing. How far distant is the cannon if sound carries 65,000 feet per minute? 8. Divide 10 into two parts, one being three times the other. EXERCISE. 1. A 5 inch line is used on paper to represent a distance of 60 feet. The paper length is what part of the true length? 2. By improvements in machinery, the output of a factory is increased 3/4. If, at the same time, the working time is reduced from 10 to 9 hours, what is the net increase in output? 3. A blend of tea consists of two grades, the first forming 3/8 of the mixture. By increasing the proportion of second grade 1/4, what part remains first grade? 4. A pipe discharges 32| gal. per sec. How many minutes will be necessary to discharge 10,000 gal.? What is its discharge rate per hour? 5. A dredge averages If cu. yard of material per minute. At that rate, it will remove how many cu. yd. in a working day of 8 hours. 6. How much water must be added to a solution containing 1/10 ammonia, to make the solution 1/30? CHAPTER XXI. INVOLUTION AND EVOLUTION. I. Involution. The products obtained by taking any number two or more times as a factor are termed powers of that number. The process of finding the powers of numbers is termed involution. If the number is taken twice, the product is termed the square of the number; if taken three times, the cube of the number; if taken more times, the corresponding power of the number, as the fourth power, fifth power, etc. Powers are expressed by writing the number with a small raised figure or exponent, showing the times it is taken as a factor. Illustrations. 2 X 2 is written 2^. 2X2X2X2is written 2*. Since the exponents represent the number of times the given number is taken as a factor, the product of two powers of the same number may be represented by writing the number with the sum of the given exponents as the exponent of the product. This principle is of value, also, in reckoning high powers of numbers. Illustrations. (1) Represent the product of 2' and 2*. Solution. 2' = 2 X 2 X 2. Short method. 2* = 2 X 2 X 2 X 2. 23 X 2< = 23+< = 2\ 23 x2< =2X2X2X2X2X2X2= 2' (2) Find the value of 3«. Solution. 38 = 3*+* = 3* X 3*. 3* = 3 X 3 X 3 X 3 = 81. 38 = 3* X 3* = 81 X 81 = 6561. 121 122 BUSINESS ARITHMETIC. ORAL EXERCISE. Determine the value of: 1. 3'. 3. 53. 5. 8». 7. 12*. 2. 2«. 4. 4». 6. 4*. 8. l(fi. 9. Find the squares of 4, 5, 6, 7, 8, 9, 10, 11, 12, 13, 14, 15, 16, 20, 25, 100, 120, 400, .2, .5, .12, .006, 1/4, 3/8, 2/3. 10. Show that a square must end in 0, 1, 4, 5, 6, or 9. 11. Show that a cube may end in any of the digits. EXERCISE. Compute, by the simphfied method, the value of: 1. 168. 3. 6.05^ 5. (f)«. 2. 121*. 4. .0036«! 6. (B*. 11. Evolution. The process of determining a number from one of its powers is termed evolution. If the number is obtained from its square it is termed the square root; if obtained from the cube it is termed the cube root. Roots of higher powers are named horn. the powers, as fourth root, fifth root, etc. Illustrations. The square of 6 is 6 X 6, or 36. 6 is the square root of 36. The cube of 4 is 4 X 4 X 4, or 64. 4 is the cube root of 64. Roots are expressed with the symbol l/ written over the number of which the root is to be found. The power to be taken is shown by a numeral, except in the case of the square root. Illustrations. The square root of 225 is expressed V225, but the square root equals 15. The cube root of 729 is expressed ^729, but it equals 9. INTRODUCTORY EXERCISE. 1. Name at sight the equal factors or square roots of 4 (4 = 2 X 2; square root is 2), 9, 16, 25, 36, 64, 81, 100, 10,000, 121, 144. 2. Find the squares of each digit. How many places in each case? INVOLUTION AND EVOLUTION. 123 3. Find the squares of 1, 10, 100, 10,000, .1, .01, .001. tiow do the numbers of places in the squares compare with those in the roots? It is evident, from these examples, that the number of places in any square is twice, or one less than twice, the number of places in the root. This fact is important, as it is possible, by pointing off the square from the decimal point into periods of two places, to determine the number of places in the root. The method of determining the square root may be under- stood, now, by studying the reverse process, using the method of multiplication by parts. Example. Find the square of 86. Solution. 86 = 80 + 6. 80 + 6 86 80 + 6 86 (80 X 6) + 62 516 802 ■!■ (go X 6) 6880 802 _|_ 2(80 X 6) + 62 = 7396 It is evident that the square of a number, composed of tens and units, is equal to the square of the tens plus twice the product of the tens multiplied by the units plus the square of the units. It is evident, also, that any number may be divided into two parts and its square determined in exactly the same way. This composition of the square is made use of in determining the root. Illustrations. (1) Compute the square root of 1296. Solution. 12'96(30 + 6 = 36, root. 302 =9 00 3 96(6 Trial div. 2 X 30 = 60 2 X 30 X 6 =360 62 = 36 3 96 Analysis. Since there are 4 places in the square there are 1/2 of 4, or 2 places in the root. Point off by 2's. 12 represents the ten's place in the root, and 124 BUSINESS ARITHMETIC. 96 the unit's place. By trial, the largest square in 12 is the square of 3. .'. the largest square of the tens is 30^, or 900. Subtracting the square of the tens, there are left 396 units. Since the square of the tens has been subtracted, there is left in the perfect square twice the product of tens and units plus the square of the units. As a trial divisor, use twice the tens, or 60. 60 is contained in 396 six times. Try 6 as the required unit. 2(30X6) =360; 6X6 = 36. The sum is 396. Evidently 6 is the required unit, and 36 the required root. Check. 36 X 36 = 1296. (2) Find the square root of 1.6384. Solution 1.63'84(1.28 root. 12 j_ 2X1.0 = 2.0, trial, 63(0.3 2X1.0X .2 =40 .22-= .04 44 2X1.20 = 2 40, tFial, .19 84(0.8 2 X 1.20 X. 08 =.1920 .082 = .0064 19 8 4 Check. 1.28 X 08"= 1.6384 or Analysis. Point off by 2*8, each way from the decimal point. If a perfect square, the root will contain one whole number and two decimals. 1 is the nearest square of the unit period and the first figure of the root. Subtract the unit and bnng down the first decimal group. Twice 10 tenths, or 20 tenths is the trial divisor. The resulting quo- tient, 3, is found to be too large, so the next lower number, 2, is taken as the next figure of the root. Complete the square, considering 1 as the ten, and .2 as the units. Subtract the value of the com- pleted square from .63 leaving .19 and bring down final group. As a trial divisor, use twice the portion of the root already found, or 2.40, obtaining the final figure, .08. Complete the square. Thus, to determine the square root: 1. — Beginning at the decimal point separate the number, each way, into groups of two figures. (The outermost period on the left may have only one place.) 2. — Find the greatest square of the left hand period. Its root is the first figure in the required root Treat as whole number. 1.63'84(1.28 root, 1« = 1 1 Trial, 2X10 63 2X10X2=40 22= 4 44 Trial 2X120 19 84 2X120X8 = 1920 82= 64 19 84 INVOLUTION AND EVOLUTION. 125 3. — Subtract the square from its period and bring down the next period. As a trial divisor use twice the root value already found, annexing a cypher. Considering the quotient as the new unit, complete the square by taking twice the product of the tens multiplied by the units plus the square of the units. 4. — Subtract this value from the last remainder; bring down the next group, and proceed as before. Noi'E. (1) In the case of fractions, either take the square root of numerator and denominator separately, or reduce the fraction to the decimal form. (2) When the remainder, at any stage, will not contain the trial divisor, write a cypher in the root and bring down the next group. If there are no further periods, periods of two cyphers may be added to the original number and the root determined to any desired number of decimal places. Since the product of a second power by itself is a fourth power, it is evident that the square root of a fourth power is the square of the number. If the square root of this square is then taken, the result will be the fourth root of the number. EXERCISE. Find the square root of: 1. 256 5. 1025.9209 9. 29,506,624- 2. 7225. 6. 52.9984. 10. 329,968,821 (2 dec. pi.). 3. 7744. 7. 134,944.81. 11. .008,907,386,72 (5dec.pl.). 4. 16,641. 8. .000,000,494,209. 12. 56,847.02938176 (4 dec. pi.). m 2209 17,161 85,264 7569 3969 23,104 1,580,049 17. Reduce the following to decimals and take the approximate square root to the thu-d decunal place: (a) 5/14; (6) 7/88; (c) 1/21. By continued square root, find: 18. The fourth root of 10,098, 039, 121. 19. The fourth root of 3.8416. CUBE ROOT. INTRODUCTORY EXERCISE. 1. WTiat is the cube of any number? What is the cube root of the cube? 2. Find the cubes of the digits. What is the largest number of places in the results? 126 BUSINESS ARITHMETIC. 3. Find the cubes of 1, 10, 100, 1000, .1, .01. How many places in each cube? It is evident that the cube root of a number is one of its three equal factors. It is evident, also, that each period of three places in the cube stands for one place in the root. As in the case of square root, the method of determining the cube root may be understood from a study of the reverse process. Illustration. Find the cube of 46. Solution. Let 46 be written 40 + 6. 40 + 6 46 40+6 46 (40 X 6) + 62 276 40^ + (40 X 6) 1840 402 _|. 2(40 X 6) + 62 2116 40+6 46 (402 X 6) + 2(40 X 62) + 6' 12696 408 ^ 2(402 X 6) + (40 X 62) 84640 40^ + 3(40* X -6) + 3(40 X 62) + 6» = 97336 From a study of the product it is evident that the cube of a two-place number equals the cube of the tens plus three times the product of the square of the tens multiplied by the units, plus three times the product of the tens multiplied by the square of the units, plus the cube of the units. It is now a simple matter to reverse the process and determine the cube root. Illustration. Find the cube root of 97,336. Solution. 97'336(46 Separate into periods of three places. 43 64 The nearest cube root of the tens group Trial 4800 33 336 is 4. Subtract the cube of 4 from 97 3X402X6=28800 and bring down the next group. Asa 3X40 X 62 = 4320 trial divisor use three times the square 6' = 216 33336 of the tens, obtaining 6 as a trial unit. Check. 46X46X46 = 97,336. The 33,336 units must be made up of three times the product of the square of the tens multiplied by the units, plus three times the tens multiplied by the square of the units, plus the cube of the units. These values are INVOLUTION AND EVOLUTION. 127 determmed separately and totalled as shown. Since they total 33,336, the 6 is the correct unit value of the root, and 46 is the cube root of the number. As in the case of square root, by always considering the figures of the root, which have been determined at any point in the solution, as tens, it is possible to determine the next figure of the root. Illustration. Find to three decimal places, the cube root of 32.890033664. Solution. 32.890'033'664(3.203 + root. 3' 27 Trial, 3X30^ 5 890(2 3X30^X2 = 5400 3 X 30 X22 = 360 2» = _8 5 768 Trial, 3X3200^ 122 033 664(3 3X32002X 3 = 92160000 3X3200 X 32 = 86400 3» = 27 92 246 427 29 787 237 remainder. Note. In trying for the third figure of the root, the trial divisor proves larger than the new dividend. Therefore a cypher is written in the root, another period brought down, and a new trial divisor used. It is evident that the steps to be followed in obtaining a cube root are: 1. — Beginning at the decimal point, separate the number, each way, into periods of three places each. 2. — For the first root figure, take the cube root of the great- est cube contained in the left hand period. Subtract the cube, bringing down the remainder and the next period of three places. 3. — For succeeding root figures, use as a trial divisor, three times the square of the root values already obtained, con- sidered as tens. Use the quotient so obtained as a trial root value arid complete the cube based upon it. If it proves too large reduce the unit value by 1. Complete the cube, subtract from the last dividend and bring down the next 128 BUSINESS ARITHMETIC. period. Continue this process until the periods are exhausted, or the desired nlimber of decimal places in the root have been obtained. EXERCISE. Find the cube root of: 1. 941,192. 6. 4/5, by reduction 729_ 2. 9.300746727. to decimal. * 9261 3. 147,197,152. 7. 8.0378843 (4 dec. 12,167 4. 25 (4 dec. pi.). pi.). 10,095,813 5. 1,083,201,548.8. 1007.65 (5dec.pl.). 12. .0000000678 (3 dec. 9. .001205 (to 3dec.pl.). pi.). GRAPHIC ILLUSTRATIONS OF SQUARES AND CUBES. Note. If deemed advisable, this section may be taken after the chapter on practical measurements. 1. Squaees and Square Root. It has been shown numerically (page 123) that the square of any number composed of tens and units is equal to the square of the tens plus twice the product of the tens multiplied by the c X D units plus the square of the units. This may be shown graphically, as ^ follows: Illustration. Compute the square of 45. Solviwm. (1) (2) 40+5 45 40+5 45 5X40+5« 225 4 0^+ 5X40+ 180 40»+2 X5 X40+52 =2025 2025 (3) To any scale, lay off 40, and then 5, in one straight line. The square on this line, ABCD, is evidently the square of 45. Construct the square of 40, AB'C'D', extending the sides meeting at D', to their intersection with the square of 45. This divides the square, as shown, into a square of 40, two rectangles measuring 40 X 5, and a small square of 5. The square of 45 plainly contains the square of the tens (40), the square of the units *-5(m) 40 (0 5U)-* 5 6(u)- / i 40(0 6(«)- i-^ — f.\ 20 1 I 5x 100 20 INVOLUTION AND EVOLUTION. ' 129 (5) and twice the product of units and tens (2X40X5). But 45 might have been any number of t tens and u units. Substituting t and u for 40 and 5 respectively, it is evident, from the figure, that the square of t+u = t^-\-2tu-\-u^. In a number containing more than two places, as 125, the principle still holds. 125 = 100+20+5. As shown by the diagram, the square of 100+20 = 1002+2X100X20+202. Now as- suming 120 as twelve tens, the square of 125 is evidently, 1202+2X5X120+52. It is evi- dent that each square, from the inmost, be- comes in turn the square of the "tens" for the determining of the square of the "tens plus units." In the reverse process of obtaining the square root of a number (see page 124) it should be evident, now, that in taking as the highest figure of the root the square root, of ^"^ the greatest square contained in the left hand group of two figures, one obtains the edge of the innermost square. Con- sidering this as the "tens" value, the next place is computed as the "unit," thus obtaining the next larger square. Then this square is taken in turn as the square of the "tens" and a new "tens plus units" square is obtained, and so on. In the process of computing the square root, at each step, after the square of the known "tens" is subtracted from the total square, the remainder equals 2 X tens X units plus the square of the units. The new unit is unknown, but, by dividing the remaining area by the known factor of the large term, "2 tens," as a trial divisor, there is obtained an approach to the exact imit. This imit is then tested to see if it is exact* 2. Cubes and Cube Roots. It has been shown numerically (page 126) that the cube of any two-place number equals the cube of the tens plus three times the product of the square of the tens multiplied by the units, plus three times the product of the tens multiplied by the square of the units, plus the cube of the units. As in the case of squares, this may be shown graphically as well as numerically. Illustration. Compute the cube of 15. 10 130 BUSINESS ARITHMETIC. ions. (1) (2) 15 10+5 15 10+5 75 5X10+52 15 102+ 5X10 225 102+2X5X10+52 15 10+5 1125 5X102+2X52X10+5' 225 103+2X5X102+ 52x10 3375 = 103+3X5X102+3X52X10+5' (3) To any scale, construct a cube of one "ten" edge. Extending the edges meeting at one point, construct over the cube, a second cube of an edge equal to one " ten " and five " units." Assume the faces of the smaller cube extended to cut the faces of the larger. A study of the subdivided figure will show that the entire cube is made up of a cube of 10 edge; three rectangular solids. A, B and C, each measuring 10X10X5; three rectangular solids, D, E and F, measuring 10X5X5; and a cube, G, of 5 edge. Thus, as in the numerical statement, the cube contains the cube of the tens, plus three solids whose contents equal the square of the tens multiplied by the units (10X10X5) plus three solids whose contents equal the pro- Original cube 1, 2, 3, 4, 5, 6, 7, 8 shown duct of the tens multipHed by hy th.Q light Wne^. the square of the units (10X5 X5) plus the cube of the units (53) , Since this is general for two place numbers, or for numbers treated as tens and units, the cube, in general, will equal t^-\r^t'hj,-\rZtu^-\-v?. In reconstructing this cube from the known cube of the tens, it is evident that there must be added to the cube of 10, 3375 - 1000, or 2375, cubic units. These additions must be in the form of three solids having an outer surface area of 10X10, or 100 square units; three solids having an outer surface length of 10 and for width the unit value still unknown; and a cube having the unknown unit for an edge. All these additions have a uniform thick- INVOLUTION AND EVOLUTION. 131 ness equal to the desired unit of the root. Since the numerical value of the cubic contents, when divided by the numerical value of surface area, gives a Unear measure as a quotient, the trial process in finding the second and succeeding figures in a cube root consists in dividing the cubic contents, remaining after the cube of the known root is subtracted by the surface area of the new additions, so far as known. The known part is 3t^-\-St. Since this does not represent the complete outer surface area of the addi- tion, it is used as a trial divisor, and the resulting quotient is tested to see if it is exact, or too large or small. It is tested of course in the formula CHAPTER XXII. DENOMINATE NUMBERS. Standard Measures. The modern systematizing of business, and the simplifying of methods of handling merchandise, have had a marked effect on our systems of measures. It is rarely, now, that commercial quantities are expressed in several denominations. Instead, some measure of a table is selected as a unit or standard and quantities are expressed as multiples, decimals, or simple fractions of that unit. The unit varies for different trades, professions, or different classes of merchandise. Thus the civil engineer measures by units of 1000 ft., 100 ft., 10 ft., tenths of a foot, etc.; the mechanic, in inches and decimals. The contractor often estimates in cubic yards and decimals. Scales are now made to weigh in tons and decimals, and clocks to mark time in hours and hundredths. The grocer uses frac- tional pounds rather than ounces, as the dry goods dealer uses the yard and its fractions. Many milk dealers use the pint as a unit. Grain, and some vegetables, may be measured by the pound unit instead of by dry measure. Practically all forms of statistics use a decimal system based on a single unit. The tendency toward unit measures is aided and reflected by the increasing sale of merchandise in unit packages, and by the increasing use of automatic weighing and computing machines, practically all of which work on a decimal basis. The simplicity in actual use is reflected, naturally, in com- putations. Except in computations of time and English money, it is seldom necessary to handle three denominations of any table. 132 - DENOMINATE NUMBERS. 133 Tables of Measures. This section includes the standard tables in general use, and a few units and special values common to certain type businesses. MEASURES OF CAPACITY. Dry Measure. 2 pints = 1 quart. 8 quarts = 1 peck (pk.). 4 pecks = 1 bushel (bu.). 1 dry qt. =67.2 cu. in. 1 bushel = 2150.42 cu. in. A heaped bushel, used for corn in the ear, apples, etc., contains 2747.71 cu. in. Liquid Measure. 4 gills (gi.) = 1 pint (pt). 2 pints = 1 quart (qt.). 4 quarts = 1 gallon (gal.). 1 gallon = 231 cu. in. Barrels and hogsheads vary in size. For general estimates. 3U gal. = 1 barrel (bbl.). 63 gal. = 1 hogshead. MEASURES Avoirdupois Weight. 16 ounces (oz.) = 1 poimd Qh ). 100 pounds = 1 hundred- weight (cwt.). 2000 pounds or 20 hundredweight = 1 ton (T,). In measuring mining products and in custom house business, the long ion of 2240 lb. is used. Comparative Table. Troy and Avoirdupois Weight. 1 oz. Avoir. = 437^ gr. 1 lb. " = 7000 gr. 1 oz. Troy = 480 gr. 1 lb. " = 5760 gr. OF WEIGHT. Troy Weight. 24 grains (gr.) = 1 penny- weight (pwt.). 20 pennyweights = 1 ounce. 12 ounces =1 pound. Troy weight is used in the measurement of the precious metals. Jewels, such as diamonds and pearls, are weighed by the carat which = 3.2 Troy grains. The term "carat" is also used to express the proportion of pure gold in composition metals. If pure gold, the metal is "24 carats fine." If one-half alloy, it is "12 carats fine," etc. Apothecaries' Weight. 20 grains = 1 scruple (sc. or 9) . 3 scruples = 1 dram (dr. or 5)- 8 drams = 1 oimce (oz. or 5)- 12 ounces or 5760 gr. = 1 pound (lb.). 134 BUSINESS ARITHMETIC. Special Unit Weights — Avoirdupois. 1 bbl. flour = 196 lb. In most states: 1 bbl. salt = 280 lb. 1 bu. potatoes = 60 lb. 1 bbl. pork = 200 lb. 1 bu. wheat = 60 lb. 1 cu. ft. fresh water = 62^1b. 1 bu. com = 56 lb. 1 bu. oats = 32 lb. COUNTING. 12 things = 1 dozen (doz.). 24 sheets (paper) = 1 quire (qr.). 12 dozen = 1 gross (gro.). 20 quires, or 12 gross = 1 great gross (g. gr.). 480 sheets = 1 ream (rm.). 20 things = 1 score. (Commercially, 500 sheets are often used for a ream.) MEASURES OF EXTENSION. Linear or Long Measure. Surveyor's Long Measure. 12 inches (in.) = 1 foot (ft.). 7.92 inches = 1 link (Ik.). 3 feet = 1 yard (yd.). 25 links = 1 rod. 5^ yards, or 4 rods, or 16^ feet = 1 rod (rd.). 100 links = 1 chain (ch.). 320 rods, or 80 chains = 1 mile. 5280 feet = 1 mile (mi.). City property is measured in feet and decimals thereof; large Sea Measure. tracts of farm, or unimproved land 6 feet = 1 fathom. by surveyor's measure. 120 fathoms = 1 cable length. 6086.7 feet, or about 1.15 miles = 1 knot, nautical or geographical mile. 3 knots = 1 league. Circular or Angular Measure. 60 seconds (") = 1 minute ('). 90 degrees = 1 right angle. 60 minutes = 1 degree (°). 360 degrees = 1 circumference. 1 minute = 1 geographic mile. This measure is used in geography, navigation and higher surveying, for the computation of differences of time, locations on the earth's surface, longitude and latitude; and for the measurement of angles. Square Measure. Surveyor's Square Measure. 144 square inches = 1 square foot. 625 square links = 1 square rod. (sq. in.) (sq. ft.) 10 square rod« = 1 square chain. DENOMINATE NUMBERS. 135 Square Measure. 9 square feet 30j square yards 160 square rods 640 acres 100 square feet = 1 square yard. = 1 square rod. = 1 acre (A ). = 1 square mile. = 1 square. Surveyor's Square Meaeure. 10 square chains = 1 acre. 640 acres = 1 square mile. 36 square miles = 1 township. Cubic Measure. 1728 cubic inches = 1 cubic foot. 128 cubic feet or (cu. in.) (cu. ft.) 8 cord feet = 1 cord (wood). 27 cubic feet = 1 cubic yard. 1 cubic yard = 1 load (earth). 16 cubic feet = 1 cord foot (cd. ft.) . MEASURE OF TIME. 60 seconds (sec.) = 1 minute (min.). 360 days = 1 commercial year. 60 minutes = 1 hour (hr.). 365 days = 1 common year. 24 hours = 1 day. 366 days = 1 leap year. 7 days = 1 week (wk.). 10 years = 1 decade. 30 days = 1 commercial 100 years = 1 century. month. 12 months (mo.) = 1 year (yr.). 365 days, 5 hours, 48 minutes, 49.7 seconds = 1 solar year. Centennial years divisible by 400 and other years divisible by 4 are leap years. MEASURES OF VALUE. U. S. Money. Canadian Money. See pages 50 and 53. The same as for the United States. 100 cents = 1 dollar = $1. English Money. 4 farthings (far.) = 1 penny (d.). 12 pence = 1 shilling (s.) = $0,243 +. 20 shilUngs = 1 pound sterling (£) = $4.8665. French Money. 100 centimes = 1 franc $0,193. German Money. 100 Pfennigs = 1 Mark = $0,238. Mexican Money. 100 centavos = 1 peso = $0,498. 136 BUSINESS ARITHMETIC. DENOMINATE NUMBERS. INTRODUCTORY EXERCISE. 1. Give illustration of abstract numbers, concrete numbers, like and unlike numbers, denominate numbers, simple and compound numbers. (See pp. 1 and 2.) 2. Name simple denominate numbers expressing: time, distance, area, value, weight. 3. Name compound denominate numbers expressing: cubic contents, liquid capacity, area. 4. Name a very common denominate measure used in each of these trades, businesses or professions : grocery, provision, oil, produce, dry goods, plumbing, mechanical engineering, contracting, banking, jewelry, coal and wood. 5. Name common denominate measures used in speaking of such things as — (1) The distance from New York to Chicago. (2) The distance across the street. (3) The quantity of a meat order. (4) The time required to do an errand. (5) The amount of imports of the United States for last year. 6. Name the common measures used in your household and its trading. 7. What is the advantage of buying by some measure or fraction of it, over buying by two or more denominations of a thing? 8. What are the advantages to the retailer in selling goods in de- nominate packages, rather than in bulk? It is evident from the tables that a number expressed in one denomination may be expressed in another denomination of the same table, and sometimes of other tables, without altering its value. The process of changing the form of expression of a quantity is termed reduction ascending, if the denomination chosen is larger than the original, and reduction descending, if it is smaller. REDUCTION DESCENDING. Illustrations. (1) Reduce 10 rds. 3 yd. to feet. Analysis and Solution, (a) 1 rd. = 5.5 yd. 10 rd. = 55 yd. 55 yd. + 3 yd. = 58 yd. 1 yd. = 3 ft. 58 yd. = 3 X 58 ft. or 174 ft. DENOMINATE NUMBERS. 137 (6) 1 rd. = 16.5 ft. 10 rd. = 10 X 16.5 ft. or 165 ft. 3 yd. = 3 X 3 or 9 ft. 165 ft. + 9 f t. = 174 ft. (2) Reduce .45 gal. to lower denominations. Analysis and Solution. 1 gal. = 4 qt. .45 of 4 qt. 1 qt. =2 pt. 1 pt. =4 ^. .-. .45 gal. = Iqt. 1 pt. 2.4 gi. 1.8 qt. .8 of 2 pt. = 1.6 pt. .6 of 4 gi. = 2.4 gi. 1 yd = 3 ft. Find missing values: 1. 3 gal. = qt. 2. 2 cd. = cu. ft. 3. 2 sh. 6 d. = d. 4. 2\ gal. = pt. 5. 3/4 bu. = pt. 6. 2 m. = rd. 7. 2 bu. = pt. 8. 1 gr. 3 doz. = units. ORAL EXERCISE. 9. .4 pk. = qt. 10. 3.5 sq. yd. = sq. f 11. U' 6" = ". 12. 65 score = units. 13. 1 mi. 40 rd. = rd. 14. 1.3 T. = lb. 15. £2.25 = sh. EXERCISE. 3. 5.6 gal. to gills. 4. 1.375 sq. mi. to A. Reduce: 1. 7.3 cu. yd. to cu. in. 2. 31 T. to oz. 6. 7.475 mi. to ft. 6. Find the cost of 5f A. @ 5c per sq. ft. 7. What will it cost to excavate 942.5 cu. yd. @ 15c per cu. ft.? 8. Express 2 mi. in equivalent engineer's unit of 1000 ft. 9. Reduce 11 gt. gross to a fraction of 1000 units. 10. What part of 1 rd. are 6 in.? 11. What part of 1 mi. are 2 y4.? REDUCTION ASCENDING. Illustrations. (1) Express 729 ft. in higher denominations. Solution. Since 3' = 1 yd. 3)729 1 rd. = 5i ft. 5.5)243 = no. yd. (mult, by .2) or 11)486 729 ft. = 44 rd. 2 yd. 44 = no. rd. 2 yd. rem. 138 BUSINESS ARITHMETIC. (2) Express 2 ft. 9 in. as a decimal of a yard. Solution. ■: 2 ft. 9 in. = 33 in. and 1 yd. = 36 in. .-. 2 ft. 9 in. = 33/36 of a yd. 33 -^ 36 = .916 +. .-. 2 ft. 9 in. = .916+ yd. ORAL EXERCISE. Reduce to the next higher denomination: 1. 52 ft. ' 3. 12 yd. 5. 65 in. 7. 8^ sh. 2. 27i pt. (dry). 4. 27,000 lb. 6. 148 oz. 8. 50 cu. ft. Find the cost of: 9. 2 lb. @ 6c per oz. 13. 3i gal. @ 5c per pt. 10. 5 qt. @ 8c per pt. 14. 3 pk. @ 60c per bu. 11. 4 yd. @ 5c per in. 15. 40 ft. @ 30c per yd. 12. 2 bu. @ 30c per pk. 16. 900 lb. @ $36 per T. EXERCISE. (Decimals to 4 places.) 1. Express as decimals of a mile: 1 rd.j 1 ft.; 1 yd.; 7.35 in.; 8.4 yd. 2. Express as decimals of a bu.: 1 pk.; 3 qts.; 1 pt.; 3^ pt. 3. Express as decimals of a cu. yd.: 1 cu. ft.; 20 cu. in. 4. Express as decimals of a £: 1 sh.; 1 d.; 7 sh. 6 d.; £ 3 8 sh. 5 d. 5. Express as decimals of a T. : 11 lb.; 1 oz.; 745 lb.; 212 lb. 8 oz. 6. Express as decimals of an acre : 1 sq. rd. ; sq. yd. ; 96 sq. ft. ; 45 sq. yd. 7. Reduce to higher denominations: 326,425 in.; 8429 ft. (dry); 11,927 cu. in 8. What quantity (gallons) of ohve oil must be bought to fill 90 gross of pint bottles? The Fundamental Processes. The fundamental pro- cesses, as applied to denominate or compound numbers, differ relatively little from the same processes in simple numbers. The main point of difference is in the substitution of an irreg- ular reduction scale for the decimal scale. ADDITION. Illustration. (1) Add 567 and 329. (2) Add 5 bu. 3 pk. 5 qt. and 3 bu. 2 pk 4 qt. DENOMINATE NUMBERS. 139 Sohdions. (1) (2) (2) Condensed. 567 5 bu. 3 pk. 5 qt. 5 bu. 3 pk. 5 qt. 329 3 2 4 3 2 4 896 8 bu. 5 pk. 9 qt. 9 bu. 2 pk. 1 qt. or 9 bu. 2 pk. 1 qt. (Reduction ascending.) Note. (2) In the condensed form the reduction is performed mentally, as the total of each measure is obtained. Thus the total of the quart column is 9 qt. Since 9 qt. = 1 pk. 1 qt., the 1 quart is written in the sum, and the 1 peck is carried forward to the next column. ORAL EXERCISE. Find the sum of: 1. 2. 3. 5 gal. 2 qt. 1 pt. 6 bu. 1 pk. 1 T. 12 cwt. 2 1 1 13- 3 5qt. 3 9 4. 5. 6. 15 cwt. 84 lb. 7] mi. 46 rd. 6sq. yd. 5 sq. ft. 7 19 2^ 300 1 2 1 8 7. To 3 gal. add consecutive pints. % EXERCISE. 1. Find the total if the items of a purchase from a London merchant are : £5 3 s. 6 d.; £ 18 11 s. 4 d.; £12 5 s. 8 d. 2. In St. Clair County the following stretches of country road have been macadamized in the past year. 1 mi. 160 rd., 2 mi. 145 rd., 8 nai. 176 rd., 1 mi. 46| rd., a total of ?. 3. A lumberman, who already owns stumpage rights on 3 sq. mi. 260 A., buys rights to neighboring tracts of 84 A., 1 sq. mi. 52 A., 526 A. 40 sq. rd., and 64 A. 120 sq. rd. He then has rights to ? sq. mi. ? A. or to ? A. For the rights to lumber these new tracts, he paid $85 per A. or $ ? m aU. SUBTRACTION. Subtraction involves reduction descending. Illustrations. (1) (2) 5 bu. 3 pk. 6 qt. 5 T. 7 cwt. 16 lb. 2 1 3 2 5 46 Differences 3 bu. 2 pk. 3 qt. 3 T. 1 cwt. 70 lb. Reduce 1 cwt. of the minuend to pounds. 116 lb. — 46 lb. = 70 lb.; 6 cwt. — 5 cwt. = 1 cwt., etc. 140 BUSINESS ARITHMETIC. ORAL EXERCISE. Find the difference between : 1. 2. 3. 4. 5 bu. 3 pk. 5 gal. 2 qt. 16 cwt.18 lb. 5 lb. 11 oz. 12 23 4 20 32 Find the difference between the larger and smaller quantity in ex. 1-5 of the last oral exercise. EXERCISE. 1. What check can you suggest for compound addition and subtraction? Illustrate with examples. 2. On a contract for the laying of 752 cu. yd. of masonry, 15 cu. yd. 7 cu. ft. are laid one day, 31 cu. yd. 14 cu. ft. the second day, 68 cu. yd. 8 cu. ft. the third and 88 cu. yd. 11 cu. ft. the fourth. How much work remains to be done? 3. From 15.0739 mi. subtract 2 mi, 126 rd. 2 yd. 4. Make the extensions in this form: Gross Cost. Discount. Net cost. £ 16—5— 8 £ 2— 6—4 £ 4—9— 3 8—7 84—7—11 3—15—8 ————■ Totals ? ? ? MULTIPLICATION. That no new principle is involved is seen from this Illustration. Find the product of 3 bu. 2 pk. 1 qt. by 8. Solution. Analysis. 3 bu. 2 pk. 1 qt. 8 X 1 qt, = 8 qt. = 1 pk. S_ 8 X 2 pk. + 1 pk. = 17 pk. = 4 bu. 1 pk. 28 bu. 1 pk. qt. 8 X 3 bu. + 4 bu. = 28 bu. EXERCISE. (Solve mentally if possible.) Multiply: 1. 3 ft. 2 in. by 6. 2. 1 yd. 2 ft. by 8. 3. 3 lb. 2 oz. by 30. 4. 2 pk. 3 qt. by 12. 9. How much silver is required for five ornaments each containing 1 oz. 7 pwt.? 5. 2 qt. 1 pt. by 8. 6. 3 rd. 27 Unks by 9. 7. 3 cwt. 60 lb. by 10. 8. 1 T. 4 cwt. by 20. DENOMINATE NUMBERS. 141 10. A factory manager reduces the length of a metal bar in a machine he manufactures 3f in. This means a saving of how much metal in 12,000 bars? 11. Determine the cost of 75 yd. extra broadcloth, purchased at £1 2 8. 3 d. per yd. 12. A factory expert discovers that the length of a bolt used in the company's product may be reduced 1/4 in. This means a saving of how many feet if 250,000 bolts are used per year? 13. Compute the cost of a car load containing 60 bbl. of apples, each containing 2 bu. 2 pk., at 75c per pk. DIVISION. In division of denominate numbers the divisor may be either abstract or denominate. Illustrations. (1) Compute the cost, per yd., of 80 yd. cloth invoiced at £180. Solution (a). Solution and Analysis (5). 80 ) £180 The cost per yd. is evidently 1/80 of £ 2.25 = £2 5 8. (cost) £180. 1/80 of £180 = £2 and a remamder of £20. £20 = 400 s. 1/80 of 400 8. = 5 8. .*. 1/80 of £180 = £2 5 8. (2) At a price of £1 2 s. per yd., how many yards may be purchased for £89 2 8.? Solution. £ 1 2 s. = 22 8. The numbers being concrete, re- £89 2 s. = 1782 s. duction to a common denominator is 81 = no. of yd . necessary. ~22)1782 ORAL EXERCISE. Divide : 1. 6 bu. 3 pk. by 3. 2. 5 gal. 2 qt. by 2. 3. 3 T. 6 cwt. by 200. 4. 41b. by 8. 5. 3 pk. 6 qt. by 10. 11. 5 bu. 1 pk. by 1 bu. 3 6. 5 yd. 1ft. by 4. 7. 3 sq. yd. 3 sq. ft. by 6. 8. 3 gal. by 2 qt. 9. 4 lb. by 8 oz. 10. 2 ft. 6 in. by 3 m. 142 BUSINESS ARITHMETIC. EXERCISE. 1. Divide 960 cu. yd. 5.4 cu. ft. by 11.5, checking by multiplication. 2. A carriage wheel revolves two times in 7 yards, or times in one mile. 3. Divide 5 bu. 7 pk. 3 qt. by 1 pk. 2 qt. 1 pt. Check the result by multiplication. 4. A dealer has 2 bbl., 31| gal. each, of olive oil, to be bottled in half-pint bottles. He requires — bottles. 5. A newspaper press printed 57,384 copies in 2 hr. 5 min., at the average rate of — copies per minute. Individual Original Work. Report on "Common Measures Used in Business." Disregard text- book classifications, and find out what parts of tables are in actual common use. Classify under: (1) Common to business in general; (2) limited to particular trades; (3) non-text-book measures. Give illustrative examples. CHAPTER XXIII. PRACTICAL MEASUREMENTS. Many simple measurements in business and science are based on elementary geometric principles. A slight knowledge of some of these principles, and of the geometric figures to which they apply, is of value to everyone. Geometric Conceptions — Plane Figures. A geometric line is considered to have extension but neither length nor breadth. A straight line is the shortest distance between two points. Lines are parallel if they are the same distance apart throughout their entire length.ZZ An angle is the divergence of two lines having a common point. AXB is the angle of the lines AX and XB, If the two lines meet so that the two angles on the same side of one line are equal, the angles are right angles, and the lines are perpendicular. Thus AYC and BYC are right angles and CY is perpendicular to AB. An acute angle is less than a right angle. An obtuse angle is greater than a right angle. A geometrical surface has length and breadth but no thick- ness. A plane, or plane surface, is a level surface, such as that of still water. A figure in a plane is a plane figure. 143 144 BUSINESS ARITHMETIC. A triangle is a plane figure bounded by three straight lines. It is called equilateral, isosceles, or scalene, according as it has three sides equal (A), two sides equal {B),ot no sides equal (C). A right angled triangle has one right angle (D). A quadrilateral is a plane figure bounded by four straight lines. If the opposite sides of a quadrilateral are parallel, the figure is called a parallelogram (E-H). The parallelogram is a rectangle if it has four right angles (F). It is a square if the sides are equal and the angles right angles (G). It is a rhombus, if the four sides are equal but the angles are not right angles (H), PRAlCTICAL MEASUREMENTS. 145 A circle is a plane figure bounded by a curved line, called the circumference, every point of which is equidistant from a point within called the center. The diameter is any straight line passing through the center and terminating in the cir- cumference. The radius is one-half of the diameter. An arc is any part of the circumference, and is measured by degrees of angular measure. The circumference is approximately 3.1416 (approximately 3y) times the diameter. This factor is called by the Greek letter ''tt," (pronounced, "pi.") If the diameter is known, the circumference may be determined by multiply- ing it by 3.1416. If the circumferenpe is known, the diameter is determined by finding the quotient of the circumference divided by 3.1416. The perimeter of a plane figure is the distance around it. The ha^e is the side on which it is assumed to rest. The altitude is the perpendicular distance from the base to the most distant point of the figure. EXERCISE. 1. Draw four circles of 4", 5", 6", and 8" radius, respectively. In each case, measure carefully the circumference and divide by the re- spective diameters. . Take the average of the resulting quotients. How close to the value of "x" do you approach? By computation, find your error in measurement of each circumference. To what is this error due? 11 146 BUSINESS ARITHMETIC. 2. Compare the diameters and circumferences of two circles, one of which has a diameter of 10' and the other a circumference of 42'. 3. What is the diameter of a circular path a mile in circumference? 4. Compute the cost of the fencing around a circular park of 90 ft. radius, at $1.50 per foot. 5. In racing on a circular track, what gain may result from securing an inside position? Illustrate. 6. A man has 1000 ft. of movable fencing for a chicken nm. Give dimensions of a square field, a rectangular field and a circular field that he may enclose. Areas of Plane Figures. The accompanying rectangle is divided by parallel lines into unit squares. How many unit squares in each row? How many rows? Which dimension shows the number of rows? Which the units per row? What is the total number of units? How obtained? Show that the number of square units would be the same if "5'' were considered the base and "6" the altitude. It is evident that the area of the rectangle is equal to the product of its length and width — its two dimensions. In the case of the parallelo- gram,by cutting off the section X and placing it in the posi- tion X', it is evident that the figure is equivalent in area to a rectangle having the same base and an equal altitude. Try this experimentally with a piece of paper. In the case of the triangle, as shown by the figure, the area is equal to one-half the area of the rectangle erected on the same base, and with equal altitude. The area is equal, therefore, to one-half the pro- duct of its base and altitude. PRACTICAL MEASUREMENTS. 147 If a circle is divided as shown, and the parts rearranged as a series of approximate triangles, it is evident that the area will equal the sum of the areas of the triangles. If the triangles are made small enough they have altitudes equal to the radius of the circle. The sum of their bases equals the circumference of the circle. The area of the circle, therefore, equals one- half the product of the radius by the circumference. Since the circumference equals tt X the diameter (2 X radius) the area equals ^RX 2RX 3.1416, or 3.1416 X the square of the radiiLS. The area of irregular right line figures may be obtained by dividing them into triangles and parallelograms and deter- mine the areas of these parts. ORAL EXERCISE. Compute the areas of the following: Figure. Altitude. Base. 1. Triangle 14 ft. 16 ft. 2. Rectangle 24 ft. 2^ ft. 3. Square 15 ft. 4. Parallelogram 16 in. 2 ft. 5. Rectangle 2\ii. 4Ht. 6. Triangle 20 in. 1 ft. 6 in. 7. Parallelogram 6i ft. 2 ft. 3 in. 8. Find the difference between 3 sq. yd. and an area 3 yd. square. 9. How does a 5 in. square compare with an area of 5 sq. in.? 10. Name two factors of 360. Find two dimensions for a rectangle of 360 sq. ft. area. Find other dimensions for a second rectangle of the same 148 BUSINESS ARITHMETIC. area. How many rectangles have the same area? If one dimension is 4' what is the other? If one dimension is 12 ft. what is the other? 11. A series of rectangles have a common area of 480 sq. ft. Find the bases if the altitudes are respectively 4, 16, 12, 6, 8, 15, 20, 40, 1/2 feet. Compare the perimeters. Do rectangles of the same area have the same perimeter? 12. Give the dimensions of several rectangles that have a common area of 1200 sq. in. 13. To what is the area of a triangle equal? If the area and one dimension are known, how is the other determined? 14. Find the altitudes of a series of triangles having a common area of 180 sq. ft. and bases of 4, 3, 20, 9, 18 and 30 ft., respectively. 15. Name the bases and altitudes of ten triangles that have a common area of 720 sq. in. EXERCISE. 1. Determine the area of a floor measuring 25 ft. 4 in. by 16 ft. 8 in. 2. Determine the area of the end section of a metal bar measuring 3j in. by 1| in. 3. How many 3-in. squares may be cut from a piece of card board measuring 25 in. by 36 in.? 4. What is the acreage of a field measuring 720 rd. by 96 rd.? 5. A field measuring 88 rd. by 126 rd. produces 280 bu. of grain, an average of bu. per acre. 6. Subdivide the field repre- sented by the accompanying dia- gram into rectangles, as shown by the dotted lines. Name the dimen- sions of each rectangle. Determine the total area. Check the com- 10/ putation by subdividing into an- other group of geometric figures and determining area. 7. How many oblongs, 2 in. by 3^ in., can be cut from sheets of composition metal 24 in. by 36 in.? Does it make any difference which way the oblongs are laid off on the sheet? 12' 12' 6'- to •o PRACTICAL MEASUREMENTS. 149 80' 8. The diagram here shown rep- resents a park with a circular flower bed. The walk is 5 ft. wide. Find the area of the flower bed. Find the number of square yards of sod- ding required for the parking. How is this area determined from the area of the rectangle and the area of the circle? Compute the cost of constructing the walk, at $1.25 per sq. yd. Com- pute the cost of curbing for the inner and outer edges of the walk, at 38c per linear foot. 9. Find the acreage of the plot of land here represented . Express dimen- sions in linear measure. Check computation of acreage by using dimensions just found. Applications of Square Root. Square root frequently is employed, in connection with geometric figures, in de- termining missing dimen- sions. 1. Squares. Since the area of a square is equal to the product of its two equal sides, it is evident that the length of one side is equal to the square root of the area. 2. Right Triangles. It has been proved, geometrically, that the square of the hypotenuse, or side opposite the right angle, is equal to the sum of the squares of the other two sides. This is illus- trated graphically in the drawing. It is evident, then, that the length of the hypotenuse is equal to the square root 150 BUSINESS ARITHMETIC. of the sum of the squares of the other two sides; and that the length of either other side is equal to the square root of the difference of the squares of the hypotenuse and the known side. 3. Since the area of a circle equals tt X the square of the radius, it is evident that the radius equals the square root of the area divided by 3.1416. EXERCISE. 1. Determine the side of a square having the same area as a rectangle measuring 54 rd. by 17.3 rd. 2. To have an area of 60,000 sq. ft., a square must have an edge of how many feet? 3. Find the edge of a square that has the same area as a circle of 50 ft. radius. 4. Determine the radius of a circle that has the same area as a square of 80 ft. side. 5. How many linear feet of fencing are required for a rectangular chicken run, 80 ft. by 120 ft.? What is the difference in amount of fencing required for a square run of the same area? 6. Compare the perimeters of a square, a rectangle of 20 ft. altitude, and a circle, if all have equal areas of 600 sq. ft. 7. If the two sides of a right triangle are 8 ft. and 16 ft., what is the length of the hypotenuse? Determine the missing values in the figures shown below: 8. 10. PRACTICAL MEASUREMENTS. 151 Gables. The pitch of a roof is the amount of rise of a rafter for each foot in the base of the gable. Illustkation. In the cut, the rafter rises 10 ft. for a base of 20 ft. This is a pitch of 10/20, or 1/2. The familiar Gothic pitch is 5/8. EXERCISE. Fmd the missing values: Base Width. Height. 1. 30 ft. 18 ft. Pitch. ? Length ? of Rafter. (Allow 1 ft. Overhang.) 2. 40 ft. ? Gothic ? (< H 11 3. 45 ft. 20 ft. ? ? " li ft. 4. ? 16 ft. 1/4 ? (t tl tt 5. 36 ft. ? 1/2 ? tt U tl PRACTICAL APPLICATIONS OF SQUARE MEASURE. The following applications are selected to illustrate the use of square measure as modified by business custom. 1. Flooking. Considerable structural work, such as flooring, roofing, tiling, etc., is measured frequently by the square, or 100 sq. ft. More rarely the measurement is expressed directly in sq. ft., or in units of 1000 sq. ft. In properly matching and fitting tongue and groove flooring, there is considerable waste of material. Thus, for every foot 152 BUSINESS ARITHMETIC. of flooring in place, 1^ to 1^ ft. of flooring material may be required. EXERCISE. 1. Compute the cost of flooring a hall measuring 80' by 35', at $2.95 per square. 2. Compute the cost of flooring with hard wood, the two front rooms shown in this floor plan of a bungalow, at $40 per M, allowing 1/3 waste, and $10.50 for extras. DINING ROOM / ^ .o'«'/ 10' X 12'6 BED ROOM JO' X 10 6' -* -r-i b^ 3. How many tile, 6" square, are required for the right rear room, allowing 1/20 extra for breakage? 4. A gable roof, 42' long, and having rafters 21' long, is to be covered with roofing tile. At the rate of 300 per square, how many tile are re- quired? How many shingles are required for the same purpose, at the rate of 800 per square? How many bundles of shingles, 250 each? Note. Carpenters and builders have tables showing the numbers of tile, shingles, etc., of different sizes, req^uired per square. These are often used as a basis of computation in practical work. PRACTICAL MEASUREMENTS. 153 2. Plastering. Plastering is commonly measured by the square yard. Allowances for openings are made in various ways, such as (1) by actual measurement, (2) by arbitrary allowance, etc. If working plans for a building are not given, dimensions of rooms are usually stated in this order: Length — width — height. Example: 21' X 16' X 9' 6''. EXERCISE. 1. Compute the cost of hard finishing the walls and ceiUng of the two front rooms of the building, page 152, at 43c per sq. yd. Height of room 9'. Allow 20 sq. ft. for door openings and 18 sq. ft. for windows. 2. Compute the cost of rough finishing the bedroom and dining room at 26c. per sq. yd., allowing by actual measurement for the doors, which are 1' by 4', and for windows, which are 6' by 4^'. 3. Painting. Painting is usually measured by the square yard, no allow- ance being made for windows or openings of a similar nature. EXERCISE. 1. Compute the cost of painting a fence, 6' high, around the lot shown on page 148, both sides, at a cost of 24c per sq. yd. 2. Compute the cost of painting with two coats the walls of a building 48' long, and having ^-^''^'^N^ this end section. Price for double coat, 45c per sq. yd. 4. Papering. Although dimensions vary, the width of wall paper is usually assumed to be 18 inches. The length is 8 yards for single rolls and 16 yards for double rolls. Complete rolls must be bought. Allowance is made for openings in various ways, such as CEILING PLAN 154 BUSINESS ARITHMETIC. by (1) exact measurement; (2) uniform allowance for each opening; (3) allowance for width of opening, and full height of room, the assumption being that remnants of rolls, after cutting full strips, will be sufficient to paper the wall over and under openings; and (4) an arbitrary allowance according to area, as " one roll to 24 sq. ft. regardless of openings.'* ORAL EXERCISE. 1. If paper is laid on the ceiling represented in the accompanying il- lustration parallel to the shorter dimension, what is the length of each strip in yards? How many strips can be cut from each 24 FT. double roll? How many strips are needed? How many rolls? What is the cost, at 45c per roll? 2. The walls of the room, the ceiling of which is shown, are also to be papered. What does the border cost, at 15c per yd? The strips of papering on the walls must be 8 ft. long. How many strips are required? How many strips can be cut from each double roll? How many rolls are needed? What is the cost, at 50c per roll? It is evident that the simplest method of approximation is to divide the perime- ter of the room, less the width of all openings, by the width of a strip, and to divide the number of strips by the number of complete strips in one roll, in order to determine the number of rolls. EXERCISE. 1. A room 30' by 28' has five openings, each 2\' wide, and one opening 4' wide. It is 9' from baseboard to ceiling. Find the cost of papering sides and ceiling, without border, at 55c per roll, allowing 6" per strip for matching design. 2. Compute the quantity of ceiling, border and wall paper required for your class room. 3. Find the quantity of wall, ceiling and border paper required for each of the rooms shown in the diagram on page 152, assuming the wall height, allowing for border, as 9', and allowing 6" per strip for matching design. Assume width of doors 3'; of windows, 2\'. PRACTICAL MEASUREMENTS. 155 5. Carpeting. Carpeting is measured by the yard 'per length of strip, regardless of width. Oil cloth, linoleum and carpet linings are sometimes measured by the square yard. In order to compute the quantity of carpet for a room, it is necessary to determine the number of strips and their length, somewhat after the manner of the computations for papering. The number of strips depends on the direction in which the strips are laid. Allowance must be made, also, for matching design. Illustration. Compute the quan- tity of carpet required for the room shown, if laid lengthwise, the width of the carpet being three feet. Solution. 19-T-3 = 6^, no. of strips. Therefore, 7 strips are required. 24' = length of one strip = 8 yd. 7X8 =72, no. of yd. required. Note. Since each strip is 3' wide, the total width of 19' divided by 3 will give the number of strips, a full strip being purchased for each fraction of width. EXERCISE. 1. Find the quantity of carpet required for the room shown above if laid across the shorter dimension? 2. Compute the cost of Brussels carpet, 27" wide, for the above room, laid as shown, at $1.50 per sq. yd., allowing 6" per strip, after the first strip, for matching. Why need no allowance be made for matching the first strip? 3. The above robm is to be carpeted with Ingrain carpet, 1 yd. wide, laid lengthwise, and with a border around the room 2' wide. Determine the quantity of border and carpet, allowing 1/20 for matching. Compute the number of square yards of lining required. 4. Find the cost of carpeting the dining room (page 152) with linoleum, 1^ yd. wide, at $1.45 per sq. yd., allowing 2 sq. yd. for matching design. 5. Compute the cost of carpeting each of the rooms with Wilton carpet, 3/4 yd. wide, at $1.25 per yd. Allow 9" for matching design, and allow for an 18" border, at $1 35 per yd. Lay the carpet in the direction that will be more economical. 156 BUSINESS ARITHMETIC. Geometric Conceptions — Solids. A solid is that which has three dimensions, length, width and thickness. It is bounded by surfaces. A polyhedron is a solid bounded by plane surfaces called faces. A poly- hedron having two parallel and equal bases, and three or more sides which are parallelograms, is a prism. A prism is a rectangular solid (A) if it is bounded by six rectangular sur- faces. It is a cube if these surfaces are all squares. A pyramid (B) is a polyhedron whose faces, with one exception, meet in a common point called the vertex. The faces of a pyramid are therefore triangles. A cylinder (C) is a solid bounded by two eqUal parallel circles and a uniformly curved surface. A conical surface is a surface generated by the motion of a straight line that continu- ally intersects a fixed curve, frequently a cir- cle, and passed through a fixed point. A cone (D) is a solid bounded by a closed conical sur- face and a plane surface. A spherical surface is a curved surface every PRACTICAL MEASUREMENTS. 157 point of which is equidistant from a point called the center. A sphere (E) is a solid bounded by a spher- ical surface. A hemisphere is one- half of a sphere cut by a plane pass- ing through the center. INTRODUCTORY EXERCISE. A cubic foot is a solid 1 foot long, 1 foot wide and 1 foot thick. How many cubic feet in figure (6) if each edge is 1 foot long? How many cubic feet in each horizontal row of (c)? Hovv' many rows? How many cubic feet in all? How many rows and units per row in one vertical slice of (d)? How many sections? How many units in all ? What is the area of the end face ? Of the top 2 What is the total area of all the faces? It is evident that the volume of a rectangular solid is equal to the product of its three dimensions. These dimensions must ordinarily be expressed in units of the same denomi- nation. The total surface area is equal, evidently, to the sum of the areas of the individual faces. ORAL EXERCISE. Compute the cubic contents, or volume, of: 1. A cube of 6" edge. 2. A rectangular solid measuring 6' by 5' by 8'. 3. A rectangular solid measuring 8' by 4^' by 10'. 4. An excavation, 10' by 20' by 30'. 5. A piece of metal 6" wide, 12" thick, and 10' long. 158 BUSINESS ARITHMETIC. EXERCISE. 1. Compute the number of cubic yards of earth that must be ex- cavated to sink this cellar to a depth of 8'. Note. Find area and multiply by 32 FT. depth. Check, by dividing into separate prisms as shown by dotted lines. Why do these processes bring the same result? 2. Determine the number of cubic feet in a stone column 4^' by 2' by 18'. 3. Compute the cost of a masonry wall, 40 long, 6' high and 2' wide, at $4.36 per cu. yd. 4. Determine the cubic contents of a room 60' by 25' by 10'. Determine the surface area of walls, ceiling and floor. 5. A school room, 36' by 30' by 10', seats 42 people. How many cubic feet of air space is allowed each person? 6. At the rate of 22 brick per cu. ft., compute the cost at $2 per M. of the bricks necessary for a wall 36' long, 20' high and 24" wide for half the height and 18" wide for the remaining height. 20 FT. u. vo 12 FT. ORAL EXERCISE. 1. Find the cubic contents of this rectangular soUd? effect on the volume of doubling any one dimension? 2. If the volume of a rectangular solid is 180 cu. in., and the product of two dimensions is 30 sq. in., what is the third dimension? 3. If two dimensions are 8' and 12', what is the third dimension of a solid having a volume of 960 cu. ft.? Note. It is evident that the third dimension may be determined by divi- ding the volume by the product of the two known dimensions. What is the 8 FT. EXERCISE. 1. 8' X 4.5' X ? = 720 cu. ft. 2. Select two dimensions to be used with a height of 9', to determine PRACTICAL MEASUREMENTS. 159 a rectangular solid of 120 cu. ft. of volume. How many solutions are possible? 3. Name three dimensions of a solid of a cubic content of 5600 cu. in. 4. Determine the height of a room 30' long and 20' 6" wide, if it is to have a cubic content of 5535 cu. ft. The Cylinder and Sphere. If the surface of a paper cylinder is cut along its length, and the paper rolled back, it will be seen to have the form of a rectangle whose height is the altitude of the cylinder and whose base is the length of the circumference of the base circle. Evidently, the lateral area is the product of the altitude by the circumference. To this must be added, to determine the entire area, the areas of the two base circles. ORAL EXERCISE. 1. If the area of the base of a rectangular solid is 14 sq. ft., what is the volume per foot of height? 2. If the area of the circular base of a cylinder is 30 sq. in., what is the volume per foot of height? What is the volume if the height is 12 ft.? As seems evident from the above, the volume of a cylin- der equals the product of the altitude and the area of the base. EXERCISE. 1. Allowing for 1^" overlap, how much sheet metal is required for the lateral surface of a hollow cylinder of 16' altitude and 3' radius of base? 2. Determine the cubic contents of this cylinder. 3. What must be the altitude of a cylinder of 4' diameter to have a cubic content of 600 cu. ft.? 4. How many cu. ft. of water will a circular cistern of 12' depth and 5' radius contain? 5. Determine reasonable dimensions for a cistern to contain 5000 cu. ft. 6. Metal pieces for the lateral surfaces of cans of 2" radius and 6" height are to be cut from sheet metal 24" by 32", no allowance being made for overlap? How many can be cut from each sheet? 160 BUSINESS ARITHMETIC. The surface of a sphere is equal to ^ X tt X ^^^ square of the radius; and the volume of the sphere is equal to 4/3 X ir X the cube of the radius, EXERCISE. 1. Find the surfaces of spheres of radii (a) 2"; (6) 6"; (c) 21''. 2. Find the volume of spheres of radii (a) 5"; (6) 15"; (c) 26' 6". 3. Find the volume of a hemisphere of .4' 6" radius. 4. Find the radius of a sphere of 750 sq. in. of surface. 5. How many square inches of gilding must be done to cover a sphere of 16" radius? SPECIAL APPLICATIONS. Cord Measure. Fuel wood, tan bark, sometimes stone, etc., are measured by the cord, a unit pile 8' long, 4' wide and 4' high, containing 128 cu. ft. In many localities, a cord of fuel is a pile 8' long and 4' high, the price varying with the width — as half sawed, quartered sawed, etc. It is evident that if a pile is 4' high and 4' wide, the number of cords may be determined from the length, as measured by the unit length of 8'. Thus a 36' pile contains 4^ cords. In the case of irregular dimensions, the measure is the number of cubic feet in a cord. EXERCISE. 1. A wood lot produces piles of wood of regulation 4' length (width), of dimensions as follows: 10' long, 3' high; 12' long, 5' high; 16' long, 4' high; 20' long, 4' high. How many cords are produced? 2. A pile of hemlock bark measures 80' by 4' by 21' and contains how many cords? 3. Sixteen acres of woodland produce a pile of cord wood 12' high and 47' long. What is the product in cords per acre? 4. Five cords of wood, regulation size, are to be piled in a shed along a wall measuring 15'. How high should the pile be? Lumber. In the measurement of boards of less than 1" in thickness, square measure of surface is used. In the measurement of PRACTICAL MEASUREMENTS. 161 boards of 1" thickness, or over, such as heavy boarding, planks, joists, scantUng, and heavy dressed timber, the unit measure is the hoard foot. The hoard foot is a hoard 1 ' long, 12" wide and 1" thick , or its equivalent. Since the end area is 12 sq. in., the board foot may be said to have a length of 1' and an end area of 12 sq. in. Board Feet. In measuring width, except in the more valu- able woods, the next lower quarter inch of width is sometimes taken. Thus a 6|" width is reckoned 6f and a ^\" width, 9'', etc. In stating quantity and dimensions of an order, or esti- mate, the arrangement is: Number of pieces — thickness in inches — width in inches — length in feet, thus 30 pc, 6" X 8" — 16'. The price is generally stated per M feet, board measure. In computation, both on paper and orally, the method of cancellation is valuable. Illustration. Compute the number of feet, board measure, in 36 pc, 2\" X 10" —16'. Solution. 3 2| X 10 ^ no. bd. ft. per ft. of = 1200, no. bd. ft. 12 l^"^^^' This no. X length X no. pieces = no. bd. ft. 21X10X16X ^6^ 1 Evidently the number of board feet is the product of the width in inches, hy the thickness in inches, hy the length in feet, by the number of pieces, divided hy 12. The quantity to cancel is always 12. Notice that this is one of the rare cases where dimensions are not expressed in the same unit. 12 162 BUSINESS ARITHMETIC. ORAL EXERCISE. Determine the number of board feet per foot of length in timbers of the following dimensions : 1. 13"X4". 4. 2"X9". 7. 3"X6". 10. 3"X9". 2. 8"X16". 5. 2"X8^ 8. 2h"X&'. 11. 2"X3". 3. 4"X12". 6. 4"X6". 9. 6"X9". 12. 2"X2". Determine the number of board feet in these timbers: 13. 3"X4"— 16'. 16. 4"X8"— 12'. 14. 4"X6"— 12'. 17. 2"X9"— 16'. 15. 8"X10"— 6'. 18. 3"X6"— 12'. Determine the board feet in these items: 19. lOpc. 2"X6"— 10'. 20. 20pc., 3"X4"— 12'. EXERCISE. Determine the number of feet, board measure, in: 1. 162 pc. hemlock, S"X10"— 32'. 2. 40 pc. white pine, 2" X 10"— 20'. 3. 360 pc. maple, 1|"X3"— 18'. 4. 456pc. oak, 3"X6"— 18'. At $26 per M., compute the cost of: 5. 150 joists, 2"X8"— 16'. 6. 42 beams, 6"X9"— 24'. 7. 78 scantling, 2"X4"— 16'. 8. How much timber is required for a plain board fence around a lot 180' by 246', if it consists of two strips of 2"X4" scantling on which is nailed 1" boarding 6' high. This does not include posts. Allow 1/8 for waste in cutting. Capacity of Bins, Tanks, Etc. In measuring the capacity of bins and tanks, it is often necessary to express the common dry and liquid measures in terms of cubic feet. Illustrations. (1) Determine the capacity in gallons of a rectangular iron tank measuring 4' by 8' by 3'. Solution. (Abstract.) 4'X8'X3' =96 cu. ft., capacity. 96 X 1728 = capacity in cu. in. PRACTICAL MEASUREMENTS 163 ^^oqV^^ =^o. of gal. (since 1 gal. =231 cu. in.) =718+ gal., capacity. (2) What is the weight of the water the tank will hold? Solution, 96 cu. ft. = contents. 1 cu. ft. of water weighs 62^ lb. 96X621 lb. =6,000 lb., the weight of water. EXERCISE. 1. Determine the capacity in bushels of a rectangular bin measuring 8' by 4' by 4'. Determine the capacity, also, by approximate measure (1 bu. = li cu. ft.). 2. A bin in a certain bam can be allowed a floor space of only 4' by 9' 6". What must be its height if it is to have a capacity of 250 bu.? (Approximate measure.) 3. Determine reasonable dimensions for a bin to contain 400 bu. of potatoes. Use 1 c\i. ft. as equivalent to .63 of a heaped measure bushel. 4. In order to have a capacity of 1,000,000 gal., a reservoir must have a capacity of how many cu. ft.? 5. Compute the capacity, in gallons, of: (a) A rectangular tank, 4' X 12' X3' 6". (b) A cylindrical tank of 6' diameter and 8' high. (c) A cylindrical cistern of 2' 6" radius and 16' deep. (d) A vat 6' square and 4' 6" high. 6. A metal tank, known to have a capacity of 60,000 gal., will hold for temporary storage how many bushels? 7. Construct a table to give the number of standard gallons in volumes from 1 to 1000 cubic feet. SPECIAL INDIVIDUAL PROBLEMS. 1. Estimate the quantity and cost of materials for a cheap board fence to enclose a vacant lot in your neighborhood. Prepare a plot of the land and a diagram showing character of fence. 2. Prepare plans and estimates for a modem earth croquet court (or tennis court) to be constructed on a vacant lot in your neighborhood. 3. Report on the total number of square feet of lighting surface in the school building. Tabulate results (1) by rooms, (2) by floors, (3) by com- 4. Report on the number of cubic feet of air space in the building, by rooms, corridors and floors. CHAPTER XXIV. MEASUREMENT OF TIME. 1. Longitude and Time. Longitude and time are matters of common interest. They are also fundamental to navigation, astronomy and geography. Nortli Pole Cape Town South Pole A meridian is an imaginary line on the surface of the earth, extending due north and south and terminating in the poles. It is therefore a semi-circumference of the earth. Any me- ridian used as a reference Hne, from which to locate places on 164 MEASUREMENT OF TIME. 165 the earth's surface by determining their distances east or west of it, is a 'prime meridian. It is now the common custom to use the meridian through the Royal Observatory at Green- wich, England, as a prime meridian. Distances east or west of the prime meridian are termed respectively east longitude and west longitude. They are measured by degrees, minutes and seconds of circular measure, as shown on page 164, where the meridians intersect the equator. Each place has its own meridian, but two places may have the same meridian even when thousands of miles apart. Longi- tude west is designated by the symbol ' +,' and longitude east by the symbol ^ — .' Thus the longitude of New York, which is west of Greenwich is +73° 58' 25.5'', while the longi- tude of. Brussels, east of Greenwich, is — 4° 22' 9". From a study of the diagram, it is evident that the difference in longitude of two places may be obtained by subtraction, if the signs of the individual longitudes are the same; and by addition if they are different. Since the earth revolves on its axis once in twenty-four hours, every point on the surface revolves in a circle, or passes through 360° of circular measure, in that time. Hence there is a close relation between time and degrees of circular measure. Since 360° correspond to 24 hours. 1° is equivalent to 1/360 of 24 hr., or 4 min. 1' is equivalent to 1/60 of 4 min., or 4 sec. 1" is equivalent to 1/60 of 4 sec, or 1/15 sec. Since 24 hours correspond to 360°, 1 hr. is equivalent to 1/24 of 360°, or 15°. 1 min. is equivalent to 1/60 of 15°, or 15'. 1 sec. is equivalent to 1/60 of 15', or 15". Regarding the sun as stationary with reference to the earth, the earth, revolving from west to east, carries the meridians under the sun in succession, the latter seeming to move toward the west. Since all meridians pass under the sun in suc- cession each twenty-four hours, they really mark, by differ- 166 BUSINESS ARITHMETIC. ence in longitude, the difference in time of places on the earth's surface. In science, difference of longitude is often expressed in time. Local time is determined at each place by calling the instant the sun crosses its meridian 12:00 noon. Illustration. If the difiference in longitude of two places is 84° 30', what is the difiference in time? Solution. 84° = 84 X 1/15 hr. Since 1° corresponds to 1/15 hr. = 5fhr. =5hr. 36min. 30' = 30X4 sec. = 2 Since 1' corresponds to 4 sec. 84° 30' =5 hr. 38 sec, difiference in time. EXERCISE. 1. Do points in the east or west of a country receive sunlight first? Which have earlier local time? 2. If two places have different west longitudes, which has earlier time? Which of two places having east longitude has the earlier local time? Evidently the more western points have the later time; the more eastern the earlier. Since time bears an intimate relation to degrees of longitude, it is a simple matter to com- pute one from the other. Illustrations. (1) The longitude of Albany is 73° 44' 48", and of Ann Arbor, Mich., 83° 43' 48". Determine their difiference in time. Solution. 83° 43' 48", long, of Ann Arbor, 73° 44' 48", long, of Albany, 9° 59' = difiference in longitude. 9X1/15 hr. =3/5 hr., or 36 min. 5dX4sec. =236 sec, o r 3 min. 56 sec. 9° 59' = 39 min. 56 sec, difif. in time. (2) The difiference in the time of two places is 6 hr. 18 min. 12 sec Determine the difiference in longitude. Solution. 6 hr. 18 min. 12 sec = difiference in time. 6hr. = 6X15° =90° 18 min. = 18X15' = 4° 30' 12 sec =12X15" = 3' 6 hr. 18 min. 12 sec. =94° 33', difiference in longitude. Athens, -23° 43' 55.5". Berlin, -13° 23' 43.5". Chicago, +87° 36' 42.0". New York, , +74° 0' 3.0". Paris, - 2° 20' 15.0". MEASUREMENT OF TIME. 167 Note. Both these solutions are based directly on the tables of com- parative values given on page 165. Longitude of Well-Known Places. Philadelphia, +75° 9' 45.0". Rome, -12° 27' 14.0". San Francisco, +122° 25' 40.8". Tokyo, -139° 42' 30". EXERCISE. 1. If the difference in longitude of two places is 71° 18' 0", what is the difference in time? 2. Compute the difference in local time between (a) Berlin and Phila- delphia; (6) Chicago and San Francisco; (c) Rome and Paris, and (d) Chicago and Tokyo. 3. A man coming to Chicago found that his watch, set by local time at his starting point, was 48 minutes slow by local Chicago time. Did he come from east or west? From what longitude? 4. When it is 5:00 P. M. at Chicago, it is what time at (a) San Fran- cisco, (6) Berlin, (c) Rome and (d) New York ? 5. A ship sailing in longitude 183° 40' 9" is how far in difference of longitude from the port of New York. Near what great body of land is the ship? The International Date Line has been selected as the 180th meridian, with slight changes at certain points. This meridian in the Pacific Ocean passes through no important land areas. Going westward ships set their calendars forward one day at this point; going east they set them back one day. Standard Time. If every place used its strict local time, it is evident that endless confusion would result in travel, especially from difficulties in constructing time tables. On this account the railways of the United States and Canada agreed on four time belts for the two countries. The belts are approximately 15° wide, and each belt takes the time of its center meridian. Thus the guide meridians are 15°, or one hour apart. The belts are the Eastern, Central, Moun- tain and Pacific, the meridians being +75°, +90°, +105° and 168 BUSINESS ARITHMETIC. + 120°. Thus, when it is 10:00 A.M. in any town in the Eastern time belt, it is 9:00 A.M. in the Central belt, 8:00 A.M. in the Mountain belt and 7:00 A.M. in the Pacific belt. Foreign countries are adopting similar belts, based largely on meridians at 15° intervals from Greenwich. 2. Commercial Time. The accurate computation of time between dates, and of dates of maturity, is of vital importance in business. All interest charges depend on time. Many business and com- mercial papers run for fixed times to dates of maturity. Considerable merchandise is sold on limited time. Failure to correctly compute a payment date may cause a business man to lose standing or credit, by being caught unprepared to meet just demands. Many seemingly minor details affect time and date com- putation. Time may be computed either approximately or by exact days, and dates of legal maturity are affected by varying state laws affecting holidays and Sundays (see page 312). I. Time Between Dates. Since the common business time is six months, or less, it is possible to make most time computations without use of paper. Where paper is used, the method is often different. (a) Approximate Time. (Paper.) Illustration. Find the time between June 21, 1908, and Jan. 17, 1910. Solution. Follow the method of denomi- nate numbers. In the subtrahend, 1 mo. is reduced to days, making 30+17, or 47 da.; 1 yr. 6 mo. 26 da. and 1 yr. is reduced to 12 mo. Note. In reduction, one month is considered 30 days, but if a given date contains the thirty-first day it is retained. Yr. Mo. Da. 1910 1 17 1908 6 21 MEASUREMENT OF TIME. 169 EXERCISE. Find the time between: 1. Jan. 27, 1906, and Aug. 29, 1909. 4. 3/ 3/ '07 and 6/ I/'IO. 2. Oct. 21, 1907, and Dec. 15, 1912. 5. 7/ 5/ '06 and 10/ 17/ '06. 3. Jan. 31, 1908, and Sept. 12, 1909. 6. 3/ 21/ '07 and 5/ 18/ '09. 7. Money borrowed Apr. 16, 1907, and due to-day, has drawn interest for how long? 8. I am given until Oct. 1 to pay a debt due Feb. 11. For how long is time of payment extended? (b) Approximate Time. (Mental.) Illustrations. (1) Find the time between Mar. 12, 1906, and May 17, 1908. Solution. Reason as follows: From Mar. 12, 1906, to Mar. 12, 1908, 2 years; from Mar. 12, 1908. to May 12, 2 months; from May 12 to May 17, 5 days. Total time, 2 yr. 2 mo. 5 da. Note. The process involves adding to the earlier date, on the principle used in making change. (2) Find the time between Aug. 28, 1913, and July 12, 1914. Solution. Aug. 28 (8th mo.) to June 28 (6th mo.) = 10 mo. June 28 to July 12 = 2 + 12 da. = 14 da. Total time, 10 mo. 14 da. EXERCISE. Compute the time between : 1. Feb. 11 and Apr. 29. 5. Mar. 16, 1912, and Dec. 15, 1913. 2. June 16 and Dec. 31. 6. Feb. 28, 1912, and Nov. 29, 1912. 3. May 31 and Nov. 24. 7. Aug. 13 and to-day. 4. Mar. 17 and Oct. 9. 8. May 19 and to-day. Exact Time. In computing by exact time, the true number of days in each month is taken into account. Usually only one of the extreme days is included, although some bankers include both. Exact time is computed for periods less than a year. Longer terms are reckoned in calendar years and exact days. (c) Exact Time. (By Table.) The following table shows the calendar year and the day of the year of each calendar day. Thus Feb. 17 is the 48th day of the year. Other forms of tables are very common. 170 BUSINESS ARITHMETIC. Table FOR THE Calculation of Time Jan. 1 1 Feb. 1 32 Mar. 1 60 Apr. 1 91 May 1 121 June 1 152 2 2 2 33 2 61 2 92 2 122 2 153 3 3 3 34 3 62 3 93 3 123 3 154 4 4 4 35 4 63 4 94 4 124 4 155 5 5 5 36 5 64 5 95 5 125 5 156 6 6 6 37 6 65 6 96 6 126 6 157 7 7 7 38 7 66 7 97 7 127 7 158 8 8 8 39 8 67 8 98 8 128 8 159 9 9 9 40 9 68 9 99 9 129 9 160 10 10 10 41 10 69 10 100 10 130 10 161 11 11 11 42 11 70 11 101 11 131 11 162 12 12 12 43 12 71 12 102 12 132 12 163 13 13 13 44 13 72 13 103 13 133 13 164 14 14 14 45 14 73 14 104 14 134 14 165 15 15 15 46 15 74 15 105 15 135 15 166 16 16 16 47 16 75 16 106 16 136 16 167 17 17 17 48 17 76 17 107 17 137 17 168 18 18 18 49 18 77 * 18 108 18 138 18 169 19 19 19 50 19 78 19 109 19 139 19 170 20 20 20 51 20 79 20 110 20 140 20 171 21 21 21 52 21 80 21 111 21 141 21 172 22 22 22 53 22 81 22 112 22 142 22 173 23 23 23 54 23 82 23 113 23 143 23 174 24 24 24 55 24 83 24 114 24 144 24 175 25 25 25 56 25 84 25 115 25 145 25 176 26 26 26 57 26 85 26 116 26 146 26 177 27 27 27 58 27 86 27 117 27 147 ^ 27 178 28 28 28 59 28 87 28 118 28 148 28 179 29 29 — — 29 88 29 119 29 149 29 180 30 30 — — 30 89 30 120 30 150 30 181 31 31 — — 31 90 — — 31 151 Illustration. (1) The time from Aug. 17 to Dec. 21 is ? days. Solution. From table. Dec. 21= 355th da. Aug. 17 = 229th da. The difference equals elapsed time =126 da. (2) Nov. 11 to Jan. 26= ? da. Solution (a). Nov. 15 is 315th day. 365 da. -315 da. = 50 da. Jan. 26 is 26th day of next year 26 Total time 76 da. Solution (6). The complementary time between Nov. 11 and Jan. 26 is the time between Jan. 26 and Nov. 11. Nov. 11= 315th da. Jan. 26= 26th da . Complementary elapsed time =289 da. True elapsed time =365 da. -289 da. = 76 da. MEASUREMENT OF TIME. 171 Table for the Calculation of Time. July 182 Aug. 1 213 183 2 214 184 3 215 185 4 216 186 5 217 187 6 218 188 7 219 189 8 220 190 9 221 191 10 222 192 11 223 193 12 224 194 13 225 195 14 226 196 15 227 197 16 228 198 17 229 199 18 230 200 19 231 201 20 232 202 21 233 203 22 234 204 23 235 205 24 236 206 25 237 207 26 238 208 27 239 209 28 240 210 29 241 211 30 242 212 31 243 Sept. 1 2 3 4 6 7 8 9 10 11 12 13 14 15 16 17 18 19 20 21 22 23 24 25 26 27 28 29 30 244 Oct. 1 274 Nov. 1 305 Dec. 1 245 2 275 2 306 2 246 3 276 3 307 3 247 4 277 4 308 4 248 5 278 5 309 5 249 6 279 6 310 6 250 7 280 7 311 7 251 8 281 8 312 8 252 9 282 9 313 9 253 10 283 10 314 10 254 11 284 11 315 11 255 12 285 12 316 12 256 13 286 13 317 13 257 14 287 14 318 14 258 15 288 15 319 15 259 16 289 16 320 16 260 17 290 17 321 17 261 18 291 18 322 18 262 19 292 19 323 19 263 20 293 20 324 20 264 21 294 21 325 21 265 22 295 22 326 22 266 23 296 23 327 23 267 24 297 24 328 24 268 25 298 25 329 25 269 26 299 26 330 26 270 27 300 27 331 27 271 28 301 28 332 28 272 29 302 29 333 29 273 30 303 30 334 30 — 31 304 — — 31 335 336 337 338 339 340 341 342 343 344 345 346 347 348 349 350 351 352 353 354 355 356 357 358 359 360 361 362 363 364 365 EXERCISE. Compute the time between ; 1. Feb. 11 and Oct. 16. 2. Mar. 19 and Nov. 10. 3. Jan. 15 and Oct. 29. 4. Apr. 24 and Dec. 27. 5. Feb. 28 and Aug. 7. 6. July 15 and Oct. 1. 7. Mar. 27, 1910, and Oct. 16, 1911. 8. Apr. 14, 1909, and Nov. 22, 1911. 9. May 29, 1910, and Dec. 19, 1911. 10. Nov. 16, 1912, and Apr. 26, 1914. 11. Show how to use the table in leap years. (d) Exact Time. (Without Tables.) Illustrations. Find the time between Aug. 18 and Oct. 27. 172 BUSINESS ARITHMETIC. Solution (a). Addition method. Aug. 18 to 31 = 13 da. Sept. 1 to 30 = 30 Oct. 1 to 7 = 27 Total time =70 da. Note. Mentally, each group is added to the previous total, as found. One would say 13, 43, 70. Solution (&). From approximate time. Reason as follows: Aug. 18 to Oct. 27, 2 mo. 9 da., or 69 da. But this period includes a 31st day in August. Therefore the exact time is 69 da. + 1 da., or 70 da. Note. Since February has 28 days, this method involves subtraction when the period includes the last of this month. EXERCISE. Find the exact number of days from : 1. Aug. 28 to Dec. 29. 6. Jan. 5 to Mar. 8. 2. Sept. 19, 1907, to Jan. 25, 1908. 7. Nov. 11 to date. 3. Oct. 1 to Jan. 13. 8. Oct. 16 to date. 4. Aug. 12 to Dec. 3. 9. Feb. 21 to June 9. 5. Mar. 28 to Apr. 14. 10. Mar. 3 to Aug. 2. II. Dates of Matukity. In computing final date? and dates of maturity, it is neces- sary to know the initial date and the "term" or period of time. If the term is stated in months and days, approximate methods are used; if stated in days, as "90 days," exact methods are used. "Day" terms are seldom over 90 days. (a) Approximate Method. Illustration. A payment due 1 yr. 7 mo. 19 da. from Aug. 11, 1912, is due on what date? Solution. (Paper.) Write in denominate form. 1912 8 11 (Add and reduce upward.) 1 7 19 1914 3 30 Date due, Mar. 30, 1914. Solution. (Mental.) Reason as follows: Aug. 11, 1912, + 1 yr.=Aug. 11, 1913; Aug. 11, 1913,+7 mo. = Mar. 11, 1914; Mar. 11 + 19 da. = Mar. 30, 1914. MEASUREMENT OF TIME. 173 EXERCISE. Find the final date. Initial Date. Term. 1. Jan. 27, 1907 1 yr. 3 mo. 18 da. 2. Feb. 16, 1913 2 yr. 5 mo. 11 da. 3. Mar. 9, 1908 1 yr. 11 mo. 3 da. 4. Apr. 14, 1908 2 yr. 5 mo. 29 da. 5. May 30, 1908 1 yr. 2 mo. 11 da. 6. Jmie 14, 1912 9 mo. 19 da. 7. Aug. 21, 1908 1 yr. 5 mo. 25 da. 8. Sept. 4, 1913 2 yr. 5 mo. 6 da. ORAL EXERCISE. 1. What date is 1 mo. from Jan. 28? From Jan. 29? Jan. 30? Feb. 1? Mar. 30? Apr. 6? June 20? Aug. 31? Note. If there is no corresponding date, the last date in the month is taken. Thus, 1 mo. from Oct. 31 is Nov. 30. 2. Mar. 6+lmo. = ? • 6. Nov. 30+3 mo. =? 3. Apr. 20+3 mo. =? 7. Dec. 31+2 mo. = ? 4. June 30+4 mo.=? 8. Dec. 31+6 mo. = ? 5. July 31+4 mo. =? 9. July 14+8 mo. = ? (b) Exact Method. {By Table, pp. 170, 171.) ORAL EXERCISE. 1. What day of the year is June 14? What day of the year is 60 days later? What date? 2. Find a date 90 days in advance of June 29. {Solution. June 29 = 180th day. 180 da. +90 da. =270 da. By table, the 270th day is Sept. 27.) Find the final dates. Initial Date. Term. Initial Date. Term. 3. June 26 • 90 da. 7. Oct. 14 125 da. 4. Aug. 31 90 da. 8. June 11 45 da. 5. Jan. 30 30 da. 9. July 29 40 da. 6. Feb. 26 60 da. 10. Aug. 27 165 da. 174 BUSINESS ARITHMETIC. (c) Exact (Periodic) Method. ORAL EXERCISE. 1. What date comes 30 days after Jan. 31? After Mar. 31? After April 20? After July 16? 2. When, for periods of 30 days, is the final date the corresponding date of the proper month? When is it earlier? When later? 3. Find the date of maturity of a paper running 90 days and dated Jan. 26. Solution. Jan. 26+30 da. = Feb. 25, since Jan. has 31 da.; Feb. 25 +30 da. = Mar. 27. (Why?) Mar. 27+30 da. =Apr. 26. Therefore, the paper is due Apr. 26. Find the dates of maturity. Initial Date. Term. Initial Date. Term. 4. Jan. 29 60 da. 8. June 27 90 da. 5. Feb. 17 90 da. 9. Aug. 14 75 da. 6. Apr. 30 60 da. 10. Sept. 19 60 da. 7. May 19 40 da. 11. Nov. 30 60 da. INDIVIDUAL ORIGINAL WORK. 1. Report on "The Arithmetic of Time Table Construction." Give illustrations. Give some idea of the character of information obtainable from time tables. 2. Prepare a time table and car schedule for a double track street railway. Estimate the number of motors, trailers, and the working force necessary for efficient service. Length of trip 40 minutes from 6 A.M. to 9 P.M.; 35 minutes from 9 P.M. to 6 A.M. 3 minute schedule, 7 to 10 A.M.; 3 to 7 P.M.; 5 minute schedule, 6 to 7 A.M.; 10 A.M. to 3 P.M.; 7 to 10 P.M. 8 minute schedule,' 10 to 12 P.M. 30 minute schedule, 12 P.M. to 6 A.M. 3. You are to leave Washington on April 30 for a tour of the following places to look after business interests for me: Boston, Portland, Me., Albany, Buffalo, New York, Cincinnati, Memphis, St. Louis, New Orleans? Charleston, S. C, Charlestown, W. Va., and Minneapolis. Examine railway time tables and submit detailed report showng routes selected ^ provisional time table, etc. If possible make an estimate of expense. Plan for at least ten business hours in Boston, New York, Cincinnati, and St. Louis, eight hours in Minneapolis, and at least four hours in each of the other places. BOILING POINT OF WATER +80° CHAPTER XXV. PRACTICAL MEASUREMENTS— Coniinwed 1. Measure of Temperature. Ordinary temperatures are measured by thermometers, graduated on one of three scales: (1) the Fahrenheit, commonly used in business and private life; (2) the Centigrade , used in scientific work, and in countries employing the metric system and (3) the Reaumur, used on the continent of Europe, especially in Germany. The common thermometer, con- sisting of its column of mercury in a glass tube, cannot be used for extreme high temperatures. For such temperatures, especially those — . over 500°, instruments of many varied type5 are constructed. Sev- eral, for example, measure tem- perature by the relative expansion of two metals, such as copper and iron. The level at which the mercury stands, when the thermo- meter is in contact with melting ice is marked 0° on the Cen- tigrade and Reaumur scales, and '+32° on the Fahrenheit scale. When in contact with water commencing to boil, the mercury level is marked +80°, Reaumur; +100°, Centigrade; and +212°, Fahrenheit. These limits being determined, the degrees of the scale are uniformly marked off. Degrees below 0° are marked " -," those above " +." t ORAL EXERCISE. 1. Name the number of degrees between freezing and boiling point on each scale. 175 176 BUSINESS ARITHMETIC. 2. Rank the individual degrees in order of size. 3. 1° R. is what part of 1° F.? Of V C? 4. rF.iswhatpartofrC? OfTR.? 6. 1° C. is what part of 1° R.? Of 1° F.? Scale Reduction. Owing to the lack of uniformity on the use of scales, the problem of reduction from one to another is very common. Illustrations. (1) 40''F. = ?° c. (2) 12° F. = ?°C. Solution. Solution. '.' 180° F. = 100° C, between limits. 32° -12° = 20° F., distance below rF.-^^^g^Jc.,or5/9°C. 0°C. 20° F. = -20X5/9° C. 40° F. = 40° -32° above zero, or 8° = -11.11° C. above zero. (4) -5°C. = ?°F. 8 X 5/9° = 4.44 + ° C. Solution. Solution. 5°C. = 5X9/5°F.= 9°F. '.- 100° C. = 180° F. But 0°C.=32°F. 1° C. = 9/5° F. .-. -5°C.=32°-9°F.=23°F. 25° C. =25x9/5° F. ; = 45° F., above freezing. But 0° C. = 32° F. .'. 25° C. = 45° F. + 32° F. = 77° F. EXERCISE. Express each of the following temperature reading on two other scales: 1. +196° F. 5. +18.6° F. 9. +182° C. 13. +67.2° R. 2. +108° F. 6. -18° C. 10. +46.5° C. 14. -50° R. 3. +64° F. 7. +64° C. 11. +18° R. 15. +112° R. 4. -13° F. 8. -30° F. 12. -66° C. 16. +100**R. 17. Express the following table in Centigrade scale: Approved Cold Storage Temperatures. Degrees Fahrenheit. Beef 36 to 40. Pork 29 to 32. Lamb, mutton 32 to 36. Fish 25 to 28. Oysters 33 to 45. Fruits 32 to 36. Poultry, frozen 25 to 30. Poultry, to freeze 5 to 10. Eggs 32 to 34. Butter 32 to 38. Vegetables 34 to 40. Canned goods 38 to 40. PRACTICAL MEASUREMENTS. 177 18. Find the missing temperatures in these tables: (a) Boiling Points. (6) Melting Points. Substance. C. F. Substance. C. F. Benzine 176" Copper 2012° Chloroform 140" Platinum 1775" Mercury 676" Sulphur 115" Sulphur 570" Zinc 779° 19. Construct a conversion table, Centigrade-Fahrenheit, for 0" to 50° Centigrade. 2. Composite Units of Measure. It sometimes happens that an accurate measure of values cannot be made by a simple standard unit, because of some special influencing condition. Thus the freight traffic of a railway cannot be measured accurately by tons, because some tons are carried only short distances, while others are carried great distances; while if measured by freight earnings, the factor of quantity is lost sight of. To meet the difficulty, a composite unit, the ton-mile, was adopted. This unit is one ton carried one mile. If the ton is carried ten miles the traffic amounts to 10 ton-miles; if 5 tons are carried 8 miles, the traffic amounts to 5X8, or 40 ton-miles. Measured by such a unit the full effect of both quantity and distance is taken into account. Moreover, it makes possible a com- parison of the traffic of different branches, or of independent roads. Composite measures of a similar order are increasingly used in science and in statistics. A Few Other Composite Unit Measures. Unit of work. A foot-pound — the work done in raising one ■pound one foot. Unit of power. The horse-power. It is equal to 33,000 foot-pounds per minute, or 550 foot-pounds per second. The irrigation unit. The acre-foot. The quantity of water required to cover one acre to a depth of one foot. 13 178 BUSINESS ARITHMETIC. The, unit of freight traffic. The ton-mile. The equivalent of one ton of freight carried one mile. The unit of passenger traffic. The passenger-mile. One passenger carried one mile. The unit of train traffic. The train-mile. One train run one mile. Illustration. What horse-power in an engine will raise 4,950,000 lb. 10 ft. in 5 min.? Solution. 4,950,000X10 = 49,500,000, no. of ft. p. 49,500,000 4- 5 = 9,900,000, ft. p. per min. 9,900,000^33,000=300, no. of horse power. ORAL EXERCISE. 1. How many foot-pounds of work are performed in raising 48 lb. 15 ft.? 2. How many acre-feet of water are required to supply a 2^ ft. depth to an area of 4^ A.? 3. What is the ton-mileage of a car load of 60 T., carried a distance of 56 mi.? 4. Compute the passenger mileage of 32 people, carried an average distance of 15 mi. EXERCISE. 1. Compute the horse-power employed in raising 30,000 lb. an average distance of 48 ft. per min. 2. Compute the horse-power performed in raising 5000 gal. of water per sec, a distance of 10 ft. Note. Express the water in terms of weight. 3. The owner of the tract of land shown on page 149 contracts for a supply of water of 2^ ft. per annum, for irrigation. What will it cost him, at $46 per acre-foot? 4. A storage reservoir is to be built with a capacity of 50,000 acre-feet. What must be its capacity in gallons? 5. A metal casting weighing 48 T. 150 lb. is carried a distance of 169.3 mi. Compute the ton-mileage. 6. In a recent year, the railways of the country carried 1,309,899,165 T. of freight an average distance of 133.23 mi. Compute the ton-mileage. PRACTICAL MEASUREMENTS. 179 7. In the year above mentioned, 715,419,682 passengers were carried a passenger mileage of 21,923,213,536. What was the average distance each passenger was carried? 8. In the year above referrred to the passenger train mileage was 440,464,866 train miles. What was the passenger mileage per train-mile? 3. The Formula. In estimating of every kind, it frequently occurs that a general rule for the solution of problems of a certain type is reduced in statement to a simple form of an equation, em- ploying words, or more frequently, symbols. Illustration. The fact that the area of a triangle is equal to one-half the product of base and altitude may be expressed: . r X • 1 Altitude X base Area of a triangle = ^ . We may need, however, to compute areas of many different figures. In such a case, we may let "A" stand for the area of any figure, "a" for altitude, "6" for base, "c" for circumference and "i2" for radius, the Greek letter "tt" for the ratio of diameter to circumference. Using symbols, certain common rules for area reduce to the following: 1. For area of a triangle, A = —^ , or simply -^. 2. For area of a rectangle, A=ab. 3. For area of a circle, A = irE^. Frequently the determination of the formula is a matter of summing up many experiments (in arbitrary rules — called empirical rules), or of complicated computations by higher mathematics. But once formulated, computations may often be made by simple arithmetic if in any problem numerical values are known for all except one symbol. Illustrations: (1) Find the area of a circle of 6" radius. Solution. Formula, A = ttB^. But TT =3.1416, R = 6, fi» = 36. Substituting in the formula, A =3.1416X36. = 113.0976 sq. in. 180 ' BUSINESS ARITHMETIC. (2) Find the base of a rectangle whose area is 135 sq. in. and whose altitude is 9". Solution. Formula, A=ab. But A = 135and6=9. Substituting in the formula, 135 = 9a. 15 = a, the altitude. (Dividing by 9.) Note. Care should be taken that denominations are correct. Thus, in above case, if area had been stated in square feet while other factors were in inches, both factors should have been reduced to a common de- nomination of feet or inches. EXERCISE. 1. Express in symbols the value of the circumference of a circle, in terms of the radius and constant ratio. 2. Express in symbolic form (a) the area of a parallelogram, (6) the surface of a cube, (c) the volume of a rectangular solid, (d) the surface area of such a solid, (e) the perimeter of a parallelogram. 3. The safe weight of a floor for a certain type of bridge is 80 lb. per sq. ft, Express this fact in formula for any sized bridge of the type. 4. Find by the bridge formula, the safe load of a bridge 90 ft. long and 17 ft. 6 in. wide. xb 5. If a: = 12, 6 = 6 and c = 5, find A, in the formula A- — . c 6. Find S, in the formula S = iTrR^, if R = 17. 7. The number of board feet in a rectangular timber = length in feet X breadth in feet X thickness in inches. a. Express this formula in symbols. h. Use the formula to determine the number of board feet in a timber 12' long, 9" wide and 2" thick. 8. The pressure of wind blowing directly against an object is sometimes measured by the formula P = .0023 7^ in which P= pressure in pounds per sq. ft., and 7 = velocity in feet per second. Find the pressure per square foot under a wind blowing 40 miles per hour. 9. A roof is so inclined that the wind pressure is .45 of what it would be if the wind struck it perpendicularly. What is the total wind pressure on the roof, which measures 18' by 60', when the wind is blowing 32 miles per hour? PRACTICAL MEASUREMENTS. 181 10. The volume of boards that may be sawed from a round log may be computed by the formula L{d-^)dl2 V = 8 in which d = diameter in inches, L = length in feet, and y= volume in feet, board measure. Compute the board measure in a 60 ft. log of 15 in. diameter. CHAPTER XXVI. THE METRIC SYSTEM, i The metric system is a decimal system of measures adopted by the French government about 1800. It is used, scien- tifically, the world over, and commercially in many civilized countries. It is authorized in the United States and Great Britain, but is used only in a part of their foreign trade. The system is based on the meters a unit of length of approxi- mately 39.37 in., which was assumed to be exactly 1/10,000,000 of the distance from equator to pole. Later measurements, however, have proven this computation incorrect. The unit of capacity, the liter, is a cube of -^^ m. edge. The unit of weight, the gram, is the weight of a cube of water of 1/100 m. edge. The tables contain multiples and decimals of units. Multiples are known by the Greek prefixes: myria (10,000), kilo (1000), hekto (100) and deka (10); decimals by Latin prefixes: deci (1/10), centi (1/100), milli (1/1000). Owing to the decimal system, few measures are commonly used. Those used are shown in black-faced type. Note. Literature on the Metric System is published by The Bureau of Standards, Washington, D. C. Linear Measure 1 myriameter = 10 kilometers = 10,000 meters. 1 kilometer (Km.) = 10 hectometers = 1,000 1 hektometer = 10 decameters = 100 1 dekameter = 10 meters 10 1 meter (m.) = 10 decimeters = 1 " 1 decimeter (dm.) = 10 centimeters .1 " 1 centimeter (cm.) = 10 millimeters .01 1 millimeter (mm.) = .001 " 182 THE METRIC SYSTEM. 183 sq. Square Measure. myriameter = 100 sq. kilometers = 100,000,000 sq. meters. 1 sq. kilometer (Km.') = 100 sq. hektometers = 1,000,000 1 sq. hektometer = 100 sq. dekameters 10,000 1 sq. dekameter = 100 sq. meters 100 1 sq. meter (m.^) = 100 sq. decimeters 1 1 sq. decimeter (dm.') = 100 sq. centimeters .01 1 sq. centimeter = 100 sq. millimeters .0001 " 1 sq. millimeter Land Measure. .000001 " 1 hektare (ha) = 100 ares = 10,000 sq. meters. 1 are = 100 centares = 100 " 1 centare Cubic Measure. 1 " 1 cu. myriameter = 1000 cu. kilameters =1,000,300,000,000 cu. meters 1 cu. kilometer = 1000 cu. hektometers = 1,000,000,000 " " 1 cu. hektometer = 1000 cu. dekameters = 1,000,000 " " 1 cu. dekameter = 1000 cu. meters 1,000 " " 1 cu. meter = 1000 cu. decimeters = 1 " " 1 cu. decimeter = 1000 cu. centimeters = .001 " " 1 cu. centimeter = 1000 cu. millimeters = .0000001 " " 1 cu. millimeter Table of Weight .000000001 " " 1 metric ton (t.) (t onneau) =10 quintals = 1,000,000 grams. 1 quintal (q.) = 10 myriagrams = 100,000 1 myriagram = 10 kilograms = 10,000 1 kilogram (Kg.) = 10 hektograms 1,000 1 hektogram = 10 dekagrams 100 1 dekagram = 10 grams 10 1 gram (g.) = 10 decigrams = 1 1 decigram = 10 centigrams .1 1 centigram (eg.) = 10 miUigrams .01 1 milligram (mg.) = .001 " The metric ton or tonneau= weight of 1 n I. of water; the kilogram = weight of 1 hter of water. Table of Capaciti; 1 hektoliter (HI.) = 10 dekaliters = 100 liters. 1 dekaliter = 10 liters = 10 1 Hter (1.) = 10 deciliters = 1 1 deciUter = 10 centiliters = .1 " 1 centiliter (cl.) = 10 milliliters = .01 " 1 milliliter (ml.) = .001 " 184 BUSINESS ARITHMETIC. Table of Equivalents. Note. Importers and customs officials have to perform a relatively slight amount of reduction to and from the metric system. Common equivalents follow. Measure. Meter Liter Gram Kilometer Hectare Sq. meter Cu. meter Metric ton Kilogram Mile Yard A. Sq. yd. Cu. yd. For fairly accurate work. For approximation. 39.37 in 3J ft. or 1.1 yd. f 1.0567 1. qt. ■) 1 .9081d. qt.J ^'^*' 15.432 gr 15^ gr. .6214 mi 6 mi. 2.47 A 2^ A. 1.196 sq. yd T. 1^ sq. yd. 1.308 cu. yd. . . ?f 1^ cu. yd. 1.1023 T 1.1 T. 2.2046 1b 2.2 1b. 1.6093 km. Quart (liquid) .9436 1. .9144 m. Quart (dry) 1.101 1. 40.47 a. Pound .4536 kg .8361 m2 Ton .9072 t. .7645 m3 ORAL EXERCISE. 1. Read as meters: a. 7 km., 5 dekameters and 3 cm. b. 9 km., 4 cm., 8 mm. 2. Read as square meters: a. 9 km2, 2 m^, 3 dm^. h. 5 hm2, 8 m^, 9 cm^. 3. Read as grams: 4 kg., 8 dkg., 7 eg. 4. Read as liters: 9 hi., 7 1., 4 cl., 8 inl. 5. Use decimal values: 100 m. = ? in. 1000 m. = 1000 1. = ? 1. qt. 200 t. = 2 km. = ? mi. 10,000 m2 = 6. Use approximate equivalents: 20 m. = ? ft. 20 1. 68.5 1. = ? qt. 200 ha. = 80 km. = ? mi. 600 m^ = 40 kg. = ? lb. 200 g. = 7. Express in metric system: 100 1. qt.; 1000 lb. 10 T.; 200 gr.; 100 cu. yd.; 10 A.; 200 sq. yd. ? cu. yd. ? T. ? sq. yd. = ? T. = ? A. = ? sq. = ? gr. 100 mi, yd. ; 1000 d. qt.; THE METRIC SYSTEM. 185 8. A "hundred meter" dash is equivalent to a race of ? yd. 9. A man who orders 1000 m. of dress goods should receive ? yd. EXERCISE. 1. If a merchant desires 1200 yd. of French silk, he must order ? meters. 2. A consgt. of 50 1. olive oil will fill how many pint bottles ? 3. A measurement on a foreign map is found by scale to be equivalent to 87.56 km. The equivalent distance is ? miles. 4. An importer is required to pay a tax of 15c per gal. on 120 1. of oil. The tax amounts to S?. 5. How does a speed of 71 km. per hour for a German express train compare with a record of 71 miles in 69 min. made by a train in this country? 6. Granite is 2.7 times as heavy as water. Estimate the weight of a piece measuring 3.1 m. by 6 m. by 4.55 m. 7. The United States requires foreign letters weighing 15 g. or less to pay 5c postage. What is the Umit of weight in our standards? 8. What is the mileage rate in cents for first class passage from Rouen to Paris, 136 m., if the fare is 15.20 fr. (1 fr. =$.193). 9. A man who weighs 172 lb. weighs ? kg. 10. Determine the weight of an aluminum bar 1 m. long and 30 cm. cross-section (specific gravity, 2.71)? 11. A tank measuring 5'X4'X8' has a capacity of ? m*. 12. Determine reasonable dimensions for a rectangular tank to have a capacity of 500 1. 13. How many tons of coal will fill an order for 5000 t.? 14. How many pieces 8.1 cm. long, may be cut from 70 m. of wire? 15. A 400 meter running track is ? yd. long. Individual Original Work. 1. Prepare a brief on the metric system contrasting it with our own system. Illustrate by parallel computations. 2. Construct a six-place table for converting Linear Measure to Metric Measure. 3. Show how an increasing error is caused in the construction of a working table, by a minor error at the start. 4. Compare computation by table and by process, giving illustrations to show the value of tables. CHAPTER XXVII. RATIO AND PROPORTION. INTRODUCTORY EXERCISE 1. Find the difference between 150 and 30. 2. 30 is what part of 150? 3. 150 is how many times 30? 4. $3000 worth of a novelty were sold in one year, and $12,000 worth the next year. Find the increase in sales. The second year's sales were how many times those of the first year? It is evident that division oflFers one effective means of comparing numbers. By it one may determine how many times as greaty not how much greater one number is than another. The numbers so compared must be Hke numbers, and conse- quently the quotient is always abstract. This quotient, whether represented or computed, is termed a ratio. The ratio may be expressed by any of the signs of division, or by the colon. The numbers involved are called terms, the first term being the antecedent and the second the consequent. Illustration. The ratio of a payment of $500 to one of $2500 may be expressed (a) $500/$2500, (6) $500 -^ $2500, or (c) $500 : $2500. The value of the ratio is determined by performing the repre- sented division. In the above illustration, the value is 1/5. If the second payment had been referred to the first, the ratio would have been expressed $2500 : $500, and its value would have been 5. The ratio of the first quantity to the second is termed the direct ratio; that of the second to the first the inverse ratio. ORAL EXERCISE. Express and determine the ratios of: 1. 6 to 42. 2. 2 yd. to 2 ft. ' 186 RATIO AND PROPORTION. 187 3. 5 qt. to 6 gal. 8. 1' to 1/4'. 4. 2 sq. ft. to 1 sq. yd. 9. 2 lb. to 4 oz. 5. $720 to $900. 10. 900 to 1500. 6. 75c to $2. 11. Measure a rod by a yard. 7. 1" to 1'. 12. Measure a bushel by three pinta 13. Name groups of two numbers having the ratios 12, 4, 1/2, 1^. 14. Add 1 to the antecedent of the ratio 2:5. What is the effect on the value of the ratio? 15. By experiment, find the effect on the value of the ratio, from: (1) Increasing its antecedent. (2) Decreasing its antecedent. (3) Increasing its consequent. (4) Decreasing its consequent. (5) Multiplying or dividing the antecedent. (6) Multiplying or dividing the consequent. (7) Multiplying or dividing the consequent and antecedent by the same value. 16. Compare the above cases with the corresponding cases of simple fractions. ^. Antecedent (dividend) -r^ . , .... bince -i^ ^ . ,. . — ^ = Ratio (quotient), it is evi- Consequent (divisor) dent that the antecedent equals the product of ratio value and consequent, and that the consequent equals the quotient of the antecedent divided by the ratio. It is therefore a simple matter to find a single missing term. Illustration. A manufacturer issues one design of oblong calendars in assorted sizes, but with the dimensions of length and breadth in a constant ratio of 2|. (a) If the length of one is 15" what is the width? (6) If the width of one is 8" what is the length? Solution (a). „„?^, = ratio = 2|. Widtn Width =Length-^2^ = 15"X2/5 = 6'^ Solution (6). Length = 2^ X the width = 2^X8" =20". ORAL EXERCISE. Determine the missing values in each of these ratios : 1. 24 :4= ? 2. ?/18 = 3. 3. 36/? = 2. 188 BUSINESS ARITHMETIC. 4. ? : 4 = 8. 6. 3/4 : 2/3 = ? 8. 1/4 yd. : ? = 1/8. 5. 1/2 : 1/4 = ? 7. ? : 3 mi. = 1/4. 9. 2h ft. : U ft. = ? Note. If two variable (changing) quantities or dimensions have an unchanging or constant ratio, one is said to vary as the other. Thus the ratio of the circumference of a circle to its diameter is always 3.1416, no matter what the size of the diameter. Quantities are said to vary inversely if one increases as the other decreases, in a constant ratio. In common speech, we refer to approximate changes in the same way. 10. What do we mean by saying that summer travel from the big cities varies with the heat? 11. What do we mean when we say that prices vary with demand? 12. What do we mean when we say that the cost of production of a certain article varies inversely as the quantity produced? The ratio method is often used in geometry and its appli- cations, in physics and in the solution of many business principles. It is really a method of fractional analysis and is used, without the name, throughout this book. EXERCISE. 1. The specific gravity of copper is 8.9. What is the weight of three cubic feet? See page 134. 2. Water is composed of hydrogen and oxygen in the ratio of 2 : 1. How many parts of each gas in 15,000 parts of water? 3. If a 3" line in a drawing is used to represent a real length of 4' 9", what is the ratio of real to represented length? 4. A piece of ore weighs 1.3 kg., while the weight of an equal volume of water is .2 kg. Determine the specific gravity of the ore. 5. If the circumference of a wheel is to be 16 ft. what must be its diameter? 6. One of two cubes is 3/4 of the height of the other. Determine the ratio of surfaces and volumes. 7. Show by ratio, the effect on volume of reducing all dimensions of a bin measuring 16' X 8' X 4', by one-third. The Lever. If a bar is supported on mri 1 J, a fulcrum, F, a weight, W, attached at the point Y RATIO AND PROPORTION. 189 may be balanced by the application of a sufficient pressure at the point X. If the weight is between the pressure point and the fulcrum, pressure must be applied upward. .k. Otherwise, it is applied ^ "T ~~J\ downward. If the dis- D3 ^ tances x and y are ex- pressed in units of measure, and W and P in units of weight, it is a law of physics that the ratio of the product of the weight by its arm to the product of the pressure by its arm is equal to unity, when the lever balances. In other words. Weight (W) X weight arm (y) Pressure (P) X pressure arm (x) = 1. Illustration. If the fulcrum in the first figure is 4 ft. from Y and 6 ft. from X, what pressure supports a weight of 12 lb.? Solution. By the law of levers, p = 1. 48 = 6P. S = P. .'. the pressure must be 8 lb. EXERCISE. 1. The fulcrum being at one end of a 6 ft. bar, what pressure bal- ances a weight of 80 lb., 1 ft. 6 in. from the end? 2. A pressure of 20 lb., applied 9 ft. from the fulcrum, supports what weight 2 ft. beyond the fulcrum? 3. Illustrate how the principle of the lever is applied in the use of a crowbar. Write and solve an illustrative example. 4. Find, by investigation, six common applications of the lever prin- ciple, and prepare and solve simple problems illustrating them. 5. Where must one place a support under a 12 ft. bar, in order that it may balance weights of 16 and 40 lb.? 6. A brake chain is connected with a brake lever 1 ft. below the sup- porting pin. The brake handle is 6 ft. above this support. If the brake lever is pulled with a force of 40 lb., what pressure does the brake exert? What pressure per square inch, if the brake shoe measured 8 in. by 8 in.? 190 BUSINESS ARITHMETIC. Proportion. An equality of ratios is termed a proportion. Thus 6 lb./21 lb. = $12/$42 is a proportion because the abstract value of the ratios is the same. An equality of several ratios, as 3/4 = $9/$12 = 18 lb./24 lb., is termed a continued proportion. An equality of products of ratios, as 1/12 X 5/4 = 5/6 X 1/8, is termed a compound proportion. Continued and compound proportion are seldom used in practical arithmetic. In any proportion, as 1 : 2 = |8 : $16, the first and fourth terms (1, $16) are the extremes, and the second and third (2, $8) are the means. The proportion is read " 1 is to 2, as $8 is to $16." Since the two ratios are equal in numerical value, and since both the extremes and the means include a numerator of one ratio and a denominator of the other, it is evident that the product of the means equals the product of the extremes. Thus 1 X $16 = $16, and 2 X $8 = $16. It is evident, also, that any term of a proportion may be found if the other terms are known. The first or third terms are always the products of the opposite ratio by the second or fourth term respectively. The second term equals the product of the first term hy the opposite inverse ratio. To avoid multiplying by concrete numbers, each term of either ratio may be made abstract. X 15 Illustrations. (1) In the proportion ^ = j^ find x. Solution. ^ = ^-^^ = 2. Evidently 1/6 of x = i|, and a; = 6 X ^f . 40 40 46 (2) In the proportion = -^ find x. X D Solution. 6 24 X = $480 X 24 = $120. Evidently $480 = a; X ^ , andx = $480 -^ ^ = $480 X ^^ = $120. b z4 Note. Since the product of the means equals the product of the ex- tremes, it is evident, also, that any mean may be found by dividing the RATIO AND PROPORTION. 191 product of the extremes by the other mean; and any extreme may be de- termined by dividing the product of the means by the other extreme. Thus, in illustration (1), writing the proportion in the form a; : 6 = 15 : 45, it is seen that one extreme is missing and that its value is equal to 6 X 15, the product of the means, divided by 45, the other extreme. This principle affords a simple method of solution. EXERCISE. Find the missing terms: 1. a; : 12 = $72 : $18. 2. 5:8:: 14:? 3. 275 lb. : 125 lb. = $11 : $x. 4. 3 sq. ft. : 16 sq. ft. : : 48c : ? c. 5. 1/4 :l/2 ::? :3/8. 6. a;:12 ::7yd. :480 yd. Two quantities are directly proportional if one varies directly as another. They are inversely proportional if one varies inversely as the other. Illustrations. (1) If one is traveling on mileage, the cost of his railway passage varies directly as the distance, or is proportional to distance. (2) Temperature being constant, the volume of a gas varies inversely as the pressure, that is, grows less at the same rate that the pressure in- creases. Thus doubling the pressure halves the volume. The common applications of proportion are those already mentioned under simple ratio. In solutions by proportion, it will be found more convenient to place the unknown quantity in the first term, although any term may be used. Solutions may be checked by unitary analysis, and by fractional methods. Illustration. A contractor is to receive $4872 for the construction of 2140 ft. of concrete pipe. What has he earned after he has completed 535 ft.? Solution. Let X equal the sum earned. X : $4872 = the ratio of amount earned to contract price. 535 : 2140 = the ratio of quantity done to total quantity. Since the amount earned depends on the work done, the ratios must be equal. 192 BUSINESS ARITHMETIC. :.x :$4872 = 535 : 2140. X X 2140 = $4872X535. (Products of means and extremes.) ^^ 848^X535 =«218. Check Solution. The part of work done is 535/2140, or 1/4. .-. the contractor is entitled to 1/4 of $4872, or $1218. . EXERCISE. . 1. The weight arm of a lever is 16 ft.; the pressure arm is 9 ft.; the weight is 4000 lb. Find the pressure or power to balance it. 2. A farmer found that he required 7 lb. of a certain seed for a 1/4 acre lot. What proportional amount of seed is required for a 3^ acre lot? Note, In electricity, the resistance of a wire to the flow of a current varies directly as its length, or inversely as the square of its cross section. 3. Determine the resistance of If mi. of a certain copper wire, if the resistance of 100 yd. is 5 ohms. (The ohm is an electricity unit of re- sistance.) 4. If the resistance of iron is seven times that of copper, find the re- sistance in 1000 ft. of iron wire, if 20 yd. of copper wire of same diameter has a resistance of 2.6 ohms. 5. If a metal sphere of 8 in. diameter weighs 214 lb., what is the weight of a sphere of the same metal having a diameter of 12 in.? 6. It is a principle of mechanics that the power acting parallel to an inclined plane will support a weight proportional to the ratio of the length of the incline to the distance the weight is raised. If the height is 5/6 of the length, what power will support a weight of 840 lb.? If the height is 1/4 of the length? Note. For the principle of partitive proportion, see page 387. CHAPTER XXVIII. GRAPHIC ARITHMETIC. A simple drawing often does .away with the necessity for a compUcated numerical statement. A line diagram, drawn to scale, will often convey its meaning instantaneously. A num- ber statement, on the other hand, may convey little meaning, or require time for interpretation, or be less likely to attract attention. Diagrams or graphs, therefore, have become a common form of number expression, or language, especially in the presentation of statistical matter. Thus the accom- Production of Barley: 1900 MILLIONS OF BUSHELS 8 12 16 20 24 CALIFORNIA MINNESOTA WISCONSIN IOWA 8. DAKOTA N.DAKOTA WASHINGTON NEW YORK NEBRASKA OREGON KANSAS MICHIGAN OHIO IDAHO MONTANA ILLINOIS COLORADO ARIZONA VERMONT OKLAHOMA INDIANA MAINE UTAH NEVADA PENNSYLVANIA TEXAS 14 193 194 BUSINESS ARITHMETIC. panying graph from a Census Report gives the average reader a far clearer impression of the relative production of barley, in different states, than would a column of figures stating the production in bushels. Moreover, if desired, the graph may be read to approximate numerical value by means of its scale. General diagrams, such as building plans, plots of land, maps, etc., commonly are used for reference. It is far easier and surer to say "Find the area of the floor shown in the accompanying diagram" than, without a diagram, to describe the shape and dimensions in such a way as to enable one to compute its area. Experiment. Try to describe the plan here shown, in words so that one could compute its area accurately I. Scales and Plotting. Ability to read and construct explanatory drawings and statistical graphs depends on a thorough knowledge of the arithmetic of scales, and of the fundamentals of plotting. GRAPHIC ARITHMETIC. 195 A linear scale is either the ratio of the plotted length to the true length or distance, or the length that is to represent on paper a fixed quantity of any denomination. If every line in a drawing is one-fourth of the corresponding true length of the object represented, the diagram is said to be drawn on " 1/4 scale. " If a one-inch line is used to represent a quantity of 10,000 bushels, the scale is said to be *' 1 inch = 10,000 bushels." Linear scales are represented, commonly, in one of three ways: (1) As a simple fraction. Illustrations: 1/4, 3/8, 5/4. (2) By corresponding dimensions. Illustration: V = T. This means that one inch on paper represents one foot of the object. (3) By diagram. Illustration: 10 5 10 20 30 40 I I r I I I I I I I I \ I I I FEET From zero to the right the scale shows diagram lengths corresponding to true lengths of 10 ft., 20 ft., 30 ft., etc. A true distance of 22 ft. is represented by a paper distance from "20" (on the right of zero) to the second sub-division on the left. (Why?) Notice that the paper scale reads to true lengths. In diagram scales decimal sub-divisions are common. Linear scales of distance may be expressed in any of the three forms. Thus a scale of 1/3 may be written V = S", or 1 ft. = 3 ft.; or . 3 INCHES QUESTIONS FOR DISCUSSION. 1. What is the advantage m having the diagram scale read to true lengths? 2. Why is a linear scale used on a drawing that represents an area ? 196 BUSINESS ARITHMETIC. 3. How does the "line" scale, if carefully constructed, save compu- tation, in making or reading drawings? 4. Name a common scale used by architects. 5. How does the graduated edge of a ruler correspond to a scale? EXERCISE. (Answer orally, if possible.) Express each of these scales in two other forms : 11. 1" = 100'. 12. 2' = U". 13. Which of the above scales could be used to make a diagram larger than the original? 14. Express the following scales in numerical form. (Use ruler to find ratio.) (a) Q>) t 5 FT.- J 2p^ ,,,.,,,. 0^ . 2,0' 9 1 (c) ? f MILES id) -J 1 // 75- 15. Which of these scales will make the largest diagram: i-- 1" = 4'- 9. 16' ' FEET 16. An engineer uses a ruler divided into units, tenths and hundredths of feet. If he uses the smallest subdivision to represent two feet, find his scale. Express it in two ways. 17. If the smallest subdivision on your ruler is used to represent one foot, what is the scale? 18. An architect uses a ruler having inches divided into sixteenths. If he uses one of these subdivisions to represent a foot, what is his scale? GRAPHIC ARITHMETIC. 197 19. Two towns, known to be twelve miles apart, are represented on the map at an interval of three inches. What is the map scale? EXERCISE. 1. Construct the scale 1/20 in diagram form. 2. Determine a fractional scale for this (ji_ i^o line scale. rods - 3. Construct a line scale for the scale V = 1200 bbl. 4. Construct a line scale for the scale 1" = 2500 lb., such that each division shall represent 1000 lb. 5. Find and illustrate four methods of representing line scales. Reduction to Plotted Lengths or Distances. Since a true linear scale shows, in fractional form, the part that the scale length is of the length to be plotted, computations in scale reduc- tion are simple fractional reductions. A scale of one-sixth means that the plotted lengths are one-sixth of the real lengths. Real lengths, in this case, are reduced to plotted lengths by dividing by six. When scales are expressed in other forms, they may be used directly, or in difficult n 5 cases, reduced to fractional form. In the f'^ET scale shown five feet of true length are repre- sented by the given line. Twenty-five feet would be represented, evidently, by a line five times as long. A scale of V = 4' reduces an 8' true dimension to a plotted dimension of 2". (Why?) On a scale of 1" = $600, the amount of $1200 is represented by a 2" line. (Why?) EXERCISE. (Answer orally if possible.) Find the scale lengths for true dimensions of* 1. 4' 6", on a scale of 1/2. 2. 3' 9", on a scale of I" = 1'. 3. 100 yd., on a scale of 1/600. 4. 35 ft. on a scale of 1" = 10 ft. 198 BUSINESS ARITHMETIC. 5. $2000, on a scale of 1" = $400. What length lines should be used in graphs to represent these quantities: 6. 4500 bu. on a scale of 1" = 250 bu. 7. 4 tons, on a scale of 1/2" = 500 lb. 8. 1600 cans, on a scale of 1/4" = 200 cans. 9. 2,400,000 bales, on a scale of 1" = 60,000 bales. EXERCISE. Find the missing values: No. True dimension. Scale. Scale Dimension. 1. 4' 6" ' ' ? X. t. u 2 FEET * 2. 2' 3" 1/12 ? 3. 14' 9" 1/4" = 1' ? 4. 270' 0" 1" = 50'. ? 5. 4 mi. 80 rd. ' TmilI ' ? 6. 8" 5/3 ? 7. 2 1/2" 1" = 1/3" ? 8. Which of the above scales will make enlargements? PRACTICE EXERCISE. 1. List twenty true dimensions taken in the class room. In a parallel column, write the corresponding scale lengths. Use the smallest sub- division on your ruler to represent one inch. 2. Make a freehand sketch of the outline of the class room floor. Take measurements and determine plotted lengths on a scale of 1/16. Dimensioning is the process of noting on a drawing the true dimensions of the parts represented (page 194). The symbols (') and {") are commonly used in place of the abbrevi- ations (ft.) and (in.). If the true length contains no fraction of a foot, zero inches is written. For example, 5 ft. is written 5' 0". Light dimension lines are drawn parallel to the lines of the drawing to which the dimensions refer, the dimension figures being entered in a break in the center. Dimension GRAPHIC ARITHMETIC. 199 lines are limited by arrow points that just touch short perpendiculars erected at the extremes of the line to be dimensioned. Dimensions are generally written up and to the right, in order that all may be read from one position. A circle or arc is dimensioned by locating its center and. giving its radius. "Over all" or "total" dimensions are written as a check on a series of intermediate dimensions. EXERCISE. 1. Point out the different principles of dimensioning illustrated on page 194. 2. What is the real value of an "over all" dimension? 3. What is the advantage in writing dimensions outside, rather than inside a small drawing? 4. Collect from newspapers, periodicals, or other sources, six specimens of dimensioned drawings. Make a list of any unusual details of dimen- sioning. Plotting, as related to drawings, is the process of locating the main points of a proposed diagram preliminary to drawing in the details. With the actual mechanical construction of drawings we have little to do, but certain mathematical preliminaries should be noticed. Before plotting, it is neces- sary to choose a paper large enough to contain the drawings of the object at the scale required. The size of paper and the dimensions of the object to be represented will necessarily affect the scale to be chosen. EXERCISE. 1. The greatest dimensions of a tract of land are 1800 ft. by 1450 ft. If plotted on a scale of 1/450, how large a sheet is required, allowing for a two- inch margin all around? 2. What is the largest scale that could be used in reproducing, on this page, the map of a piece of land measuring 4 miles by 3 miles 40 rods? 3. What is the largest scale on which the land diagram on page 149 could have been drawn? 4. What is the largest scale on which a diagram of your class room could be drawn, on a paper measuring 17 in. by 24 in.? 200 BUSINESS ARITHMETIC. 5. A rectangle measures 16 ft. by 5 ft. 2 in. How large a sheet (A paper is required for a drawing of it, on a scale of 1/4" = 1', if a 1^" margin is to be left between the drawing and the edges of the paper? Coordinates. While the general principles of scientific projection and plotting, as used in projection drawing, have no place in this discussion, the Y system of plotting by coor- dinates has a direct use in +3 Pa the graphic phases of ap- 1+5 p plied arithmetic. If we as- ffs 1+4 sume two fixed lines, Y and ^^\! j T X, at right angles, any I ■iv III ^. T I II I _-5 i I point, P, may be exactly located if its perpendicular distances from X and Y are known, together with the particular angle, I, II, III, ^ or IV, in which it is situ- ated. If P is in angle (quad- rant) I, 4 units from X and 5 from Y, it is located by measuring on X from Y 5 units; then up from X, parallel to X, 4 units. The coordinates, or locating distances, are always taken parallel to the axes of coordinates {X and F), so that it makes no fundamental difference whether the axes are at right angles or at a slant. In writing (not plotting), the position of a point with respect to the distance parallel to X is stated first, then the distance parallel to Y. Thus the point P is the point (5, 4); the point Pi is (2, 1); the point P2 is (?, ?); the point Pe is (0, 3). If the point is located in other quadrants, its position is located in the same way, except that the dis- tances parallel to X and to the left of Y; or parallel to Y and below X are marked minus. Thus P3 is the point (—2, 5); P4 is the point (—3, —5), and Ps the point (3, —5). In the simpler applications, with which we have to deal, it is possible GRAPHIC ARITHMETIC. 201 so to locate the axes as to have all our points in the same angle, and thus disregard the plus and minus signs. EXERCISE. Draw the axes and locate these points: 1. 2 in. from X and 3 in. from Y, first quadrant. 2. 4 in. from Y and 3 in. from X, first quadrant. 3. On X 2 m. from Y. 4. (3", 3"). 7. (2", -4"). ■ 10. (0', 00. 5. (4", 2"). 8. (0", -U"). 11. (3/4-, -2"). 6. (2^", 5'0. 9. (5", 0"). 12. (1", 2h"). 13. (8", 5"), on 1/2 scale. 14. (2 mi., 7 mi.) on a scale of = 2 miles. If one locates the corners of a field with respect to axes, he may plot the points and, by connecting them, obtain the out- line. In taking measurements of an irregular room, one may find the distances of each cor- ner from any side or sides, taken as axes, and then pre- pare a plot. Points on a curve may be taken at such close intervals as, after plotting, to outline the curve on paper. As a help in plotting, cross- section paper, similar to the il- lustration, may be used, any two lines at right angles being taken as axes. The sides of the sub-squares may then be taken as units of scale, and diagrams plotted speedily. The large squares are usually divided decimally. "~ _J _^ ^ EXERCISE. 1. Plot the points (4, 3), (7, 3) and (6, 6). the resulting figure? Connect them. What is 202 BUSINESS ARITHMETIC. 2. Plot to scale the points (2, 2), (3, 4), (6, 2) and (7, 4). What is the resulting figure? 3. Make a diagram on a scale of 1" = 6', of a room whose corners, taken in order, are (0', 0'), (0', 11'), (8', 11'), (8', 13'), (13', 13') and (13', 0'). Which sides are taken as axes. 4. Make a diagram of your class room by the same process. 5. Plot and find the area of a field whose comers, taken in order, are the points (50 rd., 70 rd.), (200 rd., 70 rd.), (250 rd., 200 rd.) and (50 rd., 200 rd.). 6. Plot one side of a curve of a driveway from the points (0', 10') (1, 8), (2i 6), (5, 4), (6, 3), (8, 2), (10, 1). Reduction of Plotted Lengths to Real Lengths. Few drawings or maps, except draftsmen's construction drawings, are so fully dimensioned as to supply all dimensions desired. It is often necessary to find the real length corresponding to a plotted length. If a plotted length is drawn on a one-fifth scale, the true length is naturally ^ve times that plotted. If the drawing scale is V = 4', a 2'' line on paper represents 8' of real length. (Why?) If a scale is 2 feet, a line six and one-half times as long represents a true length of 13 feet. (Why?) EXERCISE. (Oral and written.) No. Length on Drawing. Scale. True Lengths 1. 3i" 1" = 1000'. ? 2. ' 5.3" 1/1200 •? 3. 2.7" 1/63360 ? 4. 1', 3.4" 2 INCHES' ^ 5. 9.2" 1" = 2 mi. ? 6. 1', 3^' 1/16 ? 7. 11.45" 1/400 ? 8. 2^' 1" = $5250 ? 9. 4i" 1/2" = 10,000 bu. ? 10. 1" = 10'. ? GRAPHIC ARITHMETIC. 203 11. Determine the scale of the figure on p. 155. Find the true distance diagonally across the room; and the distances of any point C from each comer, and perpendicularly, from three sides. STATE LINE Avondale Larsliall 12. This map is drawn at a scale of 1" = ? mi., or what fractional scale? Which towns are in a radius of eight miles of the center of Marshall? By airline, Mason ville is ? miles from Avondale; Norwood is ? miles from the nearest point on a railway; Mason ville is ? miles from the state line, and Norwood is ? miles from the nearest railway junction. An airline electric road from Ashland to Marshall would be ? miles long, approxi- mately ? miles shorter than the nearest railway connection. Are distances computed by scale from this map absolutely accurate? What factors tend to affect the results? 204 BUSINESS ARITHMETIC. O o> 00 CM CM CM CM CN CN P^ CN O o OF C SHELS 8 19 2 PRODUCTION SD MILLIONS OF BU 3 14 15 16 17 1 Eh d 5 ^^ ESTIM 7 8 9 10 1 to c c : I I c i c a a r-. 00 1 g GRAPHIC ARITHMETIC. 205 ^ .& z 2 n_ 9? s o > 1 1 1 ^ q •a § p ^ o 0) m OS ?;- m -a ^•3 ^ CJ 3 1 1 QQ y •c-^ b Frt .5 ^ u. 1 ■§ 11 > ■sj o ^ ^ s a o ^ •s .gt^ « s F •5S (S i 0) 1 1 1 a> £ a ^ ?» ^ •< "2 M) S .a .3 ^ ^ o 1 -s 206 BUSINESS ARITHMETIC. h- UJ UJ cr 5 H O Ui g I •" (- ro ADAMS AVE. Scale? 13. The owners of the plot here shown desire to buy additional land, fronting 60' on 13th Street and 80' adjoining on Adams Ave., and running back in each case so as to make the entire lot a rectangle measur- ing V by ?'. Plot the figure and show the additions. Locate on the plot a building with a frontage par- allel to Adams Ave., of 80', set back 20' from the street line, and running back, parallel to 13th street, to a depth of 60'. The building is 15' from the 13th Street line. The area of the original plot is ? sq. yd.; of the addition, ? sq. yd. ; and of the improvement ? sq. ft. 14. Sixth Street, north, meets Ave. A at right angles. On a scale of 1/1200, make a plot of the land on the northwest corner, from this de- scription: From the corner, the boundary runs north 180'; thence west, 40' ; thence north, 60' ; thence west, 80'; thence diagonally to a point on Ave. A 100' from corner. The area is — sq. ft., and the valu- ation, at $1.95 per sq. ft. is $ . II. Graphs. Reference has been made to the graph as a means of conveying general impres- sions of relative value. It deserves, however, more ex- tended notice, now that the principles of scales are un- derstood. t What We Have What We Grow What We Use OUR TIMBER SUPPLY AND ITS DEPLETION. The large tree represents the amount of timber that we have, the second tree the relative amount of the annual growth, and the third tree the relative amount of annual use. From The World^s Work. GRAPHIC ARITHMETIC. 207 The graph may be used in the form of: (1) lines drawn to scale in lengths proportional to the quantities of bushels, dol- lars, acres, etc., which they represent; (2) proportional areas of any shape; (3) lines based on the principle of coordinate; or (4) figures, varying in one or more dimensions. xAn illustration of (4) is shown on page 206. The Parallel Line Graph is often drawn superposed on a reading scale that enables one to read approximate numerical values at sight. In the graph of the production of corn (p. 204) in the United States, based on Government figures, each scale division represents one hundred million bushels. Thus the crop for 1910 was approximately 3,125,000,000 bushels and for 1850 it was just under 600,000,000 bushels. Approximate values are all the general public desires, hence for large values round numbers are sufficient. Such graphs also show at a glance the general increase of crop from period to period and any period of unusual crop or crop shortage. By using rectangles of a uniform height so that areas vary with widths, the principle of the line graph is maintained, but changes in the filling of the different sections are made to add details of information. Thus in the figure on p. 205 each entire space represents the full arid area of a state, but, by a change of shading, five facts concerning this area are made clear. Area graphs are of endless variety but they usually conform to geometrical figures. In the circular graphs (p. 208), areas vary with represented values. Both were constructed to scale, although none is stated. The temperature and precipitation chart shown on p. 209 illustrate a coordinate graph. The subdivisions of the scale background vertically read represent (1) temperatures in degrees above or below the normal for the periods shown, and (2) the precipitation in tenths of inches. The solid lines show i w H < ^ t- m \o / Q § S / ITE GOO 3,39 §!=- Q w «^ < UJ 01 H ^ "^ 1— - ^ > \ l-l y P^ H \ PCH 02 H P5 o Pui s gf r S "^ o « .2 ^ S o d ^ •eo,. >'^rf T5o '^/A/^ ^ GRAPHIC ARITHMETIC. 209 the variation in average temperature from week to week, from the normal as shown by the horizontal line. The dotted line shows the amount and variation (departure) in rainfall. (A < 2 OH hi \- LO UJ + 25 + 20 + 15 + 10 + 5 JAN. 1st TO APRIL 3RB INCLUSIVE APRIL MAY JUNE JULY AUGUST SEPT. H g 10 17 24 1 8 15 22 2fl 5 12 19 26 3 10 17 24 31 7 14 21 28 4 11 18 26 2 • ; , I 1 fi 1 ' ■I 1 1 1 H J — 7" Q — d '^ L r ' — -^ A / =^ ~7 T^ -^^ <^ Vf ]— ~^^ -^ ^v L/l 1^^ — ^^ ^ y ■ zz z= pE ^ ^ =: 3i ^^ zz 1= = -5 St V ^L. — zz ^z. •J^ — =t V Hh ^ ^ ^I ^ zi £ !zz / \/ , '^ i' ^/ "V^ || TEMPERATURE, IN DEGREES, PRECIPITATION, IN TENTHS OF IN DHE3, ^_ j| Temperature (degrees Fahrenheit) and precipitation (inches) depart- ures for the season of 1905 from the normal of many years for the Middle and South Atlantic States and Gulf States. From the Year Book of the U. S. Department of Agriculture for the year, 1905. EXERCISE. 1. Read at sight the figure for the production of com, page 206, 2. Reduce to a number table the graph shown on page 206. 3. Read the following graph at sight. PRODUCTION OF COTTON: 1850-1900. 10 ( 1900 1890 1880 1870 1860 1850 3 1 \ MILLIONS OF 3 4 CO MME ) :rc AL 5 BAL ES J J ( ) ^ "" *" -f^^ • "* _^i n " "^ ^ z ^^^ " " ^ n 15 2m BUSINESS ARITHMETIC. 4, The corresponding 1910 figure for the above cotton graph may be taken as 10,386,000 bales. Determine the length of the proper line on the graph. 5; A man, receiving a wage of $25.00 per week, estimates his expenses as follows: Rent, $25 per month; food, $360 per year; fuel and Ught $80 per .year; clothing $100 per year; amusements, $2 per month; charity $2 per month; miscellaneous expense, $6 per month — the balance remaining representing a savings and emergency fund. Draw an outline to represent his $20 bill or weekly wage, and show graphically how it is apportioned to meet his expenses. 6. The following have been estimated to be the approximate expendi- tures of anthracite coal in New York City in a year: Anthracite. Tons. Domestic, private houses and small stores 2,500,000 Flats and apartment houses .' 2,850,000 Hotels, clubs and theaters 1,250,000 Gas and electric plants 1,300,000 Elevated and surface roads 650,000 Harbor shipping 400,000 Department stores and office buildings 650,000 Municipal departments 400,000 Total 10,000,000 Represent these figures graphically in some other way than by a line graph. 7. By means of properly proportioned outline, to suggest " skyscrapers," represent the relative amount of new building operations in different cities, in 1911 based on the following figures: Baltimore, $9,325,000; Buffalo, $10,365,000, Chicago, $105,270,000; Cincinnati, $13,485,000; Cleveland, $17,000,000; Detroit, $19,000,000; New York, $173,500,000; Pittsburgh, $11,700,000. 8. Represent in some other graphic form the graph on page 211. 9. Using round numbers, to the nearest "ten thousand," prepare a graph to illustrate these facts concerning immigration: Immigration from 1830 to 1906. 1831-1840 599,125 1841-1850 1,713,251 1851-1860 2,598,214 ■ 1861-1870 2,314,824 1871-1880 2,812,191 1881-1890 5,246,613 1891-1900. ; . . 3,844,420 1901-1910 .8,795,386 GRAPHIC ARITHMETIC. 211 ^ o t3 f^ o M M ^ rr, < o o ^ t— 1 r/3 H tf 2 •s :: S. s: I S i -i I I i I 212 BUSINESS ARITHMETIC. 10. Design a graph to illustrate this statement: "The llama will cany from 50 to 200 pounds; a man from 75 to 150 pounds; the donkey, 100 to 200 pounds; an ox, 150 to 200 pounds; a horse from 200 to 250 pounds; the camel from 350 to 500 pounds; the elephant, from 1800 to 2500 pounds." 11. Construct, on a coordinate or cross-section plan, a graph to rep- resent the recapitulation of daily sales by departments, shown on page 13. Design a form that may be extended day by day. Individual Original Work. Prepare a set of statistics of some class of the high school, compiled from office records by direction of Principal. Show same facts graphically. CHAPTER XXIX. PERCENTAGE. In the expression of numerical relations, 100 is a common basis of comparison. The process of percentage, or of com- puting by hundreds, is one of the most important of all the working processes of arithmetic and has most varied applica- tions. INTRODUCTORY EXERCISE. 1. Find 1/100 of 200, 2472, 36, 592 bu., h .006. 2. Find 3/100 of 400, 70 lb., 9000 tons. 3. Find (a) 4/100 of 720; (b) 25/100 of 800 sheep. Per cent, is a standard term for hundredths, derived from the Latin per centum, meaning " by the hundred." The term is represented by the symbol "%." Illustration. 4 per cent, of a number = 4% of it = 4/100 of it = .04 of it. ORAL EXERCISE. 1. What per cent, of a quantity is one-hundredth of it? 3/100 of it? 246 hundredths of it? .03 of it? .92 of it? 3/4 of it? 2. Express as rates per cent. : 36/100, 270/100, .40, .02, 1.25, 3.205, .6. Note. The rate per cent, is the number of hundredths taken. 3. Express as hundredths: 8%, 12%, 120%, ^%, 331%, 600%, 121%. 4. i = ? hundredths. ^ of a quantity = ? % of it. 5. Express as per cents.: 1/3, 3/4, 1/4, 5/8, 3/20, 1/40, 3/5. 6. 20% of a quantity equal how many himdredths of it? What fractional part of it? 7. Name simple fractions corresponding to these per cents. : 50%, 5%, 25%, 40%, 36%, 1331%, 100%, 106%, 96%.. 8. Express as decimals: 1/4, 1/3, 5/4, 3/25, 4.3%, 125%, 20%, 12i%, 92%. 9. Express as condmon fractions .24, .4, 3.2, .0125, 90%, 120%, 43%. 10. . Express each of these rates per cent, in at least two other forms: .26, .8, 1 1/4%, .004, 9/5, 3/4, 1/2%, .50, 48%, 75%. 213 214 BUSINESS ARITHMETIC. 11. Express these statements in other arithmetical forms: (a) I pay 33^% of a debt. (6) The use of wood pulp paper in this country has increased 49.9% in five years. (c) 94% of the men returned to work yesterday. (d) He saves 25% of his salary. {e) Thompson's plurality was cut down 37^% at the last elec- tion. 12. 1/2 = how many per cent.? What is the difference in per cent, between 1/2 and 1/2% of a quantity? 13. Find the difference in per cent, between: 1/4 and 1/4%. 1/4 and 1/3%. 33i% and 3/8. .002 and 1/4%. EXERCISE. Find the missing values in this table: Table of Comparative Value. Decimal. Per Cent. Hundredths (Fraction). Common Fraction. .0125 .02 2% 2/100 .05 1/20 .061 12% — m% — — 15% — 161% 20% 25% 33i/100 37i/100 .80% 1.75 90% 1/2 11/20 3/5 5/8 2/3 7/8 9/4 PERCENTAGE. 215 To Find the Percentage. The percentage of a quantity is the value of the hundredths taken. The base is the number on which the percentage is reckoned. Illustrations. (1) I collect 45% of a debt of $720, or what sum? Solution. (Decimal method.) 45% = .45 $720 .45 3600 2880 $324.00 the am't collected. (2) What profit is made by selling a $42 article at 3% advance on cost? Solution. (1% method.) 1% of $42 = $.42 3% of $42 = 1.26 (3X1% of $42) The profit = 3 X 1% of $42, or $1.26. (3) On a $420 purchase, I am allowed 25% discount, or $ ?. Solution. (Aliquot method.) The discount is 25% or 1/4 of $420. 1/4 of $420 = $105, discount. Note. The general tendency is toward the use of the decimal method. It is evident that the percentage equals the product of the base by the rate. FOR DISCUSSION. 1. What rates per cent, in the preceding table are aliquot parts? 2. Is the base concrete or abstract? 3. Is the 'percentage concrete or abstract? ORAL DRILL EXERCISE. 1. Fmd 1% of 24, 36, 500, 4.65, 1/5, 72.5 yd., 246 lb. ' 2. By the 1% method, find: 3%, of 2400; 8% of 20 bbl; 12% of $300; 4% of 1/3; 15%) of .2. Note. 100% of a number = 100/100 of it, or the number itself. 200% of a number is how many times the number? 3. Find: 200% of 2640; 300% of $48; 400% of 1/3; 250% of 2.4. 4. Find: 1/4% of 5600; 3f% of 1400; 2/3% of 1/2; 1/4% of 1/4; 3/4% of .4. 5. Find, by aliquot parts: 25% of $3640; 83^% of .006; 33i% of 1/4; 12^% of 624 yd.; 90% of $7.20. 6. Compare 72% of $50 with 50% of $72. Conclusion? 7. Find:18%of $250;84%of 75T.;19%of66|A. 216 BUSINESS ARITHMETIC. EXERCISE. Find: , 1. 25% of 27,240 lb. 5. 4^% of 5/36. 2. U% of $7654.20. 6. 75% of 15.02". 3. 18% of .00624. 7. 3/8% of $95,240. 4. 333i%of645T. 8. 62|% of 11,200 shares. EXERCISE. Each problem illustrates a different use of percentage. State use in each case. Re-word each problem in question form. Find missing values. 1. The directors of the Rand&ll Mfg. Co. pass a resolution to increase all salaries and wages 15%. My own salary of $1200 is thereby raised to $ . The advance will cause an increase of $ in a monthly pay roll of $24,600. 2. I decide to set apart 12^% of my monthly income of $96, or $ , for minor personal expenses, and to deposit in the bank, 25%, or 3. On a purchase of 40 O. C. stoves, listed at $4.80, a customer is allowed 40% off the price, because of the size of his order. The discount is $ and the customer pays $ each. 4. The purchaser of the stoves marks them at an advance of 25% on what they cost him. His selling price is $ , of which $ is profit. 5. A. C. Crane and Thomas Drew form a partnership with a capital of $12,600, Crane supplying 33^%, or $ , and Drew %, or $ . 6. C. B. Brown fails in business and compromises with his creditors, agreeing to pay at once 66f % of his debts, and a month later 15% more. His debts total $46,240.20. His payments are $ and $ . 7. It is reckoned that the machinery equipment in a factory decreases 5% of its original value in each six months. If worth $26,400 originally, it is worth $ , 12 months later. 8. An architect charges as a fee for the design of a building, 5% of its cost. His fee on a building costing $11,380 is $ . 9. Of 700 pupils in a school, 90%, or pupils, are present on Janu- ary 11. Of the number present, 40% are boys and are girls. In an arithmetic test, 73% or pupils make a passing average. 10. "The newspapers predict that Black's majority of 42,400 two years ago will be cut down 75% at the coming election. If this is true he will still win by majority." PERCENTAGE. 217 ORAL DRILL EXERCISE. (The teacher should prepare similar exercises.) 1. Base, 240. Find 1%, 3%, 25%, 33i%, 10%, 11%, 16|%, 87^%, 1/4%, 20%, 160%, 500%. 2. Base .6. Find 1/3%, 3/4 of it, 2/3, 2/3%, 150%, 3%, 400%, 2i%. 3. Base, 3/4. Fmd x% of it, 3%, 20%, 200%, 8%, 12^%, 400%. 4. 60 decreased 33i% of itself = ? (Called the difference.) 320 increased 25% of itself = ? (Called the amount.) 5. Defins amount and difference. Which implies subtraction and which addition? If the rate is 46%, name the difference per cent, and the amount per cent. 6. Decrease. 240 bu. 33|%; 1/4 by 1/3 of itself; 3/8, 33^%; 2460, 10%; 800 by 1/4 of itself; .6 by 40%; 4.2, 14f %. 7. Increase: 12, 33^%; 900, 2/3%; 2172 lb., 1%; .04, 125%; 1/4, 25%; 3.6 T., 16|% In the following, use each intermediate answer as the base for the next step. 8. Increase 20 by 50% of itself; increase that result 33^%; that result, 25%; that result 20%; that result, 33i%; decrease that result, 50%; decrease that 10%; increase 10%. Answer? 9. 4 increased 50%; that result increased 33^%; increased 12|%; decreased 66f%; increased 100%; decreased 50%; multiplied by 4; decreased 25%; multiphed by 1/3. The final result is ?. Finding the Rate Per Cent, introductory exercise. 1. If 20 = 5% of a given number, 1% of it equals ? and 30=-?%. 2. 16= what part of 20? What per cent, of 20? 3. 1% of 400= ? 36= ? % of 400. 924= ? %. Illustrations. (1) What per cent, is 536 of 1200? Solution. (Decimal.) Since it is desired to know how many hundredths of 1200 is 536, divide 536 by 1200 ■44f=44f%. 1200)536. (2) By selling for $2.27 an article costing $2.00, what is gained in per cent? 218 BUSINESS ARITHMETIC. Solution. (1% Method.) A gain of 1% =$.02. $2.27 Sell pr. $.27 (total gain) divided by $.02 equals 2.00 Cost number of per cent, gained. $.02)$ .27 Gain. 13|, or per cent, of gain. (3) The Charleston basket ball team wins 18 out of 24 games or ■ % of its gains. Solution. {Fractional.) The question is: 18 is how many hundredths of 24? 18 3 75 ^^^ , 24 = 4 = 100 = ^^^«'^"- It is evident that the rate equals the quotient of the percentage divided by the base. ORAL DRILL EXERCISE. Find missing values: 1. 1/4= % of 1/2, of 1, of 3/4, of 1/8, of 2, of 1/16, of 3/8. 2. 4.8 = % of 2.4, of 4, of 1.2, of 9.6, of 48, of 6. 3. What per cent, of 2400 is 16? 1.6 is .4? 4 is .8? 2/3 is 1/2? 300 is 29? .2 is .06? 7^8 1? 4 is 12? 4. $2.40 are % of $60. 6 qt. are % of 20 qt. 760 lb are % of 300 lb. 90 sq. yd. are % of 2000 sq. yd. 5. What per cent, of 3 is 4? 25 is 30? 2.5 is 5? 1/4 is 1/3? 4 is 3? 25 is 5? .25 is .5? 3/8 is 5/8? 6. 4 is what per cent, of 5? 5 of 3? 3 of 4? 4 of 6? 6 of 4? 6 of 8? 8 of 6? 8 of 10? 10 of 15? 15 of 20? 20, mcreased 20 %, of 30? 30, decreased 1/5, of 40? GENERAL EXERCISE. 1. What per cent of the cost of a $160 horse is lost by selling it for $120? 2. I invest $6000 in a partnership capitalized at $27,000, under an agreement to share in profits according to investment. What per cent. of any profits should I receive 3. If $64 is the cost of the premium on a $4000 fire insurance policy, what is the rate of premium? 4. What per cent, has a city's population increased in a year, if it now has 240,000 inhabitants, and a year ago had 220,000? PERCENTAGE. 219 5. How does a profit of $4 on a $20 investment compare in rate with a profit of $16 on a $60 investment? 6. What per cent, is made in a class test if 943 credits are obtained out of a possible 1140? 7. A new printing press has a maximum output of 35,000 copies per hour, compared with an output, for the machine it replaced, of 27,500. What is the per cent, of increase? Note. Percentage is frequently employed in technical writing and statistics, to show proportions, rates of increase and decrease and for comparisons of values. 8. Find the missing rates in this table to three places decimally: Where Exports Find Markets. Countries. Total Imports. Share from U. S. Per Cent. United Kingdom, $2,958,289,000. $638,006,000. Germany, 1,696,660,000. 236,082,000. Italy, 398,463,000. 45,956,000. China, 349,913,000. 36,304,000. Canada, 290,361,000. 175,862,000. Australia, 217,676,000. 22,549,000. Japan, .208,554,000. 34,834,000 . Spain, 204,401,000. 23,006,000. Argentina, 197,974,000. 27,908,000. Denmark, 166,837,000. 26,832,000. Brazil, 161,587,000. 18,518,000. Mexico, 109,884,000. 72,509,000. ORIGINAL EXERCISE. Write and solve original problems illustrating the use of rates per cent.: 1. In measuring the increase of sales of a business during one year. 2. In measuring the loss of a firm through bad debts. , 3. In comparing the expense of repairs on a machine with its original cost. 4. In comparing the votes of successful and defeated candidates for office, in a political election. EXERCISE. Compute missing values in this article. BASEBALL AVERAGES. Baseball records furnish an excellent illustration of the use of rates per cent, in showing relative standing. Thus, the standing of any team is 220 BUSINESS ARITHMETIC. determined by the per cent, of games won out of games played. This rate is computed to three places decimally. For example, the standing of the American League on Sisptember 25, of a recent year was: American League Standing. Team. Won. Lost. P. C. Team. Won. Lost. P. C. Philadelphia. . .84 51 .622 Detroit 70 69 .504 Chicago 82 54 ? New York 66 66 ? Boston 69 66 ? Washington ... 56 80 ? Cleveland 70 69 ? St. Louis 48 90 ? Owing to bad weather, or other causes, some scheduled games may be postponed, so that on any particular date, the number of "games played" by each team may vary. (Is this true above?) It is evident that per cent, rates offer a better means of comparison than common fractions. (Show why.) Of course, averages change every day when teams are playing. On Sept. 26, Washington added a game to its "won" column by defeating Chicago, whose column was increased by 1. Detroit lost to Phila- delphia; New York defeated St. Louis; and Cleveland beat Boston. At the close of September 26, the standing would appear as follows: American League Standing. Team. Won. Lost. . P. C. Team. Won. Lost. P. C. Philadelphia. .85 51 ? — — — Towards the close of the season possible results are often forecasted. On Sept. 26, for example, these possibilities might be considered: 1. Chicago and Philadelphia have each eight more games to play. If Chicago wins six and loses two, while Philadelphia wins four and loses four, will Chicago be in the lead? 2. Philadelphia plays the first four of these games in Chicago. If Chicago should win three would Philadelphia lose the lead? 3. A Philadelphia enthusiast asks " How many of these last eight games must Philadelphia win to be sure of the championship? " In a similar way the records of pitchers are determined. Thus a pitcher winning 25 games and losing 18 would have an average of games won of . — . In batting, the player who makes 93 safe hits in 240 times at bat is said to have a batting average of . — . PERCENTAGE. 221 The average of the players "in the field" are based on three factors: (1) the number of opposing players they "put out"; (2)' the number of plays in which they "assist" in putting out a player; (3) any attempted plays in which the player fails to complete a play within his power (errors). The players' fielding averages are expressed decimally as the per cent. * that (1) plus (2) is of the sum of (1), (2) and (3). Fielding Averages. Player. Outs. Assists. Errors Percent. C. Baker (Outfielder) 217 109 21 ? J. Town (1st base) 824 103 37 ? S. Crane (2d base) 162 219 33 ? T. Parker (Shortstop) 124 137 41 ? FINDING THE BASE. INTRODUCTORY EXERCISE. 1. If 14 = 2% of a number, 1% = ?; 100% or the number = ? 2. Find 100%, if 2% =4, 32, 420, 6. 3. Find the numbers of which 33i% =35, 1240, 520, 9, 1/4. 4. Find 100%, if 300% = 700, 45, 6. Illustrations. (1) What is the cost of an article that sells for $618, at 3% profit? Analysis and Solution. Let the cost equal 100%. The selling price equals 103% of the cost and the cost equals 100/103 of $618. $600, cost. 1.03)$618 (2) 40 pounds of Mocha coffee will form 66 f% of a mixture of how many pounds of Mocha and Java? Solution. 40 lb. = 66f % or 2/3 40 = 2/3 of mixture, of the mixture. 20 = 1/3 of mixture. 60 = 3/3 of mixture, or the total quantity. It is evident that the base equals the percentage divided by the rate. ORAL DRILL EXERCISE. 1. When is the "1% method" superior to the aliquot or fractional method? Which is better in the following? 40 = 10% of ?; 27=9% of ?; 62=50% of ?; 28=4% of ?; 75=33i of ? 222 BUSINESS ARITHMETIC. • 2. 24 =4% of?; 25% of?; .4% of?; 1/3% of?; 150% of? 3. 420 = 6% of ?; 250% of ?; 40% of ?; 25% of ?; 1/4% of ? 4. 2/3=25% of ?; 3% of ?; 10% of ?; 400% of ?; 1/2% of ? 5. 426 = 25% of ?; 6750 = 50% of ?; .06 = 75% of ?; 1/4 = 800% of ? 6. 16 = 1/4% of ?; 480 = 2/3% of ?; 12,000 = U% of ? .006 = 4/3% of ? EXERCISE. Find missing values: 1. 297.6 = 3/5% of — . 4. 45.297 = 7% of — . 2. 97,485 = 20% of — . 5. 1605 = 120% of — . 3. 2.0406 = 1/8% of — . 6. 29,623 = 47.2% of — . EXERCISE. (Each problem illustrates a practical use of this phase of percentage.) Find missing values. 1. What sum must I invest at a profit of 7^% per year to earn a profit of $900 yearly? 2. A certain cloth shrinks 6^% in dyeing. In order to have 640 yd. of finished material, one must dye — yd. 3. "Thus the price has fallen 16% in two months, and the money that would buy 100 shares two months ago, will now buy — shares." 4. A merchant sends a manufacturer an order for 600 articles, enclosing a draft in payment, and writing, "In case you grant me a discount from your last quotation, send all the goods the money will buy." The manu- facturer allows a discount of 20% from his last quotation. What number of articles should he ship his customer? (Note: " grant a discount " means "lower the price.") 5. For a certain contract a carpenter requires 72,000 ft. of flooring. As he knows the waste in cutting will average 25%, he should order — ft. 6. If a farmer ofifers to sell his land "at 75% of its real value" for $8250 — in order to dispose of it at once — he places what valuation on it? ORAL DRILL EXERCISE. All Cases. Find missing values: 1. 16=4% of — ; that result =20% of — ; that result =200% of — ; that = 500% of — ; that =20 times — ; that =25% of — ; that result =400% of — ; that = l/2%of — . PERCENTAGE. 223 2. A percentage study of 36. (a) (&) (c) 1% 1 " 4%of — r 1200 5% 100% of — 200 20% 80% of — 3000 150% 7% of — 30 400% ►of 36^ ;36 = - 3% of— 36 =- - % of - 6 33^% 33i% of — 1/2 1/2% 2/3 of — 3.6 2 3% 2i% of - .9 U% L U%of- • 40000 EXERCISE. 1. 121% of 372 = li% of — . 2. 8% of 33i% of 960 = — % of 1200. 3. 175% of 36 = 1/3 of 24% of — . EXERCISE. An Argument for the Panama Canal. Trip. New York to San Francisco via Panama, 5278 M. ; via Cape Horn, 13,340 M. San Francisco to Liverpool via Panama,. 7907 M.; via Cape Horn, 13,678 M. New York to Manila via Panama 11,412 M.; via Cape Good Hope, 13,555 M. New York to Yokohama via Panama 9692 M.; via Cape Good Hope, 15,178 M. New York to Sydney ■ via Panama, 9560 M.; via Cape Good Hope, 12,218 M. Present these figures in tabular form. Express also, the distance via Panama, (o) as a per cent, of other routes; (6) via the other routes as a per cent, of the Panama route. ANALYSIS DRILL. MIXTURES AND COMPOUNDS. Oral. 1. 20 lb. of one grade of tea are blended with 30 lb. of a second grade, forming a mixture of ? lb., of which ?% is first grade. The quantity of first grade is ? % of that of second grade. 2. 30 lb. are blended in the ratio of 5 : 4 with ? lb. of a second grade. The second grade forms ? % of the mixture and equals in quantity ? % of the first grade. 224 BUSINESS ARITHMETIC. 3. How much witch hazel is required for 8 gal. of a 12% solution? 4. ? lb. Mocha and ? lb. Java are required for a mixture of 180 lb., if the quantity of Mocha exceeds that of Java 25%. Written. 1. What quantity of each ingredient is required for 25 gal. of a mixture of hard oil diluted 40% with turpentine? 2. A mixture of tallow and rosin is to contain 15% tallow. How much tallow is required for 40 lb. of rosin? 3. 22 lb. of standard bulhon, 80% pure, will make — lb. of standard purity (90%). 4. The products of the distillation of 100 lb. of average gas coal of a certain mine approximate: 64 lb. coke, 6| lb. tar, 12 lb. ammonia liquor, 14 lb. purified gas, and — lb. minor products and waste. Express com- position in per cents. What amount of coal (2240 lb. to the ton) must be distilled to manufacture 1,500,000 cu. ft. of gas, if gas measures 10,540 cu. ft. to the ton? 5. The composition of Babbitt's metal is 3.7% copper, 89% zinc, and 7.3% antimony. Determine the quantity of each metal required for an order of 16 T. 6. The analysis of a metallic paint, per 5000 parts, is: 3943 parts iron peroxide, 165 parts alumina, 598 parts silica, 254 parts water, 16 parts lime, 14 parts manganese, — parts minor elements. Express the paint formula in per cent. form. 7. A common asphalt varnish contains 3, 4 and 18 parts respectively of asphaltum, boiled oil and turpentine. Give formula in percentage form. What quantity of asphaltum is required to combine with 600 gal. of the proper mixture of oil and turpentine? 8. A mixture of three grades of material prepared in proportions of 20%, 30% and 50%, is found to be markedly improved by reversing the proportions of first and second grades. It is decided to change 120 lb. of the old mixture to the new by adding whatever quantities are necessary. What quantities should be added? What is the quantity of the new mixture? CHAPTER XXX. ELEMENTARY PROFIT AND LOSS. INTRODUCTORY. 1. What do I gain by selling an article that costs me $400 at a price 25% higher? 2. If I sell for $40 a desk that cost me $60 what do I lose? What per cent, of the cost do I lose? 3. Compare these terms with the corresponding terms of percentage: Cost, selling price at a loss, selling price at a profit, rate of gain or loss. Practically all transactions in business are carried on directly or indirectly for profit, or to prevent or reduce loss, or to decrease expenditure. A business man continually faces the question, "Does it pay?'* The question may concern the selling price of an article, the results of some business policy, the comparison of investments, or endless other conditions. In most cases arithmetic is used to determine the answer. Percentage is very commonly used to measure results where profit may be expressed in money. ILLUSTRATIVE EXERCISE. Find missing values: 1. Tables bought at $8 are sold at $10, at a profit of — % on cost. 2. I sell apples costing $2 per barrel at $3 and dispose of 6 bbl. per day. By selling at $2.50 I find my sales average 20 bbl, per day. Lowering my selling price increases my profits $ — or — % per day. 3. By the installation of new machinery', a manufacturer reduces the force required for the same output from 500 to 420, thus affecting a re- duction of — %. 4. A manufacturer, by a change of process reduces the cost of manu- facture of a rug from $4.00 to $3.80. If his sellmg price remains $4.80 his profits increase — %. 16 225 226 BUSINESS ARITHMETIC. Note. The principle of projit as a saving influences our daily acts as individuals. 5. I can buy potatoes at an up-town store for $3.00 per bbl., and at a down-town market for $2.65. By buying at the latter, I save — c. 6. I can buy a working bench for $16.00 or I can buy material for $4.00 and make it myself, at a saving of $ — , or — %. If, however, my time is worth $1.50 an hour for the four hours required, I save only $ — . Ordinarily, 'profit or loss is reckoned as a percentage of cost. The cost, however, is an indefinite quantity. In addition to what he pays the manufacturer for goods, the merchant pays freight, insurance and storage, the expense of sale, and the cost of keeping bookkeeping records, etc. Thus the question, often arises as to what incidental charges shall be added to first cost, or subtracted from selling price to determine a base. Partly to avoid this difficulty, some dealers reckon individual profits as a per cent, of sales. Illustrations. (1) I buy 6 desks @ $31 each, paying $6 freight. Find the selling price each at 25% profit. Solution. Net cost as hose. 15% or 1/4 of $31 =$7.75 profit. $31'+$7.75 =$38.75 selling price +$1.00 freight = $39.75, gross selling price. Solution. Gross cost as base. The freight is $1.00 per desk. $31 +$1 =$32, gross cost per desk. 5/4 of $32 = $40, sell, price. (2) One desk is sold for $36 or ? % profit. Solution. Net cost as base. The profit, $36— $31 =$5. The rate, 5/31, or 16.1%. Gross cost as base. The profit, $36- $32 = $4. The rate, 4/32, or 12.5%. Sell, price as base. The profit, $36 - $32 =$4. The rate, 4/36, or 11.1%. Note. In this hook profit ivill be reckoned as a per cent, of net cost unless otherwise stated. Read problems carefully. The department store offers a good illustration of profit reckoning. Here the selling price is fixed by the merchandise manager., or by the department buyer. Computations are on a percentage basis. The rate of profit, however, varies with ELEMENTARY PROFIT AND LOSS. 227 the article or department. Rapid selling articles yield a lower rate than slow selling goods. The latter may sell at 30% profit, where the rapid seller brings 5%. The department, however, that does $60,000 worth of business annually on $10,000 capital can deal on a smaller margin of profit than one that does $80,000 business on $15,000 capital. By the more rapid "turning over" of capital, goods sold at low profits may bring a department as high an average as others sold at a high rate. Suppose an article sold a month after purchase at 5% profit, the original cost being reinvested in the same goods, and the process continued for a year. The profit on original capital is then 5 X 12, or 60% for the year. The tendency is to make all departments yield the same rate annually by means of higher marking of slow selling goods. To the price at which the goods are billed at the store, are added the fixed charges for rent, bookkeeping, selling, delivery, etc. These have been reduced to an average per cent, of cost for each store or department — ranging from 15% to 25%. Many of the stores reckon the profit as a per cent, of sales. It is then a simple matter to approximate any day's profits from the amount of sales. I. Gains, Losses and Selling Prices. EXERCISE. Illustrating simple phases of profit and loss. Answer orally if possible. 1. Find the loss on desks costing $25 if sold at 10% loss; at 20% loss; at 12^% loss; at 5% loss; at 40% loss. 2. Find the gain on toys costing $1.60 and sold at 25% gain; at 20% gain; at 5% gain; at 15% gain. 3. Find the gain or loss if goods cost $1.20 a doz. and are sold at 15 c each; at 20 c each; at $2.00 per doz.; at 5% profit; at 20% loss; at $1.05 per doz. 4. From what known values may the following be computed: (a) Selling price at gain, (6) gain, (c) selling price at loss, (d) loss? 228 BUSINESS ARITHMETIC. 5. Find the missing values by inspection. Cost. Rate Gain or Loss. Selling Price. Gain or Loss. a. $56. 20% gain $ $ 6. 9.60 16f%loss. c. 4260.00 25% gain d. 3.88 doz. e. 1.80 % 39 c EXERCISE. 1. What profit is realized by buying gloves at $9.20 per doz. and selling them at $1.15 per pair? 2. The goods in the crockery department of a store are marked at 20% profit and 18% store expense. What is the selling price of plates costing $9.60 per doz.? 3. The book buyer of a department store pays $7.50 for ten copies of a popular novel which he disposes of at $1.08. As each lot is disposed of he re-invests the original sum in a new lot, until he has disposed of 120 copies. He has " turned over " his capital — times, and gained a total of $ — . 4. Apples are bought © $2.40 per bbl. (3 bu.) and are retailed at 90 c. per bu. Determine the final loss or gain, on a lot of 60 bbl., the average loss by decay being 15%. 5. A dealer retails at $6.50 a grade of flour that costs him $5.00 per bbl. The wholesale price is raised 10%, and the dealer immediately raises his price to $7.50, but his sales fall off 10% in amount. How does this affect his net profits? II. Profit as' a Rate of Selling Price. (Omit if desired.) ORAL EXERCISE. 1. If 30% of selling price is profit, how many per cent, is cost? 2. If $14 is the cost, in ex. 1, find the selling price and profit. 3. The goods in a grocery department are sold at a profit of 15% on selling price. What are the profits on sales of $6400? 4. Find the selling price of a table costing $6.00 to gain 25%. To gain 33i%; to lose 20%; to lose $1.40; to gain 20%. EXERCISE. 1. Find the selling price of desks costing $16.00, to allow for 20% store burden (based on cost) and 25% profit. ELEMENTARY PROFIT AND LOSS. 229 2. Find the profit on an invoice of $2400 worth of goods, sold at 20% profit, the expense of sale being $98.60. 3. Find the selling price of silks costing $1.80 per yd., if store burden is 20%, profit allowance 20%, waste in cutting 5%. 4. Which base for profit, cost or selling price, yields the higher profit at any given rate? Illustrate. III. Per Cents of Gain or Loss. ORAL EXERCISE. 1. By selling a $2.00 engraving for $2.40 a profit of — c or — % of the cost is realized. 2. A profit of 75 c on a $5.00 article is a profit of — %. 3. A carriage-maker buys a cart for $40.00, expends $15 in repairs and sells it for $69.00, making — % profit. 4. .Does it pay a dealer to reduce his profit from 15% to 9%, if his sales are thereby doubled? 5. Compute the rate of gain or loss. Cost. Selling Price, (a) $3.20 $3.60 (d) (6) 4.00 3.98 (e) (c) 8.40 9.60 (/) Selling Price Gain. (jg) $8.00 $2.00 (i) (h) 15.00 5.00 (/c) 7. Find the rates in g-k, ex. 5, if based on selling price. EXERCISE. 1. Skates purchased at $11.40 per doz. are retailed at $1.60 per pair, yielding a profit of — %. 2. A real estate dealer bought a house at auction for $3600. He expended $160.00 in repairs, $95.00 in painting, $5.00 for advertising and then sold the property for $4250. Compute his gain per cent, on invest- ment. 3. A dealer purchased 60 doz. straw hats @ $9.00 a doz. He sold 42 doz. @ 40% profit, and 6 doz. @ $1.25 each. If he disposes of the balance at cost, what is his net rate of gain or loss? 4. A novelty, manufactured at a cost of 40c, is to be retailed at a price to yield manufacturer, jobber, wholesaler and retailer each a profit of 10c. What rate of profit is realized by each? Cost. Gain. Loss. $600 $ .90 15.00 1.20 4000.00 $180.00 jlling Price. Loss. $ 12.00 $ 3.00 150.00 30.00 230 BUSINESS ARITHMETIC. 5. A dealer sells Mocha coffee, costing 25c, @ 32c; and Java, costing 28c, @ 36c. A mixture of the two grades containing one-third Mocha, readily sells @ 40c. Compare rates of gain on straight grades and blends. 6. If a dealer sells at 30% below his regular price of 30% profit, he loses what per cent, of profit? IV. The Cost. (Answer orally if possible.) 1. By selling a coat for $6.00 a dealer gains 20% on cost. What is the cost? Suggestion: $6.00 = 120% of cost. 2. If a profit of 45c each is realized on hats sold for $1.65 what do they cost per dozen? 3. $40.00 represents 5% profit on what sum? 25% loss on what sum? 4. Find the cost from the values given: Selling Price. Gain. Selling Price. (a) $ 5.40 $1.40 (c) $ 2.50 $.18 (6) $240 25% (d) $960. 33 1/3% Gain. Rate. ' Loss. Rate. (e) $ .84 7% (g) $ .52 4% (/) $30. 20% (h) $84. 25% 5. Markward offers to sell a piece of land at "40% below cost," for $750. He evidently values the land at what price? 6. If the retail price of an article is fixed by competition at $5.40 the cost of manufacture must be kept down to what figure to allow for 15% profit to the manufacturer and 20% to the dealer? Examples in Savings. 7. A manufacturer, by a change in process, reduces the cost of office desks, which he sells direct, for $30.00, from $21.00 to $18.60. The saving in cost is ? % and the increase in profits ? %. 8. By the installation of new machinery, the time required for certain work was reduced from 17 hours to 14| hours. If labor costs 48c per hour this represents what saving on each piece of work? 9. By a change in the boiler equipment of a factory, the coal con- sumption was reduced from 34 tons per day to 31i tons, or ? %. GENERAL EXERCISE. 1. Does it pay to buy apples @ $2.40 per bbl. (2J bu.) and to sell them @ 32c per pk., if 10% is allowed for decay? ELEMENTARY PROFIT AND LOSS. 231 2. What rate of profit is obtained by retailing eight articles at the price they cost per dozen? 3. A blend of 32c and 18c coffee, mixed in the ratio of 3 to 4, is to be put on the market in two-pound packages, and sold at 25% advance on cost. Find the selling price per package. 4. Does it pay to lower the selling price of tea (costmg 48c) from 80c to 72c, if the change in price leads to a 10% increase in amount of sales? 5. What is the per cent, of gain on cost if 16% of what is received for an article is gain? 6. What retail selling price, on an article costing $8.00 to manufacture, will yield profits of 25% to the retailer and 20% to the wholesaler and to the manufacturer? 7. At a price yielding 6% profit, the sales of a certain article equal 3600 pkg. per week; at a price yielding 15% profit, sales average 2500 pkg. per week. Which is the more profitable price and why? 8. A 60c grade of pepper is blended with a 30c grade in the ratio of 3 to 5, and the blend is retailed at 50c By increasing the proportion of the fiirst grade to 50%, sales increase 20%. Does the change of proportion pay, if there is no increase in selling price? ORAL DRILL EXERCISE. I. The cost of a certain make of chair is $6.00. 1. What is the selling price each at 10% gain? (Give analysis.) At 5% loss? At 8% gain? At 20% loss? At 15% profit? At a profit of 36c? At a profit of $1.62 for three chairs (analysis)? What is the selling price of two chairs at 20% gain? Of half a dozen at 20% loss? 2. What is the selling price of one chair if 20% of the selHng price is profit? If 25% of the selling price is loss? If $1.40 is loss? If 87 cents is gain? 3. What is gained by selling one chair at 5% profit (analysis)? On three at 10% profit? On one dozen at 20% profit? On three sold for $20.60? What is lost on one at 5%? On six sold at 12|% below cost? By selling three for $17.00? What is the profit if 20% of the selling price is profit? (Analysis.) 4. What per cent, is gained by selling one chair for $6.50? Three for $24 00? Two for the cost 6f three? Five for what one-half dozen cost? What per cent, is lost by selling at 18c below cost? At $4.73? By selling three for $15.00? By selling three for the cost of two? What is the rate of gain or loss, based on selling price, on one chair 232 BUSINESS ARITHMETIC. sold for $7.20. (Give analysis.) On three sold for $21.00? On two sold for $10.00. On four sold at price equal to the cost of three? 5. The selling price of an article is $3.60. The cost must be limited to $ ? to gain 20%? To gain 25% based on selling price? To gain 40%. To lose $1.23? What per cent, of profit on cost is realized by buying at $2.40? At $3.00? At $3.20? II. Give and solve an origmal example to illustrate each case. 1. From what known values may cost be computed? (Several cases.) 2. From what known values may selling price be computed? Rate of loss? Rate of gain? Gain? Loss? 3. What values can be computed from cost and loss? From gain and rate? From selling price and rate? From cost and selling price? 4. Cost and ? determine rate of gain. 5. Selling price and ? determine rate of profit FOR DISCUSSION AND ILLUSTRATION. 1. Why may a dealer lose, who sells an article for the price he paid for it? 2. Could a dealer lose if he sold an article for more than he paid for it? 3. What may increase the first or direct cost of an article? 4. Why may a dealer have less of a certain lot of mdse to sell than he bought? 5. How may the following affect the selling price of an article: over- stocking; competition; style; fire; depreciation; season of the year, locality? 6. What may lead to the sale of parts of a lot of mdse. at different rates? 7. What is meant by "quick sales and small profits"? 8. May sales at a fixed price increase, and yet profits decrease? 9. Under what circumstances may profits increase, if selling price decreases? 10. Under what circumstances may profits decrease, if the selling price increases? MARKING GOODS. In many businesses it is a common custom to mark goods, directly or by means of a tag, with the selling price and, at times, with the cost. The cost, and sometimes the selling ELEMENTARY PROFIT AND LOSS. 233 price, is written in a key or private symbol, so that it may be known only to certain employees of the business. While some keys are exceedingly complicated, consisting of arbitrary characters, the usual form consists of a word or phrase con- taining ten different letters, each of which stands for a par- ticular figure. The letters may be numbered forward, back- ward, or arbitrarily. Illustration. Key: Now be quick. (1) NOWBEQUICKor (2) NOWBEQUICK 123 4567890 098 7654321 By using different keys, the cost may be concealed from all except marker and manager, and the selling price from all except marker, manager and selling force. There is a tend- ency, however, to mark the selling price in plain figures. In writing with a key, the last two letters represent cents. If any figure occurs twice in succession, an arbitrary letter, called a repeater, is used the second time. Arbitrary letters are sometimes used for fractions, but are seldom needed, as fractions of a cent are uncommon in the marking of such goods as are commonly "tagged." Illustration. Example. Write the marking tag for canes, no. 221, costing $2.80 and selling for $3.60, using the key (1), as above. Solution. Substituting the corresponding letters for the figures, $3.60 is written "wqk," and $2.80 "oik." The tag is written Note. If the selling price were $4.tX), and s were repeater, the price would be written "bks." No. 221 wqk oik EXERCISE. 1. Interpret the following, based on the key "hypodermic " : peo^ hoc^ iy^ poxc^ opd^ hmr, "x" is repeater, yic hex er ydcx pyc rm 2. Write, using the key "Mayflower," and any repeater: 37 4m 675 19.20 1.10 99 4J5 12.32 5' 3.80' 5.40' 11.85' 65' 80' 2.60' 9.64* 3. Repeat (2), writmg sellmg price in figures, and the cost in the key Now be quick." 234 BUSINESS ARITHMETIC. Write the cost and selling price in the following examples, using these keys: Cost Key. - I \ / T + ± =F 1 n O 1 2 3 4 5 6 7 8 9 SelUng Key. WASHING TUB M R ' 1234567890R First Cost. Charges. Gain. x 1. $2.40 $ .30 20% 2. 3.25 .45 25% • 3. .88 m% 15% 4. 14.56 .60 30% First Cost. Sell. Ex. Gain or Loss. 5. $2.40 5% of S. P. . 20% ofS. P. 6. 3.82 10% of S. P. 25% ofS. P. 7. 9.60 10% of S. P. 20% 8. 15.00 $1.00 25% Using the key "M ayflowers," write cost and selling price per article for the following: Cost. Per. Gain. Cost. Per. Expense. Gain. 9. $9.00 dz. 25%. 12. $15.00 dz. $.72 20% 10. 4.80 C. 20% 13. 7.20 gr. 5.00 25% 11. 6.40 cas. (2 dz.) 15%. 14. 1.26 1/2 dz. .12c 20% 15. Using the key "Washing tub," mark the goods in the following invoice at an advance of 25% on gross cost. Messrs. Topham and Chase, Pittsburgh, Pa. Bought of C. R. Newman & Co. Terms : 60 days. Philadelphia, January 29, 19 — . 1829 47 639 426 358 2i U 3 4 3 dz. Neckwear, Outing Shirts Caps, Less 10%. 3.60 2.80 4.50 9.60 3.25 Expressage, $2.80. Selling Expense, 10%. ELEMENTARY PROFIT AND LOSS. 235 Individual Original Work. Design a marking sheet showing first cost of each class of goods in bulk, cost per article, discounts, gains, terms, selling and list prices, etc. Make entries for at least fifteen articles. CHAPTER XXXI. . COMMERCIAL DISCOUNT. One of the most comm9n and important business appli- cations of percentage is the expression and reckoning of trade discounts. ILLUSTRATIVE EXERCISE. 1. On Jan. 6, J. P. Robertson buys 60 bbl. flour @ $5.00. He agrees to pay in 90 days, but is offered 3% off (discount from) the debt if he pays within ten days. Up to Jan. 16, $ ? will settle the bill; after that, and until April'O, $?. Note. These terms may be written "90 days net, 3% 10 da." or "Net /90; 3/10." In business, the credit terms are usually 30 da., 60 da., or 90 da. Rates of discount vary with firm and customer. Interest is sometimes charged for overdue payments. 2. A Chicago mail order house allows 2% discount on orders of $10; 3% on orders of $20; 5% on $50 orders, etc. What discount should a customer receive on a $20 order? (Why?) 3. The Virginia Brick Co. allows 5% discount on orders for over 10 M. brick. 26 M. @ $12 per M. cost what sum? 4. The catalog price of 5" metal piping is $1.40 per foot. The market price is 20% less. 1000 ft. at the market price cost what sum? Note. Market prices of some articles vary with demand and with the cost of raw materials used in their manufacture. To avoid constant re- printing of trade catalogs, prices are listed high, and customers are notified of current discounts on goods that they desire. 5. The Sandford Pub. Co. issues a nature book to retail at $1.80. The book is sold to dealers at 20% discount, enabling them to realize what per cent, of gain? Note. Publishers and some manufacturers fix the retail price of their products, selling to dealers and jobbers at a discount, and to private customers at 'list.' Solution. $1.80 Retail price. 20% or 1/5 of $1.80 = $.36, discount. $1.80 - $ .36 = 1.44, cost to dealer. $ .36 -^ $1.44 = .25 or 1/4, or 25%, the rate of gain. 236 COMMERCIAL DISCOUNT. 237 It is evident that a commercial discount is an allowance or subtraction from the list or marked price of goods for such business reasons as: (1) payment before due; (2) size of order; (3) reduction of list to market price; (4) to meet competition; (5) to allow for profits to middlemen who are expected to sell 'at list,' etc. These discounts commonly are expressed as rates per cent. Many discounts are aliquot rates, thus permitting short methods of computation. EXERCISE. (Answer orally if possible.) 1. To what terms in percentage do the following correspond: Rate, of discount, list price, discount, reduced or net price? 2. Find the per cent, of list price paid for goods bought at one of the following discounts: 40% (suggestion, 100% - 40%), 30%, 28^%, 63%, 75%, 1/3%. 3. Determine the net cost of: 200 ft. of piping @ $1.20, less 20%. 200 pr. gloves @ $3.60, less 12^%. A $500 piano, less 40%. 40 valves @ $.75, less 15%. 4. How do terms of "90 days, 4/30" affect the settlement of a $950 bill of goods, bought February 7 and settled March 1? 5. Check up the Randall Mfg. Co.'s bill of $552, as follows: 6 sewing machines @ $40, less 25%, $180; 4 washing machines, @ $27, less 30%, $72; 3 farm wagons, @ $85, less 20%, $202. EXERCISE. A bilhng clerk handled and checked the following transactions on Oct. 24. Find missing values. 1. Received of S. P. Adams, check for $ in full settlement for 1200 lb. Japan tea, @ 23c, bought on Oct. 18, on terms of 60 da. net; 3/10. 2. Mailed Waukeska Canning Co. check for $ , for invoice of Oct. 22, 300 bbl. Cerota Flour, 1/4 s. (i. e., 1/4 bbl.) @ $5.80 per bbl. Terms, 30 da.; 1%, 10 da. 3. Checked Parker Bros, invoice of 120 bbl. S. T. Flour @ $4.10; 12 sacks Graham Flour, @ $2.80; 40 doz. C. Tomatoes @ $.92; 144 bx. K. 238 BUSINESS ARITHMETIC. Soap, @ $7.00. Discount, 12^%. Terms: 30 day note, $400; cash for balance. Mailed check for $ . 4. Received check for $ from Franklin Cereal Co., being rebates on September sales of $746.20, at 12^%. Note. Manufacturers a certain lines of groceries sell to wholesalers at prices at which the latter sell to retailers. At intervals the manu- facturers return wholesalers a discount or rebate, based on the amount of their monthly sales. Series of Discounts. Often two or more discounts are allowed on the same item; one, for example, for quantity, and a second for early payment. In such cases, one discount is reckoned on the list price; the second on the list price minus the first discount and so on. Illustration. Successive discounts of 33^%, 20%, 5%, are allowed on a shipment of 4000 ft. iron pipe @ $1.50, reducing the cost to $ . Solution (a). 4000 X $1.50 331% = 1/3 20% = 1/5 5% = 1/20 Solution {b). 1/3 of 6000 1/5 of 4000 1/20 of 3200 100% = $6000 list price. = 2000 first discount. $4000 second price. = 800 second discount. $3200 third price. = 160 third discount. $3040 net cost. 5% of 80% 33i%of76% = iof76 =list price. 100%, list price. 20 one discount. 80% second price in per cent. = 4% second discount. 76% =_25i% third discount. 50f % net cost in per cent. 501% of $6000 = $3040. Note. 100% — 50|% = 49|%, which is called the single discount equal to the series. FOR PROOF OR DISCUSSION. 1. Why does it pay a dealer to allow discounts for the causes men- tioned? 2. Proposition. The order in which the discount rates for a series 3xe used does not affect the final result. COMMERCIAL DISCOUNT. 239 3. Illustrate with the series 33^%, 40%, 10%, the possible arith- metical advantage of changing the order of discounts. 4. Proposition. The single discount equal to a series is always less than its sum. 5. Proposition. The single discount equivalent to a series of two discounts, equals the difference between the sum and product of the series. ORAL EXERCISE. 1. Find the cost, in per cent, of list price, if goods are bought at discoimt of: (See solution (6), illustration 2, page 238.) 40%, 10%, 5%. 5%, 10%. 80%, 3%. 50%, 20%, 10%. 161%, 10%, 25%. 10%, 10%, 10%. 10%, 33i%, 40%. 3 20s (i: e., 20%, 20%, 20%.) 2. Find the single discoxmts equivalent to the following series: 40%, 25%. 20%, 25%. 10%, 25%, 33^%. 16|%, 20%. 50%, 10%. 50%, 50%. 10%, 15% 20%, 25%, 5%. 3. Required, the net cost: (a) Of 400 lb. castings @ 10c, less 20%, 25%. (6) Of 50 pr. gloves @ $2.00, less 10%, 25%. 4. Compare these offers for goods of the same quality: (a) List price $800. Discounts 25%, 33i%, 10%. (6) List price $600. Discounts 16|%, 25%. 5. What case of percentage is involved in these examples. EXERCISE. 1. Find the net receipts from the following sales of merchandise: $326.50 at discounts of 10%, 20%. $415.10 at discounts of 30%, 5%. $300.00 at discounts of 40%, 20%, 10%. 2. Bought, Aug. 30, on terms 90 da., 3/30, 5/10, 4 dz. knives @ $8.00, less 15%; 2 dz. sauce-pans @ $3.40, less 10%, 10%; 3i dz. wash boilers @ $38.80, less 30%, 5%. Paid, Sept. 16, $ . 3. A library has an opportunity to purchase books at 40%, 5% off. A purchase fund of $420 per year will buy $ worth of books at list prices. 4. In selling a bill of 80 rockers @ $8.40, less 25%, 10%, a clerk reckons the equivalent discount at 31%. What overcharge may the customer claim? 5. Both BrowTi & Co. and A. C. Lewis offer me 8000 ft. of choice cypress @ $80 per M., but Brown & Co. quote discqimts of 33^%, 5%; while 240 BUSINESS ARITHMETIC. Lewis quotes 20% and 20%. WTiich is the better ofifer? By accepting it, $ is saved. Rates of discount. In wholesaling and manufacturing, there is constant necessity to calculate rates of discount — owing to fluctuations in market prices and other causes. Illustration. An article listed at $9.00 drops to a market value of $4.00. 33i% discount has already been quoted customers. What further discount rate must be allowed? Solution. The list price =$9.00 33i% discount is 1/3 of $9.00 = 3.00 First discount price 6.00 Required price 4.00 Additional discount to be allowed 2-00 $2.00 dis. must be reckoned on $6. It = 1/3 of $6, or 33|%. Therefore the second discount is also 33 i%. ORAL EXERCISE. 1. Discounts of 20% and ? % yield a selling price of 60% of list. 2. Discounts of ? % and ? % enable one to sell at 50% of list. 3. Discounts of $20.25% and ? % lower a list price of $380 to a selling price of $180. EXERCISE. 1. What discount in addition to one of 15% will lower a list price of $7.20 to a market price of $5.00? 2. The Essex chair manufactured at a cost of $5.00 retails at $8.40. The manufacturer lists at retail price. If he is to gain 20%, he can allow the retailer what rate of discount? Suggestion. Find the manufacturer's selling price. Compare with list. 3. The same manufacturer Hsts at $10 an article costing $6.40 to make. If dealers sell at list price, they should buy at what discount in order that they and the manufacturer may make an equal money profit? EXERCISE. (Rates of profit and loss.) Find missing values. Questions of profits or losses on individual items sometimes arise. Thus a dealer may wish to determine his rate of profit on carpet bought at $2.00 and sold at $4.00 less 20%, 30%. By comparing net selling price COMMERCIAL DISCOUNT. 241 with cost, he finds the rate to be %. Or he discovers that a clerk has lost him $ by discounting at 45% a $120 sale on a series of 20% and 25%. Sometimes the difference of measure affects the problem, as in com- puting the rate of profit on step-ladders bought @ $12.00 per dz. less 50%, 5% and selling at $1.20, findmg it to be %. Perhaps a case arises in which part of an invoice has been sold. Say, 20 out of 35 bookcases, bought @ $8.00 less 40%, 10% have been sold at list, and the balance at 20% profit. This, he finds, yields him a profit of %. Questions of comparative sales arise. Goods costing $6.00 may be fisted at $16.00. An increase of discount rate from 40% to 50% results in increasing average sales 30%. "Does it pay?" he asks, and finds that . Or, a $20.00 chair, costing $8.40 to make is usually discounted 30% to the trade. By experiment over a long period, the dealer finds that his average sales per week increased from 40 to 50 when he allowed 10% ad- ditional discount. By percentage he finds that his present rate of profit is %, where formerly it was %. Some cases of marking goods arise. Illustration. Mark a desk, costing $15.00 to manufacture, at a price to yield the maker 33 §% profit, after allowing discounts of 40% and 16f % to the trade. Solution. Let 100% =fist price. 100% - 40% =60%, 1st discount price. 161%, or 1/6 of 60% =10% 60% — 10% =?50%, second discount price, or selling price per cent. 33i% of $15.00 =$ 5.00 profit to maker. $15.00 +$5.00 =$20.00 selling price. .-. 50% of list =$20.00 100% =$40.00, fist price. EXERCISE. 1. $8.00 rockers, bought at discounts of 25% and 5%, must be sold at $ to gain 20%. 2. At what price must articles costing $18.00 be marked to allow for 331% profit and 5% bad debts? 3. A manufacturer's patent extension table cost $8.40 to manufacture. If he is to allow for 20% profit for himself, and for 20% discount to the wholesaler, he must name what list price? 17 242 BUSINESS ARITHMETIC. 4. The public does not like to pay more than $7.20 for a certain article. Allowing for a manufacturer's profit of 20%, and a dealer's discount of 20%,- the maker must strive to keep the cost of manufacture within $ — EXERCISE. 1. Sold to Robt. M. Drake, Morristown, N. J., via DLW frt. 30 days 2/10; 40 doz. files @ $2.60, less 40%, 10%; 3i doz. hand saws, No. 82, @ $17.50, less 25%, 10%. Write bUl. 2. Compare bids received for the purchase of 100 C machine bolts 3/8" X 6 1/2, needed for stock. R. C. Shafer offers them @ $4.00 less 50%, 10%, 5%. James Casler offers them @ $3.90 less 60%, 10, Newark Bolt Co. offers them @ $4.10 less 60%, 15%, 5%. 3. Extend this bill. Pittsburgh, Pa., , 191— The Morris Hardware Co. New York City. Bought of The Randall Iron Co. Terms: 60 days; 3/10 2 4^ 2i C Machine bolts, 1/2X5 C Machine bolts 7.20 7.52 5/8X4" 4^' 50%, 15%, 5% dz. pr. Hinges, No. 382, 8' Less 60%, 10% dz. Steel Squares No. 11 (< Less 30%, 10%, 5.60 -I — 4.20 2. — — * Note. 4^ hundred machme bolts 5/8" X 4" @ $7.20 per 100, and the same quantity 5/8" X4^" @ $7.52. EXERCISE. (Give an illustration in each case.) 1. What values must be known to determine: (a) list price, (6) missing discount rates, (c) net cost, (d) discount in money, (e) rate of profit or loss? 2. What values can be determined from: (a) list price and series of discounts; (h) series of discounts, two as rate, one also as money; (c) cost and series; (d) cost, desired profit, and rate of discount; (e) two put of three rates, cost and list price? COMMERCIAL DISCOUNT. 243 It is not at all necessary, in the study of many questions of profits, discounts, and even of questions of business policy involving direct financial results, that the fundamental calcu- lation, or the calculations to determine an advisable course, should be based on money values. Percentage is a valuable means of studying business actions, somewhat abstractly, and of deducing results as rates per cent., that are practically formulas. Illustration. A question like this may arise: If merchandise is bought at 40% discount from list, and sold at list, how much of the selling price may be allowed for discounts and selling expense, and yet realize 25% profit. The cost equals 60% of Hst. 25% profit on cost equals 15% of list. The desired selling price equals 75% of list. 100% list —75% = 25%. Therefore selling expense and discounts must be kept within 25%. EXERCISE. Each example in this exercise should be solved without assuming any money value — although the resulting answer may be applied as a rate per cent, to an infinite number of prices and money values. Find missing values. (a) Questions Primarily of Discounts. 1. The portion of the list price paid for goods bought at 34 1% discount is -% . Note. The answer, 65 |%, is general for the given discount, regard- less of price concerned. Apply this answer to finding the cost of articles listed respectively at $4.20, $5.00, 20c, $340. 2. How do terms of "90 da., 3/60, 5/10" affect the settlement of any bill paid within five days of date of purchase? 3. % of the list price is paid for goods bought at discounts of 40% and 10%. Apply, to finding the cost of goods listed at $486, and at $56. 4. The difference between a series of 10% and 40%, and a single discount equal to their sum is %. What is the money difference on a list price of $840? 5. Which is the better *f or a seller to offer — any series of discounts or its sum? 6. Discounts of 25% and x% will lower a list price 50%. 244 BUSINESS ARITHMETIC. 7. Determine a series of two discounts to lower a list price 45%. Suggestion. Assume any rate less than the total discount, for example, 20%. Calculate the other rate. How many solutions are possible? 8. The separate discounts in a series of 3 20s compare how in real value? Illustrate, after obtaining the proportional value, by application to a list price of $8000. 9. Determine a series of two discounts, equivalent to a single discount of 40%, which shall be equal in numerical value. Suggestion. Divide the single discount into equal parts, and find the equivalent rates. The principle applied in this, and in one or two of the following examples, is applied by producers in allowing for middlemen's profits. 10. Determine a series of three discounts equivalent to a single discount of 48%, and equal in numerical value. 11. A series of two discounts is equal to 45% off list, but the first dis- count is twice the second in real value. The series is — % and — %. (6) Questions Primarily of Profit and Loss. 12. % is reaUzed by buying at 10% and 33 i% discount from the fist price at which the goods are sold? 13. When a publisher sells his net books to dealers at 40% discount, the latter reahze ? % profit. 14. Is it better for a retailer to sell at 30% advance on cost, or to sell at a list price from which his wholesaler has allowed him 30% discount? 15. Compare these offers, based on the same Ust price: (a) Discount 20%, 5%, 10%. (6) Dis. of 3 20s. (c) Dis. of 30%, 10%, 15%. 16. My stock is bought at 40% discount from the list price at which I sell, but the expenses of sale average 15% of the selling price. What net rate of profit is realized? 17. A 50% increase in sales, resulting from allowing 20% discount, has what effect on former profits of 30% made by selling at list? 18. Merchandise is bought at 40% discount and sold at 20% discount from the same list price. What is the effect of interchanging rates? CHAPTER XXXII. AGENCY. An agent is one who does business for another. The person whom he represents is termed the principal. Illustrations. (1) Jones, at your request, buys you a saddle horse. (2) Jefferson buys for Chase 400 shares P. R. R. stock. (3) A farmer ships a carload of potatoes to a city produce dealer who returns the farmer the money proceeds from their sale, less his charge for selling. EXERCISE. 1. Give other illustrations of agency in everyday affairs. 2. Name the agent and the principal in each illustration. 3. Why are agents required in business? 4. State some of the advantages and disadvantages of dealing through agents. 5. Name common businesses that are agency businesses. In certain businesses the agent is called a broker (stocks, bonds, notes, etc.), or a factor (cotton), or a commission merchant (produce). Drummers, buyers, attorneys, auction- eers and salesmen are agents. The agent may act without pay, may receive a salary, or may receive a payment, often termed a commission, based on the quantity or value of goods bought or sold. He may receive both salary and commission. The pay of a broker is termed brokerage and that of a factor, commission. The terms fee, share and allowance are also com- mon. Per cent, commissions are based on the exact amount paid (net cost), or received (gross proceeds) by the agent for the goods — regardless of charges. The principal pays his agent for a 'purchase, the net cost plus all charges for freight, drayage, commission, etc. (the 245 246 BUSINESS ARITHMETIC. gross cost). He receives /rom a sale the gross proceeds minus all charges (the net proceeds). Guaranty is an additional fee paid the agent for taking the risk of securing payments from customers to whom he sells. Warranty is an additional fee paid the agent for ensuring the quality of goods. The agent pays, but charges to his principal, freight, drayage, storage, insurance, inspection charges, etc. The agent acts for his principal. The principal, as a con- signor, consigns or ships goods to the agent, or consignee, to be sold. The principal calls the goods a shipment; the agent calls them a consignment. EXERCISE. 1. After carefully reading the preceding sections, define agent, broker, commission, guaranty, warranty, principal, gross proceeds, gross cost, the sum earned by the agent, the sum the principal receives from a sale. 2. Compare the following with the terms of percentage: agent's commission, rate of commission, gross proceeds, net proceeds (if com- mission is only charge). 1. PAYMENT OF AGENTS. The rate of commission is expressed as a lump sum, as a rate per cent, based on the price at which the agent buys or sells, as a fractional part of this value, or as a rate per article. Illustrations. I pay Scott $25 for selling my horse for me. My agent sells 200 bbl. of my potatoes at $2, and charges me 5% of the selUng price for his services. I offer one-fifth of the sum he obtains to a collector who secures payment of a debt due me. Example. What does an agent earn for buying or selling 200 bbl. of flour @ $4.80, on 5% commission? Solution. The selling price = 200 X $4.80 = $960. The commission = 5% of $960, or $48. It is evident that the agent's per cent, of commission is reckoned on the price he pays or receives for the goods. AGENCY. 247 ORAL EXERCISE. Find the commission: 1. On 400 bbl. @ 2c per bbl. 2. On 2000 bu. @ l/4c. 3. On a $3600 purchase at 2|% commission; at 4%; at 5%; at 25%= 4. On 1200 bbl. at 15c per bbl. 5. On 800 bbl. purchased @ $2 on 5% commission. What is the agent paid for his services? 6. For the collection of a $1200 debt on 5% commission? 7. For auctioning off $1800 worth of goods on 3% commission? EXERCISE. 1. Compare a 5% rate of commission with a rate of l/2c per bu. on merchandise costing 30c per bu. Suggestion. Reduce the second rate to a rate per cent. 2. Compare a rate of 3% with a rate of 15c per bx. on goods selling at $2.40 per bx Is it possible to make a comparison in the above case if no money value is given? 3. C. Johnson. $ 40.00 $102.10 $400.00 $206.40 An agent succeeds in collecting three-fourths of this debt for me. If his commission is 6%, he earns $ — . 2. SALES. INTRODUCTORY EXERCISE. 1. You sell for me 10 T. timothy on 5% commission. The customer pays you what sum? You charge $ ?, sending me what sum? Note. The principal receives the selling price (gross proceeds) — charges, or the net proceeds. 2. An agent secures a purchaser for a house and lot who pays $12,600. The agent pays $30 for survey, $10 for transfer, and charges 5% commission. What does the agent receive and what sum does he pay to other parties? 3. Whose money does the agent use to pay for charges? Explain. The standpoint of each party to an agency transaction is different, and the calculations in which he is directly inter- 248 BUSINESS ARITHMETIC. ested vary. Recognition of "viewpoint" is often essential to accurate analysis and solution of agency problems. Illustration. A. B. Newton ships to Robert Kent, commission merchant, 200 bbl. apples which the latter sells at $2.40. He pays freight of 15c per bbl., and charges 5% commission. He mails to Newton a check for $ ?, net proceeds. From the standpoint of Newton the problem is: "I ship Robert Kent 200 bbl. apples which he sells at $2.40, on 5% commission, paying 15c freight per bbl. I should receive $ ? " From the standpoint of the freight agent: "What is the freight on 200 bbl. at 15c?" From the standpoint of the agent (To customer) : "What must I receive for 200 bbl. at $2.40?" (To freight agent): "What freight is due on 200 bbl. at 15c?" (For himself) : "What is my commission on this sale at 5%?" (To principal) : " Considering these previous transactions, what do I owe Newton?" From the standpoint of the customer: "Two hundred bbl. apples at $2.40 will cost me$?" Solution. 200 X $2. 40 = $480 gross proceeds, or sum paid by customer. 200 X 15c = $ 30 freight paid to freight agent by agent. 6% of $480 = $ 24 commission. $ 54 total charges. Gross proceeds — charges = net proceeds, due principal. $480— $54 = $426, net proceeds. ORAL EXERCISE. Memoranda of Sales. (a) 400 bu. oats, @ 40c, commission l/4c per bu. (6) 5 copies Marston's Business Encyclopedia, $14, on 5% commission. (c) A horse, $240; wagon $90; and harness $30. Commission 3%; misc. expense $4. (d) 20 crates eggs, 30 doz. each, 30c per doz. Commission 5%; express, 50c per crate. 1. What do agent and principal receive in each case? 2. Any increase in general charges affects what parties? Illustrate. EXERCISE. Before solving, re-word the transaction from each party's standpoint, and state necessary and unnecessary values for each computation. 1. The consignor should receive a check for $ , as proceeds of 20 crates eggs, 30 doz. each, @ 18c. Commission 6%. AGENCY. 249 2. Find the sum due the principal on a shipment of 60 T. timothy hay @ 90c per C. What sum does the agent earn at 4% commission? 3. S. P. Naylor sells for James Carter 12,000 bu. com at 50 l/4c, commission l/4c per bu. What sum does Naylor receive? Note. In the following, name the new factors, and discuss their effect, and find missing values. 4. I shipped M. C. Parker 360 bbl. apples which he sells at the market quotation of $2.90; on 4% commission and 1% warranty. The customer pays $ ; Parker receives $ ; he earns $ ; and he pays me $ . 5. J. C. Jones receives from his agent a check for $60.40 as net proceeds from a sale of three hundred pounds of butter at 27c, commission 53^%, miscellaneous charges $4.75. Check up the returns. The customer paid 6. 250 bbl. apples, shipped to be sold at the market quotation of $3.20, arrive on a falling market and are sold at $2.90, drayage, 10c per bbl., commission 5%. What effect has the fall in price on the net proceeds? 7. The consignor should receive $- net proceeds from a shipment of 16,000 bu. oats, disposed of at 41c. Freight, 2c per bu.; insurance l/2c; storage, l/2c; commission, l/4c. 8. Extend Account Sales. From John C. Carter, Baltimore, Md., Jan. 17, 1913. To Robert G. Mason, Hyattsville, Maryland. For Merchandise Received Jan. 2, 1913, via B. & O. Consigment No. 4726 FoHo 88 R. M. Green Clerk. Jan. Sales 100 lb. Dressed chicken 450 lb. Dressed turkey 620 lb. Dressed chicken .16 .18 .17 Charges. Expressage $2.45 Drayage $1.10 Storage $3.15 Inspection $ Insurance $ Commission $ (4^%) Other Charges $ Total Charges Net Proceeds 250 BUSINESS ARITHMETIC. 3. PURCHASES. INTRODUCTORY EXERCISE. 1. You buy through an agent, on l/8c per bu. commission, 6000 bu. wheat at 90c. The agent pays $ — for the grain, charges $ commission, and you pay $ . What effect have charges on the cost to the prin. cipal? 2. An agent buys for his principal 80 bbl. winesap apples @ $2.50 on 5% com., and 50c per bbl. for drayage and storage. The net, or prime cost of the good is $ ; the total charges are $ . The principal must pay the agent in full settlement, $ . 3. If buyer and seller had dealt direct, in example 2, who would have saved money? How much? Illustration. CD. Rand sends Robert Gates a draft for $3000 with orders to invest in Greening apples, market quotation $2.40. Com. 5%, drayage 20c bbl., storage 10c. Find all values. Solution. $2.40 net cost per bbl. , (5% of $2.40) .12 com. The number of bbl. = gross cost -_ J gross cost per bbl. .20 drayage .10 storage 1063 = no. bbl. $2.82 gross cost per bbl. 2.82)3000 1063 X $2.40 = $2551.20 prime cost 282 1063 X .12 = 127.56 com. 1800 1063 X .20 = 212.60 drayage 1692 1063 X .10 = 106.30 storage 1080 $2997.66 gross cost 846 $3000 - $2.34 (surplus) = $2997.66, gross cost. 2.340 The principal receives 1063 bbl. and $2.34 surplus. The agent receives $3000, returns $2.34, keeps $127.56 com. and pays $2551.20 for the apples, $212.60 for drayage, and $106.30 storage. ORAL EXERCISE. (Memoranda of Purchases.) (a) 8000 bu. oats @ 40c, commission l/8c. (5) 200 sh. stock @ $80, brokerage $1/8 per sh. (c) 2000 lb. lard @ 10c, com. 2c per 100 lb., misc. charges $5. (d) 80 A. land @ $50, com. 4%, transfer and survey $35. 1. Compute agent's commission in each case. 2. Determine amounts paid and received by each party. AGENCY. 251 Extract from Quotation List. Chicago markets yesterday: Wheat Opening. Highest. Lowest. Closing. July 89i .90i .89 .90^ Sept.. 91 .921 .901 .92| Dec 94| .961-1 .94f .96^ May 991 1.01 .99i 1.00^ Com July 51| .52^ .511 .52^ Sept 51f .521 .51f .52f Dec 48^ .491 .48^ .49^ May 50i .50|-.51 .50 .50| Oats. July 42 .43 .41i .43 Sept 371 .381 .37f .38| Dec 38^ .39i .38| .39i May 401 .41 .40 .40|-.41 Pork.. Sept 16.55 16.55 16.52 16.52 EXERCISE. (See Quotation list where necessary.) 1. Invest $5000 in July wheat, opening quotation, brok. l/8c — no fraction of 100 bu. to be bought. Number of bushels? 2. What is the purchasing power of $2000 in September com, at highest quotation, brok. l/4c. 3. Determine gross cost of 500 bbl. pork, lowest quotation, com. 2|c a bbl. 4. An agent, charging 4^% com., draws on me for what sum to cover the full cost of 200 cr. tomatoes — $2.25 expressage and drayage 40c each? Using account sales form as a model, prepare an accoimt purchase for this transaction. 5. My draft for $700 will pay for how many crates of eggs, 30 doz. each, @ 18c a doz.; com. 4%; drayage 25c? 6. A receives from his principal a draft for $12,000 with orders to secure three mgnths' options on as many acres of certain coal lands as possible. A allows $50 for travelling expenses and 4% for commission. He secures options on 80 A. at 15 % of the selling price, which is $420 per A. 252 BUSINESS ARITHMETIC. The balance he spends m the purchase of 12% options on land priced at $210 per A. On how many acres of this latter tract can he secure options? 7. I bought through an agent 60 cr. choice tomatoes @$2.75; com. 4%; expenses $20. I sold 25 crates direct @ $3.50 and the balance through an agent at $3.80 on 5% com. Find net returns. 8. In accordance with orders, an agent disposes of 200 A. of a tract of 280 A. @ $68, and the balance @ $43, investing the net proceeds in options on timber lands @ $10 per A. His commission being 4% each way and his expenses $50, he can secure options on how many acres? 9. An advertising soUcitor agrees to secure advertising for a book- let, on 25% commission. He secures 35 full page advertisements @ $11.50 per inch. The pages are two column width and 10 inches depth of column. His total commission is $ . 10. Extend this statement of a real estate agent to his principal, charging 5% on collections. Washington, D. C, Feb. 2, 191-. Mr. James Parker, 1382 13th Street, N.W. Dear Sir Enclosed find my check per statement below. Respectfully Robert Crane, Agent. Tenant Premises Explanation Receipts Charges C. P. Lewis 1491 N.St. N.W. Rent to Feb. 1, 19— Repairs to range 60 — 3 60 James Farmer 1280 L St. Rent to Feb. 1, 19— Plumbing repairs Commission 5% Balance due 85 50 7 35 PROFIT AND LOSS. EXERCISE. (Solve mentally when possible.) Determine the rate of gain or loss on : 1. 500 lb. Rio coffee bought @ 8c, on 5% com., and sold @ 9c direct. 2. Quinces bought @ $2 and sold @ $3 on 5% commission. 3. Determine the profit or loss on 1000 bu. com bought @ 70c and sold @ 71^c; brokerage l/4c each way. 4. Spring bran bought @ $19.60 per ton, com. 5% and drayage $1.00 per ton, must be retailed at what rate to yield 15% profit? AGENCY. 253 5. What per cent, of profit is obtained by selling @ $3.20 apples bought through a commission merchant @ $2.50, on 5% commission? Expenses of sale equal 5% of the selling price. 6. What profit is realized in retailing @ $1.50, through an agent charging 20% commission, a subscription book that costs 92 cents to publish? 7. An article, costing $1.80 to manufacture, is to be sold at a price to yield the manufacturer 20% profit, after allowing selling agents 25% commission. Find list price. 8. A sewing machine costs $11.90 to manufacture. Find marked price to allow for 25% profit and 20% commission to agents. 9. How much must the sales of an agent increase to make it profi- table to lower his rate of commission from 5|% to 4^%? 10. A book costing 90 cents to publish is sold @ $1.50 through agents who are paid 30% commission. How is the publisher's rate of profit affected by lowering the commission to 20%? REVIEW DRILL EXERCISE. 1. Buy 40 bbl. @ $5, com. 5%. Find: gross cost; prime cost; commission; sum agent pays seller; sum agent charges principal; sum agent earns; largest sum involved. Repeat, if commission rate is (1) 8c. bbl.; (2) 1/4%. 2. Sell 2000 bu. @ 60c, com. l/4c, storage l/2c. Find: sum buyer pays; sum agent earns; sum handled by agent; net proceeds; the cost of the sale to the principal. 3. State the values that may be computed from these facts. For- mulate and solve simple problems involved in them. State whether purchase or sale. (a) Commission, $4; gross proceeds, $40. (6) Commission, $12; net proceeds, $188. (c) Commission, $60; prime cost, $2000. {d) Commission, $90; quantity 400 bbl. (e) 5% commission on a sale is $22. (/) An agent sends his principal $388 after reserving $12 for himseK. 4. Give illustrations showing what values can be determined from: (a) The commission and a "cent" rate. (6) The commission and a rate per cent. (c) Number of articles and price. (d) The net and gross proceeds. 254 BUSINESS ARITHMETIC. (e) The sum for which an agent sells gdods and the sum he remits his principal. 5. Give illustrations showing from what known values the following may be computed: (a) The commission on a sale; on a purchase. (6) The sum an agent receives in making a purchase. (c) The sum the principal receives from a sale. (d) The gross cost. (e) The sum paid by an agent in making a purchase; a sale. CHAPTER XXXIII. INSURANCE. To Pupils. Collect and bring to class, for reference, old fire, health, accident and life policies, insurance folders, and magazine advertisements that give insurance information. INTRODUCTORY EXERCISE. 1. What is your own definition of "insurance"? 2. Into what two classes can you divide all the classes of insurance? 3. Name several kinds of personal and of property insurance. 4. Give five reasons for taking out life insurance. 5. Give other uses for life insurance than simply to produce a fixed sum at death. 6. Read over a life, accident or health policy. Are any conditions imposed on the person insured, as to occupation, place of residence, etc.? Why should such conditions be made? 7. Give several reasons for insuring one's property. 8. Distinguish between general and special property insurance. 9. Study a fire insurance poHcy and name at least eight conditions imposed on the insured. State good reasons for these conditions. 10. State several ways in which one might "forfeit" his poHcy. 11. Does property insurance protect the insured property against damage? What does it do? Has an insurance comp ny a right to make repairs rather than pay damages? 12. How do strict policy conditions tend to prevent losses? 13. Why do insurance rates vary on different kinds of property? 14. Why should a person always read a policy carefully before taking out insurance under it? A contract is an agreement between two or more parties to do or not to do a particular thing. It is evident that the parties must agree as to the thing desired. It is generally considered, also, that there must be proper consideration — that 255 256 BUSINESS ARITHMETIC. is, a fair return in service or value, for what is received. Naturally, contracts may be of most varied kinds. Among the commonest are those for the sale or rental of property, for the payment of money, for employment and for insurance. Contracts in the form of notes, bonds, etc., have been touched on elsewhere. Contracts for insurance are agreements whereby one party agrees to pay another party a specified sum of money in the event of certain happenings, such as the death or injury of specified persons, or the destruction or loss of property through fire, water, theft or accident. The written or printed document containing the terms is the insurance policy. The face of the policy is the sum, or limiting sum, which the insurer agrees to pay. The premium is the payment made by the insured to the insurer, for protection. Payments of premiums after the first period are termed renewals. Life insurance policies run for a term of years, or for the life of the insured; health and accident insurance poUcies generally for one year; transit insurance (on goods being trans- ported) generally for the period of .travel; fire insurance for terms of one, two, three or five years. The rates of premium on personal insurance are usually expressed in dollars per thousand; on property insurance in dollars per $100 or $1000, or as a per cent, of the face of the policy. The computations involved in the general offices of insurance companies are exceedingly involved and require a high degree of mathematical knowledge. From the standpoint of the insurance clerk and the insured, the computations are very simple and require no new principles. It is evident that insurance is naturally divided into two great groups — personal insurance and property insurance. These will be noticed briefly. INSURANCE. 257 PERSONAL INSURANCE. Besides general life insurance, there are many classes of personal insurance, of which the following are well known: Accident Insurance policies provide for payment to the insured, or to specified beneficiaries, of sums varying with the character and severity of the injury to the insured. There are provisions for payments of surgeons' fees. Payments may be made in a lump sum, or weekly for a specified number of weeks. Health Insurance may be written in connection with acci- dent insurance, or as a separate contract. One form pro- vides for weekly or monthly indemnity for illness arising from a limited number of diseases; others are much more strict in limitations. The contract usually provides for payments for a period not greater that twenty-six weeks and for a specified general payment in case of permanent disability. Liability Insurance is written for employers to cover settle- ments with employees for injuries for which the employer is responsible. A policy covering all or certain classes of em- ployees in a manufacturing plant, for example, is paid for by a premium based on the total pay roll. The insurance company defends for the insured any damage suits brought by employees on account of injuries, and settles all legal or just claims. Automobile liability insurance is also written for any owner of an automobile to indemnify him for liability on account of injuries he inflicts on others while running his machine. Fidelity and Surety Insurance. Fidelity insurance is written to protect the insurer against loss through employees who do not faithfully perform their duties. It covers the bonding of employees who handle money, and is often paid for and taken out by the ones bonded. Surety insurance involves also the issuing of general court bonds, and bonds for the faithful performance of any service or contract. Here the insurance 18 258 BUSINESS ARITHMETIC. company may demand from the person it bonds not only a premium but also certain approved securities on which it can realize funds with which to complete the contract in case of the failure of the bonded person or company. Credit Insurance is in a sense, also, a class of property insurance. This is issued to merchants to protect them from excessive loss from bad debts. Merchants who sell on credit and desire such insurance submit their books to the credit insurance company which averages the losses from bad debts for a series of preceding years. A premium rate is then fixed, in return for which the insuring company agrees to make good any loss from bad debts during the year, in excess of the average loss. The credit insurance company places restric- tions on the merchant, however, to prevent the careless grant- ing of credit to unreliable customers. EXERCISE. 1. Compute the cost of a combined accident and health policy for $7500, if the premium is $5.60 per unit of $500. 2. What should the insurance company pay the holder of the above policy for a serious illness of 9 weeks 3 days, at the rate of $37.50 per week? 3. Compute the cost of a $75,000 bond for a cashier, secured from a bonding company at a rate of $2.45 per $1000? 4. The pay roll of a manufacturing company amounts to $175,000 per annum. A liability insurance company agrees to assume the em- ployer's liability for this force at an annual premium of 3i%. What is the premium? 5. What does the insurance company gain in the above case, if it pays damages of $4580 and incurs legal expense in defense and examinations amounting to $547.50? 6. A credit insurance company examines the books of a mercantile house and finds the normal loss from bad debts to be $4500. It issues a policy for an excess loss of $8000, at a premium of 4%. What is the premium? For the year covered by the policy the loss is $7896. What sum does the insurance company pay? 7. Compute the cost of a $7500 excess policy at 3J%. Compute the normal loss in this case if the respective losses for the six preceding years was $2512, $2681, $2512, $2800, $3126, and $2107. INSURANCE. 259 Life and Endowment Insurance. Insurance on the life of a person is issued under policies of most varied form. The most common classes might be termed straight life, limited payment life, endowment and term. Life policies guarantee the pay- ment of a specified sum at death of the insured, to the bene- ficiary named in the policy, in consideration of premiums paid during the life of the insured, or during a given period. Ten- payment and twenty-payment life policies are common. Term policies limit the guarantee payment in case of death within a fixed number of years. Endowment policies, in consideration of annual premiums paid for a fixed number of years, guar- antee a fixed payment to the beneficiary at death of the in- sured, or to the insured at a certain age. Premiums are usually expressed on a basis of $1000 of insurance, and vary with the age of the insured and the character of the insurance agreement. They are most carefully compiled and are based on the chances of life and the earning power of the premiums or investments of the company. The processes of computation are exceedingly complicated. ^ The reserve of an insurance company is the portion of income from premiums that it is required^ by law to set aside as a fund for the payment of policies when they fall due. The surplus of a company is the excess of assets over liabilities. In the case of mutual companies, or those issuing participating. policies, a certain portion of the surplus is returned to policy- holders whose contracts entitle them to share in it. Many causes may lead or force a policyholder to give up his policy, or to cease paying premiums on it. (Name some.) However, as a rule, if he has paid two or more annual pre- miums, he receives some return for premiums already paid. Thus, he may get cash, on surrendering his policy, equal to practically its reserve value. He may secure a smaller policy in exchange, all "paid up,'' for .whatever amount the reserve 260 BUSINESS ARITHMETIC. will purchase; or he may secure extended insurance for a certain number of years. Many companies will also lend money to the insured, on his policy, up to its cash value. Some companies allow a fixed number of days of grace, say thirty, for the payment of premium after due. Some rein- state, after lapse of a payment, on proof of insurability and payment of back premiums with interest. A good, modern twenty-year endowment policy, for example, at age 35 years, for $2000, in a certain company, if the insured live, will cost $100.72 per year for 20 years, at the end of which time it will produce to the insured $2000 and its share of surplus or accumulation. Statement of Guaranteed Values. No. of Paid up Upon Surrender. lual Prem. Cash or Loan. Insurance. Participating Term. Paid. Value. (Participating.) Insurance for 2 $112 $180 6yr. 21 days. 3 186 292 10 259 4 262 400 14 258 5 342 ■ 512 15 6 424 • 623 14 7 508 724 13 8 596 828 12 9 688 932 11 10 784 1032 10 11 884 1136 9 12 988 1236 8 13 1096 1336 7 14 1210 1432 6 15 1328 1528 5 16 1448 1620 4 17 1572 1722 3 18 1762 1804 2 19 1838 1892 1 20 2000 and accumulations, Bhaxes in dividends, ii addition. INSURANCE. 261 EXERCISE. Assume that the above policy was taken out on July 17, 1910. 1. What sum could be borrowed on the policy at the end of 7 years? On August 18, 1917? 2. If the insured surrenders the policy on May 14, 1916, what amount of paid up insurance should he receive? 3. What sum can he borrow on the policy after 12 premiums have been paid? 4. If the insured surrenders the policy on May 19, 1918, he can secure for it insurance for what term? How much is paid then in case of death 6 months later, no allowance being made for accumulations? 5. In 1912 his dividend was $19.66. He allowed it to go to reduce his premium. What was his net premium in this year? 6. What is the corresponding premium per annum for a policy of $17,000? 7. What cash surrender value has this new policy after 9 annual premiums have been paid? 8. What is the premium on a $12,000 poUcy at same rate? If the insured dies after the payment of five annual premiums, what sum do the heirs receive? What sum in excess of premiums paid in? What sum in case accumulations during the period of insurance have amounted to $1,345.61? Annual Premium Rates for Different Types of Policies. Per $1000 of Insurance. , Ordinary. 10 Pay. 20 Pay. 10 Year 20 Year Age. Life. Life. Life. Endowment. Endowment. 21 $18.50 $46.40 $28.30 $101.68 $47.70 25 20.26 49.34 30.17 102.00 48.10 28 21.73 51.82 31.76 102.29 48.47 30 22.90 53.62 32.92 102.52 48.78 35 26.48 58.68 36.27 103.25 49.80 40 31.12 64.69 40.43 104.33 51.43 EXERCISE. 1. Find the cost of a poHcy of each kind for $4500 issued at age 28. 2. A man at age 30 takes out a 20-payment life policy. When he has completed payments on it, how much less than the face of the policy has he paid in? 3* WTiy does the age of the insured affect the amount of the premium? 262 BUSINESS ARITHMETIC. 4. Give general reasons for the difference in amount of premium for policies of different kinds, at any fixed age. 5. Suppose a man, at age 21 years, had taken out a $5000 policy of each of the above kinds. Suppose that he died at the age of 50 yeajB. What benefit would he have had from each policy during his life? What benefit would his heirs receive in each case? 6. Why is an insurance company able to pay large sums at death of the insured when it receives only small premiums? 7. Name several advantages of insuring at as early an age as possible. 8. Why are intending poUcyholders required to undergo a strict medical examination? Income Insurance. In return for the payment of annual premiums varying as usual with the age of the insured, the insurance company agrees to pay the beneficiary, on the death of the insured, a fixed income per month, for a fixed number of years, or for life, in place of paying a lump sum. Under some policies both a lump sum and an income is pro- vided. If the income is payable during the lifetime of the beneficiary, then the premium rate depends in part on the age of this person. Income insurance can be arranged in income multiples of $10. Old Age and Savings Insurance. This insurance is increasing in this country and is common abroad. For the payment of small monthly premiums, ordinary straight life and endow- ment policies are issued. There are also combined insurance and annuity policies, involving payment in full at death before 65, years of age, and in full, less annuities paid, for death between, say, 65 and 69. Annuities may be paid after 65 years, the annual payment being a certain fraction of the face of the policy. Straight old age pension policies are also issued, wherein a certain annuity is given after the age of 60 or 65, in return for annual premiums paid up to that age. In case of death before the annuity age begins, the amount of the premiums paid in is turned over to the heirs. The maximum annuity is often a small sum, say $200, and the maximum INSURANCE. 263 insurance limit is very small. No medical examination is required for old age pensions. EXERCISE. 1. Compute the annual cost of an old age pension policy ensuring an annuity of $200, purchased at an age of 36 yr., at the rate of $3.14 per month. How much will have been paid in in premiums by the time the insured reaches the age of 60? 2. Anderson takes out an old age policy guaranteeing $400 annuity after. the age of 60 years, and the face of the premiums in case of death at an earher age. He is 34 years old and pays a monthly premium of $1.12 per $100 annuity. Compute his monthly and yearly premiums. In case of the death of the insured at the age of 57 years, what sum should be paid his beneficiaries? 3. Roberts, aged 26, takes out a $400 savings endowment policy payable in twenty years or at death. What is his monthly premium, if the rate is $2.19 per $500 insurance? What is his annual premium? 4. Newton, aged 40 years, takes out an income policy, guaranteeing a monthly income of $60 at his death, to his beneficiary for 20 years, at a premium rate of $5.76 per $10 income unit. What is his annual premiimi? 5. Osgood, aged 30 years, takes out an income policy, calling for the payment to his beneficiary after his death, of a monthly income of $75 for 20 years, at a premium rate of $33.47 per $10 income imit. What is his annual and his quarterly premium? In case of death, at age 47, what will he have paid in premiums? What siun will his beneficiaries receive during the following 20 years? Why is the insurance company's loss less than the difference between these two amounts? PROPERTY INSURANCE. EXERCISE. 1. Is death sure, sooner or later, to every insured person? 2. Is loss by fire certain to occur, sooner or later, to every building insured against it? Is lightning sure to strike every building? 3. Name differences between life and property insiu-ance, as regards probable losses payable by insurance companies? 4. Why are property insurance rates lower than personal insurance rates? 5. Why should fire insurance rates vary with the (a) use of a building; (&) with its surroundings; (c) with the character of material and construction of it? 264 BUSINESS ARITHMETIC. 6. Why is a brick paper mill a greater "risk" than a brick residence^ 7. Why do property insurance companies make restrictions as to how an insured building shall be built or used? 8. Why do fire insurance companies favor and work for strong fire departments? 9. Why are country dwellings insured at a higher rate than city dwelhngs of the same class? 10. Read the conditions of a policy and determine some of the duties of the person whose property is insured, in case of damage to it which is covered by insurance. As in the case of life insurance companies, property insur- ance companies may be either stock companies, owned and managed for the stockholders' profit, under certain legal restrictions; or mutual companies, in which the policyholders share in gains and losses. Some companies are partially mutual. Policies vary greatly in form and conditions. They may be valued policies, stating a fixed amount to be paid in case of loss; or open policies, in which a maximum payment is named but the actual payment is based on evidences of loss. Illustrations. Insurance on ships is frequently covered under valued policies Insurance on cargoes, and ordinary fire insurance is generally covered by open policies. In case of total or partial loss under open policies, the insurance company pays that part of the maximum stated in the policy that will indemnify the insured. Thus, if $2500 damage is caused by fire to a building insured for $6000, the entire damage is paid; but if the damage is $6800, only the maximum of the policy, $6000, is paid. Some policies contain a co-insurance clause requiring that the property shall be kept insured up to a certain per cent, of its value — otherwise that per cent, only of the loss is paid that the insurance named is of the per cent, required. Illustration. If 80% is the per cent, required to be insured then, on a $15,000 building, for example, $12,000 insurance must be carried to receive full benefit. In case of $10,000 insurance, five-sixths of $12,000, only five-sixths of any loss, up to the face of the policy will be paid. Some policies contain an average clause, specifying the INSURANCE. 265 payment of that part of the loss that the insurance is of the value of the property. Illustration. If the insurance on a piece of property is four-ninths of its value, four-ninths of any loss will be paid. Insurance companies reserve the right of replacing or re- pairing damaged property covered by their policies, in place of paying cash for losses. In case the property is insured in several companies, the loss is usually apportioned among them according to the faces of the policies they issued on it. Some- times a single company issues a policy on a valuable property, for a heavy amount, and re-insures a part of its "risk" in another company. Losses are appraised by special inspectors and often agreements are reached with owners, as to the loss payable. The rate of premium varies with the character of the risk and the period of the policy. It may be for a specified number of days, for a year or for a period of years, or in the case of merchandise in transit, for the length of a certain voyage. "Short rates" are often charged for terms under one year, and are relatively higher than the yearly rates. Rates are expressed in cents per $100, or as a rate per cent. Many of the computations of premiums and settlements of losses are based on the simple principles of ratio and of per- centage. No new process is involved in any problems in this book. SOME CLASSES OF PROPERTY INSURANCE. Fire Insurance is insurance of property against loss by fire. The liability covers damage by water in fighting the fire, or destruction of property in the same cause. Sometimes, it covers damage caused by Hghtning, or by wind storms. Marine Insurance is insurance of vessels and cargo against the perils of navigation. This includes damage by fire, or storm, shipwreck, etc. Marine policies frequently contain the average clause. 266 BUSINESS ARITHMETIC. Transportation Insurance is insurance of goods in transit by land, or by land and water, against loss resulting from the ordinary dangers of such traffic. Live Stock Insurance is issued to cover loss by casualties to horses, cattle, etc. Miscellaneous Special Types of Insurance. It is said that it is now possible to insure any property against any damage. The following titles illustrate common types, and are practi- cally self-explanatory: Automobile and equipment insurance, tourists' (personal effects) insurance, motor boat and equip- ment insurance, steam boiler insurance, explosion insurance, theft insurance, plate glass insurance, derailment insurance, fly wheel insurance, etc. ORAL EXERCISE. Find the cost of insurance: Face of Policy. Premium. Face of Policy. Premium. 1. $ 2,000 U% 5. $ 4,000 $1.25 per $100 2. $ 3,600 f % 6. $ 3,000 .80 per $100 3. $50,000 2i% 7. $12,000 1.75 per $100 4. $ 8,000 11% 8. $ 7,500 2.50 per $1000 Find the rate of premium and express in two ways: Face. Premium. Face. Premium. 9. $4,000 $120 11. $12,000 $150 10. $6,000 45 12. $ 5,000 75 13. $400 will purchase $ ? insurance at 2%. , 14. $600 will purchase $ ? insurance at 1\%. 15. $90 is the premium on a poHcy of $ ? at $1.50 per $1000. EXERCISE. 1. Johnson takes out $2500 insurance on his touring automobile at 2% at a cost of $ . 2. What will it cost Robinson to place $1500 insurance on his stable for one year at 25c per $100, and $800 on his horses and carriages at 40c per $100? 3. Newton takes out an 80% policy on his brick house, valued at $7500, at a rate of $2.75 per $1000. It costs him $ . INSURANCE. 267 4. A factory owner takes out $40,000 insurance on his buildings at 3^%; and $3000 special insurance on his fly-wheel, at $12.50 per $1000. His total payment is $ . 5. James Sanborn insures his property through a broker, as follows: Brick and stone house, valued at $24,000, at 75% of its value at $3.75 per $1000; a broad plate glass window, for $400 at lf%; furniture, valued at $12,500, at $5.75 per $1000 for five years; a brick stable and garage, for $2000, at $2.50 per $1000 one year policy; horses and vehicles, $2000 at $4.25 per $1000; automobile and equipment, $2000 at 3^%; frame boat house $600, at 60c per $100; motor boat and equipment, $1000 at 2f %. Compute his total annual cost of insm'ance. 6. A shipment of goods to Manila, valued at $85,000, is insured for 90% value at 11%. What does the insurance cost? EXERCISE. 1. On a house insured for $7500, damage is caused to the extent of $3748. What should the insurance company pay? 2. Randall insures his brick house, valued at $9000, at 4/5 value at 2%. The property is damaged to the extent of $8500. What should the insurance company pay? What is his net loss and that of the company, allowing for premiums? 3. Property worth $12,000 is insured for $7500, under a policy con- taining an 80% clause. In case of proved damage to the extent of $2750, what should the insurance company pay? 4. On a stock of goods amounting to $30,000, insurance is carried as follows: Company A, $4000; Company B, $6000; Company C, $15,000. The property is damaged by fire to the extent of $16,500. What should each company pay? 5. A stock bam, insured for $2500 against damage by tornados at 80c per $100, is damaged in such a storm to the extent of $3750. What is the real loss of the owner? 6. Prepare an illustrative example showing how the losses paid to a poUcyholder are met by other poUcy holders in the same company. 7. Robinson bought merchandise abroad to the value of £3000. (£1 = $4.86). He paid a commission of 2%, and insurance of 3/4 value, at 11% (average clause). The entire shipment cost him $ . Fire broke out in transit and the goods were damaged to the extent of $5860. What should the insurance company pay? What was Robinson's net loss? 8. Fire broke out on the estate of James Sanborn (ex. 5 of previous exercise), causing loss of $19,200 on the house; -destroying all but $800 268 BUSINESS ARITHMETIC. worth of furniture; destroying the plate glass window; damaging the stable to the extent of $1500; destroying the boat house, and damaging the motor boat to the extent of $350. Xhe insurance company repaired the motor boat, and sent a check for the other losses for which it was responsible to Mr. Sanborn. The face of check was $ . 9. The "Warfield," freighter, valued at $180,000, is insured under average clause poUcies, for $40,000 in Company A and for $65,000 in Company B — the former at 21% and the latter at 2^%. (Pre- miums?) On one trip, the vessel carried a shipment of 28,000 bushels of corn, valued at 61c per bu. and insured at 1^% to cover goods and charges. (Face of policy and premium?) The vessel also had aboard some general merchandise valued at $85,000 and insured for 3/4 value at lf% — average clause. (Premium?) Fire broke out on this trip. The general merchandise was destroyed or thrown overboard. The insurance company sold the grain that was not destroyed, amounting to 7520 bu., at 31c per bu. The damage to the vessel was appraised at $57,800. Compute the payments by each company, assuming that Company B carried the insurance on both grain and merchandise. Compute the net losses of the companies and of the owners, taking premiums into account. CHAPTER XXXIV. TAXATION AND PUBLIC REVENUE. GENERAL INTRODUCTORY EXERCISE. 1. What protection does a city government give its inhabitants? 2. What besides protection does it supply? 3. What does it do for the young people? 4. How does the local government aid travel and intercourse? 5. How are your public officers chosen? 6. Who can vote? Does it cost anything to vote? 7. How IS the money for local government expenses secured? 8. What kind of property is taxed? 9. Why must many business men pay for a license to do business? 10. What does your state government do for its citizens? 11. How does it raise money for these objects? 12. Name twelve public objects for which the United States Govern- ment pays money. How are the people benefited in each case? 13. How does the National Government aid commerce and industry? 14. How does the National Government raise its funds? 15. Why does the cost of government increase steadily? 16. In what ways are governments doing more than they used to do for their people? 1. LOCAL AND STATE TAXATION. The cost of local and, to some extent, of state governments is met by a tax or charge levied on real estate, in proportion to its value; and on general personal or movable property — such as furniture, machinery not built in as a part of a building, horses and wagons, and sometimes, stocks and bonds. A further income is derived from fees for licenses, or permits to conduct special businesses; from assessments, or charges 270 BUSINESS ARITHMETIC. against private real estate, for improvements in the form of sidewalks, water mains, etc.; and from fines and 'permits. Polls or capitation taxes are levied on all qualified voters. EXERCISE. Secure a printed statement of your own, or a nearby, city showing total expenditures for a recent year for governmental purposes. Show what are the largest items of expense. Find the average expenditure per person. The greatest source of public income is realty. The process of taxing realty in one Eastern city is as follows: A complete assessment (valuation) of real property " at not less than two- thirds of the real value thereof," is made every three years by tax assessors. The chief assessor and his assistants then sit as a Court of Equalization and Review to hear owners and witnesses, readjusting any seemingly unfair assessment. Their decisions are then confirmed by the governing ofiicials. Special assessments are made annually of property that has changed in value $500 or over, owing to its improvements, or to destruction of buildings. The tax, 1|% of assessed value, is payable to the collector of taxes in May for the year ending June 30th. Tax rates on real or personal property, expressed as mills • or cents per $1.00 or $100, or as a rate per cent., are based on the assessed value. Thus, 1^ %, 15 mills (1| % of $1.00) or $1.50 (on $100) are equivalent rates. Several taxes, such as local, county and state, may be levied on the same property, and collected at one time. ORAL EXERCISE. Express each rate in two other forms: 1. 1.8%. 3. li%. .5. 12 mills ($1.00). 7. 22^ mills. 2. 3/4%. 4. U%. 6. 16 mills. 8. $1.75 ($100). Find total rates equivalent to these series: 11. 2/5%, 8 mills, $1.50. ' 13. U%, 75c, 7J mUls. 12. li%, 3 mills, $1.25. 14. 75c, $1.25, 2\%. TAXATION AND PUBLIC REVENUE. 271 The computation of the tax involves simple percentage or multiplication. Illustration. Compute state and local tax at 1|% and 8 mills, on property assessed at $4200. Solution. State tax = U% of $4200 = $63.00 Local tax = 8 mills per $100 = 4200 X $.008 = $33.60. Note. The total tax may be found directly. 8 mills = 4/5%. 1|% + 4/5% =2x\%. 2j%% of $4200 = $96.60. ORAL EXERCISE. Find total tax. Hates. Assessed Value. Rates. Assessed Va 1. 2/5% $4500 5. 8 mi. $24,500 2. U% 7280 6. 1/4% and 6000 3. $1.25 14,500 $1.50 4. .50c 8420 7. 15m., 2^m. 8400 EXERCISE. Find missing values. State each example in form of a question, and show how it differs from preceding problems of a similar nature. 1. John Sampson's property is assessed for $12,600, at 11%. His tax is $ . 2. C. P. Crane has assessed at $1.25 per sq. ft., a lot measuring 240 ft. X 80 ft. His tax, at 1^%, is $ -. 3. Robert Walter owns three lots, assessed at $8420, $9200 and $14,620. On the latter is a building assessed at $16,500. He has personal property assessed at $4500, but $1000 is exempt by law. His total tax at 12| mills is$ . 4. A Permanent Improvement Fund tax of 2§ mills, levied by a western city on property assessed at $26,450, costs its owner $ . 5. A Current Expense Fund is created, in a Minnesota city, by a tax of 6/10%. On a realty assessment of $99,547,484 and a personal property assessment of $29,049,250, this would yield what sum if every person paid? In this city, the delinquent tax averages 4|%. What net tax should the property yield? 6. A western city recently levied this general tax per $1000. What does it cost C. A. Mansfield whose property is assessed at $8455? State revenue S1..50 State School and University Fund 2.23 272 BUSINESS ARITHMETIC. County revenue 2.75 School revenue 6.00 City funds 14.37 ? 7. James Field, collector for the town of Mayfield, receives a salary of 3% of his collections. If he collects a $1.25 tax on $364,000 of property, he earns $ — -. Note. Penalties are charged for delinquent (unpaid) taxes. In one city, penalty interest of 1% per month is charged. In default of payment, the property is sold, but the purchaser gets simply a tax deed bearing 1% per mo. interest, and the owner may redeem his property within two years. 8. The property here shown is as- 120' sessed at $30 per front foot, and taxed 8 mills. The tax due on Jan. 1, was not paid until Oct. 1, incurring a penalty, at 3/4% per month, of $ . 9. A tax certificate of $241.60, bear- ing 1% interest per month, dated Dec. 10 last, is worth to-day $ . 10. J. C. Parker leaves his wife by will, $42,000 hi property; his son, $12,- 600, and a niece $5200. The state has an inheritance tax of 1?% on bequests to Imeal heirs, and 6% to others. The tax on these bequests yields the state $ . 11. The Randolph Manufacturing Co. is taxed 1% of its legal capital, on issuance of its state charter. If the authorized capital is $240,000, the tax is $ . 12. An interurban trolley road is taxed 90c per $100 on its equipment and 1/2% on its gross earnings. Last year its equipment was assessed at $8,240,000 and its gross earnings were $6,547,325.60. Its state tax amounted to $ . Many problems arise in tax offices when making up advanced estimates. A few are stated below. EXERCISE. 1. What rate in mills and per cent, is sufficient to raise $12,460 on an assessed value of $800,000? 2. In a city having a fixed legal tax rate of 15 mills, the total assessed value must be fixed at what sum, in order that an income of $172,000 may be secured? TAXATION AND PUBLIC REVENUE. 273 3. Last year, in a Nebraska town, the legal rate of 3| mills yielded $142,500. This year the income required being only $122,000, the assessed value may be reduced to approximately %. 4. A Pennsylvania town must meet the following appropriations by a tax of ? mills, on an assessed valuation of $1,246,300: salaries $2140; schools, $8720; roads, $2465; general expenses $5840. Assume 5% deUnquent tax. 5. 520 polls @ $1.50; Ucense fees, $1260; and a $1.25 tax on realty and personalty, assessed at $1,652,000, will yield an annual income of $ . • For convenience in calculation, tax computation tables are often prepared for the total tax rate of a town or city. The following table gives the tax, at the given rate, on any sum from $10 to $99. The tax on other sums is found by " pointing off " and by addition. The original table must be computed with great care, and written, preferably, in six decimal places. 7§ mills Tax Table Per $100 Unit lines 1 2 3 4 5 6 7 8 9 1 .075 ,0825 .0900 .0975 .0150 .1125 .1200 .1275 .1350 .1425 1 2 .150 .1575 .1650 .1725 .1800 .1875 .1950 .2025 .2100 .2175 2 3 .225 .2325 .2400 .2475 .2550 .2625 .2700 .2775 .2850 .2925 3 4 .300 .3075 .3150 .3225 .3300 .3375 .3450 .3525 .3600 .3675 4 5 .375 .3825 .3900 .3975 .4050 .4125 .4200 .4275 . 4350 .4425 5 6 .450 .4575 .4650 .4725 .4800 .4875 .4950 .5025 .5100 .5175 6 7 .525 .5325 .5400 .5475 .5550 .5625 .5700 .5775 .5850 .5925 7 8 .600 .6075 .6150 .6225 .6300 .6375 .6450 .6525 .6600 .6675 8 9 .675 .6825 .6900 .6975 .7050 7125 .7200 .7275 .7350 .7425 9 0123456789 Illustrations. The tax on $82 is found on the horizontal line "8" at its intersection with the vertical unit line "2." It is $.615 or 62c. The tax on $8200 (100X82) = $61.50. The tax on $972 = tax on $900 + tax on $72, etc. EXERCISE. 1. Show how the above table may be constructed by addition, how short methods may be used for construction or checking. 19 Show 274 BUSINESS ARITHMETIC. 2. Read from the table, the tax on assessed values of: $840 $24,200 $1,220 $ 815 1200 36,450 19,600 2,290 960 8,000 452 42,600 3. Use the table to read the tax on these same values at 15 mills. 4. Construct tax tables for these rates: (a) 1^%; (6) 60c; (c) $1.45; id) $27.60 (per $1000). Licenses. Many municipalities secure an income from a charge for licenses to conduct, certain businesses. An eastern city received from this source, in one year, $572,473.43. These are a few of its rates: Busmess. Rate. Remarks. Apothecaries $ 6.00 per annum. Auctioneers 100.00 per annum. Boarding houses, hotels, public. . 1.00 per guest room. Brokers, real estate 50 per annum. Carriages, for hire 6.00 one horse. Carriages 9.00 more than one horse. Circuses 200.00 per day. Confectioners 12.00 per annum. Livery stables 25.00 per annum 10 stalls. Livery stables 2.00 each additional stall. Passenger transfer lines 6.00. each vehicle seating ten or less. Passenger transfer lines 12.00. . . .each vehicle seating over ten. EXERCISE. Using these license rates, tabulate the income derived by a city from 61 apothecaries, 18 auctioneers, 529 street cars, 63 confectioneries, 17 livery stables (under 10 stalls), 8 stables — total of 286 stalls, 282 carriages for hire; 41 brokers; 21 hotels, guest rooms respectively: 149, 217, 300, 84, 29, 80, 120, 360, 400, 210, 88, 146, 97, 63, 48, 88, 95, 85, 66, 22, 38. Assessments. In some cities a portion of the cost of public improvements and of repairs, such as curbing, sidewalks, water mains, sewers, alleys, etc., is levied by assessment against property that is especially benefited. If an owner asks for improvements, he must deposit his estimated share of the cost in advance. If he does not ask for the improvements, his payments may usually extend over two years. The TAXATION AND PUBLIC REVENUE. 275 proportion paid b} private owner and the methods of assess- ment vary greatly in different cities. Several different methods are illustrated in the following problems. EXERCISE. 1. John Evans is assessed $ ? for a water main, at a rate of $1.10 per front foot, on two lots of 85 ft. and 40 ft. frontage respectively. 2. 320 ft. of curbing are laid at a cost of 90c per foot. The frontage benefited is 300 ft., of which Adams owns 65 ft. He is assessed one-half the pro rata cost, or $ . Note. The cost of grading and paving streets is sometimes paid in full by the city or town; sometimes assessed one-half on abutting property, according to frontage, sometimes one-half according to area, sometimes part by frontage and part by area. Owing to the fact that comer lots may be assessed for improvements on two streets, their rates are sometimes reduced. Areas are measured from established building lines. 3. Rockford Avenue (see page 276) is paved with asphalt from First to Second street at a cost of $2.25 per square yard. The area paved is 60 ft. by 360 ft. If one-half of the cost is assessed on property owners, what sum must be met by the owners of frontage on one side of the street? If this cost is shared according to frontage, what hts must be assessed? Andrews, who owns lots No. 266 and No. 265, pays $?; Chapman, for No. 270, and No. 271, pays $?; Eastman, the owner of No. 267, 268, 269, 270, and No. 273 pays $?. 4. Find assessments (ex. 3) if comer lots are assessed on basis of 1/2 frontage. 5. Determine assessments if 25% is levied by frontage and balance by area. 6. A curbing is laid from the alley around these lots, 25 ft. from building line, at a cost of 35c per ft. Assess one-half the cost against the lots ac- cording to frontage. 7. The property fronting on Rockford Avenue is assessed 65c per front foot for water mains. What does each owner pay? 8. The alley is paved at a cost of $1.20 per sq. yd. from 1st to 2nd street. One-fourth of the cost is assessed by area against lots Nos. 264-273. Determine the "lot" assessments. II. NATIONAL GOVERNMENT INCOME AND EXPENSE. The main sources of National Government income and expense are shown in the tables that follow. There are many minor sources also, such as profits on coinage, fees, sales of 276 BUSINESS ARITHMETIC. SECOND STREET ~ — tsj — 03 -;^ ^ N3 cn ^ 01 NJ S 05 CD 10' 05 § _» -v4 ■ ra N> c m 5 5 > N3 > r- 03 ■~ ?; m ro ■< CT} m CO -- N3 ■-vj 5^ C3 10 to N3 -vi -vj ■^ u hO 40' 40' 40' FIRST STREET public land, etc. The internal tax on liquors, tobacco, etc., is a stamp tax, and is not of such general character as to require notice. The other taxes are paid in money. TAXATION AND PUBLIC REVENUE. Ui ^.i ^«-S i (N- C^-- C--. S^. £ p^ ^ ^1 lOOSOOi-H kH i Ttl rH lO lO Tfl lOCO l^ H < coco'oc'Tjr 00 CO (N OS g w ;> << >H ^.=^.n«l OD C0O5TtHi-H Eh CO 00 (N »0 w g ^ COtNCq. H H P^ co-^ oOi-H ;z; OOC005_ o l-l g e T-Tc^TfM'TjT Q Fh > >-H ,-( CO CD o '-' w i .-H^»-^CO^rJ^^ o co'i-Tod'co' s o t-i ^T^t^CO o H i~] i-i t^Tj< (M '-13 CO CO^rHrH^eO s 1 >- 00 1OO5CO 3 6 (N(Ni-i <2 CO CO CO Tt* CQ Ot^O (M 1^ 1 cot>^o^io .2 C0<:O*S^ *s H ^ CO CO 00 (N t^ M C^^CO^O(M^ CO CO 00 CO d 12; Q -S g CO(Ni-i 1 H g 1 00 CO CO '-i I^r-H OOlO 00 (M t^ 00 -?* p^ H g & -H^OVflO Ph o M lO lO CO »-( (Nt^osco 02 ooTtCio O'^t^cO^ O CO(NrH ■ p 1 P5 1 nil o 3 . CO 00 lO 05 1-ICO crTi-H'ccr T-HfNOs co^cq^io CO .-Too ^CO 00 OiO 1-1 05 CO EXERCISE. 1. Complete the tables. 2. Define the terms in the first column. 3. Where does authority for expenditures rest? What is an appro' priation? 278 BUSINESS ARITHMETIC. Customs. The National Government levies duties or customs on many classes of articles, for revenue and for protection of home industry. Congress has adopted a tariff or schedule showing the rates of duties on imports; and a free list, or schedule of articles imported exempt from duty. Duties are either ad 'valorem (levied as a per cent, of their net value) or specific (levied by measure). Both classes of duties are imposed on some imports. In general, classes of goods varying markedly in quality, and therefore in value, are taxed '' ad valorem.'' These frequently are manufactures, such as carpets, crockery, knives, etc. Raw materials, and many articles of fairly uniform grade, are taxed specifically, for example, rivets, gold leaf, eggs, wheat. Ad valorem duties are not computed on fractions of a dollar, less than 50 cents being discarded, and 50 cents or more being considered an additional dollar. The same rule applies, in specific duties, to fractions of the unit measure. The Government employs the long ton (2240 lb.) in weighing, but makes allowances, in many cases, for tear and breakage. Rates from a United States Tariff. Camphor, refined, six cents per pound. Castor oil, thirty-five cents per gallon. Paris green and London purple, fifteen per centum ad valorem. Castile soap, one and one-fourth cents per pound; medicinal or medicated soaps, twenty cents per pound; fancy or perfumed toilet soaps, fifty per centum ad valorem. Lime, five cents per weight of one hundred pounds, including weight of barrel or package. Cast polished plate glass, finished and unfinished, or unsilvered, not exceeding three hundred and eighty -four square inches, ten cents per square foot, above that, and not exceeding seven hundred and twenty square inches twelve and one-half cents per square foot; all above that twenty- two and one-half cents per square foot. Pen knives, pocket knives, clasp knives, valued at not more than forty cents a dozen, forty per centum ad valorem; valued at more than forty TAXATION AND PUBLIC REVENUE. 279 cents per dozen, and not exceeding fifty cents per dozen, one cent per piece, and forty per centum ad valorem. Gold leaf, thirty-five cents per one hundred leaves. Shingles, fifty cents per thousand. Horses and mules, valued at one hundred and fifty dollars or less per head, thirty dollars per head; if valued at over one hundred and fifty dollars, twenty-five per centum ad valorem. Buckwheat, fifteen cents per bushel of forty-eight pounds. Carpets, woven whole for rooms, and Oriental, BerHn, Aubusson, Axminster and similar rugs, ten cents per square foot, forty per centum ad valorem. Firecrackers of all kinds, eight cents per pound, the weight to include all coverings, wTappers and packing material. EXERCISE. 1. Define: Income, duties, revenue, tariff, free list, miports, ad valorem, and specific. 2. Which is simpler to levy, a specific or an ad valorem tax? 3. State the kind of duty specified in each of the given sections of the tariff. Suggest a reason for the kind of duty. The country is divided into collection districts, each con- taining a port of entry and one or more ports of delivery. All imports must be brought first to the port of entry at which is situated a custom house for the collection of duties and for the official entry or clearance of vessels. After duties are paid and permission given to unload, goods may be discharged at any port of delivery. Thus the collection district of the City of New York has two ports of entry, New York and Jersey City, which, as usual, are also delivery ports; and has addi- tional ports of delivery at New Windham, Albany, Cold Spring, Port Jefferson and Patchogue. In the larger ports, a Collector of the Port is the chief officer, having under him appraisers to determine values, and sur- veyors ^ weighers, gaugers, etc., to determine quantities. In brief, the process of importing goods is as follows: The seller or shipper at a foreign port makes out an invoice of the marks, number, quantities, value, etc., of the goods shipped, 280 BUSINESS ARITHMETIC. weights and values being stated in the standards of the foreign country. The invoices, if over $100, are certified by the local U. S. consul who retains a copy and sends a second copy to the collector of the port at which the goods are to be entered. On loading the goods aboard ship, the shipper re- ceives duplicate bills of lading, or receipts involving a contract to deliver at a specified port The shipper forwards copies of invoice and of bill of lading to the purchaser in the United States. The latter fills out certain forms and presents them, with the invoice and bill of lading, at the custom house, pays the estimated duties, and receives a permit to land the goods subject to examination and verification. The permit goes to the Surveyor who is in direct control of the vessel and its cargo. The goods are then landed and examined and appraised, to determine And check quantities and dutiable value. One package in each invoice, one package out of every ten of the same kind and same price of each class of material, is examined carefully. The master of the vessel also submits to the Collector of the Port a manifest, or state- ment of the name of the vessel, of the names of consignors and consignees, and of the items of his cargo. If the importer does not care to use and pay duty on his goods, at the time of receipt, he may give bond to pay the duties, and may store the goods in bonded warehouses, maintained by the Government. All or part of the invoice, within certain minimum limits, may be withdrawn as desired on payment of duties and reasonable charges for storage and incidental labor. Such goods may be withdrawn for export without being subject to duty. If the goods are retained in storage over one year, however, the duty is increased 10%. After three years, the goods are subject to forfeit. If duties have been paid and imports afterwards exported, a draivback or refunding of the duties is granted. For the collection of the excise, or internal revenue tax, the TAXATION AND PUBLIC REVENUE. 281 country is divided into districts each in charge of a collector. In many cases, the duty is paid by affixing to packages of manufactured goods the requisite amount of tax in the form of adhesive stamps which may be purchased in quantities and used as needed. The stamps must be destroyed when the packages are opened. Drawbacks are allowed on goods made in this country that are exported after the excise tax has been paid. As the invoice value of imports is expressed in the coinage of the exporting country, it is necessary to reduce such values to United States standards. Coinage reduction tables are issued periodically by the Director of the United States Mint. Values of Foreign Coin in United States Gold. (Foreign gold coins unless otherwise stated.) Country. Coin. Abbreviation. U S. Value in Gold Argentine Republic Peso p $.965 Austria Krone kr 203 Belgium, France, Switzerland . Franc fr • 193 Brazil Milreis ml 546 Chili Peso p 365 German Empire Mark M 238 Great Britain Pound Sterling . £ 4.8665 Italy Lira 1 193 Japan Yen y 498 Mexico Peso, Dollar (sU.) 498 Netherlands Florin fl 402 Philippines Peso p 500 Russia Ruble r 515 Computations under this table involve simple multiplication. Illustration. Determine the IT. S. value of a shipment of hides from Argentine, invoiced at 4200 pesos. Solution. From the table, 1 peso = $.965. 4200 X $-965 = $4053, the value of 4200 pesos. ORAL EXERCISE. .Determine value of these invoices in U. S. gold. 1. From Argentine, 2000 pesos. 3. From France, 120Q fr. 2. From England, £ 600. 4. From Germany, 5000 M. 282 BUSINESS ARITHMETIC. 5. From Chili, 250 p. 8. From Russia, 400 r. 6. From Mexico, 1000 pesos. 9. From Philippines, 720 p. 7. From Netherlands, 300 fl. 10. From Japan, 10,000 yen. EXERCISE. Determine the value in U. S. gold of these invoices: 1. From Russia, 7246.5 r. 5. From London, £365.82. ■ 2. From Japan, 864.36 yen. 6. From Amsterdam, 4368.20 fl. 3. From Switzerland, 15,329 fr 7. From Havre, 32,129.6 fr. 4. From Germany, 2468 M. 8. From Naples, 673.80 1. 9. Of bone and tallow from Argentine, 6754 pesos. 10. Of nitrate of soda from Chili, 73,854 pesos. 11. Of coffee from Brazil, 785.25 ml. 12. Of knives from Sheffield, £542.10. 13. Of glass from Bohemia, 529.65 krone. Specific Duties. When specific duties are charged, the reduction of foreign measures to U. S. standards is necessary. Great Britain uses measures similar to our own, and most other exporting countries use the metric system. Once the invoice quantity is expressed in U. S. standards, the computation of the specific duty is a matter of direct multiplication, except where the wording of the U. S. tariff necessitates a further computation. Illustrations. (1) What is the duty on 50 M. shingles at 50c per M.? Solution. At 50c per M. the duty on 50 M. =50 X 50c = $25.00. (2) What is the duty on 7200 lb. buckwheat at 15c per bu. of 48 lb.? Solution. The number of bu. = 7200 -^ 48 = 150. 150 X 15c = $22.50 duty. ORAL EXERCISE. See Tariff, pp. 278, 279 for missing rates. Compute the duty on: 1. 2400 bu. wheat @ 25c. 6. 625 lb. camphor. 2. 60 gr. pens @ 12c. 7. 3000 lb. lime. 3. 2750 lb. rivets @ Uc. 8. 28 pkg. gold leaf. 4. 16 T. grindstone @ $1.75 per T. 9. 10 pc. plate glass, 24 in.X 6. 4600 lb. Castile Soap. 30 in. TAXATION AND PUBLIC REVENUE. 283 EXERCISE. Compute the duty on: 1. 246 pc. plate glass, 16" X 24". 2. 7600 ft. bar iron, weighing 1.2 lb. per foot, @ 6/lOc per ft. 3. Three eases of fire crackers, weighing respectivelySlS, 129, 142 lb., including all packing material, @ 8c per lb. 4. 1520 short tons of barley, @ 30c per bu. of 48 lb. 5. 320 bx. Castile soap, averaging 110 lb. per bx., allowance for tare 5%, 6. 22 long tons of lime. 7. 60 gr. bottles of mineral waters @ 20c per doz. 8. Find the entire duty on this invoice. Sheffield, Nov. 17, 191- Invoice of Hardware Purchased. By James Evanston of Baltimore, Md, From The Peter Mfg. Co. of Sheffield. To he shipped per Cambria. Marks. Quantity. Description. Price. Amount. Consular Corrections. ^68 <$>69 ^70 6 160 42 gr. Files, lb. Bolts, lb. Wood screws, Total. U. S. Money. Duty on files, 25c doz. Duty on bolts, l^c lb. Duty on screws, 8c lb. £ s 3 1 1 d 6 9 £ XX X X s X X X d X X X Ad Valorem Duties. The computation of ad valorem duties often involves the reduction of money values to U. S. gold standard, and always involves percentage. Illustration. Compute the duty (40%) on forty watch cases invoiced @ 36 fr. Solution. The value of forty watch cases = 40X36 fr. = 1440 fr. 1 fr. = $.193. 1440 fr. = 1440 X. 193 = $277.92. The dutiable value is $278. The duty = 40% of $278 = $111.20. 284 BUSINESS ARITHMETIC. EXERCISE. Compute the duty on: 1. An invoice of London purple, valued at £46 8s. 2. 8 doz. pen knives @ 0/1/6 per doz. , 3. 40 head of horses from Canada, @ $135. 4. An invoice of crockery, 675 fr. Duty 60%. 5. A collection of orchids from Brazil, @ 526 ml. Duty 25%. 6. An invoice of swords, valued at $4264.14. Duty 50%. 7. A set of "standard authors" valued at £64. Duty 25%. 8. Compute ad valorem duty at 60% on this invoice: Mme. S. Ren ant. Robes Manteaux Modes. 62. Place du TrocaMro Madame C. Fanning, Dr. Paris, le 18 Novembre, 191— Fr. 2 Robes, garni motif 775 Robe, en foulard 310 Robe, en draps bronze, Princesse 400 Robe, drap gris, brader 526 Blouse, en crepe de chene blanc 92 Blouse, taffeta bleu 65 Note. Both ad valorem and specific duties are levied on many imports. 9. A rug, 6' X 8', valued at 146 fr. (see tariff). 10. 86 lb. celluloid manufactures, valued at 756 fr. Duty 65c per lb. and 30%. 11. 1200 m. oil cloth, 2 yd. wide, @ 2.4 M. per m. Duty 6c sq. yd. and 15%. 12. Extend this form: Manifest No. 868. Invoiced at Amsterdam, Netherlands, Nov. 29, 19 — . Inward Foreign Entry of Merchandise. Imported by J. J. Jervis In the steamer Kriedam A. B. Roe Master From Amsterdam. Arrived Dec. 15, 19 — No. 920-1 922-3 Packages and contents 2 cas. Brussels carpet 3/4 yd. wide. 2 cas. Tap. 3/4 yd. wide. 40% of $ ? yd. @ 44c ? yd. @ 40c Quan- tity Free List 44^8q.yd. and 40^ ad valorem 40^Bq. yd. and 405t ad Talorem Duty 400m. 420m. 600 fl. $? 840 fl. $? $? ? ? Total CHAPTER XXXV. INTEREST. Interest ranks with the fundamental processes, and with simple percentage, as one of the most useful branches of arithmetic. It has more general and varied applications than simple percentage, and its use is being constantly extended. It is the great interpretative branch of arithmetic and has to do, especially, with the study of values. INTRODUCTORY EXERCISE. 1. Give business reasons for borrowing money. 2. Need a prosperous business ever borrow money? 3. Why is money paid for the use of others' money? 4. I owe Smithson $500, due to-day. If I delay payment for a month, does Smithson gain or lose? Why? 5. Does the time I keep another's money affect the charge for its use? Interest has double meaning. Strictly, it is the use of money. Commonly, it is the sum paid for that use. The sum on which interest is charged is the principal. Interest is computed as a per cent, of this principal per year, with relatively less amounts for portions of a year. Each state has a legal rate of interest, used when no rate is stated in interest-bearing obligations. By agreement, a lower rate may be charged; or in some states, a higher rate up to a maximum fixed by law may be charged. A requirement to pay higher than the legal maximum is termed usury. Commercially, the interest year is reckoned as 360 days, or 12 months of 80 days each. If, however, the time is a year or more, calendar years are taken for the full years. The United States Government, and some banks, reckon interest 285 286 BUSINESS ARITHMETIC. on a basis of 365 or 366 days. Interest tables commonly are used to save computation. For long periods, interest time is computed by compound subtraction; for short periods the exact number of days is used, as a rule, this depending, however, on the custom of the business or the character of the obligation. ORAL EXERCISE. 1. Determine the interest on $6000 for one year at 6%. At 8%. 2. Find the interest on $1200 for 1 year at 5% ; for 2 years ; for 6 months; for 2 months. 3. What part of the interest for a year is the interest for 3 mo.? 2 mo., 4 mo., 60 da., 90 da., 80 da., 120 da , 30 da.? 4. Determine the interest on $800, at 4%, for 1 yr., 3 mo., 4 yr., 2 mo., 90 da., 6 mo., 30 da. For a fixed rate and principal, how is interest affected by a change of time? 5. How does interest at 4% compare with interest at 5%, on the same principal? 6. If the interest for a fixed time, on a given principal is $32, at 4%, find the interest at 5%, at 6%, 2%, 8%, 1/2%, 4|%. What is the effect of increasing or decreasing the rate, if principal and time remain constant? 7. The interest on a given principal, at a fixed rate, for 20 days, is $1.60. Find the interest for 5 da., 10 da., 30 da., 40 da., 2 mo., 80 da., 11 da., 3 mo. What is the effect of change of time? 8. If rate and time remain unchanged, how will the increase or decrease of principal affect the interest? If the interest on $1200, for a certain time, is $36, determine the interest on $900, $800, $700, $6000, $720, $400. Most practical methods of computing interest depend on the principles just emphasized, are closely related, and are based on a year of 360 days. It is best to understand several methods and to select for general use the one that seems most natural and simple. I. CANCELLATION METHOD. (Based on 360 or 365 days to the year.) Illustration. Example. Compute the interest on $720 for 84 daya at 6%. INTEREST. 287 Analysis and Solution. The interest for 1 year is 6/100 of $720. Fol 84 davs, it is 84/360 of this amount. 2 4720rX. 06X84 ■360- 1 $2 X. 06X84 = $10.08. EXERCISE. Compute the simple interest on: 1. $720 for 36 da. at 8%. 5. $1260 for mo. 15 da. at 6%. 2. $925 for 2 mo. 6 da. at 6%. 6. $829.60 for 39 da. at 4%. 3. $456.50 for 8 mo. at 7^%. 7. $3600 for 20 da. at 5%. 4. $862.85 for 7 mo. 3 da. at 6%. 8. $240 for 18 da. at 9%. 9. $1200 from Jan. 5 to Nov. 21 at 6%. (Approximate time.) 10. $12,900 from Aug. 21 to Dec. 3, at 4%. (Approximate time.) 11. $270 from Mar. 13 to May 24 at 7%. (Exact time.) 12. $1200 for 75 da. at 5% 13. $320 for 3 mo. 5 da. at 8%. II. SIXTY-DAY METHOD. The sixty-day, or bankers' method, is one of the most effec- tive in use. By it, interest is first computed at 6% and then reduced to the required rate. INTRODUCTORY EXERCISE. 1. If the interest rate is 6%, what per cent, of the principal is the interest for 6 mo.? 2 mo.? 8 mo.? 1 mo.? 2. What per cent, for 60 da., 30 da., 90 da., 20 da.? 3. For what period, in months, is the interest 1% of the principal? For what period in days? 4. What is the simplest method of finding 1% of a number? 5. Find the interest at 6%, for 60 days, on $1200, $800, $300. Illustrative Example. Compute the interest on $7200 for 96 days at 6%. At 3%. Solution. The interest for 60 days at 6% equals 1% of the principal. $7200.00 = the principal. 72.00 = interest for 60 da. (1% of principal) 36.00= " " 30 " (30 da. = 1/2 of 60 da. .'. interest equals 1/2 of interest for 60 da.) 7.20= " " 6_ " (6 da. = 1/10 of 60 da.) $115.20= " " 96 " Ans. (6%) The mterest at 3% = ^he interest at 6% or $57.60. 288 BUSINESS ARITHMETIC. Example 2 Compute the interest on $840 for 4 mo. 24 da. at 6%. Solution. $840 =the principal. 8 40 = interest for 2 mo. (Use a line to mark decimal point.) 8 40= " 2 80= " " 20 da. (20 da. = 1/3 of 2 mo.) 56= " " ^^( 4 da. = 1/5 of 20 da.) " 4 mo. 24 da. $20 16 = $ 8 40 = 16 80 = 2 10 = 84 = 42 = $20 16 = Check. Examples may be checked by changing time components. Illustrations. (Checks on example 2 ) 4 mo. 24 da. = 144 da. (1) interest for 60 da. " 120 da. (Twice interest for 60 da.) " 15 " (1/4 mterest for 60 da.) " 6 " (1/10 " '' 60 ." ) " 3 '' (i " " 6 da. ) (2) $ 8 140 = interest for 60 da. 21100= " " 150 " (2f times interest for 60 da.) |84= " " 6 " $20116= " " 144 "~ (By subtraction.) Note. This method substitutes simple addition and division for more complicated operations, and offers positive check solutions. If times are large, 600 days may be used as a unit time in place of 60 days; if the time is small, three places may be pointed off in order to find interest for a unit time of 6 days. It is generally advisable to select such sub-times that the interest at each step may be found from the previous step, or from the unit. See ex. 1, above. ORAL EXERCISE. Subdivide these times into parts, and also suggest a "check" series: 1. 84 da. 5. 5 mo. 7 da. 9. 3 mo. 21 da. 2. 90 da. 6. 87 da. 10. 48 da. 3. 11 da. 7. 4 mo. 15 da. 11. 37 da. 4. 21 da. 8. 7 mo. 3 da. 12. 16 da. ORAL EXERCISE. Compute the interest at 6% on: 1. $1200 for 60 da. 2. $ 840 for 30 da. $ 960 for 6 da. $ 120 for 20 da. 5. $ 600 for 3 da. 9. $ 884 for 30 da. 6. $640 for 45 da. 10. $ 390 for 2 da. 7. $ 900 for 2 mo. 6 da. 11. $ 900, 2 mo. 20 da. 8. $ 126 for 600 da. 12. $1600 for 15 da. Compute the interest on: 13. $300, 4 mo., 5%. 15. $800, 45 da., 4^%. 14. $240, 30 da., 7%. EXERCISE. Compute the interest on the following, checking by change of time: Principal. Time. Rate. Principal. Time. Rate. 1. $ 542.50 17 da. 6%. 5. $2164 23 da. 6%. 2. $3690 97 da. 5%. 6. $ 980 3 mo. 8 da. 6%. 3. $ 452.80 48 da. 4%. 7. $ 452.75 5 mo. 6 da. 6%. 4. $ 725 39 da. 6%. 8. $3726. 6 mo. 13 da. 4%. Compute the amount, principal plus interest, on the following: 9. $426 11 da. 6%. 13. $1268 132 da. 5%. 10. $ 300 25 da. 8%. 14. $ 282 90 3 mo. 17 da. 6%. 11. $1285 34 da. 6%. 15. $ 562.80 1 mo. 5 da. 6%. 12. $ 950 68 da. 6%. 16. $1262.50 7 mo. 15 da. 7§%. Using approximate time, compute the interest on: 17. $9650 from Jan. 27 to Oct. 19, at 6%. 18. $248.20 from Nov. 12 to Jan. 13, at 8%. 19. $3242.90 from Mar. 28 to Dec. 19, at 6%. Using exact time, compute the amount of: 21. $1220 from Mar. 22 to Apr. 14, at 6%. 22. $356 from Feb. 12, 1909, to June 14, 1909, at 7%. In order to simplify computation, the numerical value of the principal and time (in days) may be interchanged without affecting the resulting interest. In finding the amount, however, it is necessary to remember to add the original principal. Illustration. The interest on $120 for 17 days is the same as that of $17 for 120 days. Thus in the first case, the factors are 120 X 17 X. 06, 360 290 ^ BUSINESS ARITHMETIC. and in the second case, they are simply interchanged in order, 17 X 120 X .06. 360 Example. Compute the interest on $72 for 185 da. Solution. Read the example: Compute the interest on $185 for 72 da. $1.85 = interest for 60 da. .37 = " " 12 " $2 22= " " 72 " on $185, or for 185 da. on $72. ORAL EXERCISE. By interchanging terms, compute interest at 6%, on: 1. $60 for 29 da. 4. $120 for 38 da. 7. $600 for 71 da. 2. $20 for 96 da. 5. $15 for 92 da. 8. $150 for 42 da. 3. $30 for 47 da. 6. $12 for 132 da. EXERCISE. Compute interest and amount on: 1. $126 for 197 da. at 6%. 3. $3000 for 179 da. at 6%. 2. $540 for 83 da. at 8%. 4. $ 450 for 168 da. at 6%. III. SIX PER CENT. METHOD. In case the interest term is greater than one year, the 6% method is frequently used. Some prefer it, also, for short terms. INTRODUCTORY EXERCISE. 1. The interest on $1.00 for one year at 6% = ? 2. The interest on $1.00 for 2 mo. = ? . Find the interest for 30 da., 6 mo., 4 mb. What is the interest on $1.00 for any multiple of 2 mo. or 60 da.? 3. The interest on $1.00 for 6 da. = ? . How may it be found for 2 da., 6 da., for any multiple of 6 da.? 4. The interest on $1 .00 for 4 mo. = ? . From it, determine the interest on $400 for the same time. Theoretically, the 6% method consists in applying the 60-day method to building up the interest on $1.00, multiply- ing the result by the number of dollars. For practical use, the following values should be kept in mind : INTEREST. 291 $.06 = interest on $1.00 for 1 yr. at 6%. .01 = " " " " 2 mo. " " or .005 = " " " " 1 " " " .001 = " " " " 6 da. " " or .000J= " " " " 1 " " " Illustration. Example. Compute the interest at 6% on $700, for 1 yr. 7 mo. 18 da. Solution. $.06 = interest on $1.00 for 1 yr. .035= " " " " 7mo. (S^Xint. for2mo.). ■003= " " " " 18da. (3Xint. for6da.). $.098= " " " " 1 yr. 7 mo. 18 da. 700 X $.098 = interest on $700 for given time = $68.60. Note. Interest at other rates may be obtained by ratio, applied either to the interest on $1.00, or to the .total interest. Thus 2/3 of either $.098, or of the product, $68.60, equals the interest at 4%. ORAL EXERCISE Find the interest on $1.00, at 6%, for: 1. 88 da. 5. 29 da. 2. 92 da. 6. 62 da. 3. 17 da. 7. 54 da. 4. 14 da. 8. 2 da. Compute the interest on: 13. $200 for 24 da. 14. ♦ $1000 for 3 mo. 6 da. 9. 10. 1 mo. 9 da. 2 mo. 27 da. 11. 3 mo. 14 da. 12. 1 yr. 4 mo. 11 da. 15. $500 for 2 yr. 6 mo. 12 da. 16. $400 for 96 da. EXERCISE. Compute the interest on: Principal, Time. Rate. 1. $60432 1 yr. 2 mo. 8 da. 6%. 2. $ 98.25 2 yr. 5 mo. 3 da. 6%. 3. $ 7294 3 yr. 1 mo. 15 da. 6%. 4. $ 520 8 mo. 18 da. 3%. 5. $ 2460 lyr. 16 da. 7%. 6. $ 590.84 1 yr. 4 mo. 12 da. 5%. 7. $ 1580 3 mo. 16 da. 5%. 8. $ 2050.40 5 mo. 13 da. 6%. 292 BUSINESS ARITHMETIC. Principal. Time. Rate. 9. $ 840 7 mo. 24 da. G%. 10. $ 1292 2 yr. 5 mo. 20 da. 6%. 11. $36,432, from Jan. 29, 1908, to Oct. 17, 1909, at 6%. 12-21. Solve by 6% method, examples 1 to 8 and 21-22, page (289). Find the interest on the following by the 6% method and "check" by the 60-day method: 22. $324 for 1 yr. 2 mo. 8 da. at 6%. 23. $98.40 for 2 yr. 6 mo. 15 da. at 8%. 24. $7250 for 3 mo. 21 da. at 4 %. Find the amount of the following, using 60-day method and checking by the 6% method: 25. $76.80 from Jan. 12 to Aug.. 17, at 6%. 26. $358.25 from Feb. 14 to Dec. 29 at 6%. IV. TABLE METHOD. Interest tables of many forms are in common use among bankers and brokers. They usually require decimal division of the principal and ordinary subdivision of time. The following outline illustrates one common form. Six Per cent. Interest Table. Time. $10 $20 $30 $40 $50 $60 $70 $80 $90 1 da. .00167 .00333 .00500 .00667 .08333 .01000 .01167 .01333 .01500 2 da. 3 da. 1 mo. 2 mo. Note. The practical tables give interest for each day from 1 to 30. Such tables should be constructed with the utmost care and should read to five or more decimal points, to avoid errors when applied to large amounts. Illustration op Error. Suppose an interest table is constructed to show interest at 6% for twenty days, on amounts of $1.00 to $99.00. In a three-place table the interest for $10.00 would be $.033; in a seven-place table, $.0333333. Applying the table to determine interest on $100,000, it is necessary to move the decimal four places to the right. By the small table this would give $330; by the seven place, $333.33 — a diflference of $3.33. INTEREST. 293 EXERCISE. 1. Complete the table (p. 292) by computation, and from known values. 2. Show how, from the given times, to obtain the interest for any- time. Find the interest on $10 for 17 da. (Suggestion, 17 da. = 10 da. plus 7 da.), 23 da., 4 mo. 6 da. 3. Show how to use the table in finding interest on any principal, (a) if a decimal of a given principal; (6) if an irregular amount. Find the mterest on $40,000 for 1 da.; on $43 for 1 da. (Suggestion. $43 = $40+ $3.) 4. Using the table just constructed, determine the interest in ex. 1-11 of the preceding exercise. Exact Interest. Exact interest, based on 365 days, may be computed by the cancellation method, or it may be obtained from interest computed on a 360-day basis. Thus 1 day's interest, on a business basis, is 1/360 of a year's interest; on an exact basis, it is 1/365. The difference is 1/360 - 1/365, or 1/73. That is, exact interest is 1/73 less than common interest. Illustration. Example. Compute exact interest on $450 for 84 da., at 6%. Solution. $4.50 = interest for 60 da. 1.50 = " " 20 " .30 = " " _4 " $6.30 = " " 84 " " , $6.30 ^ 73 = $.09. $6.30 - $.09 = $6.21, the exact interest. EXERCISE. Find the exact interest on: 1. $582.90 for 39 da., at 6%. 5. $2960 from Aug. 17 to Dec. 29, 2. $1254 for 87 da., at 5%. at 6%. 3. $632.96 for 215 da. at 6%. 6. $382.50 from Jan. 5 to June 26 4. $98.25 for 49 da., at 4%. at 6%. Interest Problems. Interest is a truer measure of profit and loss than simple percentage, and is practically the common instrument for 294 BUSINESS ARITHMETIC. the computations incident to financial operations. The problems that follow are selected at random to show common uses of interest. EXERCISE. 1. I lend James Brown $2400 on Jan. 6, 1909, which he repays with interest on Aug. 15, 1910. What does he pay in settlement? What does he pay for the use of the money? 2. I borrow $1365, on my written promise to pay in 120 days with 5% interest. At the end of the period, what is due in settlement? 3. What is the quarterly income from a $20,000 loan placed at 4|% interest? 4. $824, if placed in a savings bank paying 3|% interest, will amount to what sum in 6 months? 5. If money is worth 8% to a buyer, is it better for him to pay $880 cash, or to wait 3 months and pay $900? 6. Determine the per cent, of gain on a four months' investment of $1280, which yields 5% interest. 7. If goods are bought for $420, is it better to sell them in three months at 5% profit, or at 5% interest on cost? 8. I am thinking of buying a piece of property for $8750 which, it is claimed, will rent for enough to yield 9% net interest on investment. What sum must be cleared? If repairs, taxes, etc., are estimated at $240 per year, what monthly rental should be charged? 9. A bill of crockery, $752.80, was bought Jan. 27, on 2 months credit. It was not paid until June 17. What was then due? Interest, 6%. 10. I lent through a broker, for a term of 6 months : $500 at 5 % interest ; $800 at 4:1%; $1200 at 5h%. The broker charged 1/4% interest as his commission. Determine my net income. 11. If this account draws 6% interest, what is due on Aug. 10 of the year following? S. R. Ramey 191— Jan. 17 300.— Mar 15 286.— July 12 125.— Aug. 10 85.60 12. The Montrose County School Fund is invested in: $10,000 City of Minneapolis 3^% bonds. $40,000 Morristown Water Works bonds, bearing 4i%. $120,000 Lewiston Gas Co. bonds, bearing 5%. What is the annual income from the fund? INTEREST. 295 13. I owe a London exporter £266 5 s. 10 d., with interest at 6%* from Mar. 12 to Jan. 8. What is due? CREDIT PRICES. ORAL EXERCISE. 1. Is it better for a merchant to sell an article for $400 cash, or for $400 credit? 2. If he gives credit what does he lose? 3. Should a credit price be greater, or less, than a cash price? 4. If the worth of money to a merchant be known, how may a credit price, equivalent to a cash price, be determined? Note. While many merchants add arbitrary amounts to the cash price when selling on credit, others use a carefully computed rate of interest in determining credit prices. Determine credit prices for the following: Cash Price. Credit Term. Money Worth. 5. $ 8.40 2 mo. 6%. 6. 80.00 1 mo. 7i%. 7. 6.50 30 da. 12%. 8. 5.00 3 mo. 8%. 9. 240.00 75 da. 9%. EXERCISE. (Use 6% as a rate unless otherwise stated.) 1. Does it pay to buy flour on 5 months credit, at $6.50, if it can be bought for $6.00 cash, and money is worth 8%? 2. What profit results from buying a $16.50 article on 5 months credit, and immediately selUng it at the same price on 1 month credit? 3. What is a fair settlement for a bill of merchandise amoimting to $1240 paid 3 months before its 4 months credit term is up? Note. Subtract interest for period before due. This is the interest- discount method. TO DETERMINE RATE OF INTEREST. INTRODUCTORY EXERCISE. 1. If the interest on $800 for 1 year is $36, what is the rate of interest? 2. If the interest on $600 for 3 months is $15, what is the interest for a year. The year's interest is what per cent, of the principal? 296 BUSINESS ARITHMETIC. 3. What is the interest on $1200 for 9 days at 6%? At 1%? $7.20 is the interest at what rate for the same time? It is evident that if the interest for a year, on any principal, is known, or can easily be obtained, the rate of interest is determined by dividing the known interest by 1% of the principal. If the time is irregular, it is usually easier to measure the given interest by the interest on the given prin- cipal, at 1% for the given time. Illustration. At what rate does $720 earn $4 in 40 days? Solution (1). $ 4.00 = interest for 40 days. 36.00 = " " 360 " (1 yr.) $36.00 -r- $720 = .05. The rate is 5%. Solution (2). The principal is $720. $7.20 = interest at 6% for 60 da. 4.80 = " " 6% " 40 " .80 = " " 1% " 40 " $4 (given interest) -i- $.80 (interest at 1%) = 5. The rate is 5%. ORAL EXERCISE. Principal and time being constant. 1. If the interest at 6% is $48, $60 is the mterest at ? %. 2. $3.60 is the interest at ? %, if $9 is interest at 2\%. 3. 84c is the interest at ? % if 16 c is the interest at 2%. Find the rate of interest if 4. $1200 earns $72 in one year. 6. $400 earns $3 in 2 months. 5. $800 earns $20 in 6 months.. 7. $600 earns 36c in 12 days. EXERCISE. 1. If the Russian Government sells its last 6% bond issue at 10% below face, what rate of interest does it really pay on the funds that it secures? 2. Discuss the investment value of a 6% investment running one year with a 5% one running ten years. 3. The offer of $80 for the use of $1500 for 45 days is equivalent to an offer of ? % interest. INTEREST. 297 4. $800 is loaned from January 15 to September 12 at 6%, and from October 1 to January 1, at 5%. What annual rate is earned? PROFIT AND LOSS MEASURED BY INTEREST. INTRODUCTORY EXERCISE. 1. Is it better to allow 3 months or 6 months credit on merchandise sold at a fixed price? 2. Is it better to earn 5% profit, or 5% interest, on an article that is held 2 years before sale? On one held 6 months? 3. When does a rate of interest yield a higher profit than the same rate of gain? 4. Which more clearly shows the real return from an investment — the rate of interest, or the rate of gain? Why? The quickness of sale, after purchase, greatly affects the profits of a business. This is the period of "turn-over of capital." The above exercise emphasizes, as in percentage, the value of a rapid turn-over, and shows the mathematical basis for the old business motto "Quick sales and small profits." EXERCISE. 1. A merchant invests in articles costing $2.40, which sell, on an average, in 2 mo. 6 da., at 3% profit. What is his annual rate of income on investment? 2. Property bought January 29, for $5000, and sold June 26 for $5400, yields what rates of gain and interest? 3. A house, bought for $9600, is rented for $50 per month, except for 2 months of the year. Taxes are lj% on an assessed value of $8000, and repairs cost $118.60. What is the per cent, of income on investment? 4. Is it better to make 5% interest or 3% profit on a 7 months in- vestment. Illustrate. 5. A real estate dealer advertises as a 10% investment for $5000, a house that rents for $50 per month. Taxes are 1^% on an assessed value of $4200; msurance is 11% on $3000. Is allowance made for repairs? 298 BUSINESS ARITHMETIC. TO FIND THE PRINCIPAL. INTRODUCTORY EXERCISE. 1. What is the interest on $1 at 6% for 2 months? $4 is the interest on what principal for the same rate and time? 2. What is the amount of $1 for 6 months at 10%. $525 is the amount of how many dollars, same time and rate? 3. What is the measure and quantity measured in examples 1 and 2 ? . -ri 1 . T» • • 1 Total interest ,. ^. , , v 4. Explam: Prmcipai = :j— ^ (for same tmie and rate). Interest on 3pi Illustration. (1) What principal earns $18.30 in 12 days at 6%? Solution. Int. on $1 for 12 da. is $.002. If $1 under given conditions, $18.30 ^ $.002 = $9150. earns $.002, $18.30 represents the The principal is $9150. earning of as many dollars as $.002 is contained times in that amount. Illustration (2). $976.32 is the amount of what principal for 3 mo. 12 da. at 6%? Solution. The amount of $1 for 3 mo. 12 da. at 6% is $1,017.^ $976.32 -^ $1,017 = $960. The principal is $960. ORAL EXERCISE. Find the principal. Int. Time. Rate. 1. $30. 18 da. 6%. 2. $48. 6 mo. 8%. 3. $ 9.60 36 da. 6%. 7. What principal amounts to $550 in 2 years at 5%? 8. What principal amounts to $2080 in 6 mo. at 8%? EXERCISE. 1. On January 1, 1911, a corporation must meet a debt of $25,000. What sum set aside on October 15, 1909, at 4% will amoimt to the debt when due? 2. What sum must be loaned at b\% to earn a quarterly income of $180? 3. Capitalize at 6%, semi-annual earnings of $84,200. Note To capitalize earnings or expense is to determine the capital, or principal that, at the given rate, will produce the stated earnings. Int. Time. Rate. 4. $3.20 24 da. 6%. 5. $9. 1 yr. 6 mo. 3%. 6. $2.40 3 mo. 8%. INTEREST. 299 4. Capitalize at 5% annual charges for rented buildings for which a city government pays $18,600. 5. By borrowing at 4%, a city government might have how much money with which to secure its own buildings, for the $16,500 it now pays annually in rentals? The present ivorth, or true value, of a future payment or debt, is the principal that, at the " worth of money," will amount to the payment when due. Present worth is some- times used in place of interest-discount, in reckoning advance payments, in comparing bids and investments. Illustration. Compare the present value of a $960 cash ofifer and a $964.25 credit price for 6 months, money being worth 6%. Solution. The amount of $1 for 3 months at 6% is $1,015. $964.25 -=- $1,015 = 950. The present value of the credit price is evidently $950. The credit price is better for the buyer than the cash price. Question. Why is the present worth method more accurate than the interest-discount method? EXERCISE. 1. Money being worth 8% to me, what do I gain by buying property for $960, on 6 months credit, and immediately disposing of it for cash, at the same price? 2. By present worth, at 6%, compare these bids for the construction of a building: C. Smith & Son, $12,000: 1/4, cash, 1/4, 6 mo.; balance, 1 yr. James Andrews; $11,750: 1/3, 4 mo.; balance, 1 yr. A. B. Norton, $11,500: 1/2, 4 mo.; balance 1 yr. 3. If money is worth 6%, what sum fairly settles, on June 15, a payment of $1200 not due until August 20? 4. What sum settles in cash for a purchase of $4284, on terms of 1/4 cash, balance 90 days. Money is worth 5%. TO FIND THE TIME. INTRODUCTORY EXERCISE. 1. If 6c is the interest for 1 day, on a certain principal, 54c is the interest for how many days? 300 BUSINESS ARITHMETIC. 2. Find the interest of $720 for 1 day. 84c is the interest on the same principal for how many days? o T7« 1 • rr.- -J Total interest , . , , x 3. iixplam. Time, m days, = , . — fTT^ (same prm. and rate). 4. Show how to use one month, or one year, as a measure, in place of one day. Illustration. In what time does $840 earn $28 at 6%? Solution. The interest on $840 for 1 da. = 14c'. $28 -h $.14 = 200. The interest term is 200 da., or 6 mo. 20 da. ORAL EXERCISE. Find the time, using 1 day. Find the time using 1 month. Prm. Int. Rate. Prin. Int. Rate, 1. $ 600 $1.50 6%. 5. $ 420 $ 6.30 6%. 2. $1200 $6.00 9%. 6. $ 800 $90. 6%. 3. $720 $1.32 6%. 7. $1200 $80. 5%. 4. $3000 $7.50 6%. 8. $2000 $75. 9%. 9. In what time does a princii pal double itself (simple int€ Test) at 4%, 6%, 6%, 8%, 7%, 10%, 9%, 3%? EXERCISE. 1. On January 25, $7200 is invested at 6% simple interest to remain until it amounts to $7500. Find the interest period. 2. On what date will $3200, invested July 8, 1909, at 6%, amount to $5000? Periodic Interest. Periodic interest is the interest on a principal, payable at stated intervals, plus simple interest on overdue interest payments. It can be legally enforced in but few states, but it is collected indirectly by means of non-interest notes, given for the regular payments, which draw simple interest when overdue. Illustration. What is due after one year and six months on a $1200 loan, at 6% interest, payable quarterly, nothing having been' paid? INTEREST. 301 Solution. Interest on $1200 for 3 mo. = $18, the quarterly payment. Interest on $1200 for 1 yr. 6 mo. = $108, the total sunple payments. At the end of 1 yr. 6 mo., The first payment is overdue 1 yr. 3 mo. " second " " " 1 " " " third ," " " 9 " " fourth '" " " 6 " u g£^i^ a u u 3 " The sum of overdue payments 3 yr. 9 mo. The total overdue interest on the several equal payments equals the interest on one payment for the total time, or for 3 yr. 9 mo. Interest on $18 for 3 yr. 9 mo. = $ 4.05 Regular interest payments, 108. — Principal, 1200. Amount due, $1312.05 EXERCISE. 1. On June 1, 1908, $3000 was lent at 5%, payable semi-annually. Nothing having been paid, what sum is due in full settlement, on December 1, 1909? 2. Find the amount due in full settlement for a loan of $7000, at 6%, payable quarterly, on which nothing has been paid for a term of four years? 3. Repeat example 2, on a basis of semi-annual payments. CHAPTER XXXVI. COMPOUND INTEREST. Compound interest is interest computed on the sum of a principal and its due and unpaid interest. Interest must be " compounded," or added to the principal, at stated intervals, as quarterly, semi-annually, or annually. Other periods may be fixed by agreement. While not recoverable by law, in most states, the accepting of compound interest does not constitute usury. If a new obligation is given at the end of any of the fixed periods for the amount then due, the obli- gation is binding. Compound interest is used by savings banks, in reckoning interest on deposits, by insurance companies and by business men generally, in computing the cumulative value of invest- ments at a fixed rate, where earnings are not withdrawn. Illustration. $12,000 invested at 6%, and compounded semi-annually for two years, will amount to what sum? Solution. $12,000 = original principal. 360 = interest for 6 mo. (3%). 12,360 = amount, end of 6 mo. 370.80 = interest for 6 mo. on last amount (3%). 12,730.80 = amount, end 1 yr. 381.92 = interest for 6 mo. on last amount (3%). 13,112.72 = amount, end 1 yr. 6 mo. 393.38 = interest for 6 mo., on last amount (3%). $13,506.10 = final amount. ORAL EXERCISE. 1. The interest for 4 years at 8%, compounded semi-annually, equals the interest for ? semi-annual periods, at ? %. 2. In compounding quarterly for 8 years at 6%, there are ? interest periods at ? % per period. 302 COMPOUND INTEREST. 303 a, H »-H <+H g o ^ ^ ^ a 3 Q fl Z 'P h' o (rt Oi o tS U 8 ,_( €# t«-i O ,j_, CI 3 O 0} a) -^ .S O :^ ::?? O COO -^(N O COth (N 00 0(N OiCO CO 0.-H ^(NCO 050 00 05 00 l-H CC^t^ '^ lO CO 00 -^ 00 00 CO CO 05 o r-< O 05 00 05 rfH to lO COl> Cil:^ 00"* 00 05 05(M O lO (N ^ C5 05 to 00. 05 00 O5C0 O5oo:-j 05 O 1— I T^l 05 cocqooo iOcO to O to 1-H l^ CO O to 00 to 05 O to (M Oi O --I "* cooo o t^ t^ i-H 00 "^ O t^ lO (N CO -^ -^ to CO 03 CO Oi . t^ t^ 00 050 tOQC OiOOO t^,-l ,-1 t0 05 OOOCO Oi(M (M (N coco CO 00 Ci O CQ »0 i-H (M»«t toco COr-H tH O to COCO (N O to 05 (N to T-i CO to to T-i to CO 00(N t^C050 (MCO (M (M(Nl>t^CO i-H COCOtJ^ GO t^ (M COGO t^ r-l ^T-H OOl-H O i-H (N COtJh 1-t (N CO rt< "i^ -^^ lOCO^ CO (NCOi-i O rt< OCO (M CO to (MCO tOt^O tOCOt-- C/D O (N(N(N OiCO OOO 1-H Tf (N Oi Oi 00 00 i-H (N t^ toco i-i t^Tj- (N O Oi QOOiC i-iT-i OOOiCO-^ CO CO '-1 1-H <- CO t^ Oi t^ cc cr or> ci "^^ c: tc t^CC CC CC CO (N CO Tf lO O o to 00 coco O (M r-l(M 00 O CM t^ to CO to 1-H 00 t^t^ C0I>OTfi 00 to Oi Oi t^ Oi to t:^ O Oi Oi C^ICO 00 00 to OiCM CO CO o C'l r^ .-H CO .-H coco coco Tt^ O Oi CO to Oi t^ CO to Oi Tt^ Oi O Oi CO CO Oi T-H CO 00 to »0 i-H CO i-H t^ T^i to to CO CO CO CO CO CO CI CO C^l CO 1-H M Tt CC to CO i-H ,-H OI rt^ TjH to-i coco Oi T-H CO CO CO to coo 00 CO Oi-H CO CO CO CO CO CO CO CO CO COOi T-H Oi OC t^ 00 O 00 Oi CO ■* to 1> CO '^ t^co Oi CO Oi toco 00 l>I> 00 Oi Oi 001>C3ir}^ o oco o t^ O Oi t> to CO O OCO tOOi coco OiCO to OOOi-H^ CO Tji O CO CO to t~* t^ t^ 1-H ooor^t^ Oi Tti Oi CO ■* CO c:iCO COO'* ^CO CO CO CO rt< ^ Ti- or- CO CO CO Oi CO CO l> to to Oi "* to 00 coi> 00 CO CO T-H to CO ■* ■* to to CO 00 CO CO T-H O Tfl coo T-H 1^00 -* to T-H -* CO CO coco o too too COCOt>-l> 00 to CO t^'* 00 oiO 00 Oit^- co T-H to r^ t^ O CO CO C^l CO CO T-H t^ CO Ci 00 Oi OiO O 8 to T-H CO oo _ CO Oi^ O CO 00 00'* toococo^ 01 to c^ o CO OOO^-H CO o CO CO to OiOO O CO 00 coco -* 00 O Oioooo ooo to 00 i-H ■* 00 T-H T-H CO CO CO l> OiT-H Tf 00 00 00 T-H t^ Oi O 00 to Oi CO CO '* 00 CO 00 T-H T*H t^ rH rj^ CO CO CO ^ '^ CO oo CiOcO O T-H to to T-H to CO CO CO CO CM^ T-HCO Tf OC t^ T-H CO -f to to CO r^ Oi OiT-H TJH GCCO i> CO coo to cot^t>.oooo O O 'X CO -^ O O ocooo '*C0 '*0 oot-hco^; 01 "^ CO CX) O OOOOr-H CO CO OiCO "* CO 00 to Oi Oi ^ CO CO O Oi CO 00 T-H to 00 CO "* t^ Oi ^ 1-H T-H r-1 ^ CO Tfi CO t^ Oi 00 1> '* O t^ CO CO CO CO '* 00 CO 00 CO Oi to '* CO Oi T-I >* CO CO CO coco CO T-H CO T-H t^ GO -* "* T-H Tt^ t^CO CO OOOi CO Q 00 CO to t^ O CO to 00 CO ^ '* '^ '^ cOOOil>CO CO 00 Oi CO O CO OiOO >* CO to »0 CO 00 O T-H "* t^O'* to to to CO CO O tOOO"* "* QCO t^cooo OCO CO CO CO too to ^t^ O CO '* CO t^ ooooo CO toco O T-H ■* ■* Oi Oi ■* rt< OOTt^ CO to CO OiCO CO o Oi O OI '* CO Oi GC CO CO CO Tt< T-H to to CO Oi CO to t^ CO t^ to CO T-H o !>■ Oi T-H CO lO T-H ^ OI CO CO CO O T-H ,-4 to 00 CO ■* »o to Oi O CO Oi 00 00 00 t^ CO CO CO 00 OCO "* CO CO CO coco 00'*t> CO to to COt^ o ■* O toco to Oi t^i>. OOOiO CO 00 O OI to CO CO Th< tJh ■^ 88oS2 O 1-H COCOO o ooo^ T-H CO CO ■* to ooooo O rH T-; T^* rH O to t^ to OI coco to 00 CO to T-H 00 O CO T-I CO CO co^ CO r^ 00 C3i o 00 to CO Tt^ Oi CO CO Oi t^ CO CO ooo "^t- Oi to CO 00 Oi ' ^ CO CO'*' OOOt-H ^^^^r-H Oi -* 00 OiO t^O '*0 Oi to CO T-H rH ,-H CO ^COQOO t^ 00 OiO CO T-t 1-H T-l CO CO CO CO CO to CO Oi T-I CO CO CO COt^T-H t^-* OI ^ l> Oi CO CO'* to CO 00 CO coco CO CO 1-H coco-* to coi>oooiO THCoco-^to cor>-oooiO T— I 1-H T-H 1-1 fH 1—1 T-H 1— I 1— I 1— i CO »-i CO CO'* »o CO CO CO CO CO 304 BUSINESS ARITHMETIC. 3. Compute the amount of a $4000 savings deposit, compounded quarterly for one year, at 4% per annum. EXERCISE. 1. A fund of $16,000 at 6%, compounded semi-annually, amounts in 5 years to $ — . , 2. $5000 invested Jan. 1, 1906, at 4%, compounded quarterly, will amount to $ on March 21, 1907. Note. Compute interest for fractional term on last full amount. Compound interest is computed, as a rule, from tables similar to the one on page 303. Illustration. Find the amount of $2500 for 6 years at 6%, com- pounded quarterly. Solution. In 6 years there are 4 X 6, or 24, interest periods. The rate of 6% per year equals 1^% per quarterly period. The problem is to find the amount of $2500 for 24 periods at 1^%. $1.429503 = amount of $1.00 for 24 periods at 1^%. 2500 $3573.76 = amount of $2500 for 24 periods or 6 years. It is evident that time and rate should be reduced to periods and period rates, and that the amount of $1.00 for this number of periods should be multiplied by the number of dollars in the principal. If the number of periods is greater than twenty-five, find the amount for twenty-five periods, and then find the amount of this amount for the remaining periods. ORAL EXERCISE. Read from the table the amount of $1000 if compounded: 1. Quarterly at 6% per annum for 6 years. 2. Semi-annually, at 8% for 5 years. 3. Bi-monthly, at 6% for 3 years. 4. Every 4 months, at 9% for 6 years. EXERCISE. Compute the amount of 1. $5950, for 3 yr. 6 mo, at 5%, compounded semi-annually. COMPOUND INTEREST. 305 2. $12,600 for 6 yr. 6 mo., at 8%, compounded quarterly. 3. $7200, from July 1, 1904, to October 27, 1909, at 6%, compounded semi-annually. Compute the compound interest on 4. $1200 for 8 yr. 4 mo. at 6%, compounded quarterly. 5. $2500 for 2 yr. 6 mo., at 9%, compounded every 4 months. 6. $3250 from October 1, 1909, to November 10, 1913, at 5% com- pounded 33mi-annually. What sum must be invested at 7. 6%, compounded quarterly, to amount to $7500 in 6 years? 8. 8%, compounded semi-annually, to amount to $12,500 in 4 yr. 6 mo.? 9. 9%, compounded semi-annually, to amount to $8000 in 3 yr. 6 mo.? 21 CHAPTER XXXVII. LOANS AND PAYMENTS. INTRODUCTORY EXERCISE. 1. What may lead one, in his private capacity, to borrow money? 2. Name objections to such borrowing. 3. What may cause a business man, or organization, to borrow? 4. Does a prosperous business ever need a loan? 5. When might a business borrow in order to pay cash for goods bought on credit? 6. How may a loan spread the cost of an improvement, for which it pays, over a long period? 7. What is the danger of lending money on a verbal promise to repay? 8. What business paper commonly is used as evidence of a loan, or of a payment due in the future? 9. How else may loans be "secured"? ORAL EXERCISE. 1. I borrow, to-day, $1200 for 3 mo. at 6%. I receive $ ?, but must pay back $ ?, paying % ? for the use of the money. Determine interest and amount due on these loans: Loan. Period. Rate. Loan. Period. Rate. 2. $ 800 9 mo. 6%. 6. $2500 24 da. 6%. 3. 1200 36 da. 5%. 7. 540 30 da. 6%. 4. 400 5 mo. 9%. 8. 250 2yr. 4%. 5. 1500 2 mo. 4%. EXERCISE. 1. What interest has accrued to date, at 4^%, on a loan contracted Feb. 11 of last year, for $820? 2. On Mar. 18, I must meet a loan of $840, contracted Oct. 21 last, and drawing 6% interest. If I have $520, what additional sum must I raise? 306 LOANS AND PAYMENTS. 307 3. Settlement of a 3% loan of $90,000, dated Jan. 20, is demanded Jan. 25. What is due? 4. A 3% loan of $6000, payable on demand, is contracted on Feb* ruary 16, and renewed on February 25 at 4%. Settlement is made, March 2, by paying $— . 5. My brokers lend for me, on a commission of 1/2% interest: Jan. 19, to C. Warren, $900 at 6%. Feb. 3, to M. C. Poston, $1250 at 5%. Feb. 16, to R. Fallon, $1920 at 6%. Determine my net interest on the following January 1. PROMISSORY NOTES. A promissory note is a written promise to pay a specified sum of money, on demand, at a specified time, with or without interest. It is a common evidence of debt. Illustration. %560xx Cincinnati, 0., Mar. 21, 19/5. Sixty days after date I promise to pay to the order of James Fielding at 7S6 Fifth Ave Five hundred sixty X2;/100 Dollars Value received with Interest at ^ % per annum. Robert C. Evans No. 97.... Due. May W.... Note. Robert C. Evans, who signs this note, is the maker; James Fielding is the payee. If the note had read "On demand, I promise," etc., it would have been payable whenever presented to Evans. Notes are negotiable, if transferable by the payee to other parties by endorsement of his signature on the back. To be transferable, notes must read " Pay to , or order," " Pay to the order of ," or " Pay to , or bearer." If the words " or order," or " or bearer" are omitted the note is not transferable. The following are the common forms of endorsement: (1) Endorsement in Blank. (1) James Fielding. This transfers right of owner- ship to the bearer. 308 BUSINESS ARITHMETIC. (2) Full Endorsement. (2) Pay to the order oi Here James Fielding transfers Robert Anderson, the note to Robert Anderson, James Fielding, but since the expression "pay to the order of" is used, Anderson may transfer the note to others. (3) Qualified Endorsement. (3) Pay to the order of C. The note is transferred by P. Rankin, without recourse Fielding to Rankin, but the to me former by use of the phrase James Fielding. "without recourse" refuses to agree to pay the note when due, in case the maker fails to pay. The last legal holder of the note secures payment from the maker at maturity. If the maker fails to pay, each endorser in turn, unless his endorsement is qualified, may be held for payment. In case of non-payment, the note may be given to a notary public, who formally demands payment. If payment is refused, he legally protests the note, issuing notice to each indorser and thus binding them to payment. This protest is a basis for legal action to compel payment. The amounts collectible are the face, interest if any, and legal charges. Non-interest-bearing notes draw interest after due. Notes signed by two or more parties and reading " we jointly promise," are termed joint notes, each party being held for his proportional part of the note (see p. 314). If the note reads " We jointly and severally promise," each signer is held for full settlement. Some states hold joint notes as similar to Joint and several notes. Collateral notes contain a permit to enable the holder, in case of failure to pay at maturity, to dispose of certain property (collateral) of the maker, and placed in possession of the payee, LOANS AND PAYMENTS. 309 as security, at the time of giving the note. One form of collateral note is shown on page 346. When a loan is made on real estate, notes usually are given, secured by a mortgage or deed of trust, under which the property may be disposed of in order to meet payments. Receipts for goods in warehouse or grain elevator likewise, may be endorsed to the lender as security for loans. The bonds of corporations are simply corporate notes, and frequently are secured by mortgages. A commercial draft is an order written by one person re- questing a second person to pay a stated sum of money to the order of the former, or to a third person. The parties to a draft are the drawer, or maker, the drawee, or person drawn on, and the payee. When accepted by the drawee, commercial drafts usually are negotiable. Accepted drafts are termed acceptances. Illustration (1). Andrews sells Parker $646 worth of merchandise, on terms of "60 day acceptance." He sends Parker an mvoice, accom- panied by the draft shown on page 310. Parker, finding the goods satis- factory, writes "Accepted" across the face of the draft and signs his name, thus binding himself to pay the paper when due. He then returns the draft to Andrews. By writing "Accepted, Payable at the First Nat. Bank, Chas. Parker," Parker could have turned the settlement over to his bank. Practically, the draft is now a note. Illustration (2). Andrews might wish to transfer payment to Henry Connors, a creditor of his. In that case he might make out the above draft and transfer it by endorsement, or he might make out the draft in the form shown on page 311. In the latter case he would prob- ably send the draft to Connors and the latter would present it to Parker for acceptance. When drafts read "after sight," in place of "after date," the acceptance must be dated, and maturity counts from that date. Drafts reading simply "at sight," are payable at once without acceptance. They are often drawn by merchants against customers, and are sent through a bank for collection. If the terms of a sale read, "subject to draft after 30 days" it is understood that a sight draft will be drawn at the end of the period. 310 BUSINESS ARITHMETIC. LOANS AND PAYMENTS. 311 312 BUSINESS ARITHMETIC. Factors Affecting the Maturity of Negotiable Paper. The wording of negotiable paper affects the date of maturity. On papers reading " — months after date " and " days after date" maturity is reckoned from the date of the note, the former by approximate and the latter by exact time. A few states still allow " three days of grace," after date of maturity, before payment must be made. If drafts read " after sight," or " at — days sight," the maturity is reckoned from the date of acceptance. Thus the draft on page 311 is payable 60 days from March 20. After the calendar date of maturity has been reckoned, the actual payment date must be determined with due regard to the varying state laws concerning Sundays, holidays, etc. Thus, in Pennsylvania, a note coming due Saturday is not payable until the following Monday, while if Monday should happen to be a holiday, it would become payable on Tuesday. Summary of Laws Relating to the Maturity op Negotiable Instruments. Note. These laws are in process of change, in many states. Ask a banker for the latest regulation. For states allowing days of grace, or for local laws, consult a local banker or business man. Requirement. State or Country. Due Sunday or holiday, pay Ala., Ariz., Cal., Col., D.C., Fla., Hawaii, next business day. 111., Ind., la., Kas., Ky., La., Me., Minn., •Mo., Mont., N.H., N.M., Okla., S.C, S.Dak., Tex., W.Va., Wis., Canada. Due Sunday or holiday, pay Alaska, Ark., Del. (Kent and Sussex Co.), preceding business day. Ga., Miss., Nev., Philippines, P.R., Cuba, Mexico. Due Saturday, Sunday or Conn., Idaho, Md., Mass., Neb., N.J., holiday, pay next business N.Y., N.C., N.Dak., O., Oreg., Pa., R.I., day. Mich., Tenn., Ut., Vt., Va., Wash., Wyom. LOANS AND PAYMENTS. 313 Holidays. Day^ Country or State. January 1. All states (except Mass. and Kansas), Canada, Cuba, and U. S. possessions. February 22. All states and U. S. possessions. May 30. All states and possessions, except Ala., Ark., Fla., Ga., La., Miss., N. C, Okla., S. C. and Tex. July 4. All states and U. S. possessions. Labor Day (first Monday All states (except Md., Wis., Wyom.) in September). and U. S. possessions, except Philippines. Thanksgiving Da}'^ (by proc- All states and possessions. lamation, usually last Thursday in November). December 25 (Christmas) . All states and U. S. possessions, Canada and Cuba. , Many states or counties have local holidays other than those named above. EXERCISE. Find the dates of maturity of this commercial paper, consulting the above summary. Use a calendar and allow for hoUdays, etc. Maturity in Your Own State 1. 90 day note, dated January 27. 2. 60 day note, dated Dec. 24. 3. "Three months after date" paper, dated Mar. 4. 4. "Sixty days after date" draft, dated June 14, accepted June 17. 5. "At 90 days' sight" draft, dated Aug. 31, accepted Sept. 3. Maturity in Different States. 6. "60 days' sight" draft, dated May 4, accepted May 5, payable in Minnesota. 7. A "90 days after date" draft dated September 3, accepted Sep- tember 4, payable in Ohio. 8. A 4 months note date Mar. 4, 1909, payable in Texas. 9. A 120 day note, dated April 24, 1909, payable in Canada. QUESTIONS FOR DISCUSSION. 1. As a rule, which allow a longer time for payment, notes given for months, or for the same number of thirty day periods? 314 BUSINESS ARITHMETIC. 2. Is thirty day paper ever better for the payer than one month paper? 3. Give reasons for allowing days of grace. Why are they being abolished? 4. Ordinarily, do the regulations as to Sundays and holidays favor payer or payee? 5. Why should the date of payment be fixed beyond question EXERCISE. 1. What yearly saving in interest results from refimding at 3|%, $52,500 worth of 5% bonds? Note. Refunding means re-issuing instead of paying at maturity. 2. What sum should be paid on June 20, in full payment for a 90 day non-interest note for $1200, dated February 11? Interest, 6%. 3. What sum will settle this note to-day : $520xY?r Washington, D. C, August 12, 1909. On demand, after date, we jointly promise to pay to the order of C. M. Fairley Five hundred, twenty ^ Dollars. Value received, with interest at 6% per annum. A. B. Morton, Henry C. Mason. 4. I lend C. P. Rogers $2260, at 5%, taking a first mortgage on his farm. What quarterly payment should he make? At the end of two years, nothing having been paid, the mortgage is foreclosed, and the property sold for $3675, the charges for the sale being $61.90. What is due Rogers? Income and Outgo. 5. An investment of $19,700 in 5% mortgages yields a semi-annual income of $ — . 6. A broker lends for a customer $18,000 at 5%, charging a com- mission of 5% of the first year's income. What is the client's net income the first year? 7. On February 19, I borrowed $12,000, at 3i%, giving my demand note. On March 3, 1 lent the money at 5%. On December 15, my note being presented for payment, I called in my loan. Net gain? Partial Payments. Partial payments may be made, by agreement, before or after the maturity of a note or bond. Such payments should LOANS AND PAYMENTS. 315 be endorsed on the back of the paper in one of the forms shown. Face. $1800.1^^ Washington, D. C, June 10, 1909. One year after date I promise to pay to the order of C. P. Crandall Eighteen hundred Dollars. With interest at 6% per annum. Robert Davis. Back. Paid, July 12, 1909, $200. Paid, Oct. 12, 1909, $300. Paid, Mar. 10, 1910, $400. Other forms of Indorsement. (1) July 12, 1909, paid $200. (2) Rec'd, 7/12/09, $200. (3) ^ Rec'd on the within note, July 12, 1909, three hundred dollars. Question. Why is no signature to an endorsement of a partial pay- ment necessary? The partial payment introduces several factors into the computation of the amount due, the question arising as to whether payments shall be applied to principal or to interest, or to both. State laws govern the matter, but many states have now legalized the United States rule, established by the Supreme Court of the United States. The so-called Merchants' rule is also frequently used. The United States rule is very commonly used for long term paper. INTRODUCTORY EXERCISE. 1. What is due at the end of 6 months on a 6% note for $1000? 2. If $230 is then paid, what sum will pay the interest due? What is left to reduce the face? How much of the debt is still outstanding? On what sum should interest now be paid? 3. What interest is due 6 months later? What amount? 4. If, now, $324 is paid, the new principal becomes $ . 5. The debt is finally settled a year later, by paying $ . Illustration. United States Rule. Example. Find the balance due at maturity, on the note shown above. 316 BUSINESS ARITHMETIC. Solution. Face of note $1800. — Interest, June 10 to July 12, 2 mo. 2 da 18.60 Amount due, July 12 1818.60 Payment, July 12 200.— New Principal 1618.60 Interest on new principal, July 12 to Oct. 12, 3 mo 24.28 Amount due, Oct. 12 1642.88 Payment, October 12 300.— New principal, balance due 1342.88 Interest on new principal, Oct. 12 to Mar 10, 1910, 4 mo. 28 da. . . 33.12 Amount due. Mar. 10 1376. — Payment, Mar. 10 400.— New principal, balance due 976. — Interest from Mar. 10 to maturity, Jime 10, 3 mo 14.64 Amount due at maturity $ 990.64 Note. Under this rule, a payment is first applied to pay off interest, and then to reduce principal. In case any payment is less than the interest accrued, the principal remains unaltered untU such a date that the sum of Eayments exceeds the interest accrued. For example, if the first payment ad been $10, it would have been added to the following $300 payment, and interest found on the original face from June 10 to October 12. Question. What would be the result of using a payment jess than the accrued interest? EXERCISE. Use approximate time for ex. 1-3; exact time for ex. 4-5. 1. Date of loan, June 26, 1908; amount, $3000; interest 6%. Pay- ments: Aug. 15, 1908, $450; June 21, 1909, $800; Aug. 21, $300; July 16, 1909, $425. Fmd the balance due Oct. 21, 1909. 2. A two year note for $3760, dated Oct. 15, Paid, 11/8/11, $400 1911, and bearing 6% interest, has these endorsed " 12/9/11, $200 payments. What is due at maturity? " 7/3/12, $600 " 11/6/12, $ 20 " 1/7/13, $520 3. On Oct. 27, Robert Evans paid $1200 on a demand note for $3000, dated Jan. 17, and bearing 6% interest. On Nov. 5, he paid $200, and gave a new 3 mo. note for the balance due. Determine the face of the new note. 4. Determine the amount due on this note at maturity. $3500.— Baltimore, Md., July 15, 1911. Fifteen months after date, I promise to pay to the order of Elton Howells, thirty-five hundred Dollars. Value received, with interest at 5%. Jambs Elwbll. LOANS AND PAYMENTS 317 Payments endorsed as follows: Paid, Nov. 5, '11, $ 800. " Mar. 17, '12, $ 250. " Nov. 12, '12, $1500. 5. From this Loan Book record, find the balance due, Oct. 1, 1912, at 3%: Loan to C. P. Davidson, NoJ Date. Amount. Date Payment. Amount Payment- 136 Oct. 1, '10 $40,000 Jan. 15, '11 $5,000. Mar. 20, '11 10,000. Aug. 21, '11 10,500. Jan. 1, '12 3,000. INTRODUCTORY EXERCISE. 1. What is due at maturity on a 6 months note for $1200, dated June 1, and bearing 6% interest? 2. On Oct. 1, a payment of $600 was made. If this payment was allowed to draw interest, to what would it amount when the note was due? 3. What is the difference, at maturity, between the value of the note and the value of the payment? The Merchant's rule is commonly used in banks and com- mercial houses, especially for short term notes. Illustration. Merchant's Rule. Example. Determine the balance due on the note, page 315. Solution. Face of note $1800. — Interest to maturity — 1 year 108. — Amount due at maturity $1908. — First pajonent $200. — Interest on payment to maturity, July 12, 1909, to June 10, 1910, 10 mo. 28 da 10.93 Second payment 300. — Interest, October 12 to June 10, 1910, 7 mo. 28 da. 1 1 .90 Third payment 400.— Interest, Mar. 10 to Jime 10, 3 mo 6. — Total value of payments at maturity $928.83 928.83 Difference between value of note and of payments $ 979.17 Note. Under this method, the face draws interest until the date of settlement, and each payment is allowed interest until the date of settle- ment or maturity. The balance due is the difference at time of settlement, between the value of the face and the value of the payments. 318 BUSINESS ARITHMETIC. EXERCISE, Compute by both rules, the balance due March 29, 1912, on this 1. note. $2400.— Cincinnati, Ohio, Sept. 7, 1911. On demand, after date, I promise to pay- to the order of S. P. Campbell Twenty-four hundred Dollars with interest at 6% per annum until paid. A. B. Norton. Solve the following, using exact time. Rec'd, Nov. Rec'd, Jan. Rec'd, Mar. 5/11, $300. 16/12, $580. 3/12, $627.50 Loan No. Date Name Amount Rate Date Pay. Am't Settled 2 1911 Oct. 8 James Baker $36,000 3% 6% 6% Nov. 5/11 Dec. 21/11 Mar. 20/12 $4000 6000 8000 June 21/12 3 Oct. 11 Robert Jones $4,500 Jan. 15/12 Mar. 12/12 Apr. 21/12 $ 400 1150 1200 July 17, '12 4 Oct. 15 Henry Sanders $1850 Dec. 20/11 Jan. 15/12 Feb. 20/12 Apr. 14/12 $200 185 300 400 May 21/12 5-9. Solve all the examples given under the U. S. rule by the Mer- chants' rule. QUESTIONS FOR DISCUSSION. 1. Compare the illustrative examples for the two rules and describe the different treatment of payments. 2. How are interest periods obtained under the U. S. rule? Under the Merchants' rule? 3. Why does it make no difference, under the Merchants' rule, whether the payments are greater or less than the interest? 4. Explain the reason for the difference in the final amount due under the two rules. Bank Discount. A dealer buys "on credit" in order that he may dispose of his goods, in part at least, before the bill for them becomes due. He thus makes them "pay for themselves" and lessens the amount of his necessary capital. The manufacturer, however, pays cash for his heavy labor expense, and for some supplies. LOANS AND PAYMENTS. 319 Were he obliged to wait for payments from customers, while paying out cash, he might be forced to carry extra heavy capi- tal, on which he could not earn a fair return on investment. To avoid this, he takes notes from his customers, or drafts on them, and sells these to a commercial bank, at a slight discount. This discount is computed as interest for the time the note has to run. The bank collects the full amount at maturity, from the maker, the discount representing its profit, while the manufacturer, although selling on credit, gets the immediate use of most of his money. Commercial banks are chartered by law to receive and lend money and to make collections. A large part of the income of many banks is obtained by loans through discounts. The bank's directors, or some designated officials, examine all paper offered for discount, as to its makers, endorsers, or security. Some banks discount only for their regular custo- mers. In such cases, the loan, or proceeds, is placed to the credit of the discounter, who "checks out" the money as needed. The bank gains slightly from the extra use of the deposited money. QUESTIONS FOR DISCUSSION. 1. Of what advantage is it to a dealer to buy on credit, rather than for cash? 2. What are the objections to selling "on credit"? 3. What is the disadvantage of buying goods for cash and selling on time? 4. What is meant by "tying up" one's capital? Discount Terms and Rules. The term of discount is the time between date of maturity and date of discount. A note or draft is discounted for its "full term" when it is discounted for the time stated in it. The amount or value at maturity is the face, plus interest, if any. 320 BUSINESS ARITHMETIC. The discount is the sum deducted by the bank. The proceeds is the sum credited, or paid, to the person discounting. Commercial banks vary greatly in their rules for discounting. Some use exact time, others approximate time; some have a minimum discount of 25c, etc. The general rules given below may be modified to conform to local custom: 1. Discount is reckoned on the amount due at maturity. 2. Notes discounted for full terms are discounted for the time stated in them. 3. Notes discounted for partial terms are discounted for the exact number of days between the date of discount and date of maturity. Exception. When interest-bearing notes are discounted, at the same rate as the interest rate, for a full term, the proceeds equal the face. Illustrations. (1) NonAnterest note. Discounted for full term at 6%. $840^ji Philadelphia, Pa., Jan. 27, 190— Three months after date . . / . . promise to pay to the order of James C. Leonard at The Cross National Bank Eight hundred forty Dollars Value received with Interest at — % per annum. Chas. R. CarleUm. ^o. 89 . . . .Bue Apr. 27 Solution. Discount term, 3 mo. $840.00= face. Amount due at maturity, $840. — $ 8.40= discount, 2 mo. Discoimt, for 3 mo. 12.60 4.20= discount, 1 mo. Proceeds, $827.40 iri2.60= discount, 3 mo. (2) The same note, discounted February 9. Date of maturity, April 27. Discount term, February 9 to April 27, 77 days. Amount due, $840. — $840. — =base, for discount. Discount, 10.78 $ 8.40 = discount for 60 da. Proceeds, $829.22 $ 1.40 = (( " 10 " .84 = « a g « .14 = $ 10.78 = (( -t ft "Y7 " LOANS AND PAYMENTS. 321 (3) Interest note. Discounted for full term at 6%. $784^f^ Cleveland, 0. Sixty days after date ..../. to the order of .... Cha^. Morgan & Co. at. . . .1685 Payne Ave. Seven hundred eighty-Jour October U, 190— . . .promise to pay .Dollars Value received with Interest at 6% per annum. Henry T. Carmody No. 9 Due Dec. IS 1685 Payne Ave. Solution. See rule 3, exception. The proceeds equal $784. — (4) The same note, discounted for full term at 8%. Solution. Face of note $784. — Interest, 60 da., 6% 7.84 Discount term, 60 da. $791.84 =base for discount. Amount at maturity, $791.84 $ 7.9184 = discount. 60 da. 6%. Discount, 10.56 2.6394 (< " 2%. Proceeds, $781.28 $ 10.56 = (( " 8%. (5) The same note. Discounted, November 10, at 6%. Solution. Date of maturity, December 12 >. Discount term, Nov. 10 to Dec :. 13 = 33da. Amount due at ma- $791.84 = base for discount. turity, as above, $791.84 $ 7.9184 = discount, 60 da., 6%. Discount, 4.36 3.9592 = (( 30 " Proceeds, $787.48 .3959 = li 3 " $4.36 = tt 33 " ORAL EXERCISE. Find date of maturity. Find time of discount. Date Date of Note. Term. Date. Term. Discount. 1. Jan. 27. 2 mo. 6. Apr. 9. 60 da. 2. Jan. 31. 1 mo. 7. Feb. 17 1 mo. , Mar. 6 3. Jan. 30. 60 da. 8. Mar. 12. 30 da. Mar. 26. 4. Feb. 28. 4 mo. 9. Apr. 19. 60 da. May 7. 5. Jan. 30. 6 mo. 10. July 14. 6 mo, , July 29. Find the discount and proceeds of these non-interest notes for full term Face. Term of Note. Rate Discount. 11. $960 60 da. 6%. 12. $600 2 mo. 5%. 13. $420 30 da. 6%. 14. $840 60 da. 7%. 322 BUSINESS ARITHMETIC. Face. Term of Note. Rate Dis. 4 mo. 6%. 30 da. 6%. 12 da. 6%. 20 da. 6%. Term of Note. Date Dis. 8. 90 da. (6%.) Mar. 17. 9. 90 da. (6%.) July 6. 15. 16. $1200, discounted for 17. $ 840. 18. $ 920. Face. Date 19. $ 600 Mar. 20. $1200 June 19 EXERCISE. 1. Discount 5% for full term, the draft on page 310. 2. Discount, at 7%, for full term, the draft on page 311. 3. Discount at 5%, for full term, the. note on page 307. 4. Discount, April 14, at 5%, the acceptance on page 311. 5. Discount February 19, 1912, at 6%, the note on page 316. 6. A merchant discounts on May 16, at 6%, the following notes and acceptances. What sum should be placed to the credit of his account? (a) A 3 mo. note for $725.60, dated Apr. 12. (6) A 60 day sight draft, for $852, dated Apr. 20 and accepted Apr. 23. (c) A 4 mo. note for $192.68, dated May 16. (d) A 30 day acceptance, for $365.42, dated May 15. i ORAL EXERCISE. Find the amount due at maturity on the following notes: Face. Term. Rate Int. 2 mo. 3%. 60 da. 6%. 90 da. 4%. 30 da. 6%. 4 mo. 6%. 20 da. 6%. 90 da. 6%. 60 da. 5%. Find the discount and proceeds of the following notes, bearing interest at 6% and discounted at the same rate: 1. $ 246. 2. $ 88.: 3. $ 900. 4. $1260. 5. $ 425. 6. $ 960. 7. $ 528. 8. $ 720. LOANS AND PAYMENTS. 323 Face. Term. Term. Dia 9. $ 600. 4 mo. 60 da. 10. $1200. 60 da. 20 da. 11. $ 900. 30 da. 30 da. Face. Date. Term. Date Dis. 12. $ 840. Jan. 6. 60 da. Feb. 5. 13. $1200. June 5. 30 da. June 25. 14. $ 600. Aug. 10. EXERCISE. 60 da. Aug. 30. 1. Find the discount and proceeds on the note on page 321 , if discounted November 4, at 6%. If discounted October 27, at 6%. 2. Determine the proceeds of the same note, if discounted at 5% on Nov. 3. 3. Find the proceeds of the following firm note, if discounted March 17, at 5%: $582.tV7 Kansas City, Mo., Mar. 5, 1913 Ninety days after date we. , . .promise to pay to the order of_ _John Sheldon at our office Five hundred, eighty-tvjo y^^Dollars. Value received with Interest at 6% per annum. J. M. Brovmson & Bro. No. 189 Due 4. James Devlin is obliged to pay his 90 day, 5% interest-bearing note, for $960, on Nov. 19. He has only $564.30 to his credit in the bank where his note becomes payable. On Nov. 18, therefore, he discounts the note shown below at 6%. He also deposits checks for $82.67 and $91.80. After his own note has been paid, what balance remains to his credit? $468.tTnr Baltimore, Md., Nov. 18, 190— Eighty days. after date I promise to pay to the order of James Devlin at ... . Third Nat. Bank . . . .Four hundred sixty-eight xViyDollars Value received with Interest at 5% per annum. Chas. D. Warrenton. No. 82 Due.... 324 BUSINESS ARITHMETIC. 5. Extend this form: Discount Ticket. Third National Bank. March 16,190. NOTES AND DRAFTS LEFT FOR DISCOUNT. By James Fielding Note or Draft. Maker or Drawer. Face. 840 — 1765 882 1206 Int. to Maturity, 20 (6%) (5%) Amount. 30 d. n. S. P. Norton 60 d. dft. Johnson Bros. 4 mo. n. Roberts and Bro. 3 mo. n. Champlain Mfg. Co. . Total, Discount, @ 6% (full term) Proceeds, 7. Complete the extension of the bank discount register on page 325. ORAL EXERCISE. 1. If a non-interest note is discounted for its full term of 4 mo., at 6%, the proceeds equal what per cent, of the face? If the proceeds are $980, what is the face? 2. A 60 day, 6%, interest-bearing note is discounted for 30 days at 6%. The proceeds equal what per cent, of the face? 3. Knowing the proceeds of a note, how may its face be determined? EXERCISE. Note. The per cent, method, suggested above, may be used, or the proceeds of a dollar note, under the given conditions ,may be determined. The proceeds divided by the proceeds of a dollar note will determine the face. 1. What must be the face of a non-interest-bearing note for 3 mo., that will yield $750 proceeds when discounted for its full term at 6%? 2. On January 17, 1 sold A. B. Norton a bill of merchandise amount- ing to $762.50, less 20%. Terms, 10 days. At the end of the period, Norton, being temporarily short of cash, offers his non-interest, 30 day note for such an amount that when discounted for its full term at 6%, the proceeds equal the face. This offer being accepted, the note should be made out for what sum? Individual Original Work. Report on local business methods of computing discounts on notes, drafts and business paper. Give local process of discounting. LOANS AND PAYMENTS. 325 I U I i ^ ^ N Oi X ?5 ^ X «^ ^ ^1 «0 z 5 ___ ___ ___ H QQ »- Z o s < (A •? U Z 1 i°^ ki ks ^^-i 1 "^ 1 1 ■^ ki ^ ao ki 1 1 ^ \ ^ '^ ^ k^ ^> 5V ?5i ^ ^ ^ '=^ ^ fs ^ '^ ^ :*^ ^ N ^ cs CV Qo ^ ^ ^ ^ ^ <^ ^ •^ ^ ^ ^ ^ ^ ^ <^ ^ ^ ^ ^-^ ^ ^ ^ ^^ ^ !^ c^ O^ cv ^-^ ev "«^ ov o^ ^ ^ ^ ^ •^^ ts ev ^ ir >* N >^ "N _^ ^*i ^'^ 1 =^ ^ N s u I i ^ i i K ^ ^ ^ •^ ^o N «>0 ^ ^ >N SV ®^ ^ •^ k> ts g^ 5 =5 °0 ^ ^ ^ ^ •^ 5 ^ ^ ^ ^ ■^i '^ Oi , ^ 1 1 ?^ 4 ^ «) ^ 1 s i^ 5$ ^^ ^^ :; i 1 1 ' -^ 1 1 s ^ 4 cc ^ ? ^ § ^ "S § ^ ^1 ^ ^ ^ ^ ^ ^ ^ 1 ^ ^ } J 1 1^ i «<3 J 1 1 ^ ^ ^ N. N «s N N «s N ::» ■^ =0 <^ «? <^ 4 <^ ^ t- 0) g 3 \> ■N S N \s N ^ S •"N s s s >!S >s >s CHAPTER XXXVIII. SAVINGS ACCOUNTS. "The best way to accumulate money is to resolutely save and bark a fixed portion of your income, no matter how small the amount,' — The WorWs Work. INTRODUCTORY EXERCISE. 1. Name three good reasons for saving money from one's income. 2. If a journeyman carpenter saw a chance to become a contractor on a small scale, he would have what reason for saving money? 3. Direct savings increase rapidly. A salesman, setting aside ten cents per day, saves $ per month. An iron-worker, saving $3.50 per week, has $ at the end of one year. It takes an office clerk, who is saving 20% of his weekly salary of $12, years to save the $600 necessary to secure an interest in a small independent business. 4. What are the dangers of simply "hoarding" one's money? 5. What difficulties lie in the way of investing small sums as they are saved? 6. What advantages result from placing those small savings in a bank? 7. What rates of interest do your local banks and trust companies pay? 8. What is the smallest sum accepted by them to open an account? What is the smallest sum accepted thereafter as a deposit? 9. Give reasons for one's withdrawing his deposit — (a) in prosperous times; (6) in times of panic; (c) in times of business depression. Practically all investment by wage-earners starts in the savings bank, an institution chartered by the State for receiving savings deposits, on which it pays interest. Savings banks receive enormous deposits of individually small amounts. A New York bank, in a poor district, has over $100,000,000 on deposit. The savings banks usually have the primary quality of safety, and are more carefully regulated and guarded by law than are other banking institutions. Most savings banks are 326 SAVINGS ACCOUNTS. 327 partly or entirely mutual and are run, in such cases, for the benefit of the depositors. A few are stock companies. Unless run for the benefit of stock holders, the banks place the major portion of their funds in safe mortgages, or in government, or other "gilt edge" bonds. Often the state laws specify very definitely the class of investments that may be made. It is well to examine the laws, or to get reliable business advice before depositing in any bank. If good banks are not within easy reach, deposits may be sent to them by mail. Many trust companies also accept savings deposits. They generally hold deposits '^ subject to check," but are not always as closely safeguarded as savings banks. EXERCISE. 1. Eleven savings banks of Pennsylvania recently had 398,885 de- positors, an average of ? per bank. 2. Complete this tabulation concerning the savings banks of the country : Year. No. Banks. Deposits. Depositors. Av. per Bank. Av. Depos. 1905 ' 1237 $3,093,077,357 7,693,229 $ $ 1910 1759. 4,070,486,247 9,142,908 $ $ 5 yr. increase ? ? ? ? % of increase ? ? ? ? 3. Account, in some way, for these increases. Most deposits are for long periods. Withdrawals are at infrequent intervals, and are further restricted by the fact that a bank may require, if it desires, 10 days to 60 days notice before making a payment. This rule is seldom enforced except in times of money stringency or sudden panic. On notice of expected withdrawal, the bank begins to save incoming deposits to meet this sum. It thus leaves practically its entire savings receipts free for investments. Money that it cannot immediately invest, it may lend "on call" (see p. 345), or lend to commercial banks at 1% to 2% interest. Whether mutual or stock organizations, savings banks 328 BUSINESS ARITHMETIC. usually pay from 3% to 5%, the average for the country being 3f %. Interest is added to deposits at periodic in- tervals. Since the period of compounding has some effect on income, it is well to note this, as well as the method of computing interest, before selecting a bank. The method is generally stated in the rules of the bank. EXERCISE. (Use compound interest table, page 303.) Find the amount for one year: Of $400 compounded semi-annually at 4%. Of the same sum compounded annually. Of $364 compounded quarterly at 4%. Of $942 compounded semi-annually at 3|%. Find the amount of $368 for three years if compoimded annually 1. 2. 3. 4. 5. at 3%. Since, in most cases, the amount on deposit is constantly changing, a special method is necessary for computing the simple interest for each term. Many savings banks allow interest on the smallest balance during any interest period. Sums deposited early in a period draw no interest until the next period, unless deposited the first day of the period. Ffactions of a dollar do not draw interest. ILLUSTRATION I. Northern Savings Bank. In account with James P. Norman. Date. Withdrawn. Deposited. Interest. Balance. 190— 1 Jan. 1 29 10 320 310 Feb. 17 462 772 Mar. 2 90 50 1 68150 Apr. 1 3,10 684 60 4 40 724 60 June 28 160 564 60 July 1 5164 570 24 SAVINGS ACCOUNTS. 329 Explanation. The deposits and withdrawals are entered in order of dates. The rate in this bank is 4%, compounded quarterly, or 1% per quarter. The first interest period extends from Jan. 1 to March 31. The smallest balance on hand during the period is S3 10, from Jan. 29 to Feb. 17. The interest is 1% of $310, or $3.10, which is added to the balance, as a new deposit, on Apr. 1. The next interest period extends from Apr. 1 to June 30. The smallest balance is $564.60, on June 28, and the interest is 1% of that balance, or $5.64. This interest is added July 1. If compounded semi-annually, the smallest balance for the entire period is $310, and the interest is 2%, or $6.20, a loss to the depositor, by change of compounding, of $2.54. Some savings banks reckon interest on monthly or quarterly balances, but compound it, or add it to the deposit, only at semi-annual, or annual, intervals. ILLUSTRATION II. Washington Savings Bank. In account with Henry C. Brewster. Date. Withdrawn. Deposits. Interest. Balances. 191— July 1 560 Aug. 5 246 10 806 10 Sept. 8 26 780 10 19 129 80 909 90 Oct. 31 40 869 90 Dec. 14 37 43 907 33 Jan. 1 14 29 921 62 Explanation. Here assume interest at 4%, on quarterly balances, compounded semi-annually. The lowest balance in the quarter ending Sept. 30 is $560, and the interest is 1%, or $5.60. The lowest balance in the fourth quarter, ending Dec. 31, is $869.90 and the interest is $8.69. The total interest, compounded on Jan. 1, is $5.60-}- $8.69, or $14.29. EXERCISE. 1. What is the effect of a heavy withdrawal just before the end of an interest period? 2. Would Norman (illus. 1) have gained by delaying his withdrawal of June 28, until July 1? 3. When would an additional withdrawal of $400 have caused the greatest "interest" loss in Brewster's account? 330 BUSINESS ARITHMETIC. 4. Determine the interest and final balance of Brewster's account, if the interest is reckoned on monthly balances. 5. Balance this account. Rate 3%, reckoned and compounded quarterly : Thornton Savings Bank. In account with John Kensington. Date. Withdrawals. Deposits. Interest. Balances. 190— Jan. 1 8 31 40 72 50 362 40 Mar. 3 80 45 May 19 162 65 June 12 30 40 321 6. Balance this account. Rate, 4%, reckoned on quarterly balances, and compounded semi-annually. Illinois Center Savings Bank. In account with C R. Martin. Date. Withdrawn. Deposited. Interest. Balances. July 5 29 102 10 328 45 Aug. 15 107 38 Sept. 29 54 13 Nov. 10 28 127 69 24 63 Dec. 12- 18 132 46 31 50 7. Extend this account. Rate, 4%, reckoned on monthly balances and compounded semi-annually. Exchange Savings Bank. In account with Robert S. Fenwick. Date. Deposits. Interest. Payments. Balance. Jan. 1 582 61 Mar. 3 45 May 16 312 62 June 21 46 58 Sept. 17 92 46 Oct. 20 545 20 Nov. 10 416 92 Dec. 21 38 80 SAVINGS ACCOUNTS. 331 Note. Notice change in column headings. 8. Balance this "subject to check" trust company account. Interest, 2%, compounded semi-annually. $240 is on hand January 1, 1912. Name c^oie^t ^lo. ^/uyj^itkyn No. 72M^6 Address '/M29 Min^lmj^ S^^e. Occupation ^C^2vtyucto7( DATE WITHDRAWN | DEPOSITED BALANCE '/9i/2 Ja^. /7 2 3 2 4 M 2S 6 5 3 JW. /7 3 4 4 5 ^yna-tt. s / M 2 5 4 2e 4 2 7 3 8 S^/c^. A 2 9 6 V 44 / 3 2 5 t^Ma^ // 4 2 6 Jj- 3 ^. 8 Jf 6 5 3 1/6 M s 5 29 3 6 6 €cc. 47 2 5 9 / 23 / / 4 6 3 jVo^. 43 M / 6 9 24 2 V ^ec. 44 3 4 5 46 A 2 5 2S 8 / / 6 3 30 6 8 8 2 ^. 4 J. t^ ^« i^'"^ ^ ^£ . *&REE.S-IOtP»NSMIT AND ^ 6-0000000- AaCRICAN EXPRESS CO. Mm- Note. The cost of an order equals the face plus the fee. Fees for Money Orders Drawn on Domestic Form. Payable in the United States and its possessions; also for orders payable in Bermuda, British Guiana, British Honduras, Canada, Cuba, Mexico, Newfoundland, the United States Postal Agency at Shanghai (China), the Bahama Islands, and certain other Islands in the West Indies. For Orders From $ 0.01 to $ 2.50 3 cents. From % 2.51 to $ 5.00 5 cents. From $ 5.01 to $ 10.00 8 cents. From $10.01 to $ 20.00 10 cents. From $20.01 to $ 30.00 12 cents. From $30.01 to $ 40.00 15 cents. From $40.01 to $ 50.00 18 cents. From $50.01 to $ 60.00 20 cents. From $60.01 to $ 75.00 25 cents. From $75.01 to $100.00 30 cents. DOMESTIC AND FOREIGN EXCHANGE. 357 Telegraph Orders. Telegraph orders are obtainable in the larger cities. The money is paid in at the sending office, and a telegram is sent to the telegraph company's agent at the place of payment, direct- ing him to pay the sum, from any funds on hand, to the person specified, on identification. The rates are usually one per cent, of the face of the order plus twice the cost of a ten word message. The message is frequently sent in the Company's private cypher or code, each word standing for a phrase. EXERCISE. 1. Let some pupil describe, from personal experience, the purchase of a money order, giving the steps to the process. 2. Let some pupil describe, from personal experience, the cashing of a money order. 3. Compute the total cost of money orders to cover each of the following amounts. Give the denominations of money orders where more than one required to make a payment. Amounts: $1.64, $5.00, $7.00, $L50, $25.00, $48.00, $65.50, $200.00, $165.00, $78.50, $2.50, $37.28, $96.84, $299.00. 4. An express agent issues money orders on a certain day for $34.16, $345., $23.69, $50., $76.15, $20.25 and $107.10. He started the day with a balance in his money order drawer of $124.00. During the day he cashed orders for $23.90, $45.50, $120.00, $13.00, $112.00, $3.26 and $1.56. What should be his balance in the drawer at the close of the day? 5. Determine the individual and total cost of these telegraph orders from Washington, the rate being 1%: (a) To Philadelphia $600.00, plus message cost of 50 cents; (6) To New York, $850, message 50 cents; (c) To Newark, Del., $386.50, message 40 cents; (d) To Indianapolis, $590; message 80 cents and to New Orleans, $2500, message 75 cents. Checks. It is customary for business men and others to keep the major portion of their cash funds on deposit in banks or trust companies. They then make practically all payments by checks, or orders on the bank to pay to the person named in the check the sum specified. 358 BUSINESS ARITHMETIC. Form op Check. GIRARD XRUSTT CONIRANY Note. In this instance, John Doe desires to make a payment of $400 to Henry Newton. Doe draws a check on the bank, in which he has money deposited, ordering it to make the payment to Newton. Newton may either take the check to the bank and cash it, after being identified, or he may deposit it with his own bank, after endorsing it, and the bank will collect payment from the drawer's bank. The latter bank will charge the amount against Doe's deposit. The checks of others, received in the course of business are deposited to the credit of the depositor's account, as in the case of actual cash. It is estimated that ninety per cent, of all business transactions involving money settlements are settled by checks. In commercial banks, checks may be drawn against a deposit at any time, but they must not exceed, at any time, the depositor's balance. Checks are drawn payable (1) '^To , or order" — in this case,- being negotiable; (2) "To Cash" — as a rule payable to the depositor himself; (3) "To Bearer" — payable to anyone presenting the check; or (4) "To ," payable only to the person named. Depositors desiring to draw money for a specific use, as for example, to pay wages, may draw checks to the order of the particular expense, thus " Pay to the order of Pay Roll." Such checks are cashed by the depositor, or after endorsement, by his special representative. In some localities this form of check is now forbidden. As in the case of notes (pp. 307, 308) checks may be trans- ferred by endorsement to others before being finally cashed or deposited. Business men deposit in their own banks, practically in the DOMESTIC AND FOREIGN EXCHANGE. 359 same manner as cash, the checks received by them from others and drawn on local or out-of-town banks. The banks re- ceiving these checks on deposit must collect payment from the banks on which they are drawn. To avoid the necessity of sending messengers daily to all other local banks on which they, hold checks, to make collections, banks form local clearing houses at w^hich representatives of each bank meet daily at a specified hour. Here, in a systematic way, all checks drawn against a given bank, and brought by repre- sentatives of other banks are charged against it, and all checks it presents against other banks are credited to it. Balances are then struck. In each case where the debits exceed the credits, the bank concerned pays the debit balance to the clearing house officials before a certain time. Immedi- ately after the receipt of this money from the banks that are said to owe the clearing house, it is paid out to the banks that have a credit balance. The clearing house thus begins and ends the day without money. By means of this insti- tution balances of millions are sometimes settled by the actual payment of only a few thousand dollars. Illustration. Rosslyn has three banks. On a certain day their representatives meet. The First National Bank presents $12,000 in checks against the Second National Bank, and $18,756.42 against the Third National Bank. The Second National Bank presents $19,540 in checks against the First National Bank and $15,126.10 against the Third National Bank. The Third National Bank presents $14,1.18 in checks against the First National Bank and $11,121.70 against the Second National Bank. The Clearing House credits each bank with the checks it presents and charges it with the checks drawn on it and presented by other banks, thus: Cr. Bal. $11,544.40 Bank. Charges. Credits. Dr. Bal. First Nat. Bank $33,658.00 $30,756.42 $2,901.58 Second Nat. Bank 23,121.70 34,666.10 Third Nat. Bank 33,882.52 25,239.70 8,642.82 $11,544.40 $11,544.40 The Clearing House collects the amounts of the debit balances against the First and Third National and pays them over to the Second National Bank. 360 BUSINESS ARITHMETIC. The checks that each bank receives from the clearing house are taken back by its messenger and charged against the accounts of the persons drawing them. Each bank receives, with other deposits, many checks drawn on out-of-town banks. It may collect these by mail, direct from the banks on which they are drawn, or, as is the common custom, it may collect them through its correspondent bank or banks in other localities, commonly in the larger cities, with which it keeps accounts. In the same way, banks in the villages and towns of its neighborhood, and banks of other cities, may keep accounts with it. Illustration. Peter Morton draws a check on his bank, The Idaho Na- tional Bank of Washington, in favor of James Rawson of Newark, N. J., and sends it to the latter in settlement for a purchase of merchandise. James Rawson deposits the check to the credit of his account in his home bank, the First National. The First National Bank credits Rawson and sends the check to its correspondent bank nearest Washington, the Fortieth National Bank of Baltimore, charging that bank. The Fortieth National Bank credits the Newark bank, and sends on the check, properly endorsed, to its correspondent bank in Washington, say the Twentieth National Bank. The Twentieth National Bank credits the Fortieth National Bank of Baltimore, and exchanges the check with the Idaho National Bank through the Clearing House. The Idaho National Bank charges the check against Morton's account. Thus it is evident that all the checks a depositor draws against his account, and gives out in payment, either locally or at other points, find their way back to his own bank and are charged against his account. Sometimes they take very roundabout paths back, but the whole history is shown by the endorsements on the back. It will be found interesting to study the endorsements of cancelled checks. Of course, any two correspondent banks are continually exchanging checks for collection, and their debit or credit balances with each other are constantly changing. At more or less periodic intervals, however, these balances are settled in full or in part by transfers of actual currency. DOMESTIC AND FOREIGN EXCHANGE. 361 Collection Fees. Some banks charge for out-of-town col- lections made for other than their depositors, and sometimes for collections for their depositors. The fee (rate of exchange) for such collections, while still more or less variable, is becoming steadily more uniform, especially in the East, being frequently only 1/10%. Sometimes there is a minimum charge of 10 cents or 25 cents. Bank Drafts and Cashiers' Checks. It frequently happens, owing to the risk of loss, that objection is made to receiving the check of a private depositor. In such cases the payer may have his check certified by an ofiicial of his bank on which the draft is drawn. The certification is stamped across the face of the check and signed by the cashier or other bank oJBScer. The bank thus guarantees the check and protects itself by immediately charging the check against the depositor's account. Such checks are more readily accepted. In case, however, a certified check is not used, it should not be de- stroyed, but should be deposited to the credit of the account against which it was originally drawn. The payer may also make settlement by means of a hank draft, purchased at his own bank, or at any other that will accommodate him, and drawn on some correspondent bank either in the neighborhood of the payee, or in some large central city. Exchange on New York or Chicago is acceptable anywhere. These drafts are often drawn to the order of their purchaser in order that he may endorse them over to the intended payee and thus secure proof of payment. The payee may deposit them or transfer them by endorsement, the same as checks. The charge (rate of exchange) is usually the face plus a slight premium, generally 1/10%, although a bank will often issue drafts to its regular customers at a cost equal to the face. If the correspondent or paying bank has funds to the credit or issuing bank, the drafts are charged against these. If not, 362 BUSINESS ARITHMETIC. the issuing bank will send bank checks to its correspondent for collection, or will give the bank credit and balance up later. As a rule, no charge is made for the collection of bank drafts when deposited for that purpose, if they are drawn on banks at the large money centers. Form of Bank Draft. No. 6129 Seatde, Wash., Mar. 2, 1912. Johnson, Morton and Co. Bankers Pay to the order of James F. Porter $826^*/^ Eight hundred, twenty-six j%\ Dollars. To Fifteenth National Bank, Andrew Norton, St. Paul, Minn. Cashier. Form of Bank Draft. In place of a bank draft, an ordinary cashier's check on his own bank may be obtained by a depositor, its cost being charged against his account. Such checks, being official in character, and having the weight of a bank back of them, pass more readily among strangers than do private checks. Certificate of Deposit. Instead of opening a regular checking account, one may deposit money in a bank and receive therefor a certificate of deposit, transferable by endorsement. This form is often used when some fixed sum of money is to be deposited for safe-keeping. Some savings banks and trust companies will issue certificates for fixed periods, such as one year, bearing interest at a certain rate, usually higher than that allowed on deposits subject to check. A Certificate of Deposit. $6000 T^iy Chicago, Illinois, June 16, 191 — National Bank This certifies that Thomas Medway has deposited Six Thousand j%% Dollars, payable to the order of C. T. Robinson on the return of this certificate properly indorsed. ,^ . _ ^ Martin Kentj Cashier. DOMESTIC AND FOREIGN EXCHANGE. 363 Commercial Drafts. Ordinary commercial drafts are fre- quently used for settlements and collections. $825tV7 Columbus, Ohio, Sept. 19, 191— At sight Pay to the order of Ourselves . . . .Eight hundred twenty-five ^^^ Dollars. . Value received and charge to the account of To Thompson Bros. Lewis Mark & Son. No. 67 Baltimore, Md. Note. Thompson Bros, owe Lewis Mark & Son $825.67. When the account is due Lewis Mark & Son draw a sight draft on Thompson Bros, and give it to their home bank to collect. The Columbus bank sends the draft to its correspondent in Baltimore. The bank in Baltimore presents the draft to Thompson Bros, for settlement. When payment Is received the Columbus bank is notified and it credits Lewis Mark & Son's account for the amount less a small fee (collection and exchange). Time or sight drafts are used frequently to collect for merchandise at time of delivery. Goods are shipped to a customer and a through bill of lading is taken from the trans- portation company. This is a receipt for the goods and an agreement to deliver them to the consignee, or to his order. The selling firm also makes out a sight draft on the customer, if the terms are cash on delivery, or a time draft, if credit for a certain period is to be allowed. The draft and bill of lading are fastened together and given to the seller's bank. The bank endorses the draft and sends it to its correspondent bank in the neighborhood of the purchaser. This bank presents the draft to the customer for payment, if a sight draft, or for acceptance if a time draft. When payment or acceptance has been made, the bank surrenders the bill of lading and the customer may secure his goods. Time drafts frequently are left in the bank for presentation again when payment becomes due. Rates of Exchange. Postal and express money orders cost more than their face value. Bank drafts frequently sell at, or above, their face value. Bank and commercial drafts 364 BUSINESS ARITHMETIC. may sell for less than their face value. Exchange is at par if the cost of a draft equals its face; it is above par, or at a pre- mium, if the cost exceeds the face; it is below par, or at a dis- count, if it sells for less. Exchange on small sums is usually at a premium to repay cost of handling. The rate of exchange varies with the demand for money at certain centers of trade and with the cost of shipment of actual currency. Illustration. In the harvest season for grain, eastern purchasers are sending checks west in settlement for purchases. Suppose, for example, that large orders are being received in Chicago from New York customers. Checks and drafts drawn on New York banks and presented or deposited in Chicago banks will tend to seriously decrease, or to more than exhaust, the balances to the credit of the New York banks. ^ At such times exchange on Chicago will be selling at a premium in New York, because the New York banks selling such exchange will be under the necessity of shipping actual currency to Chicago to meet the drafts. At the same time, however, Chicago purchasers will find exchange on New York below par, because Chicago banks have so much due them from New York, that drafts on the latter city may be met easily without the transfer between the cities of the actual currency. The sale of such drafts really benefits Chicago by lessening the balance due from New York to Chicago. In spring, when manufactured and imported goods are going out from New York all over the country, and merchants all over the country owe New York dealers and manufacturers, conditions are reversed. There is such a demand for exchange on New York that the rate of exchange is almost sure to go above par. The rate of exchange seldom if ever goes above the rate for shipping the actual money. If the cost of shipping money between two given cities is $10 per $10,000 the rate of premium will not go above 1/10%. Collection rates for drafts are somewhat arbitrary, being gov- erned by general trade conditions. As a rule, however, bank drafts on the large money centers are collected without charge. EXERCISE. 1. Secure a bank deposit slip. Find out how to make a deposit. Explain the process of opening an account and making the original deposit. DOMESTIC AND FOREIGN EXCHANGE. 365 2. If you owed a man in Albany a payment of $760, which would probably be the cheapest safe way of making settlement? 3. C. Y. Portner collects through his bank, at a charge of 1/10% for collection, checks on New York for $4.56.89 and $887.50 and on Phila- delphia for $1,456. What is the net sum to be credited to his bank account? 4. Extend the following form showing collections made by one bank for its correspondent bank. Collection 1/10%. Fortieth National Bank. , N. Y., September 15, 191—. Mr. Robert T, Brown, Cashier. Washington, D. C, Ninth National Bank, Dear Sir: We have credited your account this day for the proceeds of the collections stated below. j^^^ Fadelet, Cashier. Your No. Payer. Amount Collection. Proceeds. 456 Robert Chase A. B. Nortins Bruce & Co. Henry Ames Besley Bros. 480 1239 2311 477 5465 00 80 45 62 00 - 461 511 512 589 .... .... 5. Banks A, B, C and D form a Clearing House. On Sept. 16, Bank A presents checks against Bank B for $12,178.80; against Bank C for $4566.75; against Bank D for $56,124.59. Bank B presents checks against Bank A for $45,000; against Bank C for $34,123; against Bank D for $19,234.59. Bank C presents checks against Bank A for $13,543; against Bank B for $24, 118.25 5 and against Bank D for $7,188.55. Bank D pre- sents checks against Bank A for $12,325; against Bank B for $35,455.60 and against Bank C for $41,157.20. Find the debit or credit balance of each bank with the Clearing House and determine what payments must be made to settle all balances. 6. What is the net sum placed to the credit of John Grandfield on deposits of checks for $435, $350, $527.50 and $56.88, on which a collection fee of 1/8% is charged? 7. Compute the cost of a bank draft on Richmond, Va., for $723.90, at 1/10%. 8. How large a draft may be purchased for $1127.13 at 1/10% pre^ mium? 9. Determine the proceeds from the collection through an express company of an account of $2,356, at a rate of $1.75 per $1,000. 366 BUSINESS ARITHMETIC. 10. Draw up a check on an assumed bank to pay for a bank draft of $2264 at 1/5% premium. 11. Robert Gaines sold H. M. Chilton a bill of goods amounting to $1764.80. Gaines drew a draft in settlement and sent it, with the bill of lading for the goods, through his bank. What sum was added to the credit of his account when the bank had collected the draft, its charge being 1/5%? 12. A. C. Crandall sold a bill of goods for $7500 to George Grimes, the terms being "30 day acceptance." • Crandall drew the draft and sent it through his bank for acceptance. After acceptance, the bank discounted the draft for Crandall at 1/10% and 6% interest. What proceeds did Crandall receive? 13. A commercial draft for $475 cost $474.05. Was exchange at a premium or at a discount? At what rate? 14. A. M. Beardsley drew a draft on John Evans, for $467.56 and collected it through his bank at a rate of collection of 1/20%. What sum should be credited to his account? 15. Was exchange at a premium or at a discount, and at what rate, if a draft for $23,450 cost $23,567.25? 16. Boston is selling drafts on New Orleans at 3/4% premium. Which way is the balance of trade? What is the cost of a draft on New Orleans face value of $1245? 17. Suppose the balance of trade between Indianapolis and Pittsburg is largely in favor of the latter. What docs this mean? In which city is it likely that exchange will be at a premium? Why? At a discount? Why? 18. Name several cities that might have a balance of trade in their favor in the spring. In the fall. Give your reasons. FOREIGN EXCHANGE. All forms of payment possible under domestic exchange are available for payments abroad. Theoretically, foreign ex- change is based on the same general principles as domestic exchange. In practice, however, all computations are affected by differences in currency denominations and standards, and by methods of quoting exchange. From the banker's stand- point, foreign exchange is, in many ways, the most intricate of all forms of banking. DOMESTIC AND FOREIGN EXCHANGE. 367 Rates of Exchange. Foreign rates are variously expressed, but may be broadly classified under three heads. 1. Rates per foreign coin. Thus a London exchange rate of "4.875" means that one pound sterling can be bought for $4,875. A rate of "40f " on Amsterdam means that one guilder exchange on that city costs $.40 f. 2. Rates on a set number of coin. Exchange on German cities is usually- quoted "per four marks." Thus a quotation of "96," on Berlin, means that a four marks bill of exchange costs $.96. 3. Rates in "coin per dollar." This form of exchange is often quoted for France and other Latin countries, as Spain, Italy, Switzerland, etc. Thus a rate of 5.15 f on Paris means that a draft on Paris for 5.15 f francs may be purchased for one dollar. Note. Form (1) is sometimes used for exchange on any country especially in connection with some forms of money orders. Illustrations. (1) Compute the cost of a £450 draft on London, purchased at 4.875. Solution. $4,875 Since £1 cost $4,875, £450 will cost 450 450 X $4,875. 243750 19500 $2193.75, Ans. (2) Compute the cost of a draft on Berlin for 1200 M., purchased at 96. Solution. 4)1200 In 1200 M. there are 300 sets of 4 M. ~~300, no. sets 4 M. If one set costs 96c, 300 sets cost 300 X 96c. $0.96 300 $288.00 cost. (3) Compute the cost of a 12,000 fr. draft on Paris, purchased at 5.15. Solution. $ 2330.09 Since 5.15 francs cost $1.00, 12,000 francs 5.15)12000. will cost as many dollars as 5.15 is contained 1030 tmies in 12,000. 1700 1545 1550 1545 5000 4635 368 BUSINESS ARITHMETIC. Foreign Money Orders. The International Postal Money- Orders and the express orders, which are similar to domestic money orders, are issued for amounts not exceeding one hundred dollars, and are usually payable in the money of the country on which they are drawn. Fees for this service are small. 1. What will an express money order for £5 6s. cost at $4.87? 2. How large a draft on London will $50 buy, at the same quotation? 3. Robbins wishes to send a friend in Rome an order for 290.5 lira. At 19.5c per lira, the order costs $ ? 4. For $40 one may buy an order on Berlin for ? M., if the rate is 24c per M. Note. Money payments may be made by cable in the same way in which they are made by telegraph. In cable orders, a charge is made for each word of the address. 5. At 20c per word, and 1% of the amount, find the cost of a 24-word cable order on London for £600, at a rate of exchange of $4.8725. Travelers' Checks. Travelers may secure from express companies and banks travelers' checks, which can be cashed Form of Traveler's Check. at hotels and banks abroad. They are issued for fixed amounts and state on their face their value in the coinage of the differ- ent countries in which they may be presented for "cashing." DOMESTIC AND FOREIGN EXCHANGE. 369 The purchaser signs his name on the face at the time of purchase, and again on face or back, for purposes of identi- fication, at time of cashing. The checks are sometimes issued in books or series. Letters of Credit. A traveler's letter of credit is an instrument issued by a bank to its representatives abroad, authorizing them to honor drafts of the person named in the body of the letter, to total not exceeding an amount stated. The person desiring a letter of credit deposits with the issuing bank money or bonds as security. On receiving the letter of credit, he places his .signature on the face, and also gives the issuing banker other specimens of his signature, which are forwarded to the correspondent banks with the formal notice of the issuance of the letter. Once abroad, whenever the holder of the letter desires money, he stops at any correspondent bank named in the letter and presents his personal draft for the amount desired. After the signature has been verified, he is paid the amount in the coinage of the country. This amount is indorsed, also, on the letter of credit. When the "credit" called for has been exhausted, the letter is taken up by the bank making the last payment, cancelled, and returned to the issuing bank. EXERCISE. 1. Determine the cost of a 12,000 franc letter of credit, at 5.18 and 1% commission. 2. On a £600 letter of credit, are endorsed payments of £120, Nov. 10; £50, Dec. 5; £180, Jan. 16; £125, Feb. 12. Purchased at 4.86 and 1% commission, and unused portion redeemed at same rate, what repayment should be made to the holder? Commercial Letters of Credit. These letters are issued to mercantile houses desiring: to import goods to enable them 25 370 BUSINESS ARITHMETIC. to get credit from exporters for the usual periods of 30, 60 or 90 days. On proper evidence of the shipment of goods, a foreign bank at point of shipment, will buy the shipper's draft against the person named in the letter, at the prevailing rate of commercial exchange, reimbursing itself by sending the draft to party drawn on for collection, or by selling the draft in the open market. Credits are issued usually in pounds sterling because such exchange is easily negotiated in any part of the world. They are sometimes issued payable in the money of the country to which they are sent. Commercial credits are issued in four parts, one being sent to the exporter, one to the bank named in the credit, and one to the importer, or purchaser, the fourth being retained. Prices vary with credit term, financial responsibility of applicant, and many other conditions. The problems of computation are the same as those involved in other draft transactions. Bills of Exchange. Drafts drawn by a person or bank in one country on a person in another country are termed bills of exchange. They include bankers', commercial and docu- mentary bills. Bankers' bills are a bank's checks, drawn against its deposits in foreign banks. Commercial bills are those drawn by one merchant against another. Documentary bills are commercial bills, secured by bills of lading, insurance policies, or other documents that control the merchandise in transit. Bills of exchange were formerly issued in triplicate. They are now issued, as a rule, in duplicate, so worded that the payment of either one makes void the other. Sometimes the ordinary single draft is used. Owing to delays and dangers of transit, the duplicate drafts were formerly sent by different routes or mails. Now the tendency is to keep the duplicate on file. 2 o S ii ^ a, « I" DOMESTIC AND FOREIGN EXCHANGE. 371 A Duplicate Bill. PHiLADBLPmA, Pa., April 19, 19 — 2450 M. At sight of this second of Exchange (first unpaid), pay to the order of Ourselves Twenty-four hundred and fifty Marks, Value received and charge to the account of To K. R. Schmidt Berhn S. T. Coles & Co. Simple Foreign Bill. £1000— Philadelphia, March 12th, 190— On demand Pay to the order of John J. Doe One thousand Pounds sterling Value received and charge to account of To J. S. Morgan & Co. No. 16 London, England. John Smith. Fluctuation of Exchange. The rate of exchange is affected by numerous causes. If exported goods greatly exceed imports in value, the rate of exchange tends to lower because there is less demand for remittances abroad. If more money is due abroad than is due the home country, the rate tends to rise. When money is in demand here, and rates rise, rates of exchange may fall because there is less demand for bills. If money rates are high abroad, exchange rates may rise because of the demand for bills in order to send money to a place of good investment. In general, the question of supply and demand seriously affects the quotations. The discount rates on commercial bills, at foreign money centers, also affect the rate of exchange. If rates of discount are low, commercial drafts may "cash'' for a greater sum than could be realized by selling them as exchange. The question of the parity of units of foreign coinage, as compared with the units of our own coinage, also has its effect. The exchange rate will seldom go beyond the limits of the cost of buying 372 BUSINESS ARITHMETIC. and shipping the actual gold — these costs including commis- sion, freight, insurance, and often interest lost while money is in transit. That is, the rate of exchange does not go beyond the point where it is cheaper to send the actual money. Newspaper quotations are given, usually, for both sight and time paper. Par of exchange is either (1) mint par — the equality of actual value of pure metal in units of money in the countries concerned ; or (2) commercial par — a rarely used term referring to a practical equality of debt between countries that reduces the rate of exchange to the face value plus the cost of shipping. ILLUSTRATIONS OF EXCHANGE. 1. ^, in New York, sells B in London merchandise, $600. A draws his draft on 5, in pounds sterling. He may then discount the draft at a bank; or he may sell it in open market to some party who wishes to make a pay- ment in England. In the former case, the bank either sends the draft to England for collection — in which case the money is probably held by the correspondent bank in England, and the home bank draws and sells its bank exchange against the sum, and like sums, to any purchaser — or it may sell the original draft to a customer. In case A sells the original draft direct, or through a broker, the party buying it, C, who may owe D in England, sends the draft, properly endorsed, to the latter, who presents it to B for payment. By such a general process, it is evident that A gets his payment at once, and that sooner or later B pays the draft on pres- entation and thus settles for the goods that he bought. Incidentally, the bill has served the further purpose of making settlements between other parties. 2. In case A had been the buyer and B the seller, B might carry through the same process shown above. The settlement might start with A, however, who might go to a bank or broker, and buy a draft on England for the amount due, sending it to B who would cash it. In this case A corresponds to C, in ex. (1). Notice, in each case, that this process permits each person to make or to receive settlement in the money of his own country. ORAL EXERCISE. 1. In what coinage would an American importer order of a French exporter? If he bought exchange, with which to make settlement, in what coinage would it read? With what money would he make settlement for the draft? DOMESTIC AND FOREIGN EXCHANGE. 373 2. An English dealer orders goods from the catalogue of an American manufacturer. He finds prices quoted in what coinage? If he pays by bank draft, the draft is made out in what money? If he is drawn upon, in settlement, the draft is made out in what money? If the manufacturer sells the draft in New York he will receive money of what country? How would you determine: 3. The cost of a draft on Paris, at 5.19? 4. The cost of a draft on London at 4.88? 5. The cost of a draft on Berlm at .958? 6. The proceeds from the sale of a draft on Lyons at 5.185? 7. The proceeds from the sale of a draft on Amsterdam at 40$%? 8. The proceeds of a draft on Berlin at 96i? 9. Which are the higher rates, 5.19 or 5.20? 96 or 95^? EXERCISE. (For Mint exchange rates, see page 281.) 1. What is the face of a draft that an English exporter should draw on a New York customer, at mint par, to cover the purchase of 240 yd. broadcloth at 0/14/6 per yd.? If the exporter sells this draft in the market for 4.87, what proceeds does it yield him? Compute the cost of drafts on: 2. Paris, for 9700 francs at 5.18 3. Berlin, for 12,500 M. at .962. 4. Glasgow, for £161 4s., at 4.87|. 5. Amsterdam, for 752 g., at .405. 6. Rome, for 768 L. at .238. 7. Geneva, for 1284.6 fr., at 5.18|, brok. 1/8%. 8. Is the balance of trade probably for or against this country if ex- change is quoted at 5.2, 4.88, .412? Exchange Rates. (From a daily newspaper.) Sight. 60 days Cable transfers 4.87? Bankers' sterling 4.87 4.85^ Commercial bills 4.84^ Francs 5.161 5.171 Reichsmarks ; " 95| .95i Guilders 40| .40i 374 BUSINESS ARITHMETIC. Note. Use the newspaper quotations on page 373 in the following: 9. James Bros. & Co. purchase an invoice of £2000 worth of woolens, paying by 60-day bank draft. Cost of draft? 10. How large drafts would $500 buy at each of the above quotations? 11. A. B. Norton drew on a Paris customer a draft for 12,500 fr. and sold it at 1/8% commission, receiving what amount? 12. What does it cost to cable an agent £120? 13. B. C Baker shipped S. Tournier, Paris, 5000 bu. wheat, invoiced at 4.8 fr. drawing a 30 day draft on the purchaser. This draft, accom- panied by bill of lading and insurance certificate, he sold to Mason and Dean, private bankers, at 5.19, and should receive what sum? 14. Mason & Dean sold the draft just mentioned to Robert Bros., at 5.18, receiving $ — , and making a profit of $ — 15. Robert Bros, mailed the same draft, in settlement of account, to a Paris customer, who discounted it at a bank at 1%, receiving — fr. The bank presented the draft to Tournier(ex. 13) who was allowed to settle for it instead of accepting it, at 1/2% discount. He then received the bill of lading that entitled him to the wheat. CHAPTER XLI. DEPRECIATION. Depreciation is the loss of value of any property due to weaj*, new inventions, exhaustion of product, or other causes. It is the difference between original value and marketable value at time of disuse. Illustrations. (1) In spite of repairs, a factory or piece of machinery- deteriorates to the point where it must be discarded or rebuilt. This deterioration is depreciation. (2) New inventions or improvements may- make old machinery valueless. (3) A lessening of demand for a product may cause the disuse of machinery, and a consequent lessening in value. (4) A mine deteriorates from exhaustion of its ore, etc. To avoid the necessity of meeting the entire cost or replace- ment in any one year, it is customary in large concerns, to allow for depreciation, setting aside each year during the probable useful life of the property, sufficient funds to equal the cost of replacement when necessary. The sum set aside is largely a matter of scientific guess-work based on past experience. It is considered a running expense and may amount to 30% of such expenses. Allowing for individual variation, the following table shows common rates. Annual Rates op Depreciation. Modern fireproof steel and tile factories 4% to 5%. Steel construction, partly fireproof 6%. Warehouses, modem, fireproof 2^% to 4%. General freehold buildings, office and general use 1|% to 3%. Cheaply constructed building, according to use 8% to 15%. Fixtures of power and lighting plants 12% to 15%. General fixtures of buildings, other than machinery 6% to 15%. Machinery (average 11§%) 8% to 15%. Horses, work 15% to 25%. 375 376 BUSINESS ARITHMETIC. Annual amounts of depreciation are computed (1) as a fixed proportion of the original value; (2) as a fixed per cent, of reduced values; or (3) by the sinking fund method. 1. Based on Original Value. Illustration. A machine in a cotton mill, costing $560, has an average working life of 10 years, when it is disposed of for $120. Determine the annual allowance for depreciation. Solution. Depreciation = Original cost — disuse value = $560 — $120 = $440. Since this must be covered in 10 years the annual depreciation is 1/10 of $440, or $44. EXERCISE. 1. Compute the annual depreciation at 2f %, on a fireproof warehouse costing originally $364,200. 2. Compute the annual depreciation on $340,200 worth of fixed ma- chinery, at 12%, and on tools originally worth $3640, at 18%. 3. A department store owner has 40 dehvery horses originally costing $5680. He reckons the depreciation, at 22^%, as $ . 4. A boiler, costing $520, is estimated to have a life of 12 years, and then to be worth $60. Estimate annual depreciation and rate of depre- ciation. 5. A contractor employs a $14,500 dredge in contract work. In com- puting cost of work, as a basis for bids, he includes interest on the cost of the dredge and depreciation. He reckons interest at 6% per annum and depreciation 1|% per month. What should he" allow for interest and depreciation on a contract that he estimates will take 7 5 months? II. Based on Reduced Values. With machinery, for example, repairs grow heavier with age. Hence, with uniform depreciation and increasing repairs, the working cost is heaviest when the machine is least useful. To equalize the yearly cost therefore, depreciation is often heavier when repairs are light, decreasing as repairs grow heavy. Depreciation of this type is often expressed as a fixed per cent, of the net value of the previous year. Illustration. Compute the third year's depreciation, at 10% on reduced values, of machinery costing $900. DEPRECIATION. 377 Solution, $900 Orig. value 90 Depreciation at 10%. 810 Net value, end 1st year. 81 Dep. 10%. 729 Net value 2d year. 72.90 Dep. 10%, for 3d year. Ans. Note. If term and rate of depreciation are known, tables similar to the following may be computed for shop use. Table of Depreciation per $100 op Original Value. Term Year. 2% 3% 4% 5% 10% 15% 20% lyr. 2.00 3.00 4.00 5.00 10.00 15.00 20.00 2 1.96 2.91 3.84 4.75 9.00 12.75 16.00 3 1.92 2.82 3.69 4.51 8.10 10.84 12.80 4 1.88 2.74 3.54 4.29 7.29 9.21 10.24 5 1.85 2.66 3.40 4.07 6.56 7.83 8.19 6 1.81 2.58 3.26 3.87 5.90 6.66 6.55 7 1.77 2.50 3.14 3.68 5.31 5.66 5.24 8 1.73 2.42 3.00 3.49 4.78 4.81 4.19 EXERCISE. 1. Compute the depreciation for the sixth year, at 8% on reduced balances, on fixtures originally worth $8240. 2. A factory costing $82,600.00 is five years old. What is its inventory value, if depreciation is reckoned on reduced balances at 4|%? By Table. 3. Reckon the depreciation for the eighth year, on machinery worth originally $6525, if the depreciation rate is 10%. 4. What is the total loss of value in seven years, of a boiler costing $1800, and depreciated 15% annually? 5. Compute the inventory value at the end of 8 years, of an equipment of cotton mill machinery, costing originally $38,250, and depreciated at 20% annually. 6. Determine a rate of depreciation for machinery that is expected to depreciate from $900 to $630 in 7 years. III. Sinking Funds. Depreciation is sometimes provided for by establishing a sinking fund, that is a fund which is invested at compound interest at the rate necessary 378 BUSINESS ARITHMETIC. to provide for renewals. Frequently this fund is based on annual invest- ments of equal amounts. Business establishments seldom establish a sink- ing fund, since, when prosperous, funds earn a higher rate by being con- tinued in the business, than by outside investment. Sinking Fund Table. Shxywing Accumulations oi Compound Interest of Annual Investments of One Dollar. Year At 4^ At 5^ At 6^ Year At45< At6?t At6)t 1 1.040000 1.050000 1.060000 11 14.025805 14.917127 15.869941 2 2.121600 2.152500 2.183600 12 15.626838 16.712983 17.882138 3 3.246464 3.310125 3.374616 13 17.291911 18.598632 20.015066 4 4.416323 4.525631 4.637093 14 19.023588 20.578564 22.275970 5 5.632975 5.801931 5.975319 15 20.824531 22.657492 24.672528 6 6.898294 7.142008 7.393838 16 22.697512 24.840366 27.212880 7 8.214226 8.549109 8.897468 17 24.645413 27.132385 29.905653 8 9.582795 10.026564 10.491316 18 26.671229 29.539004 32.759992 9 11.006107 11.577893 12.180795 19 28.778079 32.065954 35.785591 10 12.486351 13.206787 13.971643 20 30.969202 34.719252 38.992727 EXERCISE. 1. What is the sum that must be invested annually in a 4% sinking fund to provide for replacing a $12,000 equipment of machinery in 10 years, if the estimated value of the equipment at the end of the period is $1950? 2. What allowance for depreciation should be made annually for machinery that is expected to deteriorate in value $6750 in 5 years, when it must be replaced? The sinking fund is invested at 6%. What at 5%? 3. A certain factory building is estimated to have a life of 20 years. It is built at a cost of $46,000. Estimate uniform annual depreciation. Compute annual allowance for sinking fund at 6%. CHAPTER XLII. COST-KEEPING. Knowledge of the exact cost of the articles that he makes is of vital importance to a manufacturer. Owing to slight margins of profit, carelessness in estimating materials and labor, or needless waste, may lead to large losses. The determining of the exact cost of an article, or piece of work, is termed cost-keeping. It necessitates (1) the exact measurement of the material used and wasted, (2) the com- puting of the time spent on the work by each workman, and (3) the computation of the exact cost of material and labor. The labor cost is often greater and more variable than the material cost, and is considered the vital element in cost- keeping. Note. The computation of the cost of production is often exceedingly- involved, and varies with each class of work, but the fundamental principles are very simple and are worth considering, since the subject is becoming a matter of general discussion, and of frequent mention in current pul> lications. Illustration. A printing office receives an order for 2000 booklets of a specified character. At once a dupHcate docket (record sheet) is prepared (page 380) to show full particulars of cost. The original docket accompanies the work from start to finish, quantities and costs being entered at each stage, as determined, the form being completed when the work is finished. Each workman keeps his separate ticket, and marks on it the hours that he spends on each job. These tickets are turned into the office at night, and the time and labor costs transferred to the dupli- cate docket here on file. Labor costs are also summed up and entered on the original docket as it passes from hand to hand. The duplicates thus serve as checks. Each job, however, must bear its share of the expense of the office force, foremen, etc. (the non-productive labor), and its share of rent, wear on machinery, lighting, etc. (biu-dens) . This expense, after long experience, is expressed as a per cent, of direct cost, or of labor cost only. To this, if no advance price has been quoted, is added the per cent, of profit. The completed cost cards are filed to serve as a basis for estimating on future work. Errors, carelessness, and excessive cost of labor, may also 379 380 BUSINESS ARITHMETIC. be traced from them. Many improvements are suggested by a study of cost cards. 1 JOB WORK DOCKET fa/m^ ^ jVo/is/,'a, /5% .30 .^ MATERIAL COST :? *SHOP BURDEN ? COST PER PC. MAT. ? COST PER PC. LABOR F LABOR COST — 630 PC. p / COST PER PIECE F - '/2 Woo/ /a^^o-^c 4. Complete the following card. ARTICL. 3000 FORSBERG STEEL DATE CO. t/f^ ,.^.J. 7 ^^. .80 F .^... 7 A*. .25 F .^/^y F -»- 35 Vo p ^^^ F F 382 BUSINESS ARITHMETIC. Note. In the form just shown, operations axe paid for on the "piece" plan, and coal and use of machinery are charged by the hour, previous ex- periments having determined what is a fair charge. The construction engineer, or contractor, estimates costs per unit of finished work, or per day, and holds these unit costs as a basis for future estimating. EXERCISE. 1. A gang of seventeen men, working on a concrete construction, averaged 46 cu. yd. of concrete per day. Estimate labor costs per day and per cubic yard, based on the following organization: Per day. Per cu. yd. 5 men loading barrows, at $1.75 per day, $ . — $ . — 9 men mixing and placing, at $1.75 per day, . — . — 2 men tamping, at $1.75 per day, . — . — 1 foreman, 3.00 . — 2. A gang of men averaged 42 lin. ft. of concrete conduit (3/4 cu. yd. per lin. ft.) per day. Estimate the labor cost per day and per cu. yd. 12 men mixing, at $1.80 4 helpers ". 1.50 6 loading stone and sand 1.50 6 wheeling 1.50 8 pouring concrete 1.80 2 supplying water 1.50 2 placing metal 2.35 1 sub-foreman 2.75 1 foreman ' 3.50 3. Estimate the unit cost per cubic yard for 260 cu. yd. of concrete, laid in seven 8-hour days. Materials and Labor. 0.85 bbl. cement, per cu. yd., at $2.90 per bbl. .25 cu. yd. sand, per cu. yd., at $1.12. .50 cu. yd. broken stone, per cu. yd., at $.80. .40 cu. yd. rubble, at $.55. Water, for the entire quantity, $8.00. Labor, 9 men, at 18c per hour, each. Foreman, $3.60 per day. CHAPTER XLIII. BIDS AND ESTIMATES. Those desiring work done, or material supplied, often advertise for seale'd proposals, or bids, from such contractors as care to meet their requirements. These bids are based on specifications supplied by the advertiser, which state in detail the work to be accomplished, liow it is to be done, the time of completion, the character of material to be used, the conditions as to the acceptance of the completed work, payments, etc. The prospective bidder makes an estimate of the cost of the work, allows for his profit and prepares his bid in due form, usually on a blank supplied with the specifications. The sealed proposals are opened, frequently in the presence of all bidders who care to attend, on a fixed date, and the contract awarded. While the lowest bidder, if .reliable, frequently secures the contract for the work, parts of the work may be given to different contractors. The proposal often names prices for parts of the work, or for the work as a whole. Usually the bid must be accompanied by a certified check for a certain percentage of the amount bid, as security for the acceptance and performance of contract, if tendered; or a bond must be given as security. Model Proposal for a Gravel Road, proposal. To THE Board of Chosen Freeholders, County of AND State of New Jersey: Gentlemen — The undersigned hereby declare. . .that. . . .he. . . .ha. . . . carefully examined the annexed specifications and the drawings therein referred to, and will provide all necessary machinery, tools, apparatus and other means of construction, and do all the work and furnish all the material called for by said specifications in the manner prescribed by the 383 384 BUSINESS ARITHMETIC. specifications and the requirements of the Engineer and Supervisor under them, for the following prices: (1) Price per cubic yard for earth excavation, without classification, as per plans and cross-sections, throughout the length and width of the road per cubic yard. (2) Price per acre for grubbing and removing objectionable material from sidewalks per acre. (3) Price per lineal foot for completed tile drain per lineal foot. (4) Price per cubic yard for compacted gravel as specified per cubic yard. (5) Price per cubic yard for carting gravel more than one half mile and each additional half-mile or fraction thereof per cubic yard. (6) Price per cubic yard for stripping or removing earth from top of gravel bed per cubic yard. {7) Price per square yard for each ordered inch in depth in excess of thickness named per square yard. (8) Price (lump) for the whole road complete, according to the specifica- tions and plans prepared by the Engineer Accompanying this proposal is a certified check for the sum of one thousand ($1,000) dollars, payable to the order of the Director of the Board of Chosen Freeholders of .County; which check is to be forfeited as hquidated damages, if, in case this proposal is accepted, the undersigned shall fail to execute a contract with said Board of Chosen Freeholders, under the conditions of this proposal, within the time provided for by the foregoing advertisement for proposals; other- wise said check is to be returned to the undersigned. Signed Address N. J., In preparing his bid the contractor must allow for such things as: 1. Preparatory expense. Cost of preparing for the real work, such as setting up working machinery, clearing ground, etc. 2. Plant expense and supplies. Cost of maintaining tools, machinery, worksheds, etc. Supphes for equipment. Depreciation allowance. 3. Materials. Those used in the real construction. Freight and cartage. 4. Labor. Skilled and unskilled workmen. BIDS AND ESTIMATES. 385 5. Superintendence and general expense. Wages of firemen, time- keepers, superintendent, clerical force. Supplies for their use. This expense varies from 5% to 15% of labor. 6. Contingencies. Allowance for unforeseen expense, such as delays from bad weather, labor troubles, accidents, rise in price of materials- Often in excess of 10% of cost. 7. Profit allowance. At times a fixed per cent, of cost. At times, one rate on cost of materials, say 10% to 15%, and a second rate, say 15% to 25% on labor. These rates are often made high enough to cover probable contingencies. Note. Estimates of cost are based on current prices of labor, freight, materials, etc. : estimates of quantities, largely on past experience. Often the unit cost, as obtained by cost-keeping methods, is the basis for an estimate. The exercises that follow illustrate a few principles of the subject. ORAL EXERCISE. 1. A bidder for a contract for office supplies believes that he can secure the material for $3800. He reckons freight at $87.50; labor at $32.50. Estimate his bid on a profit allowance of 20%. 2. Compute a bidder's profit of 15% on labor estimated to cost $38o. 3. Compute a bidder's allowance for 12^% profits on materials costing him $1840. 4. A contractor estimates the gross cost of a prospective structure at $28,500. If he allows 20% for profits and contingencies, what should his bid be? EXERCISE. 1. A contractor estimates the cost of materials, plant, etc., for a certain work, at $46,758, and of labor at $39,200. He adds 10% to labor for contingencies. He then reckons a profit of 20% on labor and 12^% on materials. What is the amount of his bid? 2. Gross estimated cost of materials, $15,286.50; plant expense, $3495; labor, $7296.40. Profit on materials, 15%; on labor, 25%. Amount of bid? 3. Reckon quantities of material, cost of labor and arnqjint of bid, for a contract of 260 squares of slate roofing, basing values on these figures: (1) Unit cost per square, 214 pc. slate, $5.60; freight, $1.40; loading and hauling, 35c; slate waste, 2%; 16 lb. sheathing paper, 50c; 1 lb. nails, 5c; 21 lb. nails, galvanized 9c; labor, $1.70. (2) Superintendence, 10%. (3) Labor contingency, 10%. (4) Profit, on net cost of labor, 20%; on materials, 15%. 26 386 BUSINESS ARITHMETIC. 4. Compute the face of bid on 14,365 cu. yd. concrete foundation, based on a unit cost per cu. yd., as follows: Stone, $1.15; gravel S.14; sand, $.08; cement, $1.36; labor, $1.05; superintendence, $.05. Profits on labor, 25%; on materials, 15%. 5. Compute bid (ex. 4), allowing a straight profit of 20% on cost per unit. • 6. A contractor agrees to cut and supply 60,000 ft., B. M., of 3" plank, basing this bid on the following cost per unit M ft.: stumpage, $1.75; cutting, $1 40; skidding, $.75; sawing, $2.40. Compute his bid, allowing 25% for profit and contingencies. CHAPTER XLIV. PARTITIVE PROPORTION AND PARTNERSHIP. Partitive proportion, one phase of ratio, is the process of dividing a number into parts proportional to two or more given numbers. ORAL EXERCISE. 1. Compare 4 and 6. 2. A's 12 shares compare how with B's 4 shares? Together they have how many shares? 3. Divide $320 into parts proportional to 2 and 3. Suggestion. 2+3=5 parts. $320 = 5 parts. 1 part = $64. 2 " = ? 3 " = ? 4. Divide $7200 in the ratio of 5 to 7. 5. Divide 1200 yd. into two lots having the ratio 3/5. 6. Divide 6300 lb. into parts proportional to 1, 2 and 3. 7. Divide $1800 into parts proportional to 1/4, 1/3, and 1/6. 8. Divide $600 between A and B, so that A receives 3 times as much as B. 9. Two dealers unite in ordering a carload of 40 tons of soft coal. The freight bill is $24. If the first dealer, who bought 15 tons, settles the bill, what should the second dealer pay him? 10. A piece of freight is carried 200 miles on one railway, and 150 miles on a second, for a total charge of $35.00. What should each railway receive? A partnership is an association of two or more persons for the purpose of conducting a business in common, and of sharing in its profits or losses. Partnerships are formed under verbal or written agreements. The latter usually include a 387 388 BUSINESS ARITHMETIC. statement of the conditions concerning investments, with- drawals, and the distribution of profits. Collectively, the persons forming a partnership are termed a firm, a house, or a company. Individually, they are known as partners. Partnerships may be conducted under fictitious names, under the names of the partners, or under the name of one or more partners " and Co. " Partners are real or ostens- ible, if they are really partners and are known to the public as such. They are silent partners if their connection with the firm is not known to the public; nominal, if they make no investment and have no share in the proceeds of the business; or limited if they have only a partial legal responsibility for the debts of the firm. Real and nominal partners are re- sponsible for all liabilities of their firm. Silent partners when their connection becomes known must assume their share of responsibility for the liabilities of the firm. Limited and silent partnerships are not allowed in some states; in others it is often required that at least one partner shall be real. The capital of the partnership is the property invested in the business. It includes money, any tangible form of real or personal property, labor, good will, etc. Note. Good will is the term applied to the value of past custom of an old established place of business, in bringing future trade, or of confidence in a partner who "lends his name," etc. Such a partner is often credited with good will as an investment, at some money value, on which his share of profits may be based. The net capital or present worth of a firm at any time is the excess of resources or assets (property owned and debts due) over liabilities (debts owed). If liabilities are in excess there is net insolvency, FOR DISCUSSION. Compare partnerships and stock companies as to: 1. Method of organization. 5. Effect of death of partner or 2. Method of investing. stockholder. 3. Withdrawal from business. 6. Government and management. 4. Sale of interest in business. 7. Individual responsibility for debts. PARTITIVE PROPORTION AND PARTNERSHIP. 389 ORAL EXERCISE. 1. Total gains — total losses = ? 2. Total losses — total gains = ? 3. Resources =fc ? = net capital. 4. Original capital + net gain = ? 5. Original capital — net loss = ? 6. Resources — liabilities = ? . How check? 7. May a firm gain during a period and be insolvent at its close? EXERCISE. Supply missing words or values. Solve mentally if possible. 1. Gains $2400; losses $560; net -, $ . 2. Gains, $2150; losses $2680; net , $ . 3. Capital at commencing $16000; at close, $18500; net = $- 4. Original capital, $5000; gains, $1250; losses $640; net capital at close, $ . 5. Net insolvency, Jan. 1, 1907, $2500; Jan. 1, 1908— Resources, $5260; liabilities $5000. Result of business for the year a of $ . 6. A & Co. invested $12,000, and a year later had cash, $2000; mdse., $10,500; real estate $2500, but owed debts of $2000. A & Co.'s net capital then was $ . 7. Jan. 1, 1912. Net capital $5000. Jan. 1, 1913, gross gains during year $3000; gross losses, $1200. Present resources $8000; liabiUties $900. Prove the existence of an error in these figiu-es. Distribution of Profits and Assessment* of Losses. In partnerships, profits are distributed and losses met in proportions agreed upon. These may be arbitrary or graded for special service, or based on amounts of investments; or a combination of these factors. I. EQUAL DIVISION OF PROFITS. ORAL EXERCISE. 1. Divide $726 profits equally between Adams and Brown. 2. A's investment was $4000 and D's $6000. They share equally a profit of $800, but D withdraws $100 of his share. The net capital of the firm is then $ ?, and that of D, $ ? 390 BUSINESS ARITHMETIC. 3. A invests $2000, B $4000, and C $3000. Determine their present worths, after they have shared equally a loss of $960. EXERCISE. 1. July 1, 1908. Investments: A, $12,600; B, $14,800; C, $9,650. Jan. 1, 1909. Withdrawals; A, $1200; B, $400. Jan. 1, 1909. Gains: On mdse., $7260.50; on real estate $1260. Losses: Expense, $3126.50; fuel and light $568.40; taxes $165. Find the present worth, if profits are shared equally. 2. On Jan. 1, Evans invested $12,000, Brown $8000 and Thompson $15,000 in the real estate business. A year later their resources were: Cash $7290; accounts and notes receivable, $11,360; real estate $26,750, while they owed notes for $3296.50 and taxes of $561.30. Find the present worth of each member. II. PROFITS SHARED IN ARBITRARY PROPORTIONS. ORAL EXERCISE. 1. Divide $792 between A and B in the ratio of 3 to 1. 2. A receives 2/3 of the profits of a business in which he invested 1/4 of the capital of $12,000. Find his present worth if the profits are $900. EXERCISE. 1. Crown and Briggs entered into partnership on Jan. 1, each investing $8000. Owing to Crown's greater experience, he receives 2/3 of the profits. On July 1, their profit and loss account stands as follows. Profit and Loss. Expense $429.60 Rent 600. Discounts 86. Merchandise $3129.65 Discounts 128.46 Stocks and Bonds. 82.50 Find the net gain and present worth of each partner. 2. A invests $9000 and B his services, under an agreement that the latter shall receive $900 yearly salary, to be paid from any profits, and 1/4 of the profits remaining. The profits at the end of a year amount to $3126.50. Find each partner's present worth, B having withdrawn only one-half of his salary. 3. A, B and C enter into partnership to manufacture advertising novelties. A invests $6000, and the other partners $4500 each. The partners agree: (1) To share losses equally; (2) To pay A $1200 salary PARTITIVE PROPORTION AND PARTNERSHIP. 391 out of any profits, for giving all his time to managing the business; (3) To divide any remaining profits in parts the proportion of 3 (A), 2 (B), and 2 (C). At the end of the year the profits are found to be; on merchandise, $9720; discounts, $112.50; and the expenses of the business exclusive of A's salary, $4326.50. Find the present worth of each partner. III. PROFITS SHARED ACCORDING TO INVESTMENTS. ORAL EXERCISE. 1. A invests $5000 and B $7000. What is the partnership capital? 2. Each partner invests what part of the capital? Each should receive what part of any profits? 3. What should each receive of $1400 profits? What shall each pay of $350 loss? What is each partner's share, according to the following investments? (a) A, $6000; B, $10,000. (6) A, $5000; B, $4000; C, $3000. Determine individual gain, loss and present worth in the following : Investment. Gain. Investment. Loss. 4. A, $2000; B, $10,000 $1500. 6. E, $3000; H, $8000 $3300 6. C, $6000; D, $3000 $1860. 7. G, $6000; K, $1500 $4200 EXERCISE. 1. The net profits of the firm of Thompson and Harris for the last year are $3584.64. This is to be apportioned to the partners' investments. Make the apportionments if Thompson's investment was $6000 and that of Harris, $1700. 2. Brown, Kann and Eastman engaged in a retail business, investing respectively $12,000, $8000 and $19,000. Brown, who was the only active partner, was permitted to take $1500 from the profits, annually for his services, the balance of the profits being divided according to in- vestment. Net profits for the past year were $7843.12. Make the dis- tribution. At the time of apportioning profits or losses a business statement is often prepared, showing a summary of the resources and liabilities, the investments and present worth, and, at times, the losses and gains. 392 BUSINESS ARITHMETIC. EXERCISE. 1. Complete the following business statement. Partnership Statement. Harris and Davidson, December 31, 19- Resources. Cash 5280 15212 5000 8240 45 20 60 3204 2500 ? Merchandise on hand and unsold Notes Receivable Accounts Receivable Liabilities. Accounts Payable 50 Notes Payable Present worth, Harris & Davidson ? ? Gains. Merchandise 3640 1815 2116 ? ? 60 45 12760 521 ?0 Interest and Discount 60 Losses. Labor Rentals Expense Harris, Net Gain Davidson, Net Gain Check. Harris, Investments 10000 ? 13000 681 — ? ? Harris, Net Gain Harris, Present Worth Davidson Investment Davidson Withdrawal Davidson Net Investment ? ? Davidson Net Gain Davidson Present Worth Harris and Davidson Present Worth ? Note. Share profits according to investment. ^ IV. INTEREST ON INVESTMENTS. Sometimes interest on investments is paid from the profits, the balance of profits being apportioned otherwise. Illustration. Example. A invests $6000 and B $9000 in a business, realizing $3450 profits the first year. Six per cent, interest is allowed on investments, the balance of profit being apportioned in the ratio of 1 : 2. Find present worth of each partner. PARTITIVE PROPORTION AND PARTNERSHIP- 393 Solution. Interest on A's investment, 1 yr. on $6000 = $360 Interest on B's investment, 1 yr. on $9000 = 540 900 $3450 - $900 = $2550, net profits. . A's share, 1/3 of $2550 $ 850 " B's share, 2/3 1700 A's present worth, $6000 +$360 +$850 = 7210 B's present worth, 9000+540 + 1700= $11240 EXERCISE. 1. Andrews entered into partnership with Gates on Jan. 1, 1913, investing $6000, while Gates invested $11,000. They agreed to allow each other 6% interest on investments, and to share the balance of profits in the ratio of 2 to 3. Find the present worth of each member on Jan. 1, 1914, after the apportionment of profits of $6585. 2. Jan. 1, 1913, Carleton invested $12,000; Johnson, $8500. Oct. 1, 1913, Carleton withdrew $3000. Jan. 1, 1014, profits for past year $4216.50. Interest allowed at 6%, and balance of profits shared equally. Apportionment? The average interest per partner is sometimes computed, and each partner is credited with the excess of his investment interest over the average, or charged with any deficit. The entire profits are then divided by agreement. Illustration. Last illustrative example. Solution. Interest on A's investment $360 Interest on B's investment 540 2 )900 Average interest. 450 $450 - $360 = $90 A's deficit (Debit). 540 - 450 = 90 B's excess (Credit). A's 1/3 of profits = 1/3 of 3450 = $1150 B's 2/3 = 2300 A's present worth = $6000 + $1150 - $90 (deficit) = $7,060 B's present worth = $9000 + $2300 + $90 = $11,390 EXERCISE. 1. April 1, 1912, A. M. Gates and J. B. Farquhar entered into partnership for the purpose of carrying on a manufacturing business. Gates invested $16,000 and Farquhar $12,500. It was agreed that interest should be allowed and charged at 6%, and that gains and losses should be divided 394 BUSINESS ARITHMETIC. equally. On Aug. 1, 1912, Gates withdrew $1000, and on Oct. 1, Farquhar withdrew $500. On Apr. 1, 1913, the books were closed and showed the following: Gains. Losses. Mdse $15,289 Labor $7240 Stocks and Bonds 986.40 Equipment 1645.20 Mdse. discounts 560 Discounts 300 Expense 1298.75 Find the present worth of each partner after apportionment of net gain. 2, Oct. 1, 1912, M. Andrews, B. T- Chase, and T. C. Lewis formed a partnership. Lewis invested $8500 and the other partners $7500 each. They agreed to allow and charge interest at 6% and to divide profits equally. On Dec. 1, Lewis withdrew $1000 and Chase $500. At the close of the year, assets and liabilities were as follows : Assets. Liabilities. Cash $5486.50 Bills pay $1250 Mdse 6798.15 Accts. pay 1670.60 Real estate 9725. Bills Rec 3000. Accts. Rec 4520. Make statement showing condition of business. VI. APPORTIONMENT BASED ON AVERAGE INVESTMENT. The sum that has the same earning power for a stated period as two or more sums invested for different periods, is termed an average investment. Profits and losses are often apportioned by average investment. The apportionment may be made by ratio or by interest. Illustration. Example. At organization, A invests $12,000, but withdraws $2000 at the end qf 8 months; B invests $6000 withdrawing $3000 after 6 months. How should the first year's gains of $4750 be apportioned? Solution (1). A's investment of $12,000 for 8 mo. = $96,000 for 1 mo. A's investment of $10,000 for 4 mo. = 40,000 for 1 mo. A's average investment of $136,000 for 1 mo. B's investment of 6,000 for 6 mo. = 36,000 for 1 mo. B's investment of 3,000 for 6 mo. = 18,000 for 1 mo. B's average investment = 54,000 for 1 mo. A's average + B's average = 190,000 for 1 mo. PARTITIVE PROPORTION AND PARTNERSHIP. 395 A's share = 136/190 = 68/95. B's share = 27/95. 1/95 of profit of $4750 = $50 68/95 = 68 X 50 or $3400 = A's profit. 27/95 = 27 X 50 or 1350 = B's profit. Solution (2). Using any rate, say 6%. Interest on $12,000 for 8 mo. = $480 Interest on 10,000 for 4 mo. = 200 A's invest, earning power = $680 Interest on $6000 for 6 mo. = $180 Interest on 3000 for 6 mo. = 90 "$270 Total earning power $680 + $270 = $950, of which A's mvestment represents 68/95 and B's 27/95. Final step — as in previous solution. EXERCISE. 1. M. C. Barton and James Thompson form a partnership to carry on an agricultural supply store. Barton invests $6000 for 9 mo., and then adds $2000. Thompson invests $14,000, but withdraws $4000 at the end of 4 months. • At the end of a year their summary accounts stand as follows: Farm Implements. Fertilizers. Cost $6240 Sales $5268.50 Cost $5692 Sales $7218.20 On hand 3650. On hand 965. Seeds. General Expense. Cost $2132 Sales $3586.40 Cost $1296.18 On hand 1125. Apportion the profits in accordance with average investment. 2. Chas. Parsons and T. P. Newton form a partnership • to carry on the grocery business. Complete the proprietors' accounts, apportioning profits by average investment. Chas. Parsons. Mar. 1, Mdse. $2000 Jan. 1, Invest $8000. Jan. 1, Pres. worth? Jan. 1, Net gain? T. P. Newton. July 1, Withdraw $6000 Jan. 1, 1908, Investment $18,000 Jan. 1, Pres. worth? Jan. 1, 1909, Net gain ? Profits and Losses. Losses. Profits. General Expense $1503.14 Merchandise $9290.68 Rentals 900 Discounts 587.40 Labor 1650 Delivery 2160 Parson's net gain ? Newton's net gain ? 396 BUSINESS ARITHMETIC. 3, 4, Solve the problems of the last exercise by average investment. GENERAL EXERCISE. 1. Frank T. Morris, R. P. Norton and Henry Collins form a partnership on April 1, to carry on a furniture business, investing respectively, $12,000, $6000 and $20,000. On June 15, Morris withdraws $500 and on Dec. 1, Collins withdraws $2500. At the close of the first business year, the books showed the following facts: Cost of mdse. purchased, $21,690; sales of mdse, $32,169.40; mdse. on hand unsold, $6,742.10. Cost of real estate, $12,000; alterations and repairs, $3167.50; estimated value at close of year, $14,500; notes in favor of firm, $5640; accounts due the firm, $5680; accounts owed by the firm, $1520; profits on discounts, $750; notes owed by the firm, $1200, mortgage on real estate, $6000; general expense for the year, $9650 45; cash, $18,811.45. Prepare partnership statements at close of year under following con- ditions: 1. Share respectively in proportions of 2, 1 and 3. 2. Interest allowed and charged at 5% and profits divided equally. 3. $1200 salary allowed Morris, balance of profits being divided ac- cording to original investment. 4. Profits shared according to average investment. FOR DISCUSSION. 1. Which system of apportionment is most fair, considering simply investments? 2. Which permits recognition of special service? 3. When is it impossible to use the method of "average investment"? CHAPTER XLV. EQUATION OF PAYMENTS AND OF ACCOUNTS. INTRODUCTORY EXERCISE. 1. Which party to a debt loses by its delayed payment? 2. Who gains by payment before due? 3. How may one measure, arithmetically, this gain or loss? 4. $600, which I owe, is overdue 1 mo. Money being worth 6%, what ought I to pay the creditor for the delay? Why? 5. $200 worth of mdse. bought Jan. 10, on 2 mo. credit, should be paid for on what date? The creditor is entitled to how many dollars extra, ff payment is made May 10? If paid Jan. 10 would the creditor gain or lose? What might he allow off? Debts draw interest from the time they are due until paid. Discounts, sometimes equal to interest on the debt, may be allowed on payments before due. If several payments are due the same party on different dates, a single payment equal to their total may be made on such a date that the overdue interest may equal the discount on advance payments. This date is termed the average or equated date. The process of determining it is termed averaging or equating. (Why is the settlement on the equated date fair to both parties?) EQUATION OF PAYMENTS. ORAL EXERCISE. 1. The interest on $400 for 6 mo. = interest on $200 for ? mo. 2. I owe $400 due in 2 mo. By paying $200 to-day for what extra time might I fairly retain the balance? 3. I owe $600 in 3 mo., and $600 in 5 mo. When might the total debt be paid without gain or loss to either party? Illustration (1). On Jan. 10, A. C. Brown bought $900 worth of mdse., payable, $200, Jan. 20; $400, Feb. 9; the balance, Mar. 10. (1) He 397 398 BUSINESS ARITHMETIC. settles in full Mar. 20. What does he owe? (2) On what date would the face of debt be a fair settlement? Interest 6%. Solution (1). Debt paid Mar. 20. Date Due. Payment. Overdue. Overdue Interest. Jan. 20 $200 6'd da. $1.97 Feb. 9 400 39 " 2.60 Mar. 10 300 10 " .50 Debt $900 , $5.07 Int . due 5.07 Due $905.07 Note. Use exact time. (2) By paying Mar. 20, Brown pays $5.07 extra. The earlier he pays the less interest he owes. The interest on $900 for 1 da. is 15c. .". 5.07 represents 507/15 days loss of interest, or 34 days. .'. The equated date for payment of $900, is 34 days before Mar. 20, or Feb. 14. It is evident that interest is found on each payment from due date to date of assumed settlement. The total interest, divided by the interest on the total debt, gives the number of days of delayed payment. If there is discount in place of interest, the real payment should be made later. Illustration. Find the equated date for pajnments of $600 due Mar. 10; $300, April 8; $600, May 12. Solution. Assume a date of settlement, called a focal date. Select as focal date. May 12, date of last payment. Date. Payment. Overdue. Interest. Mar. 10. $600 63 da. (to May 12) $6.30 Apr. 8. 300 34 1.70 May 12 600 _0 Due $1500 .25) $8.00 due Interest on $1500 for 1 da. = 25c. 32, no. of days. .'. By paying May 12, the debt is overdue 32 days. 32 days back from May 12 = Apr. 10, the equated date. Check Solution. If Apr. 10 is correct, there should be no gain or loss by paying face on that date. Take Apr. 10 as focal date. Date Payment Overdue Before due Interest Discount Mar. 10 $600 (to Apr. 10) 31 da. 3.10 Apr. 8 300 2 .10 May 12 600 32 da. 3.20 check 3.20 = 3.20 EQUATION OF PAYMENTS AND OF ACCOUNTS. 399 Note. This means that $1500 may be paid in full settlement on Apr. 10. On any date thereafter the amount due is $1500 + interest from Apr. 10. Remember: 1. Any date may be used as focal date or trial date. 2. By using first or last date, one interest computation is saved. Why? 3. The face of debt is always due on the equated date. EXERCISE. Find the equated date, and check by using equated date or another focal date. 1. A. B. Chase, Dr. 2. S. P. Randall, Dr. 1912 1912 Jan. 16 To mdse. $300. Mar. 12 To mdse. $420. Jan. 29 To mdse. 900. May 16 To mdse. 500. Mar. 12 To mdse. 400. Aug. 15 To mdse. 320. What is due Dec. 1? 3. James Field, Dr. 1912 Oct. 16 To mdse. $520. Oct. 20 To cash 120. Dec. 15 To mdse. 800. 1913 Jan. 3 To mdse. 320. In the following, re-write the accounts by substituting the true dates when due, obtained from the credit terms. 4. Robert Everhart, Dr. 5. James Donnelly, Dr. 1912 1912 Feb. 21 30 da. credit $300 Mar. 3 To mdse. $240. Feb. 25 2 mo. credit 600 Mar. 18 To mdse., 30 da. 380. Mar. 16 1 mo. credit 900 Mar. 30 To mdse., 2 mo. 456.40 Apr. 4 To mdse., 60 da. 820.60 Apr. 16 To mdse. 900. 6. Jones & Co., Dr. ■ 1912 Apr. 16 Mdse. 60 da. $456.10 Apr. 29 Mdse. 30 da. 829.30 May 5 Mdse. 3 mo. 420.00 June 16 Mdse. 36.50 EQUATION OF ACCOUNTS. Accounts having debits and credits are equated to determine the due date of the balance. The most convenient focal date is the latest named in the account. 400 BUSINESS ARITHMETIC. Illustration. James C. Westcott. 1909 Mar. 12 Mdse. $360 Mar. 30 By cash $300 29 300 Apr. 10 By note 120 May 4 240 (Balance $480 Solution. Here we have both debit and credit interest, the former being charged on debts overdue, and the latter being credited on pay- ments made to focal date. Focal date. May 4. Date. Debit Amounts. Period (to May 4). Dr . Interest. Mar. 16 $360 49 da. $2.94 Mar. 29 300 36 da. 1.80 May 4 240 $900 $4.74 Credit Amounts. Cr . Interest. Mar. 30 $300 35 da. $1.75 Apr. 10 120 24 da. .48 $2.23 $420 Balance = $900 - $420 = $480. Net interest = Dr. int. - cr. int. = $4.74 - $2.23 = $2.51, dr. int. Since the interest is debit, payment is overdue. Interest on $480 for 1 day = 8c. 2.51 -^ .08 = 31 +, the number of days overdue. May 4 — 31 days = Apr. 3, equated date. Check. Assume Apr. 3 as focal date. Date. Amount. Period. Dr. Int. Cr. Int. Mar. 16 $360 18 da. $1.08 Mar. 29 300 5 da. .25 May 4 240 31 da. (before due) $1.24 Mar. 30 300 4 da. (before due) .20 Apr. 10 120 7 da. (delayed) 1.14 $1.47 check $1.44 Note. The difference between dr. and cr. interest = the remainder in first solution, when dividing total interest by interest for one day. EXERCISE. Find and check the equated date. 1. C. P. Newton. 1912 1912 Mar. 16 To mdse. $1400 Apr. 4 May 28 2200 June 6 By cash $ 200 1000 EQUATION OF PAYMENTS AND OF ACCOUNTS. 401 2. The Newbold Co. 1912 Nov. 19 To mdse. $26,580 30 " 41,590 f)ec. 12 " 16,250 30 " 38,845 1913 Jan. 20 " 268 1912 Nov. Dec. Dec. 30 15 31 By cash Note $200 300 500 3. Chas. p. Randall. 1912 1912 July 15 To mdse., 30 da. $360 Aug. 5 By cash $400 31 15 da. 180 Sept. 10 Note 300 Aug. 12 30 da. 420 10 Cash 300 Sept. 6 2 mo. 620 4. James Brownet.t. & Bro. 1912 1912 Feb. 28 By note $ 300 Feb. 16 By mdse. 60 da. $325 Mar. 16 Mdse. 120 20 " 30 da. 244.60 Apr. 2 Cash 400 Mar. 18 " 3 mo. 840 May 10 (( 600 29 " 2 mo. 628 Note. accounts. Account sales are often averaged m the same manner as general Washington, D. C, May 16, 1913. Account Sales op Potatoes. Sold for account of James Parker, Hyattsville, Md. By Robert Harper, Commission Merchant. 1913 Sales. Apr. 30 20 bbl. potatoes. $2.40 May 6 50 " " 3.00 8 120 " 2.60 12 10 " Charges. 2.00 Apr. 28 Freight and cartage .46 May 12 Storage .30 12 Commission, 5% of sales ? ? Net proceeds, due by equation ? 27 402 BUSINESS ARITHMETIC. CASH BALANCE. The cash balance of an account on any particular date is the amount due on that date. If interest and discount are not allowed, the cash balance is the same as the account balance; if allowed, it equals the regular balance plus or minus the net interest or discount; or it equals the balance of the account plus interest for the time between equated date and average date, if later than equated date; or minus interest, if given date is earlier. The first illustrative example, page 398, shows the cash balance on March 20. EXERCISE. Find the cash balance directly for each of the accounts in the preceding exercise, checking by using equated date. Ac. 1. What is due July 1? 2. What is due Jan. 8? 3. What is due Oct. 31? 4. What is due Jan. 1, 1913? 6. What is due.July 1? CHAPTER XLVI. BILLING. INTRODUCTORY EXERCISE. 1. Bring to class several bills for goods purchased. State the infor- mation that each heading gives. 2. What information is given concerning the articles bought? 3. Who makes out a bill for goods? What becomes of it? Trace its life. 4. Of what value is the bill to the purchaser? What might happen in case no bill was given? The hill of merchandise is the statement in detail of mer- chandise sold in the course of trade, and is prepared by the seller for the buyer. Formerly the term invoice was applied to statements of merchandise sold wholesale, and of incoming goods from abroad. Now the terms are frequently used interchangeably. I. SIMPLE BILL. Washington, January 11, 1913. C. M. Brown, Groceries and Provisions. Sold to Henry Eastman, 1426 13th Street. 1 1 sk. Flour 85 2 6 lb. Lard @.17 1 02 3 U pk. Apples .72 4 10 b. Gran. Sugar. .06^ 5 2 qt. Molasses lb. Rice .18 6 3 .09 7 2 lb. Starch .05 8 Yeast 02 9 ? 10 11 Paid, 12 C. M. Brown 13 14 403 404 BUSINESS ARITHMETIC. EXERCISE. 1. State in your own words just what took place according to the bill on page 403. 2. Are there any non-essentials in this bill? Add two. 3. If settlement is not made, how is the bill affected? How, if sent C. O. D.? 4. What is the advantage of numbering the items on the bill? 5. Rule up three bill forms and prepare threo original bills for a Satur- day's marketing. Secure reliable information as to quantities and prices. Receipting for Payment. When payment is made, bills are receipted by the creditor or by the party representing him, who writes the word " Paid," or some equivalent expression, across the face of the bill over his signature. EXERCISE. 1. Why is the creditor's signature necessary? 2. Examine the forms of receipt of the model bills in this section, and throughout the book, and determine, in each case: (a) What facts are stated in the receipt. (6) By whom receipted and under what probable conditions. (c) What part of the receipt may be printed. (d) Under which of the following classes the receipt belongs: (1) Receipt by the direct creditor. (2) Receipt in the firm name by a partner. (3) Receipt of a corporation by an officer. (4) Receipt for the direct creditor by a third party. BILLING. 405 11. SIMPLE TWO COLUMN BILL. January 31, 191 Chestnut Grove Diary, 1421 Oak Street. John C. Norton, Jr. Sanitary Cream and Milk. Sold to James Fielding 646 Pine Street. Acc't Rendered 132 Qts. Milk .09 6 Pts. " .05 Pts. Gilt-Edge Cream 7^ Pts. " " .19 Gills " Pts. Choice " Hf. Pts. Choice " Gills " 1 A Z^+o, 'Dii4^4^y>.««^i1U AO Received payment John C. Norton, Jr. EXERCISE. 1. Extend the above bill. How should the form be changed if $3.00 is paid on account? How, if nothing is paid? 2. You are directed by James Kent to deliver daily 2 qt. 1 pt. of milk and 1 pt. of cream. In addition, you deliver: Jan. 5, 3 qt. milk; Jan. 8, 2 qt. milk; 1 pt. cream; Jan. 21, 2 qt. 1 pt. milk; 1 pt. cream. Prices: 9c per qt. for milk; 16c per pint for cream. Render a bill and have your clerk receipt for payment. 3. Design an original bill for similar use for an ice company. 4. In what other businesses could similar bills be used? Terms of Sale. As is evident from the problems and illustrations given, the terms of payment on bills of goods may vary markedly with the character and quantity of sales. The list of terms that follow are selected from actual bills. Cash. Net cash. List of Terms op Sale. 406 BUSINESS ARITHMETIC. 3. Cash or exchange on New York. 4. Account. 5. Interest after 30 days. 6. 30 days; 2/10. 7. Net 30 days; note to your own order, payable at a Philadelphia bank. 8. Net 90 days; 5% discount for exchange on New York in 30 days. 9. 30 days. If paid in 10 days, 2% discount. 10. 30 days. 11. 30 days. If not promptly paid interest will be charged from date of sale. 12. Cash, 5%; 30 days, 2%; 60 days, net. 13. Ten day note. 14. 60 days from Aug. 10; 2% 10 days. 15. 5% 30 days, or 6% 10 days. Payable in New York Gty funds. 16. Due, October 2. Less 2% if paid in 10 days. 17. C. O. D. 18. 5/60; 6/30; 7/10. 19. Best discount allowed for unexpired time is at the rate of 6%. 20. Net 30 days, or 1% discount in 30 days, or 2% discount in 10 days. New York funds. 21. Net after 60 days. 22. Subject to sight draft after 60 days. Payable in gold or its equivalent. 23. Net 30 days; 2% discount 10 days. No discount allowed after , 191—. 24. Accounts due the first of the month. 25. 2% discount if paid , 191—; Due net, , 191—. EXERCISE. 1. Give your understanding of the meaning of each of the terms stated above. 2. Examine any bills that you can secure and see whether you can add to the above list. 3. State the effect of each set of terms on the settlement of a bill of merchandise amounting to $1600, purchased August 1. BILLING. 407 III. FORM OF MANUFACTURER'S INVOICE AND LETTER OP TRANSMISSION. Address Albany, N. Y. Invoice. Name John Smithson Order No. 17296 Bought of WASHINGTON FLOUR CO. Manufacturers Fancy Patent Flours Buffalo, N. Y. Ship to Order Washington Flour Co. Date 12/30/191- Destination Albany, N. Y. Via. N. Y. C. R. R. Bbl8. H. Bbls. No.Sks. Sz.Sks. Kind. Brand. Marks. Pricfi PerCwt. Amount. 180 50 200 80 40 260 24 49 98 140 Paper Cott'n Grain Jute Ames Buffalo Prime Conrad Golden Queen Harvest Bakers Pride 5.65 4.90 4.60 4.80 4.40 4.45 ? ? ? ? ? ? ? Car 47329 Initials L V. R. R. Date 12/30/191— The above is an invoice of goods shipped this date, in accordance with your order of 12/17/191— Washington Flour Co. per Temple. EXERCISE. 1. Extend the bill and compare with previous bill, as to columns, entry of items, etc. 2. Explain the advantages and disadvantages of the use of many columns. 3. From the standpoint of the customer, what portion of the heading might be dispensed with? 4. Prepare a similar original invoice for a wholesale order, using news- paper market quotations. Add terms. Receipt the bill, signing your name as officer of the selUng company. 408 BUSINESS ARITHMETIC. IV. CLOTH MANUFACTURER'S INVOICE. Goods Purchased will not be Taken Back Except for Damage or Imperfection. No Claims allowed unless reported within 10 days after receipt of goods. MORRIS & NORTON DRESS GOODS New York, May 21, 191— John Harris & Co. Terms Register 1337 N. Y. City. S per cent. 30 das., or Book 45 Folio 14 6 per cent. 10 das. THIS INVOICE PAYABLE IN NEW YORK niTY FUNDS 19 Pes NORI^OLK LT PERCALE 45-i- 45^ 45^ 45^ 45^ 45^ 46^ 46^ 45^ 46^ 46^ 45^ 45^ 45^ 46^ 45-^ 46^ 46^ 45^ 16 Pes E-QREOLK PK PERCALE 45 45-3^ 45^ 46^ 45-^ 45^ 45^ 45^ 46^ 45-^ 46^ 45^ 45^ 46^ 45-^ 45^ 17 Pes -\^ PEPPERELL BROWN 45 45^ 46^ 45-^ 45^ 45-^ 45^ 45^ 45-i- 45^ 46^ 45^ 45^ 45^ 45^ 45-^ 45^ 10* 10 09 EXERCISE. 1. Extend the bill. What amount will settle it June 19? June 24? August 23? October 16? On what date is the face value due? 2. On October 19, the Fall River Manufacturing Co. sells you, on terms of 4/60; 5/30; 6/10; 36 pc. C. K. Checks, 45, 46, 47S 48, 45^, 46S 42', 412, 452^ 46^ 44^ 453^ 442^ 46^ 47^ 431, 45', 42^, 43^, 44S 42', 46, 47, 48«, 45, 41', 42, 44, 47', 45^, 43^, 44S 45^, 43', 42', 44 @ 23c. Also 12 pc. Ginghams: 45S 47, 46^, 43, 45, 49^, 50, 48', 47, 49, 51», 49« @9f. Draw up the bill and receipt for the company for settlement on November 16. BILLING. 409 3. Ascertain by inquiry the names, common piece lengths and prices of four standard cloths and draw up an original bill. Name your own terms and receipt for payment. V. DISCOUNT BILLS. See form on page 242. Claims for errors and over- charges must be made within ten days of delivery. Terms, 30 days net; 5% cash. Louisville, October 25, 191- THE KENAWHA SUPPLY COMPANY. GENERAL CONSTRUCTION AND RAILWAY SUPPLIES. 1468 River Street. Your Order No. 6657 Your Req. No Shipped L L. Frt. 10/25/1- Our No. 'D-S456 Salesman Roberts. Sold to John P. Carey, Evanston, IlL Quan- tity Fur- nished. Description. List. Disc, i Extension. Total. 16 sq. 10X14 Gal. Sheet Roofing ft. Gal. Valley ft. Ga . Climax lb. W. P. R. Wire C. Machine Bolts, \ X3^, C. Machine Bolts, 5/8X4 C. Machine Bolts, fX8 6 Le 4 7 13 87 11 14 ss 12 72 15 85 20% 30% 50, 10 5% 164 48§ IfiO 2 1^ Trade 1 Received payment, October 17, 191 — THE KENAWHA SUPPLY CO. J. Towne, Treas. Extend and explain bill. How does it differ in arrangement from the bill on page 242? Prepare discount bills for examples on pages 237, 239 and 242. Supply names of customers and dealers where necessary. 410 BUSINESS ARITHMETIC. VI. MONTHLY STATEMENT. Retail Statement. Samuel Henderson, The Morrison. Chicago, 111., Sept, 30, Wl- IN ACCOUNT WITH HENRY NORTON & COMPANY. Items. Chargea. Credits. Sept. 1 2 To Balance (Bill rendered) 12 yd. Lawn .18 8 Lawn Mull .30 3 iDt. Ribbon 1.15 6 yd. Embroidery .28 2 yd. Elastic .12 5 Binding .15 4 yd. Linen .65 1 set Purs, 1 Ermine Cape 12 yd. Silk 1.25 1 Set Furs 1 rem. Gingham 9 yd. Calico .10 5 C. Thread .10 1 rem. Gingham 2 Shirtwaists 3 pr. Blankets 4.00 2 pr. • Pillows 2.25 2 Bolsters 2.50 — 6Q 40 165 5 6 — 12 165 240 00 00 *Cr. 24 65 00 Cr. 27 9 60 65 Le ss — *NoTE 1. Credit items are usually entered in red. Note 2. Where individual purchases have covered many items on any one date, and a bill has been rendered at the time, details are not entered on the statement, but the date of each purchase, the words "to Bill Rendered" and the amount of individual bill are entered. This ia common in wholesale statements. BILLING. 411 EXERCISE. 1. How does this statement differ from an ordinary bill of goods? 2. Extend the statement. 3. Explain just what has taken place according to the statement How does it happen that credits occur? 4. Should the statement be receipted in case of partial settlement? 5. Prepare an original statement of a month's purchase of groceries. Receipt for payment. EXERCISE. 1. Draw up a " to bill rendered " statement from the following account: Robert H. Lawrence, 1356 18th Street, N. W. 191— — 191— 1 Mar. 3 Mdse. 1 42 16 Mar. 8 Cash 12 00 18 tt 121 15 21 Mdse. ret. 540 23 Debit memo. 4 19 23 Overcharge 48 27 Mdse. 16 35 30 10 da. dft. 150 00 28 a 39 12 31 Cash 50 00 30 (I 125 04 31 11 11 85 2. Draw up an original statement to cover one quarter of a year, containing at least fifteen debit items and six credit items of different character. VIII. REQUISITION. The requisition is a combined order and voucher or bill, used in ordering supplies for offices and workshops, when the approval of a higher official is necessary. Sometimes it does not involve prices, but is simply a request from an official or foreman, drawn on the stock clerk, for supplies needed, which the latter keeps in stock ready for use as required. In the illustration (page 412), however, the articles required must be purchased. The superintendent of a certain department asks for the supplies, and the general manager approves the request and directs the purchase. Requisitions vary greatly in different businesses and government offices. 412 BUSINESS ARITHMETIC. Requisition Book. March 10, 191— THE RANKIN MANUFACTURING COMPANY. Charge to the Repair Shop; Model Department. Please furnish for use in this shop the materials specified below: Henry C. Carter, Superintendent. Items. Quantity. 400 b. f. 2000 b. f. 600 b. f. 5qt. 9qt. 2 1b. 101b. 181b. Material. W.Pine, 2X3-6', Oak, 1X9, per M. Chestnut, per M. M. Y. Varnish H. O. Finish Indian Red Vermilion Red Yellow Murdock perM. $42.00 50.00 36.00 .21 .24 .10 .24 .26 Estimated Cost. March 11, 191— Approved and ordered purchased. Jas. M. Dean, Gen'l Man. Received the above mentioned material on March 16, 191 — Henry C Carter, Supenntendent. EXERCISE. 1. Extend the form. 2. State exactly what has taken place. 3. Prepare an original requisition for furniture and material required for the equipment of some business office. Sign as subordinate and have some other pupil sign as a superior officer. FOR INVESTIGATION AND REPORT. EXERCISE. 1. Collect specimen bills and statements from at least thirty distinct businesses. Classify these {a) in order of complexity; (6) according to the difficulty of the arithmetical computations involved. 2. Write a brief on the information furnished by the bill headings. Compare headings of the bills of retail, wholesale and manufacturing houses. 3. From your study of the bills discover how the "terms of payment" vary with the character of the business. BILLING. 413 4. What knowledge of arithmetic should a bilhng clerk have? 5. Study and report on the use and value of special columns. Give illustrations. 6. Collect, and submit an illustrated report on forms which, while not bills, are worked out on the same principle. 7. Show how each bill form is designed to meet the needs of the business in which it is used. 8. Examine bills and see how many different ways of entering items you can discover. 9. Report on the different forms of receipt for payment that you can discover. Explain the exact meaning of each CHAPTER XLVII. STORAGE. INTRODUCTORY EXERCISE. 1. What circumstances may lead a housekeeper to store her fumitiu'e? 2. Why does a commission merchant place certain merchandise in "cold storage"? Why does he not own a storage plant? 3. Why is grain "stored" in grain elevators? 4. If I rent a room in a furniture warehouse from July 15 to October 25, at $9.00 per 30 day month, or fraction thereof, the storage bill is $ . 5. The cold storage of 40 cases of eggs for 50 days, at 40c per month, or fraction, costs what amount? Storage is the charge for storing goods in an elevator or warehouse. It is computed on quantity rather than on value. The term of storage is the time for which the storage rate is quoted. The rate may be for the day, week, month, or period of 30 days, etc. Fractions of terms are usually considered as full terms. Storage warehouses and grain elevators are closely regu- lated by law. Often rates are fixed by law, by Chambers of Commerce, associations of warehousemen, etc. In the storage of grain, and of certain other produce, there are strict regulations for grading receipts of merchandise, since the person storing cannot expect to withdraw the identical property he deposited. SIMPLE STORAGE. Simple storage is storage computed at the time of withdrawal of goods. EXERCISE. (Solve mentally if possible.) Compute the storage on: 414 STORAGE. 415 1. 200 cs. eggs, for 3 mo. at 10c per mo. 2. 500 cs. eggs, for 4 mo., at 10c per mo. for 2 mo., and 8c per mo. for succeeding months. 3. 800 lb. of cheese from August 17 to October 26, at 8c per 100 lb. per month of 30 days. 4. The storage rate on the following bill is 9c per 100 lb. per 30 da. Extend the bill, explaining each item. March 31, 191— Messrs. James Brown & Son, To The Morton Storage Co., Dr. Article. Quantity. Quantity. Time. Kate. Lot No. In. Out. i Amt. 6746 Cheese 20000 lbs. Jan. 16 Feb. Feb. Mar. Mar. » 15 28 6 28 5000 lbs. 8000 2000 5000 30 43 49 ? 1 2 2 ? 9c 18c ? ? 4.50 ? ? ? ? 5. Prepare a bill for the storage of 160 cs. eggs on Jan. 15, by James Quick, at the rate of 10c per month of 30 days. Deliveries: Jan. 28, 40 cs.; Feb. 19, 50 cs.; the balance. Mar. 13. 6. Compute the storage on 5000 lb. poultry, stored Mar. 15 at l/4c per lb. per 30 da. DeUveries: Mar. 30, 1500 lb.; Apr. 4, 800 lb.; May 12, 2000 lb.; June 16, the balance. 7. Extend this bill. Rates: 10c per bale, 15c per cs., per 30 da. October 31, 19— Messrs. Cates & Co., To The Commercial Storage Co., Dr. Quan- tity. Marks and Nos. Rec'd. Deliv. Bate. Amount. 8 4 12 6 bales No. 675-83 A Cs. c D No. 721-24 bales A. C. O No. 829-41 bales BC No. 726-31 June July 8 16 27 5 Oct. Aug. Sept. Oct. 20 19 30 4 ? ? ? ? In cases where goods are being constantly received and delivered, and simple storage is charged, it is assumed that all deliveries are made from goods longest in storage. 416 BUSINESS ARITHMETIC. Illustration. Robert Connor stores 500 bbl. Apr. 16; 300 bbl. May 1; 200 bbl. May 19. He withdraws 400 bbl., Apr. 30; 200 bbl. May 18; 400 bbl., May 27. Compute his storage at the rate of 6c per month, or fraction. Solution. Date Rec'd Deliv. Rate. Apr. 16 500 bbl. 30 400 bbl. Stored, Apr. 16-30, 14 da. 6c May 1 300 1R rtrjn f 100 Stored, Apr. 16-May 18, 32 da. 12c ^° "^^^ \ 100 Stored, May 1-18, 17 da. 6c 19 200 97 .^n J 200 Stored, May 1-27, 26 da. 6c —- ^^ 1200 Stored, May 19-27, 8 da. 6c 1000 1000 Total storage, Cost. S24.00 12.00 6.00 12.00 12.00 $66.00 EXERCISE. 1. A. C. Bronson's storage memorandum is as follows: Stored, Nov. 12, 2000 lb. poultry; Dec. 5, 2000 lb.; Jan. 18, 400 lb. Withdrew, Dec. 24, 2200 lb.; Jan. 10, 1500 lb.; Mar. 17, 700 lb. Compute the storage at the rate of l/4c per lb. per month. 2. John T. Bartlett's memorandum is: Stored, Aug. 18, 50 bbl. potatoes; Sept. 15, 200 bbl.; Oct. 17, 300 bbl.; Withdrawn, Nov. 5, 200 bbl.; Nov. 20, 100 bbl.; Dec. 5, 100 bbl.; Dec. 20, the balance. Compute his storage bill at the rate of 6c per bbl. per month. 3. On Oct. 5, Brown stored 100 bbl. apples; Oct. 20, 50 bbl.; Nov. 16, 180 bbl. Nov. 1 he withdrew 80 bbl.; Dec. 1, 50 bbl.; Jan. 5, 100 bbl.; Feb. 9, the balance. Compute the storage at the rate of 12c per bbl. per month. 2. AVERAGE STORAGE. When receipts and deliveries are frequent, it is a custom to average the time, and to charge average storage. Usually exact time, with 30 day periods, is used. 1. The storage of 200 bbl. for one month equals the storage of 1 bbl. for ? months. 2. The storage of 100 bbl. for 2 months equals the storage of 1 bbl. for ? months. 3. The storage of 500 bu. for 20 days equals the storage of 1 bu. for ? days. Illustrative Example. The following is a memorandum of flour stored by C. P. Dean with the Commercial Storage Co., at a rate of 4c STORAGE. 417 average storage. Receipts: Feb. 6, 200 bbl.; Feb. 21, 150 bbl.; Mar. 8, 400 bbl.; Mar. 29, 200 bbl. DeUveries: Feb. 12, 100 bbl.; Mar. 9, 150 bbl.; Mar. 21, 300 bbl.; Apr. 4, 400 bbl. Solution. Arrange entries in order of dates : Date. Rec'd. Deliv. Balance Time. Eqiiiv. to 1 bbl. Stored for Feb. 6 200 bbl. 200 bbl. 6 da. 1200 da. 12 100 bbl. 100 9 900 21 150 250 15 3750 Mar. 8 400 650 1 650 9 150 500 12 6000 21 300 200 8 1600 29 200 400 6 2400 Apr. 4 400 The storage items are equivalent to the storage of one barrel for 16,500 days. 16,500 days = 550 terms of 30 days each At 4c per term, the storage = 550 X 4c = $22.00. Ans. Note. Notice that each "one bbl." equivalent is the product of the balance on hand by the number of days that balance remains unchanged. EXERCISE. 1. Solve the examples in the last written exercises by average storage. 2. Are charges lower by average, or by simple storage? Why? 3. Compute average storage at the rate of l^c per bu. per 30 days, from the following memorandum: Received: Oct. 10, 12,000 bu.; Oct. 25, 14,000 bu.; Nov. 1, 3500 bu. Deliveries: Dec. 5, 2500 bu.; Dec. 8, 3000 bu.; Dec. 21, 5,000 bu.; Dec. 30, 8,000 bu.; Jan. 5, 11,000 bu. 4. On Jan. 16, Robert Osbom bought 2,000 bbl. of flour at $4.25 and placed it in storage at an average rate of 5^c per barrel. On Jan. 29, he bought and stored 200 bbl. at $4.30; on Feb. 16, he withdrew 300 bbl. for sale at $4.65; on Mar. 11, 500 bbl. for sale at $4.80, and disposed of the balance, on Apr. 3, at $5.10. Drayage charges amounted to 15c per bbl. What was his per cent, of profit? FOR INDIVIDUAL REPORT. 1. Prepare a brief on the knowledge of arithmetic required by the average retail clerk. 2. Prepare a brief on the knowledge of arithmetic required by the average mechanic. 28 418 BUSINESS ARITHMETIC. 3. Report on "What Knowledge of Arithmetic is Most Necessary." Secure the opinion of business men and manufacturers. 4. Report on "Arithmetic Computation Tables in General Use." Make clear their value and range of use. Give sections of typical tables illustrating their use and the principles of their constructions. 5. Write a brief on "Machines for Arithmetical Computation." Cover range, usefulness, speed, accuracy, cost and extent of use. APPENDIX I. SIGNS AND SYMBOLS. a/c . . . .account. a/s .... account sales. + addition. O.K. . . all correct. & and. @ at, each, to. and so on. B/L ...bill of lading. c/o care of. V check mark, correct. circle. c cent. ° degree. -^ division. $ dollar. = equal, equals. o equivalent. ' foot, minutes. .fourths (written, as exponents: 2^ = 2i) 1. 2. 3 > greater than. C hundred. " inches, seconds. < less than. X multiplication; incor- rect. # number (written before a figure). o/d .... on demand. % per centum. # pounds (written after a figure). £ pound sterling. : ratio. *.' since. >/". . . .square root of. — subtraction. .' therefore. M thousand. A triangle. 419 APPENDIX II. STANDARD ABBRJSVIATIONS. Note. The singular form is now commonly used for singu- lar and plural, unless otherwise noted. Ac Account. A acre. agt agent. ans answer. B.O back order. bg bag. bal balance. bl bale. bk. ...... bank, book. bbl barrel. bkt basket. bot bought. bx box. bu bushel. en can. cd card, cord. car carton. cs case. C, C.B. . . cash, cash book. Cash cashier. csk cask. c, ct cent. eg centigram. cm. centimeter. ck check. ch chest. c.o.d collect on delivery. cml .commercial. com commission. Co company. consgt consignment. cr crate, credit, creditor. cu cubic. cwt hundredweight. da day. dr debt, debtor, debit. dept department. do ditto (the same). dol dollar. dz., doz. . . dozen. dft draft. ea each. E.O.E errors and omis- sion excepted. etc et cetera (and so forth). ex example, express. exch exchange. e. g exempli gratia (for example). exp expense. 420 STANDARD ABBREVIATIONS. 421 far farthings. fir firkins. f ol folio, page. ft foot. for foreign. fw., fwd. . .forward. f r franc. f.o.b. . . . . .free on board. gal gallon. gi gill- g gram. gr gross, grain. guar guaranty, guar- antee. hf half. hhd hogshead. hr hour. ewt hundredweight. in inch. int interest. I, inv invoice. invt inventory. ins insurance. inst instant (the pres- ent month). kg keg. km kilometer. L.F ledger folio. l.p list price. If., Iv loaf, loaves. lb pound. m.p marked price. mdse merchandise. m meter. Messrs. . . . Messieurs (gentle- men. Sirs). mi mile. min minute. Mr Mister. Mrs Mistress. mo month. no number. O.B order book. oz ounce. p., PP page, pages. pkg package. pd paid. pi pail. pr pair. pay payment. d pence. per per (by the). p.c per centum, pk peck. pwt pennyweight. pc piece. pt pint. poc, pkt. . pocket. lb pound. Pres President. prox proximo (the fol- lowing month). qt quart. qr quire. R.R railroad. rm ream. recM received. M Reischsmark. 422 BUSINESS ARITHMETIC. rd rod. sk sack. S sales. sec second. sec'y- .... secretary s shilling. set settlement. ship shipment. shipt shipped. sig signed, signature. st street. stk stock. sund sundries. trc .tierce. T ton. tr., trans. . transfer. treas treasurer, treasury. tb tub. ult ultimate (last month). via via (by way of). viz videlicet (namely, to wit). wk week. wt weight. yd yard. yr year. INDEX. Abbreviations, 420-422. Acceptance, 309-311. Accident insurance, 257. Account, 72. averaging, 397. cash, 73. expense, 74. merchandise, 73. personal, 74. proprietor's, 75. Account sales, 249. Accuracy, 7. Accurate interest, 293. Acre foot, 177. Acute angle, 143. Addition, 7. checking, 11. complements, 10. dictation, 7. grouping, 9. horizontal, 12. of decimals, 15. of denominate numbers, 138. of fractions, 95. principles of, 8-12. Ad valorem duties, 278, 283. Advertising, 77. newspaper, periodical, 79, 80. billboard, 79. poster, 78, 79. records of replies, 83, 84, 85. Agency, 245. Agent, 245. Agent, payment of, 246. Aliquot parts, 110. division by^ 115. multiplication by, 110. Altitude, 145. Analysis of problems, 118. Angle, 143. Angular measure, 134. Antecedent, 186. Appraiser, 279. Approximate results, 40. time, 169. Arabic notation, 3. Arc, 145. Area of circle, 147. of parallelogram, 146. of triangle, 146. Assav office, 51. Assessment, 269, 270, 274, 389. Assessors, 270. Automobile insurance, 257. Average, 43. Average clause, 264. Average investment, 394. Average storage, 416. Averaging, 43, 397. Avoirdupois weight, 133. Axioms, 46. B Bank discount, 318. register, 325. rules for, 320 terms of, 319. ticket, 324. Bank draft, 361, 362. Banker's 60-day method, 287. Base, 145. Base for percentage, 215, 221. Base for profit and loss, 226. Base ball averages, 219. Bear, 344. Beneficiary, 257, 259. Bids and estimates, 383. Bill, discount, 409. manufacturers, 407, 409. monthly statement, 410. professional, 69. simple, 403, 405. storage, 415. Billing, 403. Bill of exchange, 370, 371. Bill of lading, 280. Bin, capacity of, 162. 423 424 INDEX. Blank endorsement, 307. Board foot, 161. Boiling point, 175. Bond, 309, 337. compared with stock, 337. coupon, 337. quotations, 339. registered, 337. Bonded warehouse, 280. Bonus, 69. Broker, 245. Brokerage, 245. Building and loan associations, 351. distribution of profits, 353. earnings, 351. loans, 351. Bull, 344. Business accounts, 72. Business terms, 72. Calculation of time table, 170-171. Call loans, 345. Canadian money, 135. Cancellation, 88, 286. Capacity, measures of, 133. of bins, tanks, etc., 162. Capital, 334. Capital stock, 332. Carat, 133. Carpeting, 155. Cash balance, 402. Cashier's check, 361. Casting out nines, 11. Centigrade scale, 175. Certificates, gold and silver, 50. Certificates of deposit, 362. Change schedule, 66. Check, 357, 358. Checking the work, 11, 118. Cu-cle, 145. Circular measure, 134. Circulation of money, 62. Circulation statement, 12. Circumference, 145. Coal consumption, 210. Coins of the United States, 51. Cold storage temperatures, 176. Collateral, 345. loan, 345. note, 346. Collection fee, 361. Collector of the port, 279. Commercial discount, 236. Commercial draft, 363. Commission and brokerage, 245. Commission merchant, 245. Common stock, 334. Complements, 10. Composite measures, 177. Composite numbers, 86. Compound interest, 302. tables, 303. Compound proportion, 190. Compounds, 223. Cone, 156. Conical surface, 156. Consequent, 186. Consideration, 255. Consignee, 246. Consignment, 246. Consignor, 246. Contract, 255. Conversion of fractions, 103. Conversion tables, 184. Coordinates, 200. Copying numbers, 8. Cord measure, 160. Com, production of, 205. Corporation, 332. advantages of, 333. Correspondent bank, 360. Cost, 226, 230. Cost by hundred and thousand, 116. Cost per inquiry, 83. Cost-keeping, 379. ticket, 380, 381. unit of, 382. Cotton, production of, 209. Counting table, 134. Coupon, 337. Credit insurance, 258. Credit prices, 295. Cube, 156. Cube of numbers, 121. Cube root, 125. graphic illustration, 129. Cubic measure, 135. metric, 183. Curb market, 343. Currency memorandum, 66. Currency of U. S., 50. Custom house, 279. Customs, 277. Cylinder, 156. INDEX. 425 Date line, 167. Date of maturity, 172. Decimals, addition of, 15. division of, 39. multiplication of, 31. subtraction of, 22. Degree, circular measure, 165. Denominate numbers, 132. addition, 138. division, 141. multiplication, 140. reduction ascending, 137. reduction descending, 136. subtraction, 139. tables of, 53, 133-135. Denominator, 91. Deposit, 359. Depreciation, 375. on original values, 375. on reduced values, 375. table of, 377. Diagram, 193. Diameter, 145. Dictation, 7. Difiference column, 20. Dimensioning, 195. Discount, 236. bank, 318. rates, 240. series, 238. Discussion — See Questions for dis- cussion. Distribution of profits, 389-395. Dividend, 25, 335. Divisibility, tests, 86, 87. Division, 35. by continued subtraction, 35. long, 37. of decimals, 39. of denominate numbers, 141. of fractions, 101. short, 36. short methods of, 38. Domestic exchange, 355. Draft, 309. Drawee, 309. Drawer, 309. Dry measure, 133. Duties, ad valorem, 277, 283. specific, 277, 282. £ Endowment policy, 259. Endorsement, blank, 307. fuU, 308. quahfied, 308. English money, 135. Equality of expressions, 46. Equated date, 397. Equation, 46. Equation of accounts, 397, 399. Equation of payments, 397. Equilateral triangle, 144. Estimates, 383. Evolution, 122. Exact interest, 293. Exact results, 40. Exact time, 169. Exchange, 355. domestic, 355. fluctuation of, 364, 371. foreign, 366. inland, 355. rates, 364, 366, 373. Ex-dividend, 343. Expenditures of U. S. Gov't, 277. Exponent, 121. Exports of U. S., 208, 219. Express money order, 356. Expressions, equality of, 46. Extension, measures of, 134. Extremes, 190. F Face, 156. Factor, agent, 245. Factors and multiples, 86. Factor, highest common, 88. Factoring, 87. Factory expense, 68. Fahrenheit (temperature), 175. Fee, 269. Fidelity insurance, 257. Figures, geometric, 143. Fire insurance, 265. Flooring, 151. Floor plan, 152. Focal date, 398. Foot pound, 177. Foreign coin, gold value of, 281. Foreign exchange, 366. 426 INDEX. Foreign money order, 368. Formula, 179. Fractions, 91-109. addition, 95. compound, 92. conversion of, 103. division of, 101. improper, 92. lowest terms, 93. multiplication of, 97. proper, 92. reduction of, 91. similar, 95. simplification of, 93. subtraction of, 96. terms, 91. Fractional relations, 106. Fractional units, 91. Free list, 278. Freezing point, 175. French money, 135. Gable, 151. Gain, 72. Geometric conceptions, 143-147: 156-160. Geometric line, 143. German money, 135. Gold value of coin, 281. Good will, 388. Graph, 193; 204-211. Graphic arithmetic, 193. Graphic treatment of squares and cubes, 128. Greatest common divisor — See Highest common factor. Gross cost, 245. Gross proceeds, 246. Grouping, in addition, 9 Guaranty, 246. Hundreds, cost of, 116. Hypotenuse, 149. Immigration, table of, 210. Import duties, 277. Imports of U. S., 211. Improper fraction, 92. Income, 347. Income insurance, 262. Individual work, 42, 142, 163, 174, 185, 212, 219, 235, 324, 412, 417, 418. Indorsement — See Endorsement. Insolvency, 388. Insurance, 255. companv, 259, 264. personal, 256-263. poUcy, 256, 259. property, 256; 263-268. Interest, accurate, 293. banker's 60-day method, 287. cancellation method, 285. compound, 302. simple, 285. six per cent, method, 290. tables, 292, 303. Interest on investments, 392. Investment, 347. Invoices, 279, 282, 283. Involution, 121. Irrigation, 206. Isosceles triangle, 144. Job work ticket, 380. Knot, 134. Health insurance, 257. Hemisphere, 157. Highest common factor, 88. Holidays, 313. Horizontal addition, 12. Horsepower, 177. Household expense, 21. How to read numbers, 4. Land measure, metric, 183. surveyor's, 134. Lateral area, 159. Lawful money, 50. Least common multiple, 89. Legal rate of interest, 285. Letter of credit, 369, 370. Lever, 188. INDEX. 427 Liabilities, 72. Liability insurance, 257. License, 269, 274. Life insurance, 259. annual premiums, 261. values, 260. Limited partner, 388. Linear measure, 134. Linear metric measure, 182. Linear scale, 195. Lines, 143. Liquidation, 338. Liquid measure, 133. Loans, 306, 345. Long division, 37. Longitude, 165. Loss, 72. Lumber, 160. Maker of note, 307. Making change, 54. Map length, 202, 203. Margin, 343. Marking goods, 232. Marine insurance, 265. Maturity, 172. amount due at, 319. of negotiable instruments, 312. Mean, 43. Means, 190. Measure, angular, 134. apothecaries', 133. avoirdupois, 133. circular, 134. comparative, 133. composite, 177. cord, 160. counting, 134. cubic, 135. dry, 133. extension, 134. linear, 135. liquid, 133. long, 134. metric, 182. money, 135. sea, 134. square, 134. surveyor's, 134. time, 135, 162. Measure, temperature, 175. Troy, 133. value, 135. weight, 133. Measuring, 136. Mensuration, 143-163. Merchant's rule, 317. Meridian, 164, Metric equivalents, 184. Metric system, 182. Mexican money, 135. Mints, 51. Minuend, 18. Mixed numbers, 92. division of, 101. multiplication of, 99. Mixtures, 223. Money, circulation of U. S., 52. recoinage of U. S., 52. table of foreign, 135. table of U. S., 53. United States, 49. Money order, 355. form of express, 356. rates, 356, 368. Multiple, 86. common, 89. Multiplication, 25. aliquot parts, 25. commutative law, 25. decimal, 31. denominate, 140. fractional, 97. shortening, 28-30. tables, 26. Multiplier, 25. Negotiable paper, 307. bank draft, 361, 362. certificate of deposit, 362. check, 357. maturity, 312. promissory note, 307. sight draft, 309. time draft, 310, 311. Net capital, 72, 388. Net cost, 246. Net gain, 72. Net insolvency, 388. 428 INDEX. Net loss, 72. Net proceeds, 246. Newspaper, advertising, 79. advertising rates, 81. circulation, 12, 80. Nines, casting out, 11. Nominal partner, 388. Normal, 43. Notation, Arabic, 3. Roman, 5. Notes, 307, 320. Notice of protest, 308. Numbers, 1. abstract, 1. composite, 86. compound, 2. concrete, 1. copying, 8. denominate, 1, 132. dictation of, 7. like, 1. powers of, 121. prime, 86. roots of, 122. simple, 2. unlike, 1. Numeration, 4, 5. Numerator, 91. Obtuse angle, 143. Old age insurance, 262. Open policy, 264. Panama canal, 223. Papering, 153. Paper table, 135. Parallel line graphs, 207. Parallel lines, 143. Parallelogram, 144. Parcels post, 57, 58. Partial payments, 314. indorsements, 315. merchant's rule, 317. United States rule, 315. Partition, 36. Partitive proportion, 387. Partners, 388. Partnership, 387. Partnership statement, 392. Par value, 333. Passenger mile, 178. Payee, 307. Payments, 61, 306. Pay roll, 63, 65. Pay roll check, 358. Percentage, 213. comparative table, 214. to find base, 221. to find percentage, 215. to find rate, 217. Perimeter, 145. Periodic interest, 300. Perpendicular line, 143. Pitch of a roof, 151. Plane figures, 143. Plane surfaces, 143. Plastering, 153. Plotting, 194, 197, 199. Policy of insurance, 256. open, 264. valued, 264. Poll tax, 270. Polyhedron, 156. Port of delivery, 279. Port of entry, 279. . Postage, 56. report, 59, 60. stamps, 57. Postal information, 57. Postal money order, 356. Post oflBces and routes, 56. Posters, 78, 79. Power of a number, 121. Practical measurements, 143-175. Precipitation chart, 209. Preferred stock, 334. Premium, 256. rates of insurance, 261. wage, 69. Present worth and true discount, 299. Present worth, of a debt, 299. of a firm, 72, 388. Price per hundred or thousand, 116, Prime cost — See Net cost. Prime factor^ 86. Prime meridian, 164. Prime number, 86. Prmcipal, 285. Prism, 156. INDEX. 429 Problem analysis and solution, 118. Proceeds of a note, 320. Proceeds of a sale, 246. Production of barley, 193. Profit as rate of selling price, 228. Profit and loss, 225. Profit and loss account, 390 Profit sharing, SSO-391. Promissory note, 307. Proper fraction, 92. Property tax, 269. Proportion, 190. compound, 190. continued, 190. direct, 191. indirect, 191. partitive, 387. principles of, 190. simple, 190. terms, 190. Proprietor's account, 75. Proposal, 383. Protest, 388. Purchase, 249. Pyramid, 156. Quadrilateral, 144. Qualified endorsement, 308. Questions for discussion, 55, 90, 105, 195, 215, 232, 238, 313, 318, 319, 388, 396. Quotation, 334, 338. Quotation list, 251, 339. Quotient, 35. Radius, 145. Rate, 213, 217. Rate card, 81. Rate of exchange, 364, 366. fluctuation of, 364, 371. Rate of interest, 285. Ratio, 186. direct, 186. inverse, 186. signs of, 186. terms, 186. value, 186. Ration list, 27. Reading and writing numbers, 3-5. Reaumur thermometer, 75. Receipts of U. S. gov't, 277. Receipts for payment, 74, 404. Reciprocal of a fraction, 101. Record, of advertising, 84, 85. of postage, 59, 60. of professional visits, 71. Rectangle, 145. Rectangular solid, 156. Reduction of denominate numbers, 136, 137. Reduction of plotted lengths, 197. Registered bond, 337. Relations, fractional, 106. Remainders, 35. Requisition, 411. Requisition book, 412. Resistance, 192. Results, exact and approximate, 40. Right angle, 143. Right angled triangle, 144. Roman notation, 5. Root of numbers, 122. S Salary, 62. Salary pay roll, 63. Sales (agent), 247. Savings accounts, 326, 328, 329, 331., interest on, 328. Savings banks, 326. Scalene triangle, 144. Scales, 194. Sea measure, 134. Selling price, 226. Services, biU for, 69. payment for, 61. Shareholder, 332. Short division, 36. Short methods, multiphcation, 28, 30, 100. division, 38. Sight draft, 309, 361. Signs, algebraic and numerical, 47- Simple interest, 285. Sinking fund, 337, 377, 378. Six per cent, method, 290. Sixty-day method, 287. Solids, 156. Solution of problems, 118 Solving equations, 45. 430 INDEX. Specifications, 383. Specific duties, 277, 282. Specific gravity, 41. Sphere, 157. Spherical surface, 156. Square, 144. Square (lumber measure), 151. Square measure, 134. metric, 183. Square of a number, 121. graphic illustration, 128. Square root, 123. graphic, 128. Standard time, 167. Standard measures, 133-135. Standard weights, 133-135. Statement, monthly, 410. Statistics! 14^ 193-211. Stock broker, 343. Stock certificate, 333. Stockholder, 332. Stock dividends, 336. Stock record, 20. Stock, 332. buying and selling, 342. classes of, 334. quotations, 334, 338. shares, 333. Storage, 414. average, 416. bill, 415. term, 414. Subtraction, 18. checking, 18. complement method, 19. of decimals, 22. of denominate numbers, 139. of fractions, 96. Subtrahend, 18. Surplus, 335. Surveyor, 280. Surveyor's measure, 134. Symbols, 419. Tables, apothecaries' weight, 133. avoirdupois weight, 133. Canadian money, 135. circular measure, 134. circular measure and time, 165. conversion, 133. Tables, counting, 133. cubic measure, 135. denominate numbers, 133-35. dry measure, 133. English money, 135. French money, 135. German money, 135. import duties, 278. insurance premiums, 261. interest, 292. linear measure, 134. liquid measure, 133. . metric, 182. metrical equivalents, 184. Mexican money, 135. multiplication, 26. paper, 134. siiJcing fund, 378. square measure, 135. tax, 273. time, 137, 170, 171. Troy weight, 133. United States money, 53. values, insurance, 260. wage, 67. Tabulations, 14. Tanks, capacity of, 162. Tariff, 277. Tariff act, 277. Taxation, 269. Tax table, 273. Telegraph money order, 357. Temperature, 175. scale reduction, 176. Temperature chart, 209. Term of discount, 319. Term of note, 319. Terms of a fraction, 91. Terms of a sale. 405. Tests of divisibihty, 86, 87. Textile manufactures, 208. Thousands, to find cost of, 116. Timber supply, 207. Time, calculation table, 170-171. commercial, 168. difference in, 166. standard, 167. Time card, 68. Time draft, 309-311. Time of discount, 119 Time sheet, 68. INDEX. 431 Ton mile, 178. Train mile, 178. Transit insurance, 266. Transportation, 47. Travelers' check, 368. Triangle, 144. Troy weight, 133. True discount, 299. Turn-over of capital, 227. Types of problems, 118, 119. Value of foreign coin, 281. Valued policy, 264. Visits, record of, 71. Volume, cylinder, 159. prism, 157. Undivided profits, 335. Unit, of freight traffic, 178. of irrigation, 177. of passenger traffic, 178. of power, 177. of train traffic, 178. of work, 177. United States customs, 275. United States money, 49, 50. United States rule, 315. Units, 1. fractional, 91. Unity, 1. Usury, 285. Wages, 62, 64. Wage earners, 61. Wage pay roll, 65. Wage table, 67. Wall signs, 78. Warehouse, 280. Watered stock, 325. Weight, apothecaries', 133. avoirdupois, 133. comparison of, 133. long ton, 133. miscellaneous, 133. metric, 183. Troy, 133. Whole life policies, 259. Printed in the United States of America. T HE following pages contain advertisements of Macmillan books on kindred subjects. TEXT-BOOKS On Commercial, Industrial and Allied Subjects Tarr & Von Engeln. LABORATORY MANUAL FOR PHYS- ICAL AND COMMERCIAL GEOGRAPHY . In press Moore. INDUSTRIAL HISTORY OF THE UNITED STATES In press Cheyney. industrial AND SOCIAL HISTORY OF ENGLAND . $1.40 CoMAN. INDUSTRIAL HISTORY OF THE UNITED STATES 1.60 King. ELEMENTS OF STATISTICAL METHOD . 1.50 BuRCH & Nearing. elements OF ECONOMICS . 1.00 Ely & Wicker. ELEMENTARY PRINCIPLES OF ECONOMICS 1.00 Blackmar. economics : For High Schools and Academies 1.20 THE MACMILLAN COMPANY 64-66 FIFTH AVENUE, NEW YORK BOSTON CHICAGO DALLAS ATLANTA SAN FRANCISCO Macmillan's Commercial Series Edited by CHEESMAN A. HERRICK President of Girard College, formerly Director of School of Commerce Philadelphia Central High School Each volume 12mo. Cloth Altmaier's Commercial Correspondence with Postal Information A simple, practical text-book widely used with great success. BiGELOw & Arnold's Elements of Business Arithmetic A new, practical, elementary treatment of the subject suitable for use in the last years of the grammar school or the first years of the high school course. Bogle's Comprehensive Bookkeeping A complete manual for the use of students in commercial courses. Blanks and a Teachers' Manual, made to accompany this text are available at moderate prices, and these books form a good working basis for the commercial course in secondary schools. Herrick's The Meaning and Practice of Commercial Education This book explains the purpose and describes the actual working of commercial schools. It treats commercial education from various points of view, and shows that this form of instruction is a result of present economic conditions and a natural step in our national de- velopment. An appendix supplies a number of curricula for schools of various grades, and there is a serviceable bibliography of the subject. Thurston's Business Arithmetic for Secondary Schools The book is particularly fitted for use in commercial courses, but it may be used in any practical course in high school arithmetic. It deals with processes and business forms used in modern commercial practice. Trotter's Geography of Commerce In this book there is a union of the two phases of thought which form the basis of the geography of commerce. It interprets the activities of men and of organizations of men as they are dependent upon physical conditions. THE MACMILLAN COMPANY 64-66 FIFTH AVENUE, NEW YORK BOSTON CHICAGO DALLAS ATLANTA SAN FRANCISCO Reference Books on Commercial Subjects Adams and Sumner . . . AVEBURY . . . Bastable . . . Carlton . . . Gheyney . . . Clare » . . . Clark . . . . Clark .... Coman . . . . COMAN .... Cunningham AND McArthur, Ely ..... Farrer and Griffin . . . Fay ..... FiSK GiBBIN .... Gow Graham and Oliver . . . Graham and Oliver . . Graham and Oliver . Labor Problems $1.60d On Municipal and National Trading .... l.OOz Theory of International Trade 1.25z Education and Industrial Evolution 1.2oa Industrial and Social History of England . . 1.40d A. B. C. of Foreign Exchange 1.2oz Essentials of Economic Theory as Applied to the Problems of Industry and Public Policy . 2.00a Control of the Trusts ,1.00a Industrial History of the United States . . . 1.60d Economic Beginnings of the Far West. . . . 4.00a Outlines of English Industrial History . . . 1.40d Evolution of Industrial Society 1.25d The State in its Relation to Trade .... l.OOz Co-operation at Home and Abroad .... 3.00z International Commercial Policies .... 1.25a History of Commerce in Europe 90f Marine Insurance l.lOz French Commercial Practice, Part I 60z Part II l.OOz German Commercial Practice, Part I : 60z Part II 1.25z Spanish Commercial Practice, Parti 60z Partn 1.25z THE MACMILLAN COMPANY BOSTON 64-66 FIFTH AVENUE, NEW YORK CHICAGO DALLAS ATLANTA SAN FRANCISCO REFERENCE BOOKS Graham and Oliver Graham and Oliver . Hatch and Valentine Hepburn . Herbertson Hooper and Graham . Hooper and Graham Hooper and Graham , Hull . . Jernigan Levy . . Lyde . . . Lyde . . MacGregor Meili . . monteverde Monte VERDE Nearinq . . Robertson Schmoller . Scott . . Sparling . Stevens . . Tarbell . . Taussig TiLLYARD Foreign Trader's Correspondence Handbook . .75z Foreign Trader's Dictionary of Terms and Phrases in English, German, French and Spanish l.OOz Weights and Measures of International Com- merce 80z Artificial Waterways and Commercial Devel- opment 1.00a Man and His Work 60x Commercial Education 1.50z Home Trade l.OOz Import and Export l.OOz Problems in Banking and Currency .... 3.50a China in Law and Commerce 2.00a Monopoly and Competition . 3.25z Man and His Markets 50x A Short Commercial Geography 90f Industrial Combination 2.50z International Civil and Commercial Law . . 3.00a Spanish and English Commercial and Technical Terms 70f Spanish Idioms and Their Equivalents 80f Wages in the United States 1.25a Trade and Tariffs 1.50z The Mercantile System 75d Increasing Human Efficiency In Business . . 1.25a Introduction to Business Organization .... 1.25a Industrial Combinations and Trusts .... 2.00d The Tariff in Our Times 1.50a Principles of Economics. Two vols., each . 2.00d Banking and Negotiable Instruments: A Man- ual of Practical Law 2.00z THE MACMILLAN COMPANY 64-66 FIFTH AVENUE, NEW YORK BOSTON CHICAGO DALLAS ATLANTA SAN FRANCISCO 459940 UNIVERSITY OF CALIFORNIA LIBRARY